Sunteți pe pagina 1din 214

©Educational Training Services, A Division of Maple Leaf International Consulting, Inc.

Not to be reproduced, copied or distributed. Intended for use only by the registrant.

MODULE II tells us that X is either 2 or –2. If X is 2,


QUANTITATIVE ASSESSMENT then it is positive; if it is –2, another
CONCEPTS REVIEW AND EXERCISES value for X consistent with our logical
interpretation of the ‘evidentiary’

G
information, then X is NOT positive.
MAT Quantitative assessment Can you see that we have two likely
measures your quantitative values for X consistent with our logical
reasoning skills and tests your ability to interpretation of the ‘evidentiary
make logical interpretation of statement’ (X2 = 4) but there is no
information that is presented to you, or ‘consistency’ with respect to the decision
conceptually known to you. You will we can arrive at across the two different
then use the ‘logical interpretation’ to ‘scenarios’? Obviously, we cannot make
make ‘logical decisions’. A ‘logical a ‘logical decision’ about whether X is
interpretation’ is one that considers all positive or not on the basis of the
valid explanations of agiven information. ‘evidence’ X2 = 4.
For example, if the information is that X
is an integer, then a logical On the GMAT, you will treat as
interpretation of this statement will be X ‘evidence’ any ‘IF’ information that is
could be any positive whole number, any given to you or any information that is
negative whole number, or Zero. known to you as ‘concept’. You will
Similarly, if the information presented to then be required to interpret this
you is |X| = 2, then the logical ‘evidence’ in a logical fashion, and then
interpretation of this information is that X use the logical interpretation to decide
is either 2 or –2. Likewise, if the whether or not any decision having
information given to you reads that the logical consistency is feasible.
‘average of X, Y, and Z is 10’, your
‘logical interpretation’ of this statement RULE: If the logical interpretation of
will read: “X, Y, and Z can be any value an ‘evidence’ presents more than one
they could be as long as they ADD UP valid scenario, then neither scenario
to 30’. alone is a logical interpretation. For
example, if the ‘evidence’ is ‘X times Y
MAKING LOGICAL DECISIONS USING is positive’, our ‘logical interpretation’ of
LOGICAL INTERPRETATION OF this statement presents two ‘valid’
INFORMATION GIVEN TO YOU scenarios: X and Y are both positive or
X and Y are both negative. Neither X nor
The process of making ‘logical decisions’ Y can be zero.
involves ‘testing for consistency across
all logical interpretations of the given Notice that our ‘logical interpretation’
information’. For example, if the gave rise to three valid scenarios about
question was: If X2 = 4, is X positive? the values for X and Y, and neither
We will have to read this question as scenario alone is a logical interpretation.
follows: Given that X2 = 4, can you For example, if you made a ‘decision’ by
determine with logical certainty that X using the scenario that ‘X and Y are
is greater than Zero? Our logical positive so that X times Y is positive’,
interpretation of the information X2 = 4 then your ‘decision’ is ‘not logical’.
Some of the materials used in this module are taken from disclosed editions of GMAT, and no monetary
value is included in the cost of this module for supply of these materials. These materials are strictly used
for illustration of concepts discussed. Page 1
©Educational Training Services, A Division of Maple Leaf International Consulting, Inc.
Not to be reproduced, copied or distributed. Intended for use only by the registrant.

Your logical decision-making process ‘assumed’ that X and Y cannot both be


must consider all ‘three’ valid scenarios negative so that X times Y is positive.
and test for ‘consistency’ across all valid
scenarios. If the question was: “If X Notice that this process of making
times Y is positive, is X+Y positive?” ‘logical decisions’ is analogous to our
you will read this question as “Can you ‘logical analysis’ of an ‘illogical
determine with logical certainty argument’ presented to us in the
whether X+Y is positive, given that X Analytical Writing Assessment or in the
times Y is positive?’ Critical Reasoning Section of the
GMAT. IF the evidence was: “John
(Remember that the ‘decision questions’ made a GMAT score of 500 out of 800”,
on the GMAT data sufficiency do not one interpretation of this information is
use English language statements. They that John did not take our prep course.
use the variant of ‘cling-on’, and you Another interpretation could be that John
must ‘translate’ the question into was not well on the day of the test. Or,
English language in order to grasp the there could be any number of other
real meaning of decision questions. ‘IS X interpretations, and neither interpretation
+ Y positive’ is cling-on for ‘can you alone is a valid interpretation. We applied
determine with logical certainty whether the same rule when we analyzed an Issue.
X + Y is positive, given X times Y is If the issue was: Why do you figure that
positive?’) courtesy is a vanishing virtue in an urban
environment?, one explanation could be
Our logical interpretation of the that the urban dwellers are prone to
information that ‘X times Y is positive’ higher levels stress brought on by the
gave rise to three valid scenarios: aggressive and competitive urban
environment. Another explanation could
X and Y are both positive, or be that there is a distrust of strangers in
X and Y are both negative, and an urban area. A third explanation could
Neither X nor Y can be Zero. be that the principle of reciprocity or
‘pay back’ does not work well in an
In the first ‘valid’ scenario, X + Y is urban environment because the ‘good
positive. In the second ‘valid’ scenario, deed’ done to a stranger is not ‘returned’
X + Y is negative. In the third valid to the urban dwellers, with the result that
interpretation of the evidence, X + Y the urban denizens have less motivation
cannot be Zero. to actively practice courtesy.

Therefore, our ‘logical decision’ is that X As you can see, our ‘logical
+ Y can be positive or negative, and interpretation’ of the evidence that
neither decision alone is a logical ‘courtesy is a vanishing virtue in an urban
decision. We must conclude that we area’ is open to at least three different
cannot determine with logical certainty ‘valid’ explanations, and neither
whether X + Y is positive, given than X explanation alone is a logical explanation.
times Y is positive. If we were tempted (That is why we asked you to come up
to do so, we would be ‘guilty’ of making with at least two different valid and
‘illogical decisions’ because we logical explanations for the ‘issue’ in
Some of the materials used in this module are taken from disclosed editions of GMAT, and no monetary
value is included in the cost of this module for supply of these materials. These materials are strictly used
for illustration of concepts discussed. Page 2
©Educational Training Services, A Division of Maple Leaf International Consulting, Inc.
Not to be reproduced, copied or distributed. Intended for use only by the registrant.

Module I so that the reader will know be scored in each of the two sections:
that you are not making ‘illogical’ Problem Solving and Data Sufficiency.
explanations.)
QUANTITATIVE ASSESSMENT is PROBLEM SOLVING
the first of the two ‘adaptive’ test
sections on the GMAT. The ‘adaptivity’ Even though the section is titled ‘Solving
is a process by which the GMAT test of Problems’, not all ‘problems’ will
continuously adjusts the difficulty level of involve crunching of numbers in the
the questions presented to you. This course of ‘solving a problem’. Some
‘adjustment’ is made using ‘artificial problem solving questions will simply ask
intelligence’ that judges your comfort you to do quantitative reasoning and
level by the way you are picking answer make a logical decision. Consider the
choices to the problems presented to following problem:
you. This manual is constantly updated
in order to reflect changes taking place What is the units’ digit of 1499?
on the GMAT and to feature GMAT
problems that are truly typical of the The exponent of 99 on 14 looks ominous
recent GMAT questions. Therefore, and daunting but the simple rule of
work through this manual, not just ‘read GMAT is that the more daunting and
through’ it. Be sure to practice with all ominous a problem appears, the more
the ‘examples’, not merely read through likely it is a simple reasoning problem.
them. Because you will be taking the You will engage in your reasoning
test in a timed-environment, you need to process by taking some ‘baby steps’:
make sure that your recognition of the
type of problem presented to you and 141 = 14 The units’ digit is 4
your ability to set up the problem 142 = XX 6 The units’ digit is 6.
expeditiously are up to snuff. This goal Notice that we did not care about the
can be accomplished through practice, tens or the hundreds’ digits here because
and through several repetitions. As the the problem is not about digits other than
saying goes, excellence is not an act but a the units’ digit.
habit, and habit is a matter of repetition 143 = xxx4 Again, we are concerned
of the same act over time. Also, use this about the units’ digit and not the other
MODULE along with the Quantitative digits.
Preprogram assignment file and with We already begin to see a pattern
Quantitative Concepts in a nutshell file. developing. The units’ digit cycles
between 4 and 6, with 4 as the units’
Assessment of your quantitative digit for exponents 1 and 3, and with 6 as
reasoning skills will be made by asking the units’ digit for exponents 2 and 4.
you to deal with about 18 to 19 problems We will see the same pattern throughout
in Problem Solving and another 18 to 19 and because the exponent 99 is an odd
questions in Data Sufficiency format. integer, we will ‘logically conclude’ that
Out of the 37 questions given to you, the units’ digit of 1499 will be 4. Notice
only 28 will be scored, and the other 9 that we did not do a significant amount
questions will be experimental questions. of ‘number crunching’ in this ‘problem’
This means that exactly 14 problems will
Some of the materials used in this module are taken from disclosed editions of GMAT, and no monetary
value is included in the cost of this module for supply of these materials. These materials are strictly used
for illustration of concepts discussed. Page 3
©Educational Training Services, A Division of Maple Leaf International Consulting, Inc.
Not to be reproduced, copied or distributed. Intended for use only by the registrant.

but relied on a ton of reasoning to arrive


at a logical conclusion.

Consider another problem in which you Therefore, the sum of the 25th and 27th
will be asked to do similar ‘reasoning’ term will be
and to recognize a pattern in order to 224 + 226 = 224 (20 + 22 ) = 5•224
reach a logical conclusion.
Did we do any number crunching here?
Consider the following PROBLEM: We did some when we did some
‘factoring’ of the terms in the final steps
In the sequence 1, 2, 4, 8, 16, 32,……… but most of the problem was about
what is the sum of 25th term and 27th reasoning and about recognizing a
term? pattern in order to make a logical
extrapolation.
We notice that the pattern is such that
every value after the first one is twice the Of course, there will be some problems
preceding but because the problem asks that will require some reasoning and
us to ‘add’ values 25 and 27, we figure some crunching of numbers, but you will
that there must be a ‘better way’ to deal never get a problem (at a high difficulty
with the problem than to extend the level) that does not involve any reasoning
series all the way down to 27th term. at all.

One of the skills that will come in handy DATA SUFFICIENCY


when you work on the GMAT is your
ability to size up the problem and to GMAT Data Sufficiency is about
‘transform’ it to an equivalent form. making a logical interpretation of the
evidence presented to you in two
If we transformed the sequence as a different statements and then to use the
sequence of exponents, we will be able to logical interpretation to decide whether a
deal with the problem more effectively. logical decision involving ‘logical
The transformed sequence is certainty’ can be made about the
20, 21 , 22 , 23 , 24 , 25, ………… ‘decision question’ posed in the question
stem.
We now notice that the value of the
exponent in the First term is 0, the value Each Data Sufficiency problem will
of the exponent in the second term 1, the consist of a Question Stem, which may
value of the exponent in the third term 2, also contain some ‘evidence’, and two
and so on. We can see that the value of different statements constituting the
the exponent is 1 less than the ordered evidence to be examined in the course of
positional value of the term. We can making a decision.
now ‘logically conclude’ that the 25th
term will be 224 and the 27th term will be
226.

Some of the materials used in this module are taken from disclosed editions of GMAT, and no monetary
value is included in the cost of this module for supply of these materials. These materials are strictly used
for illustration of concepts discussed. Page 4
©Educational Training Services, A Division of Maple Leaf International Consulting, Inc.
Not to be reproduced, copied or distributed. Intended for use only by the registrant.

FORMAT 1:
{STEM EVIDENCE}, [DECISION QUESTION] FORMAT 2

STATEMENT 1 [DECISION QUESTION]


STATEMENT 2
STATEMENT 1
If you come across a Data Sufficiency problem STATEMENT 2
having the above format, you should read it as
follows: EXAMPLE:

[DECISION QUESTION] “How old is John?”


{STEM EVIDENCE}, STATEMENT1 1. John is 3 years older than Samantha.
2. 3 years from now, John will be twice
{STEM EVIDENCE}, STATEMENT2 as old as Samantha.
REMEMBER: THE ‘STEM
EVIDENCE’ separately belongs to each Each Data Sufficiency problem will have
statement, and must be used as FIVE answer choices, which are
‘additional evidence’ presented. arranged in the SAME ORDER for every
problem.
EXAMPLE:
ST 1 ALONE IS SUFFICIENT BUT
“IF X IS AN INTEGER, IS X AN ST 2 ALONE IS NOT.
EVEN INTEGER?” ST 2 ALONE IS SUFFICIENT BUT
ST 1 ALONE IS NOT.
1. X > 0 ST 1 AND ST 2 COMBINED IS
2. X < 3 SUFFICIENT BUT NEITHER
ALONE IS.
You should read the above problem as: EACH STATEMENT ALONE IS
SUFFICIENT.
“IS X AN EVEN INTEGER?” NEITHER STATEMENT ALONE
OR COMBINED IS SUFFICIENT
1. X IS AN INTEGER AND X > 0 TO MAKE A LOGICAL
2. X IS AN INTEGER AND X < 3. DECISION.

Notice that Statement 1 alone is


considered to be GOOD for making a
LOGICAL decision with in options 1 and
4. Statement 2 alone is considered to be
GOOD for making a LOGICAL decision
with in options 2 and 4.

Some of the materials used in this module are taken from disclosed editions of GMAT, and no monetary
value is included in the cost of this module for supply of these materials. These materials are strictly used
for illustration of concepts discussed. Page 5
©Educational Training Services, A Division of Maple Leaf International Consulting, Inc.
Not to be reproduced, copied or distributed. Intended for use only by the registrant.

DATA SUFFICIENCY The question is: Can you determine a


TYPES OF QUESTIONS unique value for X on the basis of a
logical interpretation of the evidence
1. UNIQUE VALUE DECISION presented in statement 1 alone first?
2. TRUE OR FALSE DECISION
What is the logical interpretation of the
UNIQUE VALUE decision problems in evidence presented in statement 1? If the
Data Sufficiency typically start in WHAT ‘absolute value’ of X is 2, then X could
or How, and the TRUE OR FALSE be 2 or – 2. Is that a UNIQUE value?
DECISION problems typically start in IS By no means. Therefore, we must
conclude that the evidence presented in
EXAMPLE: statement 1 alone does not allow us to
make a unique value decision about the
UNIQUE VALUE DECISION: value for X. IF Statement 1 alone is not
sufficient for a unique decision, then
“What is X?” neither of the two options in the answer
1. | X | = 2 choices, option 1 and option 4, can be
2. X < 0 good answer choices. We must,
therefore, eliminate options 1 and 4. We
You should keep in mind that the have three options remaining at this stage
DECISION QUESTION in the stem is of analysis.
not worded in English language. It is a
mutant form of the ‘klingon’ language We must now proceed to examine
we are used to hearing on Star Trek. statement 2, and determine whether, on
You must, therefore, TRANSLATE the the basis of a logical interpretation of the
‘klingon’ statement to English statement evidence presented in statement 2, we
before you process the question. can make a unique value decision with
respect to the value for X.
Mutant KLINGON: “What is X?”
What is the LOGICAL
ENGLISH: “Can you determine with INTERPRETATION of the evidence
logical certainty a UNIQUE VALUE presented in statement 2?
FOR X by using a LOGICAL
INTERPRETATION of the EVIDENCE Statement 2 tells us that X is a negative
presented in statement 1 alone and on the value. How many LIKELY negative
basis of evidence presented in statement values for X can we think of? Unlimited.
2 alone, or , if need be, on the basis of Is it possible to arrive at a UNIQUE
the COMBINED information presented value decision for the value of X on the
in statements 1 and 2?” basis of a logical interpretation of the
evidence presented in statement 2?
Now that we have ‘translated’ the Hardly. WE must, therefore, conclude
‘klingon’ statement to ‘English’ that statement 2 alone is also not
statement, let us proceed to ‘attack’ the sufficient to make a UNIQUE VALUE
problem. decision about X. Statement 2 is
considered to be good for making a
Some of the materials used in this module are taken from disclosed editions of GMAT, and no monetary
value is included in the cost of this module for supply of these materials. These materials are strictly used
for illustration of concepts discussed. Page 6
©Educational Training Services, A Division of Maple Leaf International Consulting, Inc.
Not to be reproduced, copied or distributed. Intended for use only by the registrant.

logical decision in options 2 and 4. We NOTE: You must ‘COMBINE’ the two
have already eliminated option 4 when statements ONLY WHEN neither
we concluded that statement 1 alone was statement ALONE is sufficient to make a
not sufficient to make a logical decision unique value or a logical certainty
about the value for X. We must now decision. If any one statement alone was
eliminate option 2. We have two answer sufficient, you will NOT proceed to
choices remaining: option 3 and option combine the two statements.
5.

If the ‘combined’ evidence of statements


1 and 2 is sufficient to allow us to make a Example of a problem in which we will
unique value decision for the value of X, not combine the information in
then we will pick option 3. If, after statements 1 and 2:
examining the combined evidence of the
statements 1 and 2, we cannot determine If the area of a square S is equal to the
a UNIQUE value for X, then we must area of the rectangle R, what is the
choose option 5. perimeter of the square region?

Let us COMBINE the two statements 1. The length of ONE side of the
and see whether the combined evidence rectangle R is twice the length of
allows us to make a unique value the side of the square S.
decision about X. 2. The area of the rectangle R is 49.

The combined evidence reads as follows: (Translation: WE have STEM


EVIDENCE that must belong to each
3. X is either 2 or – 2, AND X is statement independently. The question
negative. is: Can you determine a UNIQUE
VALUE FOR THE PERIMETER of the
Looks like we have a winner here. If X square region S on the basis of a logical
is negative and X is either 2 or negative interpretation of the evidence presented
2, then it must be – 2. A positive value in statement 1 alone and on the on the
for X will not satisfy the statement 2 and basis of statement 2 alone, OR on the
must be discarded as not a value for X. basis of the ‘combined’ evidence of
statements 1 and 2?)
We can see that the COMBINED
evidence of statements 1 and 2 allows us
to make a unique value decision about X,
and we must pick option 3.

Some of the materials used in this module are taken from disclosed editions of GMAT, and no monetary
value is included in the cost of this module for supply of these materials. These materials are strictly used
for illustration of concepts discussed. Page 7
©Educational Training Services, A Division of Maple Leaf International Consulting, Inc.
Not to be reproduced, copied or distributed. Intended for use only by the registrant.

Because the Stem Evidence must be therefore, decide that statement 1 alone
treated as part of each statement alone, is NOT sufficient to make a logical
we have the two statements read as decision about the unique value for the
follows: perimeter of the square region because
we cannot determine a unique value for
1. The area of the rectangle R is equal the length of the side ‘a’ of the square.
to the area of the square S, AND the
length of one side of the rectangle R We must now eliminate options 1 and 4,
is twice that of the square S. and proceed to examine statement 2
2. The area of the rectangle R is equal alone and decide whether, on the basis of
to the area of the square S, AND the statement 2 alone, we can come up with
area of the rectangle is 49 square a unique value for ‘a’ and, therefore, for
units. ‘4a’.

Be sure to process the stem evidence in STATEMENT 2


the manner indicated above so that you On the basis of the redefined statement 2
will not FORGET to use stem evidence. that combines the stem evidence with the
A fatal mistake committed by the test- additional information presented in
takers is that they forget about the stem statement 2, we have
information and make some bad a2 = L•W = 49
decisions. Does this allow us to determine a unique
value for the side of the square region?
Yes, it does. WE can uniquely determine
Let us now begin to examine statement 1 that the square region must be 7 units on
alone and see whether a ‘logical each side, and the perimeter must be a
interpretation’ of the evidence as unique 28 units. Notice that although the
‘redefined’ above to include the stem values for a turn out to be 7 and – 7, we
evidence is sufficient to make a unique have to discard the negative value
value decision about the perimeter of the because the value for the side of a square
square region S. ( A unique numerical cannot be a negative value.
value for ‘4a’, the perimeter of the
square region of side S.) WE must now conclude that we can
make a UNIQUE value decision for the
STATEMENT 1 perimeter of the square region S on the
If ‘a’ is the value of each side of the basis of the redefined evidence in
square S, and if L is the length and W, statement 2 alone. We must, therefore,
the width of the rectangular region R, pick option 2.
then we have a2 = L•W and L = 2a.
Because any one statement alone was
Our ‘logical conclusion’ of the above sufficient to make a logical and unique-
evidence is that W must be ½ of a. value decision about the perimeter of the
Notice that there are 3 variables: a, L, square region S, we will NOT combine
and W, and we cannot uniquely solve for the evidence of statements 1 and 2.
any of them on the basis of TWO
simultaneous equations. We must,

Some of the materials used in this module are taken from disclosed editions of GMAT, and no monetary
value is included in the cost of this module for supply of these materials. These materials are strictly used
for illustration of concepts discussed. Page 8
©Educational Training Services, A Division of Maple Leaf International Consulting, Inc.
Not to be reproduced, copied or distributed. Intended for use only by the registrant.

TRUE OR FALSE DECISIONS in Let us proceed to examine statement 1


DATA SUFFICIENCY alone first and decide whether the logical
interpretation of the evidence presented
Data Sufficiency questions that start off in statement 1 alone is sufficient to make
in ‘is’ are typically ‘true or false’ a decision having ‘logical certainty’
decision questions. about whether or not X is a positive
integer.
EXAMPLE:
LOGICAL CERTAINTY DECISION
IS X a positive integer? is one that is CONSISTENT across all
valid interpretations of the evidence.
1. X2 > 4
2. X3 < - 8 STATEMENT 1
Once again, we need to TRANSLATE
the ‘mutant klingon’ decision question in A logical interpretation of evidence in
the stem to English statement. statement 1 tells us that X could be
greater than 2 or less than negative 2.
MUTANT KLINGON: What are the values for X consistent with
“Is X a positive integer”? the above interpretation? X could be 3
or 3¼ or – 3 or – 3¼ . As you can see,
ENGLISH TRANSLATION: we can make the EVIDENCE in
“Can you determine with logical certainty statement 1 work for both POSITIVE
whether or not X is positive whole INTEGER VALUES OF X,
number by using the logical NEGATIVE INTEGER VALUES OF
interpretation of the evidence presented X, POSITIVE NON-INTEGER
in statement 1 alone and by using the VALUES OF X AND NEGATIVE
logical interpretation of the evidence NON-INTEGER VALUES OF X. Note
presented in statement 2 alone, OR, if that the values we ‘chose’ to ‘test’ for
need be, by using the COMBINED ‘likely values of X’ were all consistent
evidence of statements 1 and 2?” with a logical interpretation of the
statement 1. Yet, we have the following
decision outcomes across those valid
Of course, any ‘stem evidence’, if given, ‘scenarios’.
must be independently considered part of If X is 3, then it is a positive integer.
each statement. In this problem, there is If X is 3 ¼, then it is NOT a positive
no stem evidence and we will have to integer. (it is positive but not an integer).
work with the evidence presented in the If X is – 3, then it is not a positive
statements alone. integer. (It is an integer, but not
positive).
If X is – 3¼ , then it is neither positive
nor an integer.

Some of the materials used in this module are taken from disclosed editions of GMAT, and no monetary
value is included in the cost of this module for supply of these materials. These materials are strictly used
for illustration of concepts discussed. Page 9
©Educational Training Services, A Division of Maple Leaf International Consulting, Inc.
Not to be reproduced, copied or distributed. Intended for use only by the registrant.

Is there any ‘CONSISTENCY’ across


these ‘valid’ scenarios, consistency with REMEMBER: You do not have to
respect to the ‘true or false’ decision answer in the affirmative in order to be
about whether or not X is a positive able to make a logical certainty decision.
integer? No, there isn’t. The question is: Is it possible for us to
make a unique decision, either
We must, therefore, conclude that strictly confirming that X is a positive integer or
on the basis of a logical interpretation of confirming that X must not be a positive
the evidence presented in statement 1 integer. This decision must be firm
alone, we cannot make a ‘true or false’ across all valid scenarios that are
decision having ‘logical certainty’ about consistent with a logical interpretation of
whether or not X is positive whole the evidence used in the statements given
number. to us.

We must now eliminate options 1 and 4, In order to understand this process


and proceed to examine statement 2. better, consider the decision along the
following lines:
STATEMENT 2
Statement 2 alone allows us to make a
X3 < - 8. logical decision about whether or not X
is a positive integer. And the decision is
Our ‘logical interpretation’ of this that X cannot be a positive integer.
evidence is that X is negative, and must
be less than negative 2. ‘Valid’ values
for X, consistent with the above logical
interpretation, are – 2 ½, - 3, and so on.

What does this interpretation tell us


about whether or not X is a positive
whole number? It tells us that X MUST
NOT BE A POSITIVE WHOLE
NUMBER, and we are able to make a
LOGICAL CERTAINTY DECISION on
the basis of the evidence presented in
statement 2 alone. Statement 2 allows us
to definitively conclude that X CANNOT
BE A POSITIVE WHOLE NUMBER.
We must pick option 2 corresponding to
the decision that a logical certainty
decision can be made solely on the basis
of the information presented in statement
2 alone.

Some of the materials used in this module are taken from disclosed editions of GMAT, and no monetary
value is included in the cost of this module for supply of these materials. These materials are strictly used
for illustration of concepts discussed. Page 10
©Educational Training Services, A Division of Maple Leaf International Consulting, Inc.
Not to be reproduced, copied or distributed. Intended for use only by the registrant.

Let us consider another problem and Let us start with the evidence in
understand how to make logical STATEMENT 1 first.
decisions about ‘true or false’ questions
in data sufficiency. Our logical interpretation of the evidence
in statement 1 tells us that ‘a’ MUST BE
“If a and b are integers, is b an even an even integer and ‘b’ could be odd
integer?” integer or even integer in order to make
the statement 1 work.
1. 3a + 4b is even integer.
2. 3a + 5b is even integer. Notice that we are adding two values to
get an even integer. What are the
different ways in which we can add two
Once again, our first step is to ‘translate’ integer values to get an even sum? Each
the Decision question in mutant klingon integer could be odd integer or each
into English statement. must be even integer. In the evidence,
the term ‘4b’ must be even regardless of
Also, notice that the problem gives us whether ‘b’ is even or odd. (IF you
‘stem evidence’ that must be treated as multiply any integer by an EVEN integer
part of each statement taken alone. such as 4, the result is always an EVEN
integer). For instance, ‘b’ could be 3 and
ENGLISH language Decision 4b is 12, an even integer. ‘b’ could also
QUESTION: be 2 and 4b is 8, an even integer. WE
must logically conclude that the term 4b
“Can you determine with logical certainty cannot be an odd integer because the
whether or not ‘b’ is an even integer, factor 4 will make the term ‘4b’ uniquely
given that a and b are both integers, by even integer. This means that the other
using a logical interpretation of the term ‘3a’ must be an even integer in
redefined evidence in statement 1 (along order that 3a + 5b will be an even
with stem evidence) alone and by using a integer. How do we make the term ‘3a’
logical interpretation of the redefined an even integer? By making ‘a’ an even
evidence in statement 2 (including the integer. Notice that ‘a’ CANNOT be an
stem evidence) alone, OR, if need be, by odd integer because, if it were, then ‘3a’
using a logical interpretation of the will be odd and 3a + 4b will be an odd
combined evidence of statements 1 and integer. That will not be consistent with a
2, taken with the stem evidence?” logical and literal interpretation of the
evidence presented in statement 1.
Our REDEFINED evidence items in
statements 1 and 2 become: Therefore, we logically conclude, on the
basis of a logical interpretation of the
1. a and b are integers, and 3a + 4b is evidence presented in statement 1 alone,
an even integer. that ‘a’ must be an even integer but the
2. a and b are integers, and 3a + 5b is value ‘be’ could be an odd integer or an
an even integer. even integer in order to make the
evidence in statement 1 work. Do we
have consistency across the two valid
Some of the materials used in this module are taken from disclosed editions of GMAT, and no monetary
value is included in the cost of this module for supply of these materials. These materials are strictly used
for illustration of concepts discussed. Page 11
©Educational Training Services, A Division of Maple Leaf International Consulting, Inc.
Not to be reproduced, copied or distributed. Intended for use only by the registrant.

interpretations of the likely values for In the second scenario, ‘a’ and ‘b’ must
‘b’? No. In one interpretation, ‘b’ could be both even integers so that 3a will be
be an odd integer and make the evidence even integer and 5b will also be an even
in statement 1 work; in another, ‘b’ could integer so that 3a + 5b will be an even
be an even integer and also make the integer.
evidence in statement 1 work.
In one of the two valid scenarios, b is
Our logical interpretation of statement 1, odd integer; in another valid scenario, b
in sum, is that ‘a’ must be an even integer is an even integer. Is there consistency
but ‘all bets are off’ about ‘b’: ‘b’ could with respect to the decision about
be an odd integer or an even integer, and whether ‘b’ is even or not across the two
be consistent with a logical interpretation valid scenarios? Hardly.
of the statement 1.
If there is no consistency with respect to
We must now conclude that the evidence the decision across at least two valid
presented in the statement 1 (taken with scenarios consistent with a logical
the stem evidence) alone is not sufficient interpretation of the evidence, then we
to make a logical determination of must conclude that no logical decision
whether or not ‘b’ is an even integer. can be made on the basis of a logical
interpretation of the evidence.
WE must now also eliminate options 1
and 4, and move on to examine statement
2. Therefore, we must now conclude that
statement 2 alone is also not sufficient to
STATEMENT 2 (taken with the stem make a logical decision about whether or
evidence) reads: not ‘b’ is an even integer. We must now
eliminate option 2, and proceed to
‘a’ and ‘b’ are integers, AND examine the combined evidence of
3a + 5b is an even integer. statements 1 and 2, taken with the stem
evidence.
Notice that we have two valid
scenarios: THE COMBINED EVIDENCE
3a and 5b are both odd integers so that READS:
their sum is an even integer. STEM EVIDENCE
OR ‘a’ and ‘b’ are integers.
3a and 5b are both even integers so that STATEMENT 1 INTERPRETATION
their sum is even integer. ‘a’ is even integer and ’b’ could be even
or odd integer.
In order to make the first scenario work, STATEMENT 2 INTERPRETATION
‘a’ and ‘b’ must be both odd integers so ‘a’ is odd integer AND ‘b’ is odd integer,
that 3a will be odd integer, and 5b also OR ‘a’ is even integer and ‘b’ is even
an odd integer so that 3a + 5b will be an integer.
even integer.
The only value for ‘a’ that will satisfy the
evidence in statement 1 and statement 2

Some of the materials used in this module are taken from disclosed editions of GMAT, and no monetary
value is included in the cost of this module for supply of these materials. These materials are strictly used
for illustration of concepts discussed. Page 12
©Educational Training Services, A Division of Maple Leaf International Consulting, Inc.
Not to be reproduced, copied or distributed. Intended for use only by the registrant.

is an ‘even’ integer value for ‘a’ . 3. Do not start with the ‘decision
According to the logical interpretation of question’ and ‘back’ into the
statement 2, if ‘a’ is even integer, then evidence.
‘b’ must be an even integer. WE can see
that ‘a’ an even integer and ‘b’ an even NEED TO KNOW set of information
integer will satisfy the stem evidence,
evidence in statement 1, and the evidence When you deal with data sufficiency,
in statement 2. Even though statement 1 you must predetermine what pieces of
allowed us to conclude that ‘b’ could be information will help you solve the
an odd integer or an even integer, but ‘a’ puzzle or answer the question posed in a
must be even integer, the only valid unique manner. Let us call it the N.T.K
scenario that will satisfy both scenarios list or the Need-to-Know list. Let us say
is an even integer value for ‘a’ and even that the question posed is: How much
integer value for ‘b’. commission did Jane make on her sales in
the first half of 1999? What pieces of
We can conclude on the basis of the information will help us answer this
combined evidence in statements 1 and 2, question? We need to know the amount
taken with the stem evidence, that both of sales she made in the first half of 1999,
’a’ and ‘b’ must be even integers in order and the commission rate at which she
to satisfy both statements. was remunerated. We have to establish
this N.T.K list and then look for these
We must pick option 3. two specific pieces of information in the
statements 1 and 2. If we do not have
We will revisit the data sufficiency drill these two pieces of information, we
later on in this file. But for now, cannot answer the question posed in a
understand that the GMAT will ask you unique fashion. Similarly, if the data
to deal with problem solving and with sufficiency question asks you: “Is X
data sufficiency. In data sufficiency, there equal to Y?”, our N.T.K list will read:
are two kinds of decision problems you • I need to know the value of X and
will come across: unique value decision, the value of Y so that I can compare
and ‘true or false’ decision. the values and determine whether
they are equal or not.
The procedure for making logical
decisions in data sufficiency is as follows: If the question posed reads: What is the
average speed for travel between
1. Make a logical interpretation of the Chicago and Indianapolis?, our N.T.K
evidence presented in statements 1 list will seek out the distance between the
and 2 (taken with stem evidence, if two cities and the time of travel between
applicable). the two cities. In real-life, we go through
2. Use the logical interpretation of the the same process, don’t we? If we want
evidence to make a decision to buy a new car, we need to know
involving logical certainty about the whether we can afford the monthly
decision question. payments for the car and the insurance,
and the parameters determining
affordability - may be our existing
Some of the materials used in this module are taken from disclosed editions of GMAT, and no monetary
value is included in the cost of this module for supply of these materials. These materials are strictly used
for illustration of concepts discussed. Page 13
©Educational Training Services, A Division of Maple Leaf International Consulting, Inc.
Not to be reproduced, copied or distributed. Intended for use only by the registrant.

financial commitments, our income, and You will follow the procedure below
our future obligations for which we must each time you deal with a data
save now. If we want to marry someone, sufficiency problem.
we need to know whether the other • Establish your N.T.K list and be clear
person has temperamental compatibility, about what information will help you
compatible life style habits, and ability to answer the question posed.
make adjustments. In the corporate • Associate choices A and D with
world, if we want to launch a new statement 1, and associate Choices B,
product, we would like to know whether C, and E with statement 2. Although
the target market has a need or want for the real test will not identify the
the product, whether the price is choices as A, B, C, D, and E, you
something that the market will bear, and will call the first choice ‘Choice A’,
whether we have the required the second choice ‘Choice B’, and so
distribution channels and resources to do on.
a successful job of marketing the • Examine statement 1 and look for
product. We might want to call this your NTK information. If statement 1
aspect of our decision-making “doing the provides your NTK information in
home-work”. It is no different when it toto, then you will consider statement
comes to data sufficiency. 1 alone as sufficient for now. In this
event, you will eliminate the choices
What is the danger if we do not B, C, and E (associated with
predetermine requirements? We will be statement 2) out of hand. On the
like a philosopher, who is defined as a other hand, if you do not find the
blind person looking for a black cat in a NTK information in statement 1, you
dark room when it is not there. The will eliminate choices A and D
probability of finding the darn cat is the associated with that statement.
same as the probability of survival for a • If statement 1 was good, then you
snowflake in hell,. You will have the will carry forward choices A and D
same chance of finding the correct to the second statement. If statement
answer if you do not predetermine what 1 was not good for a unique answer,
you want to know and establish your you will deal with the remaining three
N.T.K list in advance before you go to choices associated with statement 2.
examine statements 1 and 2 • You can see that at this stage, your
independently. odds of picking the right answer have
improved to 33% or to 50% from the
In Data Sufficiency, you will be initial 20%. Not a bad deal, huh?
presented a question along with or • SCENARIO 1: Statement 1 was
without any additional information and good. We carry forward A and D,
two statements. Your task is to examine and kill the other three choices, B, C,
the statements 1 and 2 independently and and E.
determine whether the question posed
can be answered in a unique fashion on
the basis of your NTK list.

Some of the materials used in this module are taken from disclosed editions of GMAT, and no monetary
value is included in the cost of this module for supply of these materials. These materials are strictly used
for illustration of concepts discussed. Page 14
©Educational Training Services, A Division of Maple Leaf International Consulting, Inc.
Not to be reproduced, copied or distributed. Intended for use only by the registrant.

described above. If you do not have the


• If you find that statement 2 also has method suggested behind your madness,
the information in your NTK list, you are going to be one disappointed
then you must select D, individual at the end of the test when the
corresponding to “Each statement unofficial score report pops up on the
alone is sufficient”. On the other screen. The rewards for proceeding
hand, if statement 2 does not have methodically are great, and begin with
the NTK information, then you must the popping sound of champagne cork.
select A.
• SCENARIO 2: Statement 1 was We will deal with data sufficiency
NOT good to begin with. We will kill questions as we move along and also
choices A and D, and keep choices later on as a specific area of attention in
B, C, and E to deal with. this file. For now, familiarize yourself
• IF Statement 2 has our NTK with the rules and the methodical
information, then we must select procedures starting with establishing the
choice B corresponding to NTK list.
“Statement 2 alone is sufficient but
statement 1 alone is not”. If Problem Solving questions are designed to test
your basic mathematical skills and your ability
Statement 2 also does not have the
to reason quantitatively and solve quantitative
NTK information, then we must problems. Your ability to conceptualize verbal
eliminate choice B and proceed to descriptions of situations and solve problems
combine statements 1 and 2. We have presented using basic arithmetic, elementary
choices C and E remaining at this algebra and commonly known and used
stage. concepts of geometry will be put to the test in
these sections.
• SCENARIO 3: COMBINE
STATEMENTS 1 AND 2 “Data sufficiency” section asks you to
• If the combined information provides use the same conceptual skills as do
our NTK information, then we must problem solving questions. However,
select choice C, which corresponds decision-making does not involve full
to “Statements 1 and 2 combined are implementation and verifying results.
sufficient but neither one alone is Data sufficiency is like a jig-saw puzzle,
sufficient.” and you are only required to determine
• If, after combining the two whether you can complete the picture by
statements, we still do not have the using the puzzle pieces in statements one
NTK information we are seeking, and two.
then we must select the choice E, Let us see how the same question can be
which corresponds to “I tried every presented to us either format:
trick in the book but it does not
work. Leave me alone” or words to
that effect (Neither statement alone
or combined is sufficient).

The key to doing well in the data


sufficiency section is your ability to
proceed methodically in the manner
Some of the materials used in this module are taken from disclosed editions of GMAT, and no monetary
value is included in the cost of this module for supply of these materials. These materials are strictly used
for illustration of concepts discussed. Page 15
©Educational Training Services, A Division of Maple Leaf International Consulting, Inc.
Not to be reproduced, copied or distributed. Intended for use only by the registrant.

Let us take an example and see how we The answer


choices in
Problem Solving: can establish our N.T.K list and how we
Data
“What is X, if X + 2 = 4?” can rephrase the question posed. Sufficiency
We know that we can solve for X from will always
the given information: X = 2. be arranged
“If X is a prime integer less than 7, in the same
what is X?” order for all
We will pick an answer choice that states questions.
a value of 2 for X. The first
1. X2 = 9. choice
2. 2 < X < 5 corresponds
How can the same problem be presented to a decision
in data sufficiency context? In Data The questions we will ask are:
in which you
conclude that
sufficiency, you will be asked a question,
♦ What do I know about prime Statement 1
and your task is to determine whether alone is
integers?
you can answer the question from the sufficient but
♦ What else do I need to know in order Statement 2
additional information in two different
to make a decision? alone is not.
statements independently. Choice 2
corresponds
The answer to the first question is that to a decision
We stated earlier on in this discussion
prime integers are whole numbers that in which
that data sufficiency is about making a statement 2
have exactly two different positive
decision, and not about actual alone is
divisors, 1 and itself. We also know that sufficient but
implementation. Decision-making
there are only 3 prime integers less than statement 1
involves establishing your Need To alone is not.
7: 2, 3, or 5.
Know information. In addition to Choice 3
establishing your N.T.K., you will also be corresponds
What else do we need to know in order to a decision
required to use any information you may that
to make a decision about what X is? We
know, information such as the properties Statements 1
will need some additional information
of a triangle, or the number of integers and 2
such as an equation involving X or some TOGETHER
qualifying as “digits”, or the number of
such thing. are sufficient
hours in a day. Also, be sure to use any but neither
information that may be provided along alone is.
Having established our N.T.K., let us Choice 4
with the question itself. For example, the
rephrase the question posed in the means that
question might read: either
statements 1 and 2 independently, one at
“If X is a prime integer less than 7, what statement
a time, starting with statement 1 first. alone is
is X?”
sufficient.
We must read the statement 1 as: Choice 5
We should know that there are only 3 means that
“IF X2 = 9, and if X is either 2, 3, or 5 the
prime integers less than 7: 2, 3, and 5,
(a prime integer less than 7), can we statements
and that X should be one of these three alone or
determine one way or the other what X
values. combined are
is?” not sufficient
to make a
Last, but not least, you should be able
We notice that if we take the square root unique
to redefine the information in decision.
on both sides, X could be +3 or –3. But
statements 1 and 2 in the manner
given that X is a prime integer and less
indicated below.
than 7, X cannot be negative integer.
Therefore, -3 is not a likely value for X.
Some of the materials used in this module are taken from disclosed editions of GMAT, and no monetary
value is included in the cost of this module for supply of these materials. These materials are strictly used
for illustration of concepts discussed. Page 16
©Educational Training Services, A Division of Maple Leaf International Consulting, Inc.
Not to be reproduced, copied or distributed. Intended for use only by the registrant.

The only likely value for X is +3. Is this a In order to compute the average speed,
unique value? Yes. Can we make a we need to know the total distance
unique determination about what X is on between City A and City B, and the time
the basis of Statement 1 alone? Yes. We of travel between the two cities.
will keep the choices A and D, and kill Let us rephrase the question posed in the
options B, C, and E. light of statement 1 first:

We will need to similarly rephrase that 1. If the distance between City A and
statement 2 as follows: City B is 300 Miles, can we
determine the average speed of
“If X is a prime integer having one of travel between the two cities? We
the following three values: 2, 3, 5, and cannot because we do not know how
if 2 < X < 5, can we determine what X long it took to get from A to B.
is?” Therefore, we cannot answer the
question on the basis of statement
You bet. The only integer value that lies 1 alone. We will kill choices A and
in the range between 2 and 5 is 3. We D, and keep B, C, and E only.
can make a unique determination of what 2. Let us examine statement 2 by
X is on the basis of statement 2 alone redefining it: If it took 6 hours to go
also. We notice that each statement alone from City A to City B, can we
is sufficient to answer the question determine what the average speed
posed. The choice corresponding to this was? We cannot, because we do not
decision is Choice D. know the distance between the two
cities. Remember that you cannot use

L et us see how we can establish our


N.T.K and redefine the question
posed by looking at another example.
the information you learned in
statement 1 when you examine the
statement 2 on its own merits first.
Let us kill choice B and keep C and E
“What is the average speed for the only as viable options at this juncture.
trip between City A and City B?” 3. Because we cannot make a unique
decision on the basis of Statement 1
1. The distance between City only and also on the basis of
A and City B is 300 Miles. Statement 2 only, we MUST now
2. It took 6 hours to go from combine the two statements and see
City A to City B. whether we can make a decision on
the basis of the combined
information. You must bear in mind
We need to ask ourselves: that if any one of the two statements
♦ What do we know about was sufficient to answer the question
Distance, Time, and Speed? posed in a unique way, you should
♦ What do we need to know in not go to step 3 and combine the two
order to answer the question statements.
posed?

We know that Distance = Speed X Time.


Some of the materials used in this module are taken from disclosed editions of GMAT, and no monetary
value is included in the cost of this module for supply of these materials. These materials are strictly used
for illustration of concepts discussed. Page 17
©Educational Training Services, A Division of Maple Leaf International Consulting, Inc.
Not to be reproduced, copied or distributed. Intended for use only by the registrant.

400 men and 300 women. As you can


4. Let us rephrase the combined see, the difference between the men and
information: “If the distance between women numbers is 10 in one scenario but
City A and City B is 300 miles, and If it 100 in the second scenario, and each
took 6 hours to go from City A to City B, scenario is consistent with the same ratio
can we uniquely determine the average
of 4 to 3. Are we able to determine the
speed?” Of course, we can. We have
difference between the men and women
both pieces of information we
numbers in a unique way on the basis of
predetermined in our N.T.K.
the redefined statement 1? No, we are
Therefore, the combined information
not. We should consider Statement 1
is good for making a unique decision,
deficient on its own, and kill the two
and we must pick choice 3 or C.
choices A and D associated with
Statement 1. We have just 3 options: B,
Your ability to define the Need-To-
C, and E left. Let us move on to redefine
Know information and to “rephrase” the
the question in the light of statement 2.
question posed in the light of statements
1 and 2 is a critical factor in your ability
If we know that 4 more women will
to answer Data Sufficiency questions
make the ratio of men to women 1 (or
correctly. Let us take a look at another
make the number of men and women
Data Sufficiency question and hone this
equal), can we determine the difference
skill.
between men and women numbers in the
club? Of course, we can. The statement 2
DATA SUFFICIENCY:
information tells us that there are 4 more
“How many more men than women are
men than women in the club so that if 4
in the club X?”
more women came on board, the ratio
will be 1 or the number of men will be
1. The ratio of men to women in
A,D equal to the number of women making
the club is 4 to 3.
the ratio equal to 1. We can determine
2. If 4 more women join the
the answer unique fashion by using
B,C,E club, the ratio of men to
statement 2 alone while Statement 1
women will be 1.
alone was deficient. Of the 3 remaining
choices, B, C, and E, we must pick
Our Need to Know information will be
choice B to correspond to this decision
some information that either tells us the
scenario.
number of men and women separately or
one that tells us the difference between
Let us take a look at a couple of more
the two values in some fashion.
data sufficiency problems and get
comfortable with the process of
Let us take a look at Statement 1. We
predetermining the “NTK” and of
will read the statement as: If the ratio of
redefining the question posed in the light
men to women in the club X is 4 to 3,
of statements 1 and 2.
can we determine the difference between
the men and the women numbers? We
will see later on in this file, “ratio” is a
“proportionality” concept, and is true for
values such as 40 men and 30 women or
Some of the materials used in this module are taken from disclosed editions of GMAT, and no monetary
value is included in the cost of this module for supply of these materials. These materials are strictly used
for illustration of concepts discussed. Page 18
©Educational Training Services, A Division of Maple Leaf International Consulting, Inc.
Not to be reproduced, copied or distributed. Intended for use only by the registrant.

TRUE OR FALSE DECISIONS IN “If X is a prime integer that lies in the


DATA SUFFICIENCY range between 30 and 40 inclusive, is
X+2 a prime integer?”
Data Sufficiency will ask you to
determine whether you can uniquely Once again, our objective is to try to
determine on the basis of information create a CONFLICT across scenarios, if
provided in the statements whether we can. If we are unable to create a
something is definitely true or definitely conflict, we may be able to come up with
false. The best way to deal with such a unique determination of whether X + 2
problems is to try to ‘create a is a prime integer or not.
CONFLICT’ across two scenarios using
the information provided in the We know that X must be either 31 or 37.
statements so that you can ‘bail out’ of These are the only two prime integers in
the statement. the range between 30 and 40, inclusive.

Consider the following Statement: If X = 31, then X+2 = 33, which is not a
“If 3X = 4Y, Is X > Y?” prime integer.
If X = 37, then X+2 = 39, which is also
The evidence we have is that 3X = 4Y. not a prime integer.
Notice that this is a ratio statement
involving X and Y. We can write the Notice that we cannot create any other
above ‘evidence’ as X/Y = 4/3. We will scenarios because the values for X are
learn later in this book that a ratio limited to two: 31 and 37. For either
statement is a ‘multiples statement’. 4/3 values of X, we can see that X+2 is NOT
is the same ratio as 8/6 or as –4/-3. If a prime integer. Therefore, the provided
X/Y=4/3, then X = 4 and Y = 3. In this information is sufficient to uniquely
scenario, X > Y. But X = -4 and Y=-3 determine that X+2 is NOT a prime
will also satisfy the above ‘evidentiary integer.
information’. In the latter scenario, X <
Y. Notice that we have two different Consider another example of this type of
scenarios, each satisfying the provided scenarios testing.
statement but one in which X > Y and
the other in which X < Y. Do we have a “IF R and T are integers such that
conflict across scenarios here? You bet. R•T = 12, is T positive?”
Can we make a ‘logical decision’ as to
whether X > Y or not? We cannot. We You must read the ‘is T positive’ as ‘is it
must conclude that the information 3X = possible to uniquely determine whether T
4Y is not sufficient to make a unique is positive or not positive using the
determination of which is true: X > Y or statement information?’
Y >X.
If R•T = 12, it means that both R and T
Consider another example: are positive or both R and T are
negative. It is not possible for one of the
Some of the materials used in this module are taken from disclosed editions of GMAT, and no monetary
value is included in the cost of this module for supply of these materials. These materials are strictly used
for illustration of concepts discussed. Page 19
©Educational Training Services, A Division of Maple Leaf International Consulting, Inc.
Not to be reproduced, copied or distributed. Intended for use only by the registrant.

two values to be positive and the other Let us now examine statement 1.
negative because in such a scenario, the
product of R and T cannot be positive. Statement 1 tells us that k is positive. Is
this enough information to make a
If R = 3 and T = 4, then R•T = 12. In unique decision about whether 5k is less
this scenario, T is positive. than 600 or greater than 600? It is not.
How do we arrive at this decision? By
If R = -6 and T = -2, then R•T = 12. In creating conflict across two scenarios,
this scenario, T is negative. each scenario satisfying the statement
that k is positive.
Can you see that T is positive in one
scenario and negative in the other, but If k = 3, a positive integer, 5k = 53 =
both scenarios will satisfy the evidence 125, and 5k < 600.
provided: R•T = 12. But if k = 4, another positive integer, 5k
= 54 = 625, which is greater than 600.
We have ‘major conflict’ across
scenarios, and must conclude that we Notice that we have managed to create
cannot uniquely determine whether T is conflict across two scenarios and in each
positive or negative on the basis of the scenario, we picked values for k
evidence provided. consistent with the statement 1
information. Are we in a position to
Remember: Your objective is to create uniquely determine whether 5k is less
a conflict across two scenarios using the than or greater than 600? We are not.
same statement information so that you We must conclude that statement 1 alone
can ‘bail out’ of the statement and is not sufficient to make a unique and
conclude that the statement is not logical decision about 5k in terms of
sufficient to make a unique determination whether it is less than or greater than
of whether it is definitely true or 600. We must eliminate the options A
definitely false that T is positive. and D because neither one of these two
choices can be picked if statement 1 is
Now that you understand the procedure, not sufficient to make a unique decision.
let us hone our understanding by looking
at a complete data sufficiency problem Statement 2 tells us that 5k+1 < 3000. We
providing a question and two statements. also notice that 5k+1 = 5k•51 < 3000
Let us divide both sides by 5 to get rid of
Data Sufficiency: the factor 51 on the left side of the
“Is 5k < 600?” inequality. Notice that we must do it so
that we will have an inequality in terms
A, D (1) k is a positive integer. of 5k about which we are attempting to
B, C, E (2) 5k+1 < 3000 make a unique decision.

In order to make a unique decision about We get: (5k•51) / 5 < 3000/5


whether 5k is less than 600 or not, we Or 5k < 600.
need to get a handle on the likely values
for k, the only unknown in the statement.
Some of the materials used in this module are taken from disclosed editions of GMAT, and no monetary
value is included in the cost of this module for supply of these materials. These materials are strictly used
for illustration of concepts discussed. Page 20
©Educational Training Services, A Division of Maple Leaf International Consulting, Inc.
Not to be reproduced, copied or distributed. Intended for use only by the registrant.

Statement 2 tells us exactly what we are Can we find out the ages of L and J from
attempting to determine. We must this equation? We cannot. We have two
conclude that statement 2 alone is unknowns and one equation. We require
sufficient whereas statement 1 alone was two simultaneous equations in order to
not. We must pick option B, which be able to uniquely determine the values
means that the statement 2 alone is for L and J. We conclude that statement
sufficient but statement 1 alone is not. 2 alone is also not sufficient. We must
eliminate option B, keep C and E, and
WE do not proceed to combine the two proceed to combine the statements
statements because ‘combining the two because neither statement alone is
statements’ is a ‘desperate act’ required sufficient to make a unique decision.
only when each statement alone is not When we combine the statements, we
sufficient. If either statement alone is will read: If L = 15 and if L + 5 = 2J,
sufficient or each statement is sufficient, can we determine L and J and make a
then we do not have to combine the two comparison? We notice that we have
statements. two unknowns and two equations so that
we can find out the values of L and J in a
unique fashion. We can now answer the
Consider another data sufficiency question in a unique fashion by
problem asking us to make a ‘true or combining the two statements, and we
false’ decision in a definite manner. must select choice C.

Is Lisa older than John? Notice how we did not bother to solve
for L and J. When you are dealing with
A, D (1) Lisa is 15 years of age. Data Sufficiency, do not bother to solve
B,C,E (2) If Lisa were 5 years older, she equations as long as you recognize that
would be twice as old as John. you have all the required information
Our NTK list says that we need to know with which to get the answers. If you
the age of Lisa and of John so that we were asked the same question in problem
can compare the ages and determine solving, you must, of course, solve for L
who is older. and J by substituting the value of 15 for
If, on the basis of Statement 1, we know L in the second equation.
that Lisa is 15 years old, can we If L = 15, then
uniquely determine who is older: John 15 + 5 = 2J, or J = 10.
or Lisa? We cannot because we do not Obviously, Lisa is older than John but we
know how old John is. This statement need not bother to solve for L and J as
tells us nothing about John, and we long as we know that we have the
cannot compare Lisa’s age with an required pieces of information that helps
unknown quantity. We must eliminate us answer the question in a unique sort
choices A and D, and proceed to of way.
examine statement 2. We have just three
choices remaining: B, C, or E.
Statement 2 needs to be translated into a
mathematical equation, which will be:
L + 5 = 2J
Some of the materials used in this module are taken from disclosed editions of GMAT, and no monetary
value is included in the cost of this module for supply of these materials. These materials are strictly used
for illustration of concepts discussed. Page 21
©Educational Training Services, A Division of Maple Leaf International Consulting, Inc.
Not to be reproduced, copied or distributed. Intended for use only by the registrant.

about ‘divisibility of X by 12’, we cannot


Consider another problem that pick either option A or option D.
requires ‘testing scenarios’ and
‘creating conflict across scenarios’ in Let us now take a look at statement 2
the process of making a unique ‘true and see whether we can make a unique
or false’ decision. decision using this statement alone about
whether X is divisible by 12 or not.
“Is X a multiple of 12?”
If X is a multiple of 6, then X is an
1. X is a multiple of 4. ‘integer’ multiple of 6. X could be 6, 12,
18, 24, 30, 36, and so on, all multiples of
2. X is a multiple of 6. 6.

If X = 6, a multiple of 6 value, then X is


The question is: Can we make a not divisible by 12 because 6/12 is not an
‘definitely true’ or ‘definitely false’ integer.
decision about whether X is a multiple of But if X = 12, which is also a multiple of
12 or divisible by 12 using the 6, then X is divisible by 12 because 12/12
information in the statements provided? is an integer.

(We will see more about ‘divisibility’ in Do we have a conflict across scenarios
the chapter on numbers and factors here? Yes, we do. Can we make a
coming up later in this book). unique decision in the face of conflict?
We cannot. We must conclude that
Statement 1 tells us that X is a multiple statement 2 alone is also not sufficient to
of 4. A ‘multiple of 4’ is to be read ‘an make a unique decision about whether X
INTEGER multiple of 4’. is indeed a multiple of 12 or not.
X could be 4, 8, 12, 16, 20, 24, 28, 32,
and so on. Notice that all the ‘likely’ We must eliminate option B and proceed
values for X are ‘evenly’ divisible by 4. to combine the two statements, as we
must when either statement alone is not
Can we use this statement alone to make sufficient.
a unique decision about whether X is
divisible by 12? We cannot, because if X The combined information of statements
= 8, a multiple of 4 value, then 8/12 is 1 and 2 tells us that X is a multiple of 12.
not an integer. But X could also be 24, Why did we make this ‘logical
another multiple of 4, which is divisible conclusion’? Because if X is a multiple
by 12 because 24/12 is an integer. Do of 4 and also a multiple of 6, then X must
you see a ‘conflict’ across scenarios be a multiple of 12, the least common
here? You bet. Can we make a unique multiple of 4 and 6. Notice that in the set
decision using this statement about of likely values that are multiples of 4
whether X is a multiple of 12 or not? and 6, 12 is the smallest number that is
We cannot. We must conclude that common to both sets.
because statement 1 alone is not
sufficient to make a unique decision
Some of the materials used in this module are taken from disclosed editions of GMAT, and no monetary
value is included in the cost of this module for supply of these materials. These materials are strictly used
for illustration of concepts discussed. Page 22
©Educational Training Services, A Division of Maple Leaf International Consulting, Inc.
Not to be reproduced, copied or distributed. Intended for use only by the registrant.

If our ‘logical conclusion’ on the basis of Analytical Writing Assessment section,


the combined information in statements 1 where you tried to come up with a bunch
and 2 is that X is an INTEGER multiple of ‘may be’ scenarios in the process of
of 12, then X could be 12, 24, 36, 48, looking at ‘alternative explanations’ for
and so on. the stated evidence. Objectivity is about
making decisions after examining all
Are we in a better position to make a possible explanations of a given
unique decision about whether X is a information. If the evidence is: X2 = 4,
multiple of 12 or not? We are because then a conclusion that states X = 2 is
all the values that are multiples of 12 illogical because it ignores the possibility
must be divisible by 12. that X could be –2 in order to satisfy the
same evidence. Let us consider another
The combined information is sufficient to data sufficiency problem and see how we
uniquely determine that X must be can make a decision by examining the
divisible by 12, if X is divisible by 4 and different interpretations of a stated
by 6. information.

We must choose option C. “Is X = 4?”

Get used to the ‘testing scenarios’ 1. X2 = 4X


process, and be sure to ‘create conflict’ 2. X is an even integer.
across scenarios, if you can. In fact,
your goal is to create ‘conflict’ across We are required to make a definitely true
scenarios in the process of making ‘must or definitely false decision about whether
be true’ decisions, because if you can X is 4 or not.
create conflict across scenarios, then the Statement 1 tells us that X2 = 4X. People
condition that you are testing ‘could be schooled in algebra will be tempted to
true’ but is not necessarily ‘must be cancel X on both sides and conclude that
true’. X = 4. But we must pause and examine
whether another interpretation of the
“MUST BE TRUE” type of problems same evidence exists. The statement; X2
can also be tested in Problem solving, = 4X can be satisfied by X= 4 and by
and a later chapter is devoted to this type X=0. If we ‘rush to a decision’ stating
of problems. that statement 1 alone is sufficient to
make a unique determination of whether
We have seen in the discussion on Data X is 4 or not, we will be wrong. Because
Sufficiency thus far that the focus is on X could be 4 or 0 in order to satisfy the
reasoning and on predetermining what evidence presented in statement 1, we
information will help us answer the will consider this statement to produce
question posed in a unique fashion. You two ‘different interpretations of the likely
will also be expected to make ‘logical values of X’, and must consider that
decisions’ by examining all possible statement 1 alone is not sufficient to
explanations of the stated evidence. This make a unique decision. We must now
process is similar to the ‘critique of the eliminate options A and D.
argument’ process tested in the
Some of the materials used in this module are taken from disclosed editions of GMAT, and no monetary
value is included in the cost of this module for supply of these materials. These materials are strictly used
for illustration of concepts discussed. Page 23
©Educational Training Services, A Division of Maple Leaf International Consulting, Inc.
Not to be reproduced, copied or distributed. Intended for use only by the registrant.

Let us proceed to examine statement 1 Get used to this type of analysis. GMAT
on its own merit. data sufficiency section will test your
skill in this area.
Statement 2 tells us that X is an even
integer. X could be 0, 2, 4, 6, 8, …, -2, - Let us do another one before we move
4, -6 and so on. Are we looking at a on to the next concept area.
unique decision on the basis of statement
2 here? We are not. We must conclude DATA SUFFICIENCY
that statement 2 alone is also not
sufficient to make a unique determination “If X and Y are non-zero integers, is X
of whether X is 4 or not. We must equal to 2”?
eliminate option B and proceed to
combine the two statements. 1. X+Y = X•Y
2. X = Y
On the basis of the combined
information, we see that X could be 0 or Notice that the question stem provides
4, both even integers. The values 0 and 4 one piece of information about X and Y,
will satisfy BOTH statements, and are a information that must be used in
non-unique set of values. How are we to conjunction with the information in the
be sure that X is 4 when it is only a 50% statements. We know that neither X nor
chance that it is. There is an equal chance Y can be 0. Statement 1 tells us that
that X is not 4 and equal to 0. Do you X+Y=X•Y. There are only two sets of
see a conflict across scenarios here? values that will satisfy this relationship:
Either X and Y are each equal to 0 or
We have tried all the procedural steps: each equal to 2. Because we have been
Examine the statements alone and told that X and Y cannot be 0, our
combined, and are unable to make a ‘logical conclusion’ here must be that X
unique decision about whether X is 4 or and Y must be each equal to 2. We can
not. make a unique decision on the basis of
information in the statement 1 alone and
We must choose option E, corresponding confirm in the affirmative that X is
to a decision that no unique indeed 2. We must keep options A and
determination of whether X is 4 or not D for now, and move on to examine
can be made on the basis of the statement 2. Statement 2 tells us that X
information provided in the statements 1 and Y are equal. X and Y could be each
and 2. The question was: Must X be 4? equal to 1, or 2, or 3, or 4, or 100 or
The short answer is: X could be 4 but any value at all. Statement 2 is not good
does not have to be. The best we could for a unique determination of whether X
have done on the basis of the information is equal to 2 or not, but statement 1
provided in the two statements to us is to alone is sufficient to make this unique
conclude that X COULD be 4, but does determination. We must choose option
not have to be. A. The ‘buzz phrases’ are ‘unique
determination’ and ‘alternative
explanations or scenarios’.

Some of the materials used in this module are taken from disclosed editions of GMAT, and no monetary
value is included in the cost of this module for supply of these materials. These materials are strictly used
for illustration of concepts discussed. Page 24
©Educational Training Services, A Division of Maple Leaf International Consulting, Inc.
Not to be reproduced, copied or distributed. Intended for use only by the registrant.

SEQUENCES
AND PATTERNS

GMAT will test your ability to recognize Consider the following problem:
patterns and iterations, and to determine
the value for an element in a sequence. “In a recurring sequence, -4, -2, 0, 2, 4
‘Recognition’ of a pattern is a critical are values that repeat indefinitely in the
skill that you will require in real life too. same order. What is the sum of the 27th
If you consistently fail to win the lottery value and the 29th value in the
week after week, there is a pattern of sequence?”
picking a bad set of numbers. If a sales
person consistently fails to meet his Because the 5 values specified repeat ad
quota, there is a pattern of non- infinitum in the same order, we can see
performance. If one’s lover consistently that the 26th value must start in –4. The
fails to send flowers on Valentine’s day, 27th value will be –2. The 28th value will
there is a pattern of ‘couldn’t-care-less’ be 0. The 29th value is 2. The sum of the
aspect to his or her behavior. Normally, 27th value and the 29th value is –2+2 =
we will ‘recognize’ such patterns and 0. We must choose the option that
make a ‘federal case’ out of them. corresponds to a value of 0 for the sum
GMAT problems will ask you to do the of the 27th and the 29th terms.
same in the context of numbers and
variables.
Consider another ‘higher difficulty level
Let us say that we have a sequence of problem’:
numbers as follows:
“If in a sequence T1 = 2, T2 = 5, T3 = 11,
2, 3, 5, 7, 11, 13, 17, 23, X, ………….. T4 = 23, T5 = 47, What is T7? What is
the expression for TN in terms of the
We can see that the above is a sequence preceding values in the sequence?”
of positive prime integers. The next
prime integer after 23 must be 29. Do you get the feeling that you just
Therefore, we can logically conclude that walked out of a hot tub in a sauna room?
X must be 29. You will not be alone and will have
plenty of company in this regard. But
Let us take a look at another sequence of you must bear in mind that no GMAT
values: problem is meant to be difficult. You
must also bear in mind that GMAT
1, 2, 4, 8, 16, 32, X,….. problems test your ‘reasoning’ ability and
your ‘ability to recognize patterns’.
We can see that each successive value is
double the preceding one. We must What do we see is going on here?
logically conclude that X must be twice
32 or 64.
Some of the materials used in this module are taken from disclosed editions of GMAT, and no monetary
value is included in the cost of this module for supply of these materials. These materials are strictly used
for illustration of concepts discussed. Page 25
©Educational Training Services, A Division of Maple Leaf International Consulting, Inc.
Not to be reproduced, copied or distributed. Intended for use only by the registrant.

We see that T2 is one more than TWICE TN = TN-1 + 3•2N-2


the preceding term T1. T3 is one more We can test this equation by replacing N
than twice the preceding term T2. T4 is with 2 and by checking to see whether
one more than TWICE the preceding we get a value of 5 for the Term 2.
term T3. T5 is one more than TWICE the Similarly, replace N with 3 and check to
preceding term T4 and so on. Can you see whether you get a value of 11 for the
see a pattern emerging here? term 3. You will.
We can set up the Formula as follows:
TN = 1 + 2 (TN-1) Now we can determine T7 by simply
replacing N with 7 in the above
The value for T7 will be 1 more than relationship.
TWICE T6. The value for T6 will be 1 T7 = T6 + 3•25 = T6 + 3•32 = T6 + 96
more than TWICE T5. Knowing that T5 T6 = T5 + 3•24 = 47 + 3•16 = 95
is 47, we can conclude that T6 = 1 + Therefore, T7 = 95 + 96 = 191
2(T5)
= 1 + 2(47) = 95 Notice that the second approach is
Therefore, T7 = 1 + 2(T6) infinitely more abstruse and GMAT or
T7 = 1 + 2 (95) = 1 + 190 = 191 GRE is likely to have you come up with
the FORMULA as discussed in the first
NOTE: We could have also set up the scenario. But, you will not get both the
FORMULA representing the pattern as formulas because they are both correct
follows: and you cannot pick one over the other.
T1 = 2
T2 = T1 + 3•20 Can you see how this ‘pattern’ problems
T3 = T2 + 3•21 are dealt with? Let us do a couple of
T4 = T3 + 3•22 more so that you can get ‘real
T5 = T4 + 3•23 comfortable’ with this ‘pattern stuff’.
(Never mind the fact that we used an adjective
The pattern is as described in the rows
‘real’ to modify another adjective,
above. ‘comfortable’. We are not dealing with Sentence
Correction here, are we? If it were sentence
We can ‘extrapolate’ the above sequence correction, we would have used ‘really
and make the following ‘logical comfortable’)
determinations’.

Each successive term after the first term


is the sum of the preceding term and a
multiple of 3 value. Notice that T5 is the
sum of the preceding term T4 and the
exponent on the base of 2 is two less
than the suffix of the term T5. You can
see that this pattern holds true for all the
values after the first value.

We can, therefore, conclude that TN must


be equal to:
Some of the materials used in this module are taken from disclosed editions of GMAT, and no monetary
value is included in the cost of this module for supply of these materials. These materials are strictly used
for illustration of concepts discussed. Page 26
©Educational Training Services, A Division of Maple Leaf International Consulting, Inc.
Not to be reproduced, copied or distributed. Intended for use only by the registrant.

PROBLEM 1 F(1) = 1 – ½
+
“If F(X) = 1/X – 1/(X+1), F(2) = ½ -1/3
What is the sum of +
F(1)+F(2)+F(3)+F(4)+…….+F(100)?” F(3) = 1/3 – ¼
+
The problem looks ‘intimidating’ but we F(4) = ¼ - 1/5
must bear in mind that no problem on the +
test is meant to be difficult. Challenging, F(5) = 1/5 – 1/6
perhaps, but never ‘difficult’. We notice +
that the original statement is in terms of F(6) = 1/6 – 1/7
X, but the summation involves real +
integers. *
*
Let us try to replace the variable X with *
the integers and see whether there is a *
recognizable pattern emerging. +
F(99) = 1/99 – 1/100
F(1) = 1/1 – ½ = ½ +
F(2) = ½ - 1/3 = 1/6 F(100) = 1/100 – 1/101
F(3) = 1/3 – ¼ = 1/12
F(4) = ¼ - 1/5 = 1/20 The required sum = 1/1 – 1/101 =
101/101 – 1/101 = 100/101
We notice that the problem will be ‘very
complex’ if it involves adding fractions Our ability to recognize an emerging
such as ½, 1/6, 1/12, 1/20, and so on. ‘pattern’ saved the day for us, and got us
GMAT is not a slave-driver but a test to the answer. Train yourself to
that asks you to work through the recognize patterns. The test will test your
problems efficiently and smartly. We ability to do so.
must say to ourselves that ‘adding
fractions’ is not the way to go. Let us Also, the above problem tested your ability to
‘size up’ the problem and see whether ‘associate’ values with X. Given that F(X) is in
terms of X, we ‘associated’ X with 1 when we
there is a pattern emerging. We do see a computed F(1); X with 2 when we computed
pattern. We notice that the –1/2 is the F(2), and so on. If we are given that F(X,Y) =
mirror image of ½; -1/3 is the mirror X3 + Y2, we can compute F(-3, -2) by
image of 1/3 and so on. When we add ‘associating’ X with –3 and Y with –2.
F(1), F(2), etc., each successive term will F(-3, -2) =(-3)3 + (-2)2 = -27 + (-4) = -31
cancel out leaving just the first term of
the first function and the last term of the
last function. Notice that the successive
terms cancel out diagonally.

Some of the materials used in this module are taken from disclosed editions of GMAT, and no monetary
value is included in the cost of this module for supply of these materials. These materials are strictly used
for illustration of concepts discussed. Page 27
©Educational Training Services, A Division of Maple Leaf International Consulting, Inc.
Not to be reproduced, copied or distributed. Intended for use only by the registrant.

Let us take a look at another problem


asking us to recognize patterns. What is the pattern we recognize here?
We notice that all the values are in terms
“In a sequence, the first three values are of digit ‘9’, which repeats as many times
the first three positive prime integers in as the value of the suffix. S1 is a single
ascending order, and each of the digit number 9. S2 is a two digit number
successive values is the sum of all of the 99 and so on. We can determine that the
preceding values. What is the ratio of digit 9 must recur N times in the value
the 30th term to the 25th term?” for SN. We also see another pattern:
9 is 10-1
99 is 102 – 1
We have the sequence going as follows: 999 is 103 – 1
9999 is 104 – 1
2, 3, 5, (2+3+5), (2+3+5+2+3+5), ……
We can clearly see that SN must be
Notice that a pattern begins to emerge. 10N – 1.
The fifth term is twice the fourth, and we
will expect the same pattern to continue. If you are unable to leap forward to
Let us verify that it is indeed so. this conclusion, you can ‘back-solve’
by looking at the options and by
The sequence is: replacing N with 1 and checking to see
whether you get a value of 9; Verify
2, 3, 5, 10, 20, (20+10+5+3+2) 40, by replacing N with 2 and see that you
(40+20+10+5+3+2) 80, ……….. get 99, and so on.

Each successive term after the third term Recognition of ‘patterns’ can also be
is twice the preceding term, and the same tested in data sufficiency problems. Take
iteration will continue. a look at the above problem in the
following data sufficiency setting.
Therefore, the term 26th will be twice
term 25th and so on.

Term26 = 2• T25
T27 = 2•T26 = 2•2•T25
T28 = 2•T27 = 2•2•2•T25
T29 = 2•T28 = 2•2•2•2•T25
T30 = 2•T29 = 2•2•2•2•2•T25 = 25•T25

Therefore, the ratio of T30 to T25 is


25 = 32/1.

Take a look at another problem:

“If S1 = 9, S2 = 99, S3 = 999, S4 = 9999,


……….What is SN in terms of N?”
Some of the materials used in this module are taken from disclosed editions of GMAT, and no monetary
value is included in the cost of this module for supply of these materials. These materials are strictly used
for illustration of concepts discussed. Page 28
©Educational Training Services, A Division of Maple Leaf International Consulting, Inc.
Not to be reproduced, copied or distributed. Intended for use only by the registrant.

DATA SUFFICIENCY hand, if P is 3, then all the subset values


are divisible by P. Because we do not
“If the set S contains subsets S1 through know whether P is 2 or 3, we are unable
SN such that S1 = 9, S2 = 99, S3 = 999, to make a unique determination of
…. SN = 10N – 1, are all the subset whether all the subset values are divisible
values divisible by the prime integer P?” by P or not. We have conflicting
outcomes across two different scenarios
1. P < 5 when we test P = 2 and P = 3. We must
2. Subset S5 is divisible by P. conclude that Statement 1 alone is not
sufficient, and eliminate options A and D.
The information provided is mind- These two choices cannot be picked if
numbing for sure, but we must repeat the the statement 1 turns out to be not
mantra that GMAT problems are not sufficient.
supposed to be ‘difficult’ but
‘challenging’. Also, keep in mind the Statement 2 is somewhat intimidating but
simple rule that the harder the problem we must once again tell ourselves that
sounds, the less complex it will be. the more intimidating the statement
looks, the less complex it is likely to be.
How do we know that this problem is
not as complex as it seems on first blush? We know that S5 = 99999. How do we
Our reasoning says so. Remember that know this? The pattern tells us so. If S5
all problems will involve some degree of is divisible by P, we can logically
reasoning effort on your part. We notice conclude that P must be 3. No other
a pattern in the subsets: They are all in prime integer will divide an integer with
terms of the recurring digit 9. Given that 5 recurring 9’s as digits. Knowing that
P is a prime integer, the only prime P is 3, it stands to reason that all the
integer that will divide ALL of the subsets will be divisible by P because all
subsets is 3. No other prime integer will the subsets are made up of the same digit
evenly divide the values in the subsets. In 9 in a recurring pattern.
fact, if the single-digit sum of the digits
in an integer is 9, then the integer must Statement 2 alone tells us that the subset
be divisible by at least 1, 3, and 9. values are all divisible by the prime
Knowing that 1 and 9 are not prime integer, which must be equal to 3.
integers, we need to seek information
telling us whether P is equal to 3 or not. Statement 2 alone is sufficient but
statement 1 alone is not. We must pick
Notice how we ‘transformed’ a complex option A.
problem to a ‘manageable’ one through a
simple reasoning exercise. Let us The upshot of the above analysis is the
examine the statements one by one. following: “Do not get intimidated by
how the problem looks and sounds. Just
Statement 1 tells us that P < 5. Given think through it, and remember: the
that P is a prime integer, P could be 2 or more difficult and intimidating the
3. If P is 2, then none of the values of the problem seems, the less complex it is
subsets is divisible by P. On the other likely to be.”
Some of the materials used in this module are taken from disclosed editions of GMAT, and no monetary
value is included in the cost of this module for supply of these materials. These materials are strictly used
for illustration of concepts discussed. Page 29
©Educational Training Services, A Division of Maple Leaf International Consulting, Inc.
Not to be reproduced, copied or distributed. Intended for use only by the registrant.

A certain computer program generates a


Let us plow through another Patterns sequence of numbers a1, a2, … , an such
problem and increase our comfort level that a1 = a2 = 1 and ak = ak-1 + 2ak-2 for all
with such problems. integers k such that 3 ≤ k ≤ n. If n > 6,
then a7 = ?
The infinite sequence a1, a2,…, an,… is such
that a1 = 2, a2 = -3, a3 = 5, a4 = -1, and an = an-4 A. 32
for n > 4. What is the sum of the first 97 terms B. 43
of the sequence? C. 64
D. 100
A. 72 E. 128
B. 74 This is a relatively simpler problem than
C. 75
the ones we have been previously
D. 78
E. 80 grappling with.
In order to be able to see a pattern, we
need to take some ‘baby-steps’ and set We can see that if k = 3, then
up a few more values of the ‘sequence’. a3 = a2+2a1 = 1 + 2(1) = 3
a4 = a3 + 2a2 = 3 + 2(1) = 5
a5 = a5-4= a1 = 2 a5 = a4 + 2a3 = 5 + 2(3) = 11
a6 = a6-4 = a2 = -3 a6 = a5 + 2a4 = 11 + 2(5) = 21
a7 = a7-4 = a3 = 5 a7 = a6 + 2a5 = 21 + 2(11) = 43.
a8 = a8-4 = a4 = -1 Option B is the answer.
a9 = a9-4 = a5 = a1= 2
As you can see, the sequence reverts to Take another pattern problem for a test-
the first value ‘2’ after a set of 4 values, drive.
and the sequence carries on in this
fashion. We have seen a clear pattern of
‘ad infinitum’ repetition of the first 4
values. We can logically conclude that
the 97th value will revert to 2. In the first
96 values, there are 24 sets of the first
four values. The first 4 values add up to
3. The sum of the first 96 values is 24
times 3 = 72. If we add the 97th value to
this total of 72, we get 74. Therefore, the
sum of the first 97 values is 74. Choice
B.

Let us take a look at another ‘pattern’


problem.

Some of the materials used in this module are taken from disclosed editions of GMAT, and no monetary
value is included in the cost of this module for supply of these materials. These materials are strictly used
for illustration of concepts discussed. Page 30
©Educational Training Services, A Division of Maple Leaf International Consulting, Inc.
Not to be reproduced, copied or distributed. Intended for use only by the registrant.

If the sequence x1, x2, x3, …, xn, … is such Pattern problems test your ability to
that x1 = 3 and xn+1 = 2xn – 1 for n ≥ 1, observe and to see a clear and
then x20 – x19 = recurring connection among the
values . If, in the above problem, the
A. 219 question was: what is x20?, then we will
B. 220 use the sequence 3, 5, 9, 17, 33, …. to
C. 221 see the following pattern: (21+1), (22 +
D. 220 - 1 1), (23+)1, (24+)1, (25+)1, and so on.
E. 221 - 1 We can see that the exponent 1
corresponds to value 1, exponent 2 to
Let us take some ‘baby-steps’ and set up value 2, and so on. We can logically
a few values of the sequence so that we conclude that x20 must be (220+ 1).
can begin to see a clear ‘pattern’.
X1 = 3
X2 = 2x1 – 1 = 2(3) – 1 = 5
X3 = 2x2 – 1 = 2(5) – 1 = 9
X4 = 2x3 – 1 = 2(9) -1 = 17
X5 = 2x4 – 1 = 2(17) – 1 = 33

Notice that the question is about the


‘difference’ of a particular set of
consecutive values.

On the basis of the above representation


of sequence, let us see whether any
‘clear’ pattern emerges about the
‘difference’ of consecutive values.

x2 – x1 = 5 – 3 = 21
x3 – x2 = 9 – 5 = 22
x4 – x3 = 17 – 9 = 23
x5 – x4 = 33 – 17 = 24

We are now able to see a clear pattern:


the difference of the first two values is 2,
that of next two is 22, that of the next
two is 23, and so on. We also see that the
exponent corresponds to the subscript of
the value being subtracted. We can
logically conclude that x20 – x19 must be
equal to 219, with the exponent on 2
coinciding with the subscript of the value
x19 being subtracted. We must choose
option A.
Some of the materials used in this module are taken from disclosed editions of GMAT, and no monetary
value is included in the cost of this module for supply of these materials. These materials are strictly used
for illustration of concepts discussed. Page 31
©Educational Training Services, A Division of Maple Leaf International Consulting, Inc.
Not to be reproduced, copied or distributed. Intended for use only by the registrant.

scenario in which X and Y could be anything


they want to be as long as the sum of the two
numbers is a whole number. You will also
notice that the fact the sum of X and Y is a
whole number does not mean that the sum is a
As some wise guy said, if there are no numbers, positive whole number. The sum of X and Y
then there is no math. You should expect more could be a negative whole number, because the
than a few questions about numbers and integers sum is not specified as a positive whole number.
in the GMAT.
What are the possible sets of values for
Let us say that I have a number in my hat, and X and Y such that X + Y will be an
want you to bet on it. Would you? I would not, integer?
if I were you. Because, I need to know 4 things 3
about the number so that I can get closer to the X: +1 -½ -¾ +24
1 1 1
reality. Y: -2 -2 2 -34 +14

Specifically, I need to know whether the number X+Y -1 -3 -4 +4


is a positive number or a negative number, and
whether the number is a whole number or “not a
whole number”. As you can see, X and Y need not be integers,
positive or negative so that X + Y can be an
Let us say that I have a number N. Unless I integer.
define the number to be a positive number, you
will not assume that it is positive. Unless the
number is defined as an integer or a whole
number, you will not assume that it is a whole
number. If I simply state that N is a number,
then you will understand this statement to mean All numbers can be represented on a number
an “all-bets-are-off” information. “All bets are line such that 0 is smack in the middle, and all
off” because the number could be a positive or a positive values are to the right of Zero, and all
negative number, an integer (whole number) or negative values are to the left of Zero. A
not an integer. rational number is one that can be positioned
precisely on a number line. Example of a
Convention dictates that we omit the plus sign in rational number are: +2, - 2 ½ , + 7 ¾ , and so
front of the number, if that number is deemed on. You will notice that ½ and ¾ can be
positive. For example, we do not say that the car costs converted to terminating decimals, and we can
plus 20,000 dollars. We simply say that the car costs position these rational numbers precisely on a
$20,000. Similarly, we do not say that we make plus number line.
5,000 dollars in wages per month. We simply state An irrational number is a number that has a
that our wages are $5,000 a month. This non-terminating decimal. Examples of irrational
conventional thinking should not force you to
numbers are: 6, 1/3, 5/7, 4/9, and so on. You
consider N to be positive just because a plus sign is
omitted. In the GMAT, N could be positive or will notice that these numbers are non-
negative unless it is specifically defined one way or terminating decimals, and cannot be precisely
the other. located on a number line. We can only find an
approximate location for these irrational
If you are told that there are two numbers X and numbers.
Y such that X + Y is an integer (whole number), All numbers to the right of Zero
then you will make sense of this information by
asking a question: Do we know anything else
will be steadily increasing
about X and Y in terms of whether these values; all numbers to the left
numbers are positive or negative, or integers or of Zero are steadily
non-integers? If the answer is that we do not,
then we will take this to be an “all-bets-are-off” decreasing values.
Some of the materials used in this module are taken from disclosed editions of GMAT, and no monetary
value is included in the cost of this module for supply of these materials. These materials are strictly used
for illustration of concepts discussed. Page 32
©Educational Training Services, A Division of Maple Leaf International Consulting, Inc.
Not to be reproduced, copied or distributed. Intended for use only by the registrant.

X and Y could lie on the opposite sides of Zero


the same distance from Zero. In this scenario, X
and Y have the same absolute values but are not
equal to each other.

Or, X and Y could lie the same distance from


Zero on the same side of Zero. In this scenario,
The absolute value simply measures the distance X and Y not only have the same absolute values
from Zero of a number on a number line. Since but also are equal to each other.
“distance” is a positive quantity, the absolute
value is always positive. But we do not know what scenario we must set
store by, do we?
Consider X and Y as shown on the number line
below: Therefore, if the question in data sufficiency
read:

X 0 Y Is X =Y?
- 21 2 0 + 212
(1) | X | = | Y |
As you can see from the above representation, X (2) X2 = Y2
and Y have the same distance from Zero on the
number line, and will have the same absolute
values. We will denote this information by the
following notation: Can we answer the question on the basis of
information in statement 1 alone? We cannot
because, as we have seen earlier in this section,
|X| = |Y| there are two possible scenarios: one in which X
and Y are not equal and the other in which X
We will also notice that X and Y are not equal and Y are equal. Since we are dealing with
to each other because they lie on the opposite more than one unique scenario, we must
sides of Zero, albeit the same distance from conclude that statement 1 is not sufficient to
Zero. answer the question.

If we did not have the benefit of the visual You should also know that X2 = Y2 is the same
representation above, and if we were simply told information as | X | = | Y | , and is another way
that | X | = | Y |, we cannot determine one way of saying the same thing. For example, if X is
or the other whether X and Y are equal to each +2 and Y is –2, X2 = 4 and Y2 = 4 but X and Y
other. are not equal. Alternatively, X and Y could be
both +2 or both –2, in which case X2 = Y2 and
What are the possible scenarios? X = Y.
Scenario 1: Once again, two different scenarios, and not a
X 0 Y unique solution or answer to the question.

- 21 2 0 + 212

OR, (Scenario 2)

Y X
X 0 Y

- 212 0 + 212
Some of the materials used in this module are taken from disclosed editions of GMAT, and no monetary
value is included in the cost of this module for supply of these materials. These materials are strictly used
for illustration of concepts discussed. Page 33
©Educational Training Services, A Division of Maple Leaf International Consulting, Inc.
Not to be reproduced, copied or distributed. Intended for use only by the registrant.

Also, the absolute value statement does not give


us a clue as to where the number lies on the
number line in terms of the Zero point.
EXAMPLE 1:
Let us say that the question is:
“IF | X – 3 | = 9, WHICH OF THE
IS A > 0? FOLLOWING COULD BE A VALUE FOR
X – 5?”

(1) A = - B • 6
• -1
We will read the statement 1 to mean that A and • -11
B have the same absolute values, and that A and • -15
B lie on the opposite sides of Zero. • -21
But do we know which of these two values lies We know that the absolute value for X – 3 is 9.
to the right of Zero and which lies to the left of This means that X – 3 as a value lies either 9
Zero? We do not. We simply know that if A is units to the right of zero or 9 units to the left of
to the right of Zero, then B is to the left of Zero, zero. WE have, therefore, TWO potential values
and vice versa. In other words, if B is +2, then A for X – 3 to contend with.
is –2. If B is –3, then A is +3. Two different
scenarios, one in which A is positive, and the X–3=9 or X – 3 = -9
other in which A is negative. Not a unique OR, X = 12 OR X = - 6
solution. We must conclude that we cannot IF X = 12, then IF X = -6, then
answer the question on the basis of statement 1 X–5=7 X – 5 = -11
alone.
Let us see which one of these two likely values
The important thing to keep in mind is that for X – 5 can we find in the choices.
when you deal with absolute values, you know
the distance of that value from 0 on the number We have –11, and we must select that choice.
line but you do NOT know whether the value
lies to the right of zero or to the left. You must EXAMPLE 2:
be sure to check out scenarios pertaining to:
• What if the value is to the right of zero?, “IF | X3 | = 125, WHICH OF THE
and FOLLOWING COULD BE A VALUE FOR
• What if the value is to the left of zero on the X + 1?”
number line?
• -6
• -4
• 4
Let us see how we can use these two scenarios
• 8
when we deal with some problems from the
absolute value concept area. • 10

IF the absolute value of X3 is 125, then there are


two possible values for X3.
X3 = 125 OR X3 = -125
Or, X = 5 X = -5
X+1=6 X+1=-4

+6 as a likely value for X is not one of the


answer options. We must, therefore, choose the
second answer that corresponds to the other
likely value that X can have.
Some of the materials used in this module are taken from disclosed editions of GMAT, and no monetary
value is included in the cost of this module for supply of these materials. These materials are strictly used
for illustration of concepts discussed. Page 34
©Educational Training Services, A Division of Maple Leaf International Consulting, Inc.
Not to be reproduced, copied or distributed. Intended for use only by the registrant.

Remember: IF X IS RAISED TO AN ODD We must conclude that X could be +3 or –3.


POWER SUCH AS 3. 5, 7, ETC., then the value Likewise, Y could be + 4 or – 4.
for X will have the same sign as the original
sign. For example, if X5 = -32, then X = -2, Can we get a –7 by using a sum of the likely
because we need to go with –2 five times as values of X and those of Y? Yes. We can
factors in order to get a result of –32 as a combine –3 as a value for X and –4 as a value
product. for Y to get a likely –7. The first choice IS a
likely value. The question asks us to pick a
However, if X is raised to an EVEN power such choice that is NOT a likely value.
as 2, 4, 6, 8, etc., then X can have both positive
and negative values. For example, if X2 = 25, Can we get a –1 for the sum of X and Y? Yes.
then X could be +5 or –5. Similarly, if X4 = 81, WE can combine +3 for X and –4 for Y to get
then X = 3. Also, if X is raised to an even –1.
power, the value for that expression cannot be
negative. For example, X2 = -9 does not have Can we get a 0 for the sum of X and Y? We
real roots or real values for X. Therefore, the cannot because there is no way we can combine
equation X2 + 9 = 0 does not have real roots. ± 3 and ± 4 to get a zero sum. We must select
this choice as representing an unlikely or
You should also remember the values in the impossible value for X + Y.
following table in order to improve your speed
while taking the test: We can see that X + Y could be +1 or +7
because we can put together –3 for X and +4 for
X X2 X3 X4 Y to get +1; and +3 for X and +4 for Y to get +7
for X + Y.
2 4 8 16
EXAMPLE 4:
3 9 27 81
DATA SUFFICIENCY
4 16 64 256
“WHAT IS THE VALUE
5 25 125 625
FOR | X + Y |?”
6 36 216 1,296
1. | X3 | = 125
7 49 343 2,401 2. Y2 = 36

EXAMPLE 3: Let us associate choices A and D with statement


1, and B,C, and E with the statement 2. Our
“IF | X | = 3 AND | X | = 4, WHICH OF N.T.K involves values for X and Y so that we
THE FOLLOWING CANNOT BE A can determine the distance of X + Y from Zero
VALUE FOR on the number line.
X + Y?”
Statement 1 tells us that X is +5 or –5. We know
• -7 that X is 5 units away from Zero on the number
• -1 line, but do we know anything about Y using
• 0 this statement? NO. We must conclude that
• 1 statement 1 is not sufficient and kill the choices
• 7 A and D. We have just three choices to contend
with now. Statement 2 tells us that Y is + 6 or
–6. Statement 2 tells us nothing about X. We
must eliminate choice B. When we combine the
two statements, we see that X + Y could be
either 1 unit away from Zero or 11 units away

Some of the materials used in this module are taken from disclosed editions of GMAT, and no monetary
value is included in the cost of this module for supply of these materials. These materials are strictly used
for illustration of concepts discussed. Page 35
©Educational Training Services, A Division of Maple Leaf International Consulting, Inc.
Not to be reproduced, copied or distributed. Intended for use only by the registrant.

from zero because X + Y could be +6+5 = 11 or


+6 – 5 = +1 or –6 +5 = - 1 or –6 – 5 = -11. We
are getting two likely distances for X + Y from
zero on the number line, and two different
absolute values as likely distances for X + Y.
We said that the statements examined You should also know that the consecutive ODD
independently or in a combined fashion must be and EVEN integers are 2 values apart on the
good for a unique determination. Can we make a number line. Examples of consecutive EVEN
unique determination using the combined integers are:
information? We cannot. We must, therefore, go -8, -6, -4, -2, 0, 2, 4, 6, 8, 10, 12, 14, etc.
with choice E, or the last answer-option in the
GMAT-CAT. Examples of consecutive ODD integers are:
-7, -5, -3, -1, 1, 3, 5, 7, 9, etc.

IF N is an integer, then for all values of N,


2N ± 1 is an ODD integer, and 2N will be an
INTEGERS EVEN integer.

WE can represent consecutive positive ODD


integers as follows:
An integer is a whole number. There are four 2N + 1, 2N+3, 2N+5, 2N+7, etc..
sub-classifications of integers: Consecutive positive EVEN integers are:
• ODD INTEGERS are by default not 2N, 2N+2, 2N+4, 2N+6, 2N+8, ETC.
divisible by 2. Odd integers could be
positive or negative. Examples of odd
integers are: -11, -5, +3, +111, and so on. Consecutive integers are spaced 1 apart.
• EVEN INTEGERS are by default divisible Examples of consecutive integers are:
by 2. EVEN integers could be positive, -7. –6, -5, -4, -3, -2, -1, 0, +1, +2, +3, and
negative, or ZERO, which is neither so on.
positive nor negative. Examples of even
integers are: -20, -16, -2, 0, +4, +116, and For any integer N, we can write the
so on. An EVEN INTEGER must end in consecutive integers as:
one of the following digits: 0, 2, 4, 6, or 8. N, N+1, N+2, N+3, N+4, N+5, and so on.
You must remember that ZERO is an
integer and is an even integer.
• PRIME INTEGERS are by definition
integers that have just 2 different positive
factors: 1 and itself. PRIME INTEGERS are
always positive, and the least value for a
prime integer is 2. The integer 2 is the Just remember the following “combination”
ONLY even integer that is a prime integer. rules as they apply to integers>:
All other prime integers are ODD integers.
You should know the prime integers in the
range between 2 and 50: 2, 3, 5, 7, 11, 13,
17, 19, 23, 29, 31, 37, 41, 43, 47, 53, and so
on.
• DIGITS are integers having a value
between 0 and 9. A digit could be positive
or zero. A digit cannot be negative. –5 is a
number using 5 as a digit.

Some of the materials used in this module are taken from disclosed editions of GMAT, and no monetary
value is included in the cost of this module for supply of these materials. These materials are strictly used
for illustration of concepts discussed. Page 36
©Educational Training Services, A Division of Maple Leaf International Consulting, Inc.
Not to be reproduced, copied or distributed. Intended for use only by the registrant.

Some of the materials used in this module are taken from disclosed editions of GMAT, and no monetary
value is included in the cost of this module for supply of these materials. These materials are strictly used
for illustration of concepts discussed. Page 37
©Educational Training Services, A Division of Maple Leaf International Consulting, Inc.
Not to be reproduced, copied or distributed. Intended for use only by the registrant.

For any integer N, we saw that we


can represent an odd integer as Example 2:
(2N ± 1). Notice how we turned the
integer N into an even integer by IF X is an integer, is Y an
integer?
multiplying it by 2. For all integer
values of N, 2N will be an even (1) X = Y + 1
integer, and when we add an odd (2) X + Y = Integer
integer to (or subtract an odd
integer from) an even integer, the Statement 1 tells us that X and Y are
consecutive integers because X is 1 greater than
outcome is an odd integer. Y. Remember: Consecutive integers are spaced
1 apart. We can answer the question on the basis
For all integer values of N, we can of the information in statement 1 alone. Let us
represent a series of consecutive move on to examine statement 2 and see
whether we can answer the question on the basis
odd integers as:
of statement 2 alone also.
2N ± 1, 2N ± 3, 2N ± 5, 2N ± 7, 2N ± 9,
and so on. Statement 2 tells us that X + Y is an integer.
Because X is defined as an integer in the stem,
Similarly, for any integer value N, consecutive we conclude that Y must be an integer as well.
even integers can be represented as: (If Y is not an integer, then X + Y cannot be an
integer, given that X is an integer).
2N, 2N ± 2, 2N ± 4, 2N ± 6, So. We conclude that either statement is
and so on. sufficient to answer the question, and we will
pick the 4th choice in the answer section.
You might have noticed that any
two consecutive odd integers Example 3:
are spaced 2 apart, and any two
consecutive even integers are IF X and Y are consecutive integers, is
X > Y?
also spaced 2 apart.
(1) X – 1 and Y + 1 are consecutive
Example 1: The sum of a set of six integers.
consecutive positive odd integers is 216. How (2) X and Y + 2 are consecutive
much greater is the largest integer in the set integers.
than the smallest integer?

Solution: We will start by defining the You should understand the question to mean: Is
consecutive positive odd integers as 2N + 1, X = Y + 1 ? IF X and Y are consecutive
2N + 3, 2N + 5, 2N + 7, 2N + 9, and 2N + 11. integers, then the only way that X can be greater
The sum of these values is 12N + 36. than Y is if X = Y + 1.
Our equation is: 12N + 36 = 216
Or 12N = 180 or N = 15. Let us examine statement 1.
The largest number in the set is 2N + 11 or If X is less than Y, then X = Y –1
41. X – 1 will be Y – 2.
The smallest integer in the set is 2N + 1 or 31 Statement 1 tells us that X –1 and Y + 1 are
The difference of the two integers is 10. consecutive integers. Because X – 1 is Y –2
under this scenario, we notice that Y – 2 and Y

Some of the materials used in this module are taken from disclosed editions of GMAT, and no monetary
value is included in the cost of this module for supply of these materials. These materials are strictly used
for illustration of concepts discussed. Page 38
©Educational Training Services, A Division of Maple Leaf International Consulting, Inc.
Not to be reproduced, copied or distributed. Intended for use only by the registrant.

+ 1 cannot be consecutive integers. We conclude We can see that conditions I and II must be
that X cannot be less than Y. valid.
If X is greater than Y, then X = Y + 1. What do we think of condition III ? If we are
X – 1 will be Y. dealing with three consecutive integers, at least
X –1 and Y + 1 will be consecutive integers one of them must be an even integer, and any
because X – 1 is Y and Y and Y+1 are integer multiplied by an even integer will give
consecutive integers. We conclude that an even integer as the outcome.
statement 1 lets us determine precisely that X is We conclude that condition III must be true as
greater than Y. Statement 1 alone is sufficient. well. We must pick the last choice.
Let us move on to examine statement 2.

IF X is less than Y, then X = Y – 1


NUMBER OF INTEGERS IN
Statement 2 tells us that X and Y+2 are
A GIVEN RANGE OF
consecutive integers. We notice that X is Y –1
VALUES
under this scenario, and Y –1 and Y + 2 cannot
be consecutive integers. Therefore, we conclude
that X cannot be less than Y.
IF X is greater than Y, then X = Y + 1 If N1 and N2 are integers such that N2 > N1 ,
Statement 2 tells us that X and Y + 2 are then the number of integers between N1 and
consecutive integers. Since X is Y + 1 under this N2, inclusive, is N2 – N1 + 1 . If N1 and N2
scenario, we conclude that Y + 1 and Y + 2 are are not integers, then the number
indeed consecutive integers, and that X must be integers in the range is the absolute
1 greater than Y. integer value difference of N1 and N2,
provided that the difference is an
We can answer the question independently using integer. For example, the number of
either statement. We will pick the 4th choice. integers between 1 ½ and 5 ½ is the
difference of the two values.

How many odd integers exist between


Example 4: 123 and 2451, inclusive?
Step 1: Find out the number of integers
in the range, inclusive.
If a, b, and c are consecutive
integers such that a < b < c, then Number of integers in the range is (2451
which of the following must be true? – 123 + 1) = 2329
I. c–a =2
II. b=a+1=c–1 We notice that the number of integers in
the range is an odd value. This tells us
III. abc is an even integer. that the end values in the range are both
the same kind – odd and odd or even
θ I only and even. In our problem, we notice that
θ II only the end values are both ODD integers –
θ III only 123 and 2451 are both odd integers. This
means that the number of ODD integers
θ I and III only in this range is exactly ONE more than
θ I, II, and III the number of EVEN integers in the same
range.
Solution: If a, b, and c are consecutive In our problem, we will find the number
integers, and a < b < c , then we should know of odd integers by adding a 1 to the
that number of integers and dividing the sum
a = b – 1 or b = a + 1 by 2.
and b = c – 1 or a + 1 = c – 1 or c = a + 2 Number of odd integers =
½ •(2329 + 1) = 1165.

Some of the materials used in this module are taken from disclosed editions of GMAT, and no monetary
value is included in the cost of this module for supply of these materials. These materials are strictly used
for illustration of concepts discussed. Page 39
©Educational Training Services, A Division of Maple Leaf International Consulting, Inc.
Not to be reproduced, copied or distributed. Intended for use only by the registrant.

Number of even integers in the same integers as 26, including the integers 100 and 200 –
range is ONE less than the number of both multiples of 4.
ODD integers or 1164. Notice that the
sum of the two must add up to the
REDEFINE THE RANGE WITHIN THE
number of integers in the range.
SPECIFIED RANGE IN ORDER TO
DETERMINE THE NUMBER OF MULTIPLES
If the end values in the range are such that OF N within the specified range.
one is odd and the other is even, then the Let us say that the problem asked us to compute the
number of odd integers and the number of number of multiples of 4 in the range between 99 and
even integers in the range will be each 201.
equal to ½ of the number of integers in the
range. For example, the number of odd As you can see, the FIRST value in the specified
integers and the number of even integers range that is a multiple of 4 is 100 and the last value
will be each equal to ½. Of the number of in the range that is a multiple of 4 is 200. We can
integers in the range between 24 and 473, ‘redefine’ the question to ask: HOW MANY
inclusive. We notice that the number of MULTIPLES OF 4 are there between 100 and 200,
integers between 24 and 473, inclusive, is inclusive? The answer is ¼ (200 – 100) + 1. We add
a 1 because both 100 and 200 in the defined range are
(473 – 24 + 1) = 450. The number of odd
multiples of 4.
integers = ½ • 450 = 225. = Number of even
integers in the range. Let us say that the question was:
How many integers that are multiples of 3 exist
Number of multiples in a between 100 and 300, inclusive?
range:
The difference of 300 and 100 is 200, and 200
You may come across questions in the divided by 3 will give us 66. But then, we notice
GMAT asking you to compute the number of that 300 is a multiple of 3 and we must add 1 to
multiple values of a given integer in a range the number 66 for a total of 67 multiples of 3 in
of specified values. The number of integers the range between 100 and 300, inclusive.
in the range will usually be the ratio of the
You could have also arrived at the same result
difference of the end values to the multiple
by ‘redefining’ the range in terms of the first
value itself. But then, we may have to make
value that is a multiple of 3 and the last value in
some adjustments to this ratio in the the range that is also a multiple of 3. The
following manner. ‘redefined’ range is between 102 and 300,
inclusive. The number of multiples of 3 in this
Let us say that the question asked you: ‘redefined’ range within the specified range is 1/3
(300 – 102) + 1 or 67 multiples of 3. We added
How many integers that are divisible by 4 a 1 to the quotient of ½(300-102) because both
exist in the range between N1 and N2, values, 300 and 102, are multiples of 3.
inclusive?
IF the problem asked you:
The answer is:
How many multiples of 3 exist between 25 and
Integer value of (N2 – N1) + 1, 80, inclusive?
4
if either N1 or N2 or both is evenly divisible by 4. Redefine the range to one having values
We will also add a 1 if the least of the end values is between 27 and 78, inclusive. 27 is the first
one less than an integer multiple of 4 and the other is value in the range that is a multiple of 3 and 78
one more than an integer multiple of 4. For example, is the last value in the SPECIFIED range and a
if the problem asked us to compute the number of multiple of 3. The number of multiples of 3
integers in the range between 100 and 200, inclusive, between 27 and 78, inclusive, is 1/3 (78 – 27) +
we will take the difference of 100 and 200 and divide 1= 28 multiples of 3. Again, we added a 1 to the
the difference by 4. We get an integer quotient value quotient because both 27 and 78 in the
of 25. We then take a look at the extreme values in ‘redefined’ range are divisible by 3.
the range and notice that both 100 and 200 are
divisible by 4. We need to add a 1 to the quotient
value of 25 we obtained, and state the number of

Some of the materials used in this module are taken from disclosed editions of GMAT, and no monetary
value is included in the cost of this module for supply of these materials. These materials are strictly used
for illustration of concepts discussed. Page 40
©Educational Training Services, A Division of Maple Leaf International Consulting, Inc.
Not to be reproduced, copied or distributed. Intended for use only by the registrant.

whole numbers and what if they are


Example 1: How many integers exist not? Will the answer be the same?
between 234 and 37, 456, inclusive?

The answer is: (37456 – 234) + 1 or We know that the difference of n and
37222 + 1 = 37223 m is 6.
There are 37,223 integers between 234
and 37,456, inclusive. If m and n are integers such that m
is 1 and n is 7 so that the difference
Example 2: How many integers that are
divisible by 5 exist between 20 and 200 is 6, then there are 7 integers in the
inclusive? range between 7 and 1, inclusive. If
we do not consider the end values,
We notice that the difference of 20 and then the number of integers in the
200 is 180, and 180 divided by 5 is 36. range is only 5.
But then, both 20 and 200 are multiples
Let us check the scenario in which m
of 5, and we need to add a 1 to 36 to get
a total of 37 multiples of 5 in the range and n are not integers.
between 20 and 200, inclusive. If m is 1.2 and n is 7.2, then there
are only 6 integers in the range
Example 3: How many integers that are between m and n, inclusive.
divisible by 3 exist between 100 and 200,
inclusive?
We notice that if m and n are
We notice that neither 100 nor 200 is a integers, then the number of integers
multiple of 3. Also, 100 is not 1 less than in the range is 7 (if we consider
a multiple value of 3 and 200 is not 1 inclusive situation) or 5 (if m and n
more than a multiple value of 3. are not included) but if m and n are
Therefore, the number of multiples of 3
NOT integers, then the number of
in the range is the quotient of (200 –
100)/3 or 33. integers is the same as the integer
value of difference between the two
end values.

Example 4: If m and n are Two different scenarios, and two


numbers such that n > m and n – different values. We must pick the
m = 6, how many integers exist last choice: Cannot be determined.
between m and n, inclusive?
NOTE: If the problem does not specify
θ6 “inclusive”, there is a good chance that the
θ5 end values in the range are NOT integers,
and you must be sure to consider the
θ4 scenarios pertaining to non integer end
θ3 values. IF the end values are integers, and
θ Cannot be determined we need to compute the number of integers
excluding the end values, then we need to
subtract 1 from the difference of the end
You will notice that m and n are not values. For example, the number of integers
defined as integers in this problem, between 25 and 100 excluding 25 and 100
and we are not going to assume that is 100 – 25 – 1 = 74.
they are. We must be sure to check
the scenarios: What if they are
Some of the materials used in this module are taken from disclosed editions of GMAT, and no monetary
value is included in the cost of this module for supply of these materials. These materials are strictly used
for illustration of concepts discussed. Page 41
©Educational Training Services, A Division of Maple Leaf International Consulting, Inc.
Not to be reproduced, copied or distributed. Intended for use only by the registrant.

factor of X. In mathematical terms,


the equation is:
X = p. 1/Z where p is an
integer. ( 1/Z will divide X an integer
number of times to quality as a
factor).
You will understand a Factor to be a
number that will divide a given a
If X is proportional to Y, and
number a whole number of times.
inversely proportional to Z, you will
read this information to mean that
If Y is a factor of X, then Y must
Y/Z is a factor of X. The
divide X a whole number of times.
mathematical representation of this
Or X / Y must be an integer.
information is: X = n. Y/Z where n is
an integer. In other words, Y/Z will
We can write this information in
divide X an integer number of times.
mathematical form as:
X = k, or X = k. Y
Y
Example 1:
Where k is an integer.
If X is proportional to Y and inversely
proportional to Z, and if X is 4 when
The following statements are
Y is 2 and Z is 8, then X in terms of
interchangeably used, and mean the
Y and Z is:
same thing. Y is a factor of X.
θ Y/Z
X is a multiple of Y.
θ 4Y/Z
X is proportional to Y
θ 8Y/Z
Y is a divisor of X
Y is a factor of X. θ 16Y/Z
θ 32Y/Z
Any of the above statements will
translate to a mathematical form:
X = k. Y Step 1 is to translate English into
math. If X is proportional to Y and
inversely proportional to Z, we
understand this statement to mean
that Y/Z is a factor of X or
X = k. Y/Z where k is an
integer.
If X is proportional to Y, you will
read this statement to mean that Y is Step 2 is to find a value for k. In
a factor of X. In mathematical terms, order to help us do this, the problem
you will write: X = k.Y tells us that X is 4 when Y is 2 and Z
is 8. Let us plug in these values for
If X is inversely proportional to Z, X, Y and Z to get the value for k.
you will read this statement to mean We have: 4 = k. 2/8 or k = 16
that the reciprocal of Z, viz. 1/Z, is a
Some of the materials used in this module are taken from disclosed editions of GMAT, and no monetary
value is included in the cost of this module for supply of these materials. These materials are strictly used
for illustration of concepts discussed. Page 42
©Educational Training Services, A Division of Maple Leaf International Consulting, Inc.
Not to be reproduced, copied or distributed. Intended for use only by the registrant.

Step 3 is to re-write the original We could have dealt with this


equation by replacing k with a problem by using real-life numbers,
numerical value of 16. instead of algebra.
We have X = 16. Y/Z or X = 16Y/Z If X is proportional to Y, then we
know that X is a multiple of Y. If Y is
We can see that the best answer is a multiple of Z, then Z happens to be
the 4th choice. the least value of the three values. If
X = 16Y/Z Z is 1, then Y could be a multiple of
1, say 2. If Y is 2, then X is a
Example 2: multiple of 2, say 4.
If X is proportional to Y, and Y is a
multiple of Z, then So, we have X = 4, Y = 2, and Z = 1,
1. X is proportional to Z and these values satisfy the
2. X – Y is proportional to Z. conditions specified in the problem.
3. X + Y is proportional to Z.
Let us test the condition 1. IF X is 4,
If X is proportional to Y, we can and Z is 1, is 4 a multiple of 1? You
write: X = k. Y (k is an integer) bet. Therefore, condition 1 is valid.
If Y is a multiple of Z (read this to
mean that Z is a factor of Y), then, Let us test condition 2. IF X is 4 and
Y = p.Z (p is an integer) Y is 2, then X – Y is 2. Is 2 a multiple
We have X in terms of Y and Y in of 1? You bet. Condition 2 is also
terms of Z. Can we express X in valid.
terms of Z? We can. Let us write
pZ for Y in X = k.Y to get X = k.p.z. Let us test condition 3. If X is 4 and
We notice that this statement means Y is 2, then X + Y is 6. If 6
that X is proportional to Z. proportional to or a multiple of 1?
So we have X = k.p.Z Yes indeed. Therefore, all three
And Y = p. Z conditions are valid.
What is X – Y ?
X – Y = k.p.Z – p.Z = (k.p – p)Z We could have chosen an alternate
We notice that Z is a factor of X – Y set of values of X, Y, and Z
because (k.p – p) is an integer. We consistent with the way things are
can see that this information tells us defined and still come out with the
that X – Y is proportional to Z. same results. If you find working with
What is X + Y ? algebra as an abstraction
X + Y = k.p.Z + p.Z = (k.p + p) Z intimidating, then you can choose to
We can clearly see that Z is a factor work with real-life numbers. But
of X + Y because k.p + p is an then, you must know that 2 is a
integer. In other words, (X + Y) is multiple of 1. If you have this basic
proportional to Z. understanding, then you can work
We can see that all three conditions with the most intimidating of GMAT
are valid. problems with ease.

Some of the materials used in this module are taken from disclosed editions of GMAT, and no monetary
value is included in the cost of this module for supply of these materials. These materials are strictly used
for illustration of concepts discussed. Page 43
©Educational Training Services, A Division of Maple Leaf International Consulting, Inc.
Not to be reproduced, copied or distributed. Intended for use only by the registrant.

Factors with a Wrinkle. The mathematical representation


If, when X is divided by Y, we get a of the information that a, when
remainder of R, then (X-R) must be divided by 7, gives a remainder of
divisible by Y. 2 is: a = k.7 + 2
Similarly, we can write
In other words, Y must be a factor of b = p.7 + 4
(X – R). What is a + b ?

In mathematical terms, we can write a + b = k.7 + 2 + p.7 + 4


this information as: Or a + b = (k+p). 7 + 6
(X – R) = k (k is an integer) This information tells us that if
Y (a + b) is divided by 7, we should
Or (X – R) = k. Y get a remainder of 6.
Or X = k.Y + R Or, (a + b – 6) must be divisible
by 7.
In real life terms, if 32 is divided
by 5, we get a remainder of 2. We Now take a look at the answer
must conclude that (32 – 2) must choices. You can see that the
be divisible by 5. values in choices 1 through 4 all
In mathematical terms, this give a remainder of 6 when
information will read: divided by 7. 110 cannot be a
32 = 6.5 + 2 value for (a+b) because when 110
is divided by 7, we get a
remainder of 5, and not 6.
Example 1. We must pick the last choice.
If a is divided by 7, we get a
AS A RULE, IF X AND Y ARE TWO
remainder of 2 and if b is divided
VALUES, AND EACH OF THESE
by 7, we get a remainder of 4, then VALUES IS DIVIDED BY A COMMON
which of the following cannot be a FACTOR PRODUCING SEPARATE
value for (a+b) ? REMAINDERS R1 AND R2
RESPECTIVELY, THEN (X + Y) WHEN
θ 20 DIVIDED BY THE SAME FACTOR
MUST YIELD A REMAINDER EQUAL
θ 48
TO THE SUM OF THE TWO
θ 76
SEPARATE REMAINDERS. In the
θ 97 above problem, a+b, when divided
θ 110 by 7, MUST yield a remainder of 2
+ 4 =6. Our task is to look for a choice
value that will not provide this remainder
value when divided by 7. The value 110
does not produce a remainder of 6 when
divided by 7.

Some of the materials used in this module are taken from disclosed editions of GMAT, and no monetary
value is included in the cost of this module for supply of these materials. These materials are strictly used
for illustration of concepts discussed. Page 44
©Educational Training Services, A Division of Maple Leaf International Consulting, Inc.
Not to be reproduced, copied or distributed. Intended for use only by the registrant.

Example 2:
If we get a remainder of 3 when k is
If you find working with real-life numbers is
divided by 7, then which of the less messy, feel free to use this approach.
following will give a remainder of 6 Be sure to select values in keeping with the
when divided by 7? way the problem is described

I. 2k Example 3:
II. 3k + 4
N! represents the product of
III. k + 3 consecutive positive integers from 1
through N. For example, 3! Is the
product of 1.2.3.
When k is divided by 7, we get a
If X = 21! + 17, which of the
remainder of 3. The mathematical
following cannot be a factor of X?
form for this information is:
k = p.7 + 3
I. 21
Let us test the conditions one by one.
I. 2k = (2p).7 + 6 II. 17
We can see that when 2k is divided by 7, III. 13
we get a remainder of 6
II. 3k + 4 = (3p).7 + 9 + 4 π I only
3K + 4 = (3p).7 + 7 + 6
3k + 4 = (3p+1).7 + 6 π II only
We can see that 3k + 4 will yield a π III only
remainder of 6 when divided by 7. π I and III only
III k + 3 = p.7 + 3 + 3 = p.7 + 6 π I, II and III
We can see that k+3 will also yield a
remainder of 6 when divided by 7.
Solution: 21! Represents the product of
We must pick a choice that says “I, II,
consecutive positive integers from 1 through
and III”. 21. One of the factors of 21! Will be 17.
Because X is 21! Plus 17, 17 is the only
value that will divide X a whole number of
We could have dealt with this problem by times. 21 and 13 will not yield integer values
using real life numbers. What is the least when they divide X because 17, when
possible value for k consistent with the way divided by 21 or 13, does not yield an
the number is defined in the problem? integer.
It is 7 + 3 = 10. What is 2k? It is 20. Does The answer is I and III cannot be factors of
20 give a remainder of 6 when divided by th
X. We pick the 4 choice.
7? Yes, it does. Similarly, if k is 10, what is
3k + 4? It is 34. Does 34 produce a
remainder of 6 when divided by 7? Yes, it
does. Lastly, if k is 10, what is k + 3? It is
13. Does 13 produce a remainder of 6 when
divided by 7? You bet it does. We can see
that all three conditions yield a remainder of
6 when divided by 7. We must pick a choice
that says that all three conditions are good.

Some of the materials used in this module are taken from disclosed editions of GMAT, and no monetary
value is included in the cost of this module for supply of these materials. These materials are strictly used
for illustration of concepts discussed. Page 45
©Educational Training Services, A Division of Maple Leaf International Consulting, Inc.
Not to be reproduced, copied or distributed. Intended for use only by the registrant.

Example 4: How would a simple problem


asking you to find the factors of a
If n is an integer such that 91.k.n is given integer appear in the
the product of 2.3.5.7.11.13, which GMAT? In the following fashion:
of the following cannot be a value for
k? “If N is an integer such that
K•N = 87, how many positive
π 22 integer values of K exist?”
π 30
π 55 We notice that if N is an integer,
π 60 we can rearrange the given
π 66 equation in terms of N as follows:

N = 87 / K = Integer
Solution: If n is an integer, then 91.k
must be a factor of 2.3.5.7.11.13. Our definition of factors tells us
that K must be a factor of 87
n = 2•3•5•7•11•13 because it divides 87 an integer
91.k number of times. In fact, K could
(91.k divides 2.3.5.7.11.13 a whole
number of times; therefore, 91.k must be
be any of the values that are
a factor of 2.3.5.7.11.13) positive factors of 87.

Let us try to simplify the expression to a How many positive factors of 87


more manageable form. We notice that 7
can we count? The approximate
and 13 on top will cancel out the 91 at
the bottom leaving a simpler equation: square root value of 87 is 9. Let us
n = 2•3•5•11 check out all integers between 1
k and 9 and see whether any of
The value for k must be such that the them or all of them could be
value will divide the expression on the
numerator a whole number of times.
factors of 87.
We notice that k cannot be 60 because if
k is 60, n will not be an integer. We must
th
pick the 4 choice. All other values in Factors of 87 : 1, 87, 3, 29,
the choices 1 through 3 and 5 will yield a
whole number value for n. That is it. There are just four
positive factors of 87, and K could
be any of these values. Therefore,
there are four positive integer
values of K exist.

Some of the materials used in this module are taken from disclosed editions of GMAT, and no monetary
value is included in the cost of this module for supply of these materials. These materials are strictly used
for illustration of concepts discussed. Page 46
©Educational Training Services, A Division of Maple Leaf International Consulting, Inc.
Not to be reproduced, copied or distributed. Intended for use only by the registrant.

these values cannot be factors


of 145.
10. Therefore, the factors of 145
are: 1, 145, 5, 29

What are the factors of 256? We


will check out all integer values in
the range from 1 through 16
Let us say that we want to find (square root of 256). Remember:
how many factors exist for 145 We will take pairs of values as
and what they are. We will do the factors such that the two values
following drill: will multiply to give the original
number. If a factor repeats itself,
1. We will find out the we will take just one value as the
approximate square root value factor. In this example, since the
of 145. The square root of 145 square root of 256 is exactly 16,
is roughly 12. 256 will be a perfect square, and
2. We will check out all integer any number that is a perfect
values from 1 through 12 to square will have odd number of
see if the values in this range factors.
will divide 145 a whole number
of times. If you apply the drill outlined
3. Remember: 1 and the number earlier in this section, you will find
itself are always factors of any that the factors of 256 are:
number. 1, 256, 2, 128, 4, 64, 8, 32, 16
4. 1 will divide the given number
145 times. We will take 1 and What are the factors of 171?
145 as factors. We must check out all integer
5. 2 cannot divide 145 a whole values in the range from 1 through
number of times. 2 cannot be a 13 (the closest approximation to
factor of 145. the square root of 171). The
6. Can 3 divide 145 a whole factors are: 1, 171, 3, 57, 9, 19
number of times? No. 3 cannot
be a factor of 145. Remember: If the digits of a
7. Can 4 divide 145 a whole number add up to a single digit
number of times? No. value of 9, then the number must
Therefore, 4 cannot be a factor be divisible by 3 and 9, if the
of 145. integer is an odd integer; if the
8. 5 will divide 145 twenty-nine integer is an even integer, and the
times. Therefore 5 and 29 are digits add up to a single digit sum
factors of 145. of 9, then the number must be
9. 6 through 12 will not divide 145 divisible by 3, 6, and 9. If the
a whole number of times, and digits of an odd integer add up to
6, then the integer is divisible by
Some of the materials used in this module are taken from disclosed editions of GMAT, and no monetary
value is included in the cost of this module for supply of these materials. These materials are strictly used
for illustration of concepts discussed. Page 47
©Educational Training Services, A Division of Maple Leaf International Consulting, Inc.
Not to be reproduced, copied or distributed. Intended for use only by the registrant.

3; if the digits of an even integer


add up to 6, then the integer is 2
divisible by both 3 and 6. A 3
number to be divisible by 5 must 4
end in a digit of value 5 or 0. An 5
even integer divisible by 2 must cannot be determined
end in 0. 2. 4. 6. Or 8.

This problems tests your


understanding of prime integers
and of the properties of integers.
If an integer has just two positive,
and different factors : 1 and the We know that any prime integer
number itself, then the integer is a greater than 2 must be an odd
prime number. integer. We are adding two prime
integers X and Y to get another
Remember: A prime number must prime integer. This must mean
have exactly two positive, and that X + Y is an odd integer. How
different factors. do we get an odd integer by
For this reason, 1 is not a prime adding two integers? Only when
number. one of them is an odd integer, and
the other an even integer. (If both
The smallest and the only even were odd integers, the sum of X
integer that qualifies as a prime and Y will be even, and cannot
number is 2. All other prime qualify as a prime integer). What
numbers are odd. Examples of is the only even integer that
other prime numbers are: qualifies as a prime integer?
3, 5, 7, 11, 13, 17, 19, 23, 29, 31, 2 . Therefore, we pick the first
37, 41, 43, 47, 53, and so on. choice as the answer.

Remember: All but one prime


numbers are odd integers. But all
odd integers are not prime
numbers. Examples of odd
integers that are not prime Did you know that any non-prime
numbers are: 15, 21, 25, 27, 33, integer can be expressed as the
and so on. product of prime integers, not
necessarily distinct? For
example, 6 can be expressed as
Example 1: the product of 2 and 3, prime
numbers. 27 can be expressed as
If X and Y are prime integers such the product of 3, 3 and 3.
that X + Y is also a prime integer,
then X or Y must be equal to
Some of the materials used in this module are taken from disclosed editions of GMAT, and no monetary
value is included in the cost of this module for supply of these materials. These materials are strictly used
for illustration of concepts discussed. Page 48
©Educational Training Services, A Division of Maple Leaf International Consulting, Inc.
Not to be reproduced, copied or distributed. Intended for use only by the registrant.

In order to express an integer as a If 910 is the product of m.n, r, and


product of prime integers, we will p such that 1 < m < n < r < p,
follow the drill suggested here: What is the value of mnp ?

Let us say that we want to π 70


express 910 in terms of its prime π 130
factors. We will start dividing 910 π 182
by the smallest prime number π 455
value that will divide it a whole π cannot be determined.
number of times. In this case, 2
will divide 910 an integer number We can express 910 as a product
of times: 455. But 455 can no of 4 prime integers: 2, 5, 7, and 13.
longer be divided by 2. Let us try
the next prime number: 3. 3 Because of the relationship
cannot divide 455 a whole number specified between m, n, r, and p,
of times. The next prime integer we must conclude that m=2, n=5,
that will divide 455 a whole r=7 and p = 13.
number of times is 5. 5 Will divide
455 Ninety-one times. The product of m, n and p is 130.
91 can no longer be divided by 5. We pick the second choice as the
The next prime number that will answer.
divide 91 a whole number of times
is 7. 7 will divide 91 and yield 13. EXAMPLE 2:
We notice that 13 is a prime
integer, and is no longer divisible. If 390 is a product of four integers
a, b, c, and d such that
We can, therefore, express 910 as 1 < a < b < c < d, then what is
a product of prime integers as: ab/cd?
910 = 2.5.7.13
If the problem tells us that a given
Similarly, 330 can be expressed as integer can be expressed as a
the product of 2.3.5.11 product of more than two
integers, we must treat it as a
170 can be expressed as the prime factors problem. We can
product of 2.5.17 set up 390 in terms of its prime
factors as 390 = 2•3•5•13.
NOTE: If the problem specifies Because 1 < a < b < c < d , we
that a given integer is the product know that a = 2, b = 3, c = 5, and
of more than 2 factors, each d = 13. Therefore, ab/cd = 6/65.
greater than 1, then we must deal
with the problem as “prime
factors” problem.
Example 1.

Some of the materials used in this module are taken from disclosed editions of GMAT, and no monetary
value is included in the cost of this module for supply of these materials. These materials are strictly used
for illustration of concepts discussed. Page 49
©Educational Training Services, A Division of Maple Leaf International Consulting, Inc.
Not to be reproduced, copied or distributed. Intended for use only by the registrant.

IF the problem asked you whether


X is a multiple of 30 given that it is
a multiple of 4 and 5, the answer
will be a resounding “may be”. If X
is 20, it is not a multiple of 30. If X
is 40, it is not a multiple of 30. But
We learned that if Y is a factor of if X is 60, then X is a multiple of
X, then X is a multiple of Y. In 30. In fact, if you take a look at the
real-life terms, if 5 is a factor of list of likely values of X starting
30, then 30 is a multiple of 5. with 20 and moving on in steps of
(Read ‘multiple of’ as ‘any integer 20, we notice that X is a multiple
times’). of 30 for 1 out of every 3 values.
Therefore, the probability that X
Let us say that X is a multiple of 4. will be a multiple of 30 is 1/3. Any
X could be any of the following probability of less than 1 or not
values, starting with 4 as the least equal to zero is good for a “may
positive value ( 1 times 4; then it be” answer and not for a definite
goes as 2 times 4, 3 times 4 and yes or no answer.
so on):
4, 8, 12, 16, 20, 24, 28, 32, 36, 40, IN data sufficiency, if they asked
44, 48, 52, 56, 60, and so on. us the following:

Let us also say that X is a multiple IS X A MULTIPLE OF 20?


of 5. X could be any of the
following values consistent with 1. X IS A MULTIPLE OF 5
this definition. 2. X IS A MULTIPLE OF 6
5, 10, 15, 20, 25, 30, 35, 40, 45, 50,
55, 60, and so on. Our N.T.K is that we need to know
whether X is divisible by 20 for all
We notice that if X is a multiple of 4 and likely values of X or we need to
a multiple of 5, then X must be a multiple know the actual value of X in
of 20, the least common multiple value of
4 and 5. order to determine whether it is a
multiple of 20 or not.
If X is a multiple of 20, then it can have
values starting with 20 and moving in Statement 1 tells us that X could be any of
steps of 20: the following values:
Likely values are 20, 40, 60, 80, 100, 120, 5, 10, 15, 20, 25, 30, 35, 40, 45, and so on. If X
and so on. is 5, then it s not divisible by 20. If X is 20,
then it is. We notice that for every four likely
Do we know precisely what value X has? values of X, only one of them is a multiple of
20. The probability that X is a multiple of 20
We do not. All that we know is that X can
given the first statement is ¼, which is good
be any of the values that are multiples of for a “may be” answer. Not good enough. We
20. must kill choices A and D.

Statement 2 tells us that X can be any of the


following values: 6, 12, 18, 24, 30, 36, 42, 48,
Some of the materials used in this module are taken from disclosed editions of GMAT, and no monetary
value is included in the cost of this module for supply of these materials. These materials are strictly used
for illustration of concepts discussed. Page 50
©Educational Training Services, A Division of Maple Leaf International Consulting, Inc.
Not to be reproduced, copied or distributed. Intended for use only by the registrant.

54, 60, 66, and so on. If X is 6, it is not


divisible by 20. If X is 60, then it is. In fact,
for every 10 likely values of X, only one of
them is divisible by 20. The probability that X
will be divisible by 20, given that it is a Let us try another example.
multiple of 6, is a minuscule 1/10. Not good
for a definite yes or no answer. We must kill
choice B and move on to combine the two If X is a multiple of 4 and 6, then
statements. We have left two choices: C and which of the following cannot be a
E.
value for X?
When we combine the two statements, the
least common multiple value that is in both π 24
sets of likely values is 30. This means that X
is a multiple of 30, and goes in steps of 30
π 36
starting with 30. π 48
π 120
Likely values of X are:
30, 60, 90, 120, 150, 180, and so on. π 150

Can we conclude that X is a multiple of 20 for IF X is a multiple of 4, then it can


all likely values of X, given that X is a
multiple of 30? We cannot. Because, if X is be any of the following values:
30 then it is not divisible by 20. If X were 60, 4, 8, 12, 16, 20, 24, 28, and so on.
then it is. As you can see from the list of
likely values of X, X is a multiple of 20 for
every other likely value of X. This translates
If X is a multiple of 6, then it can
into a probability of ½ that X is divisible by be any of the following values:
20. A probability of less than 1 is good for a 6, 12, 18, 24, 30, 36, 42, and so on.
“may be” answer, and we have learned that
in data sufficiency we will not accept “may
be” for an answer. We notice that the smallest value
that is consistent with both
We will say that we cannot tell definitely one
way or the other whether X is a multiple of 20
specifications is 12. We must
unless we know the actual value of X. We conclude that X is a multiple of 12.
must choose E as the answer. X could be any of the following
When you come across problems asking you
values:
to deal with multiples, be sure to proceed 12, 24, 36, 48, 60, 72, .. 120, 144,
methodically and systematically by 156 and so on.
employing the above reasoning. You cannot
go wrong.
We notice that X cannot have a
value of 150 because 150 is NOT a
multiple of 12. We must pick the
last choice as the answer.

Remember: If X is a
multiple of 12, then X is
divisible by 12.

Some of the materials used in this module are taken from disclosed editions of GMAT, and no monetary
value is included in the cost of this module for supply of these materials. These materials are strictly used
for illustration of concepts discussed. Page 51
©Educational Training Services, A Division of Maple Leaf International Consulting, Inc.
Not to be reproduced, copied or distributed. Intended for use only by the registrant.

Statement 2 tells us that N could


be any of the following values:
6, 12, 18, 24, 30, 36, 42, 48……..
Example 2: Once again, if N is 12 or 18 or 30,
it is not divisible by 24. If N is 24
Is N divisible by 24? or 48, then it is. The probability
that X will be a multiple of 24
(1) N is divisible by 4 given that it is a multiple of 6 is ¼
(2) N is divisible by 6 or not good enough. Let us kill
choice B and move on to combine
π Statement 1 alone but Statement the two statements and see
2 alone is not. whether we can come up with a
π Statement 2 alone but statement winner.
1 alone is not.
π Statement 1 and 2 together When we combine the two
π Either Statement 1 or statement statements, we notice that N must
2 alone is sufficient be a multiple of 12, the least
π Neither statement, alone or common multiple of 4 and 6.
combined, is sufficient. IF N is a multiple of 12, then N can
be any of the following values:
12, 24, 36, 48, 60, 72,……..
This is a data sufficiency
question. The question is: Is N a Can we conclude that N is a
multiple of 24? multiple of 24? We cannot. The
fact that N is a multiple of 12 does
Can we answer the question on not mean that N is automatically a
the basis of statement 1 alone? multiple of 24. The smallest
We cannot because statement 1 values that N can have is 12, and
tells us that N could be any of the 12 is not divisible by 24. Or N
following values: could be 24, which is a multiple of
4, 8, 12, 16, 20, 24, 28, 32, 36,….. 24 as well. If N is 24 (or 48 or 72),
then N is a multiple of 24. The
If N is 4 or 16 or 28, then it is not probability that N is a multiple of
divisible by 24. If N is 24 or 48, 24, given that it is a multiple of 12
then it is. More than one is ½, which is good for a “may be”
possibility here. Not a unique answer, but not for a definite yes
solution. Let us kill choices A or no answer.
and D, and move on to examine
statement 2 and see whether we We must give up and pick the last
can answer the question in a choice: Neither statement alone or
unique fashion on the basis of combined is sufficient to answer
statement 2 alone. the question definitively ( yes or
no).

Some of the materials used in this module are taken from disclosed editions of GMAT, and no monetary
value is included in the cost of this module for supply of these materials. These materials are strictly used
for illustration of concepts discussed. Page 52
©Educational Training Services, A Division of Maple Leaf International Consulting, Inc.
Not to be reproduced, copied or distributed. Intended for use only by the registrant.

Example 3:
DIGITS OF A
Exactly ¼ of the boys in a school NUMBER
walk to the school. Exactly 1/5 of
the boys in the same school play A number in a decimal notation
soccer. Which of the following consists of digits in specific
values cannot be the value for the locations. A digit is an integer
number of boys in the school? having a value between 0 and 9.

π 20 Consider any decimal number. Let


π 60 us say that we have 35.467.
π 150
π 240 We can write this number as:
π 320 35.467 = (3 X 10) + (5 X 1) +
1 1
(4 X 10) + ( 6 X 100)
1
Solution: If exactly ¼ of the boys + (7 X 1000)
walk to the school, then the
number of boys in the school As we can see, 3 takes 10 as the
must be a multiple of 4. coefficient, and is called the
The number of boys could be any “tens’” digit. Likewise, 5 is called
of the following: the units’ digit; 4 the tenth digit; 6
4, 8, 12, 16, 20, 24, and so on. the hundredth digit; and 7 the
thousandth digit.
Of exactly 1/5 of the boys play
soccer, then the number of boys If we multiply the number by 10,
must be a multiple of 5 as well. each of the coefficients attached
to the digits gets multiplied by 10,
The number of boys consistent and the digits become the digit of
with this definition could be any of one higher order. If 35.467 is
the following values: multiplied by 10, we get 354.67.
5,10, 15, 20, 25, 30, 35, …… The 3 is now the hundreds’ digit,
the 5 the tens’ digit; 4 the units’
When we take the two lists of digit; 6 the tenth digit; and 7 the
likely values into account, we hundredth digit.
conclude that the number of boys
must be a multiple of 20, the least IF we divide the number by 10,
common multiple in both sets. each of the coefficients attached
The only value that is not a to the digits gets divided by 10,
multiple of 20 in the answer and the digits of this new number
choices is 150. We must pick the become digits of one lower order.
third choice as the answer.

Some of the materials used in this module are taken from disclosed editions of GMAT, and no monetary
value is included in the cost of this module for supply of these materials. These materials are strictly used
for illustration of concepts discussed. Page 53
©Educational Training Services, A Division of Maple Leaf International Consulting, Inc.
Not to be reproduced, copied or distributed. Intended for use only by the registrant.

Example 1: Let us associate choices A and D


with statement 1, and B, C, and E
If ∇, @, #, $, ! and & represent with statement 2. We know that
non-zero digits, and if the three digits we are looking at –
∇@ @, #, and $ - are non-zero values
+ # $ (they are defined as positive), and
different.
$ & ! Statement 1 tells us that @ could
then $ is be a 1 or a 3 and # must be a 3 or
a 1 so that the sum will be a 4. But
π1 then, we have two possible values
for # and @, and we cannot
π 2 answer the question definitely
π 3 using statement 1 alone. Let us
π 4 kill choices A and D.
π 5 Statement 2 tells us that @ is
more than 1. That does not
Solution: If two two-digit numbers explain what # is about. Let us kill
are added, and the result is a 3- choice B, and proceed to combine
digit number, then the hundreds’ the two statements.
digit of the 3-digit number cannot
be more than 1. (In the extreme When we combine the two
case, let us add 99 to 99. The sum statements, we notice that @
is 198, and the hundreds’ digit is cannot be 1 because a value of 1
1). Since the digits are specified will not satisfy statement 2. The
as non-zero digits, we conclude only other value possible for @ is
that $ must be equal to 1. 3. If @ is a 3, then # MUST be a 1
so that the sum will be a 4, the
value for $.
EXAMPLE 2:
The combined information helps
DATA SUFFICIENCY us sort through the mess and
determine a unique value for #. We
If @, #, and $ represent three must choose C as the answer. (In
different positive digits such that computer format, C corresponds
@ + # = $, what is the value of #? to the third choice in the list of
choices).
1. $ = 4
2. @ > 1

Some of the materials used in this module are taken from disclosed editions of GMAT, and no monetary
value is included in the cost of this module for supply of these materials. These materials are strictly used
for illustration of concepts discussed. Page 54
©Educational Training Services, A Division of Maple Leaf International Consulting, Inc.
Not to be reproduced, copied or distributed. Intended for use only by the registrant.

Example 2:

The cost, C, of manufacturing an If the problem


item is given by the formula: requires that
you express the
C = 0.49• r•s•t2 answer as a
Where r, s and t are three different ratio or as a
component values. If, as a result proportion or as
of plant modernization and labor a percent value,
agreements, r increases by 50%, s you can choose
to work with
increases by 20% and t decreases your own ‘base-
AS a manager-wanna-be, you by 30%, then C line’ values. If
must demonstrate that you have a the problem
good sense of proportion. In fact, π is unchanged tells us that the
radius of a
GMAT is a Factors, Ratios and π decreases by 12% circle doubles,
proportions test, and they must π increases by 12% and asks how
re-christen the test FRAP test. π decreases by 20% many times the
π increases by 20% area is
Example 1: increased, we
can choose to
Solution: work with
2
If X is inversely proportional to r , values of 1 and
and if r is halved, then X is: If r increases by 50%, then r 2 for R. If R=1,
becomes 1.5r then the area is
PI. If R
π halved If s increases by 20%, then s becomes 2, the
π increased two fold becomes 1.2s area is PI•22 =
π decreased two fold If t decreases by 30%, then t 4•PI
π increased four fold becomes (0.7t) The ratio of the
π decreased four fold And t2 becomes (0.49t2) new value to
the old value is
4/1.
Solution: Let us plug in the changes to the Remember: “Y
formula: is how many
If X is inversely proportional to r2, C = 0.49. (1.5r)(1.2s)(0.49t2) times X” means
the same to a
then we can write: X = k. 1r2 C = (0.49) r•s•t2 • ( 0.88)
mathematician
(1.5 times 1.2 times 0.49 gives as the
IF r becomes r/2, then 0.88) statement ‘what
X = k. 1/(r/2)2 = k.4/r2 We can see that C is 88% of fraction of X is
As we can see, if r is the original value or a decrease of Y?”. Both
questions will
halved, then X is increased 4 12%. We pick the 2nd choice. ask you to
times. We pick the 4th choice. You can choose to deal with this problem by compute the
thinking up real life numbers. For example, R ratio Y/X.
could be 2 initially and change to 3 after the
You will deal with these “sense of increase. S could be 5 initially and change to 6
proportion” questions when you after the increase. T could be 10 initially and
go to “solid objects” in geometry. decrease to 7 after the decrease. Do this and see
whether you get the same answer as above.

Some of the materials used in this module are taken from disclosed editions of GMAT, and no monetary
value is included in the cost of this module for supply of these materials. These materials are strictly used
for illustration of concepts discussed. Page 55
©Educational Training Services, A Division of Maple Leaf International Consulting, Inc.
Not to be reproduced, copied or distributed. Intended for use only by the registrant.

PROBLEM 2:

“The volume of a cylinder is obtained by Also notice that we will ‘ignore’ the
using the formula Volume = PI•R2•H, constant PI because the final answer will
where PI is a constant, R is the radius of be in the form of a ratio, and the constant
the cylinder and H its height. If the radius is unaffected by the change. When we do
doubles and the height becomes 1/3rd its the ratio, the constant, which is
initial value, the new volume is how unaffected by the change, will cancel out.
many times the old volume?”
Old Volume (R=1 and H=3) = 12•3 = 3
Notice that ‘how many times’ statement New Volume (R=2 and H=1) = 22•1 = 4
is an invitation to making a ratio
statement. We are required to compute New Volume/Old Volume = 4/3
the ratio of the new Volume to the old The new volume is 33% more than was
Volume. Because the answer will be in the old volume.
the form of a ratio, we can set our own PROBLEM 3:
baseline values and work through the
problem. Remember: You can set your “Container X is twice the capacity of
own baseline values as long as the final container Y. If X is half-filled and Y is
answer is in the form of a ratio. If the 3/4th filled, and if all the liquid in cylinder
problem asked you to find out the Y is transferred to cylinder X, to what
value of the ‘new volume’, you cannot fraction of its capacity is X filled?”
set your own baseline values because
the answer is not required in a ratio Notice that the problem asks us to
format. express the volume of liquid in X as a
fraction of its capacity. The answer is
We will say that R is initially 1 and required in a ‘ratio’ format, and we will
becomes 2 after the change; H is initially set our own ‘baseline’ values.
3 and becomes 1 after the change. Why When we pick our baseline values, we
did we pick these values? Because we must be sure to factor in the fractions
wanted to work with reasonably small specified in the problem, as otherwise we
values and because we wanted to will be dealing with some crazy fractions
continue to work with integer values. If along the way. The two fractions
we had chosen a value of 1 for the specified are ½ and ¾. Let us simply
Height initially, then we will have to use multiply the denominators and say that X
1/3 as the new value after the change. has capacity 8 gallons and Y 4 gallons.
Instead, we chose to work with 3 and 1, X Y
integer values. Remember: You can CAPACITY 8 4
process integers more efficiently and VOLUME 4 3
easily than you can process fractions. AFTER TRANSFER 4+3=7 0
th
Also remember to work with as small a X is filled to 7/8 of its capacity. You
value as possible. 3 and 1 are more easy will get the same answer no matter what
to process and deal with than are the baseline values you chose to work with.
values 6 and 2, or 18 and 6.

Some of the materials used in this module are taken from disclosed editions of GMAT, and no monetary
value is included in the cost of this module for supply of these materials. These materials are strictly used
for illustration of concepts discussed. Page 56
©Educational Training Services, A Division of Maple Leaf International Consulting, Inc.
Not to be reproduced, copied or distributed. Intended for use only by the registrant.

value of 50%. Proportionality


Exercise: will be tested big time in the
If x and y are each two digit GMAT, and questions
numbers with the same two appearing in the GMAT will
TRIVIA: digits but in the reverse order, is have the following genre:
If X and Y are x - y divisible by 9?
two integers EXAMPLE 1:
such that X and One approach to solving this
Y are made up
of the same problem is to think up two 2- “ABC company employed 537
digits, not in digit numbers: Say, 27 and 72. more people in 1999 than they
the same order,
then the
When you take the difference, did in 1998. If the additional
difference of X we get 72 - 27 = 45. We know number of employees in 1999
and Y MUST that 45 is divisible by 9. represented an increase of 30%
be divisible by
9. For example, on the number for 1998, how
if X is 65 and How do you work with algebraic many employees were there in
Y is 56, then expressions to get the same 1998?”
the difference is
9, which is result and reach the same
divisible by 9. conclusion? We can see that a number
If X is 123 and
Y is 312, then
value of 537 corresponds to a
the difference is Let us set up x as percent value of 30%. The
189, which is x = 10 t + u .... (1) question is: What is 100%?
divisible by 9. Let us set up y as
IF X is 2576
and Y is 7625, y = 10 u + t .....(2) Our proportionality statement in
then the an equation form will be:
difference is
5049, which is
What is x - y ?
divisible by 9. x - y = 10 t + u -(10 u + t ) 30/537 = 100/ X1998
Now, go ahead = 10t + u - 10u - t Where X1998 represents the
and try to
impress
x - y = 9t - 9u = 9 ( t - u) number of employees in 1998.
someone you Is (x - y) divisible by 9 ? You
know with this bet. Let us cross multiply and
trivia.
rearrange the equation to get:
FRACTIONS, RATIOS,
PROPORTIONS, AND X1998 = 100•537/30 = 1,790
PERCENTAGES ALL REFER
TO PRETTY MUCH THE The company employed 1,790
SAME INFORMATION. employees in 1998. The value
for 1999 will be 537 more than
1,790, or 2,327.
A fraction of ½ represents a
ratio of 1 to 2 or a percentage
Some of the materials used in this module are taken from disclosed editions of GMAT, and no monetary
value is included in the cost of this module for supply of these materials. These materials are strictly used
for illustration of concepts discussed. Page 57
©Educational Training Services, A Division of Maple Leaf International Consulting, Inc.
Not to be reproduced, copied or distributed. Intended for use only by the registrant.

“A train engine’s maximum speed is


120 m.p.h when no cars are
EXAMPLE 2: attached, and the maximum speed
decreases in proportion to the
“50 fish were caught, tagged, and square root of the number of cars
returned to the pond. During a attached. IF 4 cars are attached, the
second catch of 50 fish, 2 were maximum speed drops by 30 m.p.h
to 90 m.p.h. How many cars of the
found to be tagged. If the
above type need to be attached so
percentage of tagged fish in the
that the engine will stop moving?”
second catch approximates the
percentage of all tagged fish in
The question is: How many cars
the entire pond, how many fish are
should be attached so that the
in the pond?” speed reduction will be in the order
of 120 m.p.h.

We know that a speed reduction of


We know that the pond has 50 30 corresponded to square root of 4.
tagged fish. In the second catch, the The question is: A speed reduction
percent of tagged fish is 2/50 or 4% of 120 corresponds to how many
of the catch. The problem tells us cars?
that 4% represents all tagged fish in
the entire pond. This means that 4% Because the speed reduction is in
corresponds to a value of 50. The proportion to the square root of the
question is: What is 100%? number of cars attached, our set-up
for this problem will look like this:
Our set-up will read:
30 /sqrt(4) = 120/sqrt(N)
4/50 = 100/X where N is the number of cars that
will bring the engine to a grinding
or, X = 50•100/4 = 1,250 halt.

The number of fish in the pond is Sqrt (N) = 120• sqrt(4) / 30


1,250 and the number of tagged fish = 4• 2 = 8
– 50 – represents 4% of this total. Let us square both sides to get:
Our verification shows that the value N = 82 = 64. We need to attach
we obtained makes sense. 64 cars in order to bring the engine
to a halt.
Understand the problem information
and then proceed to set it up as a
simple proportionality equation.
EXAMPLE 3:

Some of the materials used in this module are taken from disclosed editions of GMAT, and no monetary
value is included in the cost of this module for supply of these materials. These materials are strictly used
for illustration of concepts discussed. Page 58
©Educational Training Services, A Division of Maple Leaf International Consulting, Inc.
Not to be reproduced, copied or distributed. Intended for use only by the registrant.

9. How many factors or divisors do the


following integers have? What are they?
Assignment: Integers
(A) 81 ____________ _________
(B) 140____________ __________
(C) 256____________ ___________
1. Integer 9 is not a divisor of which of (D) 171____________ ____________
the following values of y?
10. The difference of two consecutive
(A) 63 (B) 108 (C) 36 (D) 200 (E) 126 even integers is _________________

Your Answer:_______________ 11. The difference of two consecutive


odd integers is __________________
2. Even integer when added to an Odd
integer results in __________ integer. 12. What is the next integer in the
following sequence of even integers?
3. Even integer when multiplied by an
even integer results in 2n, 2n+2, 2n+4, 2n+6,_________
__________integer.
13. Which of the following are odd, even
4. An odd integer when subtracted from or indeterminate integers? (n is not 0)
an even integer results in Odd Even Can’t Say
_____________ integer.
(A) 2n+3 O O O
5. A prime number n is an integer that (B) n+5 O O O
has exactly ________ different (C) 2n+24 O O O
_________ divisors and the divisors are (D) 2n-3 O O O
________________________________. (E) 2(n-7) O O O
(F) 3(2n-1) O O O
6. Which of the following numbers (G) (2n+15) -
cannot be prime number(s) and why? (2n-4) O O O
(H) (2n+64) +
(A)3 (B)169 (C) 32 (D) 83 (E) 59 (3n+5) O O O
(I) n(2n-11)÷n O O O
Your Answer:_____________________ (J) (2n+1)(2n+6) O O O
_________________________________
_________________________________ Hint: n takes on all values (except 0),
positive and negative.
7. The only even prime number
is_______. 14. If x and y are prime numbers and
(x+y) is a also a prime number, then x or
8. The expression n/n=1 is true for all y must be equal to ______________
values of n except _________________

Some of the materials used in this module are taken from disclosed editions of GMAT, and no monetary
value is included in the cost of this module for supply of these materials. These materials are strictly used
for illustration of concepts discussed. Page 59
©Educational Training Services, A Division of Maple Leaf International Consulting, Inc.
Not to be reproduced, copied or distributed. Intended for use only by the registrant.

We get: 2/7 + 3/11 = 22/77 + 21/77 = 43/77


Fractions
Example 2:

A fraction is obtained by taking the ratio of


two integers, and has the form: n/d, where n Add a/b and c/d.
is the numerator and d the denominator.
Let us multiply the numerator and the
The smallest value for the fraction is obtained denominator of the first fraction by d, and those
when the numerator is the smallest and the of the second fraction by b so that both fractions
denominator the largest values. The largest will be expressed in terms of a common
value for the fraction is obtained by making the denominator.
numerator the largest it can be and the We get:
denominator the smallest it can be. Let us say a/b + c/d = ad/bd + bc/bd = (ad + bc)/bd
that we have 5 integers in a set: 3, 13, 17, 23,
31. If we are required to make the smallest We will follow the same procedure when we
fraction using the values in the set, it will be have to subtract two fractions.
3/31. The largest fraction we can set up using
the same values in the set will be 31/3. Example:

You will be dealing with a lot of fractions in the 2/9 – 3/7 = 14/63 – 27/63 = -13/63
GMAT, and you are advised to deal with
fractions as given instead of turning them into Multiplication and Division:
decimal numbers. GMAT is, in the final Multiplication of two fractions is a simple
analysis, a factors test, and you will be able to process: We multiply the numerators and the
simplify expressions by canceling out common denominators separately and form the new
factors contained in the fractional expressions. fraction. Before we begin to multiply, we must
try to cancel out common factors so that we can
If you are required to add two fractions, you will have a fraction in its simplest form.
be required to bring both to a common
denominator value and add the numerators. Example:

Example: 3/8 • 4/7 = 3/2 • 1/7 = 3/14


Add 2/7 and 3/11.
Division of a fraction by another fraction is done
The trick is to multiply the numerator and the by taking the inverse of the denominator
denominator of the first fraction by the fraction, and multiplying the numerator fraction.
denominator of the second fraction, and the
numerator and the denominator of the second Example:
fraction by the denominator of the first fraction.
(3/5) ◊ (5/9) = 3/5 • 9/5 = 27/25
Let us multiply the numerator and the
denominator of the first fraction by 11. We get:
2/7 = 22/77 Example:

Let us multiply the numerator and the (a/b)◊(c/d) = a/b • d/c = ad/bc
denominator of the second fraction by 7. We get:
3/11 = 21/77
Sometimes, we may have to deal with a mixed
As you can see, we have expressed the fractions
number that contains an integer and a fraction.
2/7 and 3/11 in terms of a common denominator
77. What remains to be done is to add the
numerators 21 and 22 while keeping the same
denominator.
Some of the materials used in this module are taken from disclosed editions of GMAT, and no monetary
value is included in the cost of this module for supply of these materials. These materials are strictly used
for illustration of concepts discussed. Page 60
©Educational Training Services, A Division of Maple Leaf International Consulting, Inc.
Not to be reproduced, copied or distributed. Intended for use only by the registrant.

Example: IF the problem specified that we get the


1 ¼ = 1 + ¼ = 4/4 + ¼ = 5/4
maximum possible value for the given
GMAT will test your comfort level with expression, then we must assign 7 to a and the
fractions. Take a look at the following examples. other two values to b and c. The greatest
Example 1: possible value for the given expression is,
What number is 3/5 times as far away from 2/3 therefore, 7/15.
as 6 4/7 is from 5/4? Example 3:
The problem is simply one of finding a value for Which of the following expressions has
the following expression: the highest value?
2
/3 + 3/5 • (6 4/7 – 5/4)
5
/22•52
Let us work on the expression within the 6
/22•53
parenthesis first.
62
/23•53
4 5 46 5 184 35 149
(6 /7 – /4) = /7 – /4 = /28 – /28 = /28 32
/22•53
15
/2•53
Let us now deal with the given expression:
2
/3 + 3/5 • (6 4/7 – 5/4) = 2/3 + 3/5 • 149/28
Solution:
= 2/3 + 447/140
We notice that the fraction with the greatest
= 280/420 + 1341/420
numerator in the choice expressions has the
= 1621/420
denominator reading 23•53
361
=3 /420 (ANSWER)
Let us write all choices in terms of the base
denominator value of 23•53. (This means that we
have to multiply the numerator and the
Example 2: denominator of the first choice by 2•5, the
“If a, b, and c are three different digits and can numerator and the denominator of the second and

have any of the following values: 3, 5 or 7, what the fourth choices by 2 and those of the final
choice by 2•2).
is the smallest possible value for the expression
The choices begin to look as shown below after we
(a/b) / c?
have manipulated the numerators and the
The expression (a/b) / c is equivalent to
denominators so that the denominators of all
(a/b) / (c/1), which translates to:
choices will be the same value:
a
/b • 1/c = a/b•c 50
/23•53
The answer is option D,
Because we are the looking for the least possible 12
/23•53 which has the greatest
value for the expression, we must assign the 62 value for the numerator.
/23•53
Remember to express all
smallest of the three possible values to a and 64
/23•53 fractions in terms of a
assign the other two values to b and c. 60 common denominator
/23•53
when you are asked to
The least possible value for the given expression
compare fractions.
is, therefore, 3/35.
Some of the materials used in this module are taken from disclosed editions of GMAT, and no monetary
value is included in the cost of this module for supply of these materials. These materials are strictly used
for illustration of concepts discussed. Page 61
©Educational Training Services, A Division of Maple Leaf International Consulting, Inc.
Not to be reproduced, copied or distributed. Intended for use only by the registrant.

Example 4:
Example 6:
The price of a stock at opening bells was 678, and
10
the price of the same stock at closing bells was /0.0005 = ?
737. By what percent did the price of the stock
We notice that 0.0005 is the equivalent of
change during the trading day?
5
/10,000,
% Change =
and 10/(5/10,000) = 10 • (10,000/5) = 20,000.
(change in value)/(opening value)
100% We could have obtained the same results by

multiplying the top and the bottom by 10000 so
% Change = [(52/7 – 55/8) + 55/8 ]• 100%
= [(416/56 – 385/56) +55/8 ]• 100% that we can turn 0.00005 into 5. The numerator
= [31/56 • 8/55] • 100% becomes 100,000 and when 100,000 is divided
= 31/7 • 1/55 • 100% (8 cancels 56
seven times) by 5, we get 20,000.
31 1 (100 and 55 have a
= /7 • /11 • 20% Example 7:
common factor of 5)

= 620/77%
= 8% (approximately) 1 ¾ + 6 7 /8 =?
The price increased by 8%
over the opening value. 6+ 1
Note: A fraction is turned into a ¾ + 5 /9
percent value simply by the process of
multiplication of the fraction by 100.
The numerator expression simplifies to:
7
/4 + 55/8 = 56/32 + 220/32 = 276/32 = 69/8
Example 5:
The denominator expression simplifies to:
5 5
/100 is what percent of /1000? 6+ 1 = 6 + 36/47
27
5
/100 is the same as 50/1000. /36 + 20/36

50 5
= 282/47 + 36/47 = 318/47
/1000 is ten times /1000,
or 10•100% = 1000% of 5/1000. The given expression is the ratio of:

Note: 5/1000 is 10% or 1/10th of 5/100 (50/1000).


69
/8 + 318/47
= 69/8 • 47/318

= 3243/2544 = 1081/848
(We killed the common factor of 3 from the top and the bottom).

Some of the materials used in this module are taken from disclosed editions of GMAT, and no monetary
value is included in the cost of this module for supply of these materials. These materials are strictly used
for illustration of concepts discussed. Page 62
©Educational Training Services, A Division of Maple Leaf International Consulting, Inc.
Not to be reproduced, copied or distributed. Intended for use only by the registrant.

After receiving 3 marbles, Cathy will have


RATIOS 7N+3 marbles.
The new ratio, in terms of N, after the gift is:
A ratio specifies the relative proportion of the (11N – 3)
component values in a set. If the ratio of men to (7N+3)
women to children in a population is 5:4: 4, we
can conclude that for every 5 men in the
The numerical value of this ratio is
16
population, there are 4 women and 4 children /11.
respectively. Do we know anything about the Our equation is: (11N – 3) = 16
size of the population or the numbers of men, (7N + 3) 11
women, and children in the population? We do Let us cross multiply and set up the
not. All that we know is that the numbers of
equation: 11•( 11N-3) = 16•(7N + 3)
men, women, and children have a common
multiple that will cancel out leaving behind the
ratio as specified above. For example, the 121N – 33 = 112N + 48
population could consist of 5000 men, 4000
women and 4000 children; or, the population 9N = 81, or N = 9
could consist of 5 million men, 4 million Let us plug in the value for N to get the
women, and 4 million children.
number of marbles Mary and Cathy have
All that we can say from the above ratio is that after the gift. Mary has 11N – 3 = 96
there are 5N men, 4N women, and 4N children marbles. Cathy has 7N + 3 = 66
in the group. (N is a common multiple, which marbles.
will cancel out when a ratio is taken). Answer: Mary has 96-66 = 30 more
marbles than Cathy does after the gift.
Total value for the population is = 5N + 4N +
4N = 13N
Learn to set things up in the form of a matrix as
shown below so that you can see things in clear
Men are 5N/13N • 100% = 38% of the perspective.
population.
Women are 4N/13N • 100% = 31% of the Before After
population
Children are 4N/13N • 100% = 31% of the Mary 11N 11N-3
population

Example: Cathy 7N 7N+3


“Mary and Cathy have marbles in the
ratio 11: 7. If Mary gifts away 3 of
11 (11N-3)
her marbles to Cathy, then the new Ratio /7 /(7N+3) = 16/11
ratio of marbles will be 16:11. How
many more marbles does Mary have
than Cathy does after the gift?” NOTE:
As you can see, we cannot determine the new
Solution: ratio if the problem specifies that Cathy loses 3
marbles and Mary gains 3 marbles because we
Step 1: Because we do not know how many do not know to or from what initial values we are
marbles that each lady has, we will start by adding or subtracting the values. But, if we are
told the new ratio after the exchange of marbles,
saying that May has 11N marbles and Cathy has
we can determine the values initially and the
7N marbles so that we have a ratio of 11:7. values after the exchange.
After the gift, Mary will have 11N-3 marbles.

Some of the materials used in this module are taken from disclosed editions of GMAT, and no monetary
value is included in the cost of this module for supply of these materials. These materials are strictly used
for illustration of concepts discussed. Page 63
©Educational Training Services, A Division of Maple Leaf International Consulting, Inc.
Not to be reproduced, copied or distributed. Intended for use only by the registrant.

Let us do one more to understand this drill:

Ratio of more than two If the ratio of men to women in a club


quantities and how to get a is 3 to 4 and the ratio of women to
children in the same club is 5 to 2,
combined ratio?
what is the ratio of men to women to
children in the club?
If X to Y is 2 to 3 and Y to Z is 3 to 5,
what is X:Y:Z? We notice that the “women” values in the two
ratios are not the same. The drill is that we
WE can see that for every 2 X’s, we have 3 Y’s multiply the first ratio by the “women” value in
and for every 3 Y’s, we have 5 Z’s. the second ratio.
The ratio of X:Y:Z is 2:3:5. The first ratio becomes 15:20
We also multiply the second ratio by the
If X to Y is 2 to 3 and Y to Z is 5 to 7, “women” value of the first ratio.
what is X:Y:Z? The second ratio becomes 20:8

We notice that the proportional values for Y in We can see that for every 15 men, there are 20
the two statements are not the same. We need to women and for every 20 women, there are 8
bring them to a common value so that we can men.
express the ratio of X to Y to Z. The ratio of men to women to children is
15:20:8
The drill is as follows: Exercise:

X Y Z If x = 4y and 3y = 7z, what is x:y:z?


2 : 3
5 : 7 We notice that the first statement tells us that
We notice that we have two values for Y’s in the the ratio of x to y is 4:1. The ratio of X to Y is
two ratios, but these values do not have anything the same as the value of X/Y. If X = 4Y, then
in common. We NEED to bring the two Y X/Y = 4/1 or the ratio is 4 to 1.
values to the same level so that we can tell for a The second statement tells us that the ratio of y
given number of Y’s how many of X’s we have to z is 7:3 because if 3Y = 7Z, then Y/Z = 7/3.
and how many of Z’s we have. Let us do the We notice that Y has two values 1 and 7 but we
trick we will apply when we deal with two need to bring them to the same common value
fractions, with different denominators: we bring so that we can tell for a given number of Y’s,
them to a common denominator value. Let us do how many X’s and how many Z’s exist.
the same in this case. After all, a ratio is nothing
but a fraction. (The ratio of X to Y is the same Let us multiply the first ratio by 7 to get the
as the value of X/Y). ratio of X to Y as 28:7
Let us multiply the second ratio by 1 to get the
Multiply the first ratio by the proportional value ratio of Y to Z as 7:3
of Y from the second ratio: In this case, 5
The first ratio becomes X: Y =10:15 X Y Z
28 : 7
Multiply the second ratio by the proportional 7 : 3
value of Y from the first ratio: In this case 3.
The second ratio becomes X:Y = 15:21
We notice that for every 7 y’s we have 28 x’s
We notice that for every 10 X’s, we have 15 Y’s and 3 z’s.
and for every 15 Y’s, we have 21 Z’s.
The ratio of X to Y to Z is 10:15:21 The ratio of x:y:z is 28:7:3

Some of the materials used in this module are taken from disclosed editions of GMAT, and no monetary
value is included in the cost of this module for supply of these materials. These materials are strictly used
for illustration of concepts discussed. Page 64
©Educational Training Services, A Division of Maple Leaf International Consulting, Inc.
Not to be reproduced, copied or distributed. Intended for use only by the registrant.

Exercise: Sedans: Minivans: SUV’s

The ratio of men to women in a club is 2 8 5 20


2 to 3, and the ratio of women to
4 20 5 25
children in the same club is 4 to 3. If a
total of 580 members are in the club,
how many more women are there than
children? Ratio of Sedans:Vans:SUV is 8:20:25
Number of sedans = 8N
The ratio of Men to women is 2 to 3 and that of Number of Vans = 20N
women to children is 4 to 3. Number of SUV’s = 25N
We notice that the “women” numbers are the Total number of all types of vehicles = 53N
same in both statements, and we need to bring = 1,590,000
them to the same value. N = 30,000
Let us multiply the first ratio by 4 and the (Sedans + Minivans) – SUV’s = 28N – 25N =
second by 3. 3N = 90,000 vehicles. (Answer)
The ratio of Men to women becomes 8 to 12.
The ratio of Women to children becomes 12 to 3. The ratio of the number of men and women
9. in a club is 3 to 4. If 6 men resign their
We can see that the ratio of men to women to membership and 2 more women join the
children is 8:12:9 club, the new ratio becomes 3 to 5. How
We can see that there are 8N men, 12N women many more women than men are there in
and 9N children for a total of 29N. the club after the change?
We are given that 29N = 580 or N = 20. Before After
Let us plug the value of N back into the numbers
Men 3N 3N-6
to get:
Number of men in the club is 160. Women 4N 4N+2
Number of women in the club is 240.
(3N-6)
Number of children in the club is 180. Ratio ¾ /(4N+2) =
3
There are 60 more women than there are /5
children in the club.
Complete the problem using the above set-up.
Assignment: (Please show your set up.)
Answer: 20
1. If the ratio of biographies to novels in a
library is 5 to 7 and the ratio of novels to 4. Adam, Bob, and Chris agree to divvy up the
science books is 3 to 8, how many more lottery winning in the ratio of 3: 4: 5. If the
science books than the other two types combined share of Bob and Chris is
combined are in the library if the library has $180,000, What is the share, in dollars, of
a total collection involving these three Adam?
categories of 1,656 books?
Answer: 360 Answer: $60,000

2. A town has passenger cars of the following 5. A car dealership has 150 new cars in its lot.
three descriptions: sedans, minivans, and The cars are in two sizes: Mid-size and
sport utility vehicles, and the total Large. If there are three large sized cars for
population of these vehicles is 1,590,000. every two mid-sized vehicles in the lot, and
The ratio of sedans to minivans is 2 to 5 if there are no other sizes of cars in the lot,
and the ratio of minivans to sport utility how many large sized and mid sized cars,
vehicles is 4 to 5. How many more sedans respectively, are in the lot?
and minivans combined are in the town
than there are sport utility vehicles? Answer: 90 and 60

Some of the materials used in this module are taken from disclosed editions of GMAT, and no monetary
value is included in the cost of this module for supply of these materials. These materials are strictly used
for illustration of concepts discussed. Page 65
©Educational Training Services, A Division of Maple Leaf International Consulting, Inc.
Not to be reproduced, copied or distributed. Intended for use only by the registrant.

6. Two-thirds of the 23.4 million air travelers What do we see here? For every BC women,
from Chicagoland used Chicago’s O’Hare there are AC men and AD children. Therefore,
airport. If the number using Chicago’s the ratio of men to women to children is:
O’Hare was twice the number using the
Mid-way airport, and if the number using
Gary, Indiana’s municipal airport was one-
third the number using Chicago’s Mid-way Answer: AC:BC:AD
airport, approximately how many million
passengers used the Gary’s municipal
airport? 8. “If the ratio of passenger cars to mini-vans
Answer: 2.6 million in a town is X to Y, and that of passenger
cars to S.U.V’s is Y to Z, the ratio of
passenger cars to mini-vans to S.U.V’s is
what?”
We saw earlier in this discussion that if PASSENGER MINI S.U.V’s
3X = 4Y, then the ratio of X to Y is 4 to 3. CARS VANS

As a rule, if MX = NY, then the ratio of X to X Y


Y is the ratio of coefficient of Y to that of X,
or N to M.
Y Z
If 2X = 5Y, then the ratio of X to Y is 5 to2.

If 6X = 4Y, then the ratio of X to Y is 4 to 6.

Try this one for size: Answer: XY : Y2 : XZ

7. “If the ratio of Men to Women is A to B,


and that of men to children is C to D, what
is the ratio of men to women to children?" 9. There are twice as many apples in a basket
as oranges, and three times as many
Hint: oranges as pears. If there are 100 fruits of
Set up the problem as follows: the above three description, How many
MEN WOMEN CHILDREN more apples than oranges are there in the
basket?”

A B Hint: The ratio of apples to oranges is 2 to


1,and that of oranges to pears is 3 to 1.
C D
Answer: 30
NOTE: If we have to write as an equation the
statement “there are twice as many apples as
Let us bring the two values A and C to a oranges”, we would write A = 2•O, and read
common “denominator” value of AC by the ratio of apples to oranges as 2 to 1.
multiplying the first ratio by C and the second “Three times as many oranges as pears” will
ratio by A. We get: be written: O = 3•P or the ratio of oranges to
MEN WOMEN CHILDREN pears will be 3 to 1. You should have the
flexibility to deal with the same information
in either manner. GMAT is about flexibility
AC BC and not about rigid thinking.

AC AD

Some of the materials used in this module are taken from disclosed editions of GMAT, and no monetary
value is included in the cost of this module for supply of these materials. These materials are strictly used
for illustration of concepts discussed. Page 66
©Educational Training Services, A Division of Maple Leaf International Consulting, Inc.
Not to be reproduced, copied or distributed. Intended for use only by the registrant.

PROBABILITY IS A there are 40 balls in the box. Therefore, the


PROPORTION probability of not picking a Red is the same
STATEMENT as the proportion of Green and Yellow balls
in the box. It is 30/40 or 3/4th.
Probability is another way of expressing a
proportion information. If the problem Some problems will test your ability to apply
specifies a ‘ratio’, it is possible for us to use a fraction on another fraction. Such
the ratio information to compute what problems take on the connotation of
fraction of the total population is each of the ‘conditional probability’, which is nothing but
values in the ratio. a ‘fraction of a fraction’ problem. Consider
the following problem:
Let us say that the ratio of males to females
in a population is 2 to 3. A ratio is a “In a population, the ratio of males to
‘conditional statement’ and tells us that IF females is 3 to 5. Among the males, the ratio
there are 2 males, there must be 3 females of smokers to non-smokers is 2 to 5. If a
for a total group size of 5 people. Therefore, person is to be randomly chosen from the
males are 2/5th of the total group, and population, what is the probability that a
females are 3/5th of the total group. male-smoker will be picked?”

If we were to randomly pick a person from IF the ratio of Males to Females is 3 to 5,


the population, the probability that it will be then we know that the probability of picking a
male is the same value as what fraction of male is 3/(3+5) = 3/8, and that of picking a
the total is made up males. In this problem, female is 5/8.
the probability of randomly picking a male is
2/5. Similarly, the probability of randomly IF the ratio of smokers to non-smokers
picking a female is 3/5. among men is 2 to 5, we know that 2/7th all
males are smokers and 5/7th of all males are
Let us sum up the procedure for computing non-smokers.
probability using a ratio statement:
The probability of picking a male smoker
If the ratio of males to females in a group is from the total group = 2/7•3/8 = 3/28.
X to Y, then the probability of randomly
picking a male is X/(X+Y). The probability of This means that 2/7th of 3/8th of all people in
randomly picking a female is Y/(X+Y). the group are male-smokers.

Let us say that a State Congress is made up Take a look at another problem.
of Republicans and Democrats. If the ratio of
Republicans to Democrats is 3 to 4, then we “Students pick one of the three majors:
know that IF there are 3 Republicans, there English, French, or Spanish. If the ratio of
must be 4 Democrats for a total Congress English majors to French majors in a school
size of 7 people. Therefore, the probability of is 5 to 3, and if the ratio of French majors to
randomly picking a Republican is the same Spanish majors is 4 to 7, and if a person is to
as what fraction of the total Congress is be randomly chosen from this group, what is
made up of Republicans. It is 3/7. Similarly, the probability that a Spanish major was
the probability of randomly picking a picked?”
Democrat from the population is 4/7.

Take a look at the following problem: We need to express the combined ratio of
“If a box contains 10 Red balls, 13 green English to French to Spanish. Notice that
balls, and 17 yellow balls, and if a ball is to French is a common value to both ratios, and
be drawn randomly from the box, what is the before we can use this common value to
probability that it is NOT Red?” express a combined ratio of three values, we
must make the values for French equal in
“Probability of NOT RED” is the same as the both ratios. The least common multiple for 3
probability of ‘green or yellow’. We are and 4 is 12, and we must write the French
required to express the number of Green and value in terms of 12, and all other values
Yellow balls as a fraction of all balls in the using the same multiples.
box. Green and Yellow add up to 30, and
Some of the materials used in this module are taken from disclosed editions of GMAT, and no monetary
value is included in the cost of this module for supply of these materials. These materials are strictly used
for illustration of concepts discussed. Page 67
©Educational Training Services, A Division of Maple Leaf International Consulting, Inc.
Not to be reproduced, copied or distributed. Intended for use only by the registrant.

PROBLEM 4:
We get:
“A box contains 7 Red balls, 5 Blue balls,
E F S and 6 Green balls. Three balls are drawn
5 20 3 12 from the box sequentially, without
4 12 7 21 replacement. What is the probability that AT
LEAST one Red ball was drawn?”

We can see that for a group of 12 French We are required to compute the probability
majors, there are 20 English majors and 21 that at least 1 Red was drawn. “At least one”
Spanish majors. means “one or more”. The easiest approach
The combined ratio is 20:12:21 for a total of to dealing with a problem that specifies ‘at
53 units. least one’ of a type of outcome is to find out
the probability of an undesired outcome and
English majors are 20/53 of the total. subtract it from the total probability of 1.
French majors are 12/53 of the total. Remember that a probability of 1 is a
Spanish majors are 21/53 of the total. certainty. In this problem, we do not want an
outcome in which NONE of the three balls is
Therefore, the probability of randomly Red. This outcome is possible when each of
picking a Spanish major is 21/53. the three balls drawn is either Blue or Green.

The probability of drawing a Blue or Green in


each of the three successive draws without
PROBLEM 3: replacement = 11/18•10/17•9/16 = 55/272

“A State Legislature is made up of Hottentots Notice that we had 11 balls of Blue or Green
and Rottentots. If the ratio of Hottentots to color out of a total of 18 when we were about
Rottentots is 5 to 6, and if the ratio of to draw the first ball. Because the ball was
Catholics to non-Catholics in the group is 4 not replaced, the total number of balls is 17
to 5, and if a person is to be randomly for the second draw, and 16 for the third
chosen from the Legislature, what is the draw. Also, we need to subtract the total
probability that a Hottentot subscribing to number of blue and Green balls by 1 for each
the Catholic faith will be picked?” successive draw because we are assuming
that the ball drawn is not red.
If the ratio of H to R is 5 to 6, then the
proportion of H in the group is 5/11 and that Probability of picking a blue or a green in
of R is 6/11. each of the three successive draws = 55/272

If the ratio of Catholics to non-Catholics is 4 Notice that this is the probability of an


to 5, then Catholics are 4/9th of the total undesired outcome.
group, and Non-Catholics 5/9th of the total
group. Probability of picking at least 1 Red ball =
1 – probability of picking no red ball
We should expect 4/9th of 5/11th of the total =
group to be made up of Catholic Hottentots. 1 – 55/272 = 272/272 – 55/272 = 217/272
Therefore, the probability of randomly Remember: PROBABILITY IS NOTHING BUT
picking a Catholic Hottentot is 20/99 or A PROPORTION STATEMENT. Probability of
approximately 1/5. picking X in a random selection is the same
as what fraction of the total group is
represented by X. Conversely, if the
probability of picking a male in a random
rd
selection is 1/3, then 1/3 of the total group
is made up of males.

Some of the materials used in this module are taken from disclosed editions of GMAT, and no monetary
value is included in the cost of this module for supply of these materials. These materials are strictly used
for illustration of concepts discussed. Page 68
©Educational Training Services, A Division of Maple Leaf International Consulting, Inc.
Not to be reproduced, copied or distributed. Intended for use only by the registrant.

Let us define the consecutive positive


odd integers as 2n+1, 2n+3, 2n+5,
Averages 2n+7, 2n+9, and 2n+11.

The sum of the six integers is 12n + 36.


The average of a group of values is the
The average of the six integers is
ratio of the sum of all the values to the
2n + 6.
number of values in the group. The
The equation for the information that
sum of a set of values is the product of
“the average of six consecutive
the average of the set and the number
integers is 20 less than twice the
of values in the set.
largest integer in the set” is:
Example:
2n + 6 = 2(2n+11) - 20
“If the set X contains the values n,
2n + 6 = 4n + 22 - 20
n+1, 2n+1, 3n+1, and 4n+1, what is
2n = 4 or n = 2
the average of this set of values?”
The average of the integers is: 2n + 6 =
10
Average = (Sum of all values) / (No. of
values)
Example 3:
= (n+n+1+2n+1+3n+1+4n+1) / 5
“The sales in the first half of the year
= (11n + 4) / 5
were X dollars. The sales in the second
Example 1:
half of the year were on an average
“If x + y = 11z, what is the average, in
$10,000 more per month than in the
terms of z, of 3x, 3y, and 6z?”
first half. What is the expression, in
The average of 3x, 3y, and 6z is
terms of X, for the average sales per
(3x+ 3y + 6z) = x + y + 2z
month during the year?”
3
We are required to get the answer in
Sales in the First half = X dollars.
terms of z only. This requirement
Sales in the Second half = X +
mandates that we get rid of x and y in
6•$10,000
the answer. How can we do it? We are
= X +$60,000
given that x + y = 11z. Let us rewrite
Total sales for the year = X + X +
the answer by replacing (x + y) with
$60,000
11z.
= 2X + $60,000
We get:
Average sales per month during the
The average of 3x, 3y, and 6z, in terms
year =
of z, is
(2X + $60,000) / 12 = X/6 +
11z + 2z = 13z (Answer)
$5,000
Example 2:
We should look for a choice that
“The average of six consecutive
contains the expression: X/6 + $5,000.
positive odd integers is 20 less than
twice the largest integer in the set.
What is the average of the set of
integers?”

Some of the materials used in this module are taken from disclosed editions of GMAT, and no monetary
value is included in the cost of this module for supply of these materials. These materials are strictly used
for illustration of concepts discussed. Page 69
©Educational Training Services, A Division of Maple Leaf International Consulting, Inc.
Not to be reproduced, copied or distributed. Intended for use only by the registrant.

Example 4: Example 6
ABC couriers charge 75 cents for the
first pound and 50 cents for each If x+y = 3z, what is the average of x,
additional pound of package sent. 3x, 4y, and 12z, in terms of z?
What is the average cost per pound, in The average of any set of values is the sum of
dollars, of a package weighing p the values divided by the number of values in
pounds? (1 dollar = 100 cents) the set.

Cost of sending the first pound = 75 We have four values: x, 3x, 4y, and 12z.
cents
The average of the four values is:
Cost of sending the additional (p-1) (x+3x+4y+12z) / 4 = (4x+4y+12z)/4
pounds = (p – 1)• 50 cents = x + y + 3z
Total cost of sending the package = We notice that we are required to state the
75+(p-1)•50 = (25 + 50•p) cents answer in terms of z only.
Average cost per pound =(25 + 50•p) / p
Can we find a way to get rid of x + y from the
cents above answer? Yes, we can. We are given that
Average cost per pound in dollars = x + y = 3z. Let us replace x + y with 3z to get:
(25 + 50•p)
/100•p dollars The average of 4 values = 3z+3z = 6z.
25(1+2p)
= /100•p dollars
= (1 + 2p)/4p dollars Example 7:
Example 5: “The average of Sam’s three consecutive
“N students agree to share equally the cost of a weekly savings was $80. If Sam’s first
gift to their teacher. The gift cost X dollars. If M weekly savings was one half that in the
students later failed to contribute their share, second week and one third that in the third
and the total cost of the gift was shared equally week, how much did he save the second
by the remaining students, how many more week?”
dollars does each contributing student pay as
their share of the cost of the gift?”
That the average of three week’s savings was
$80 tells us that the total savings in the three
Scenario 1: weeks are $240. Let x be the savings in
Cost of the gift = X dollars week1, y that in week2, and z that in week3.
Number of students contributing = N We have : x + y + z = $240 ………. (1)
Share of each student We also have: x = ½ • y or y = 2x
= Cost / Number sharing the cost And x = 1/3 • z or z = 3x
= X/N dollars Let us rewrite (1) by replacing y and z in
terms of x only. We get:
Scenario 2: x + 2x + 3x = $240 or x = $40
Cost of the gift = X dollars Because y = 2x, we conclude that y = $80.
Number of contributing students = (N-M) If the problem asked you to find the value for
Share of each student = X / (N-M) dollars z, you will go back to z = 3x and conclude
that z = $120
Additional share of each student under the
scenario 2 = [X/(N-M)] - X / N
= X(N - N + M) / N•(N-M)
= MX
N•(N-M)

Some of the materials used in this module are taken from disclosed editions of GMAT, and no monetary
value is included in the cost of this module for supply of these materials. These materials are strictly used
for illustration of concepts discussed. Page 70
©Educational Training Services, A Division of Maple Leaf International Consulting, Inc.
Not to be reproduced, copied or distributed. Intended for use only by the registrant.

Example 8: have eliminated choice2 as a viable


Data Sufficiency: choice at this point. We have to
“If the average of a set of 4 integers is choose one of the remaining two:
45, how many integers in the set are choice 3 or choice 5.
greater than 45?” When we combine the two statements,
1. Two of the integers in the set are we have information pertaining to
50 and 35. three of the four values, and we can
2. One of the integers in the set is 20. determine that the fourth value must
be 75. We can answer the question
We notice that the sum of the four definitively now, using the combined
integers in the set is 4 times 45 or 180. information.
3. Statement 1 tells us that two We must pick choice3.
integers add up to 85. This
information tells us that the Example 8:
remaining two must add up to 180-
85 = 95. The remaining two “A group consists of 20 men
numbers could be both bigger than and 15 women. If the average
45 (say, Two of the integers in the I.Q. of men in the group is 125
set are 50 and 35. and that of women in the group
4. One of the integers in the set is 20. is 130, what is the average
We notice that the sum of the four
I.Q. of all members of the
integers in the set is 4 times 45 or 180. group?”
Statement 1 tells us that two integers
This is a weighted average problem. We need to
add up to 85. This information tells us determine the total I.Q values for men and
that the remaining two must add up to women in the group, add them, and divide the
180-85 = 95. The remaining two total by the total number of men and women in
numbers could be both bigger than 45 the group.
(say, 47 and 48) or just one integer
Average I.Q. = (20•125 + 15•130) / (20+15)
could be more than 45 (say, 80) and = (2500+1950) / 35
the other less than 45 (say, 15). We are = 4450/35 = 127 (approx.)
looking at two conflicting scenarios The average I.Q of all members in the group is
here and must conclude that the about 127.
statement 1 is not good for a unique
10. “If the average of (a+4), (b+5), (c+6), and
answer. We have eliminated choices 1
(d-11) is 11, what is the average of (a + b)
and 4 as possible contenders. We will and (c + d)?”
have to choose one of the three The question is: What is ½ • (a + b + c + d)?
remaining choices. The only unknown quantity is the value for
The statement 2 tells us that one of the (a + b + c + d). Can we determine that by using
values is 20. This means that the the first piece of information? Yes, we can.
The set up for the first piece of information is:
remaining three add up to 160. All
¼ • (a + 4 + b + 5 + c + 6 + d – 11) = 11
three could be more than 45 each, or Or, a + b + c + d + 4 = 44
just two are more than 45 or just one. Or, (a + b + c + d) = 40.
(say, 100, 35 and 25). Once again, The average of (a + b) and
multiple conflicting scenarios here. We (c + d) is ½ • 40 = 20.

Some of the materials used in this module are taken from disclosed editions of GMAT, and no monetary
value is included in the cost of this module for supply of these materials. These materials are strictly used
for illustration of concepts discussed. Page 71
©Educational Training Services, A Division of Maple Leaf International Consulting, Inc.
Not to be reproduced, copied or distributed. Intended for use only by the registrant.

Example 9:

“The average price per dozen of Example 10:


donuts during the month of July was
$5.95, and that during the month of “If John bought X books at $5 each and Y
August was $6.40. If twice as many books at $7 each, what is the average price
dozens were sold in August as in July, paid by John per book in terms of X and Y?”
what was the average price per dozen
This is another weighted average problem. We
for the two month period?”
need to find out how much money John spent
buying books and then divide the money spent
This is a weighted average problem. If by the number of books bought by John.
twice as many dozens were sold in
August as in July, then we know that the John paid 5X dollars to buy X books at $5 each.
ratio of sales in August to that in July is 2 John paid 7Y dollars to buy Y books at $7 each.
John paid a total of 5X + 7Y dollars to buy
to 1. We can set the quantities as X for (X + Y) books.
July and 2X for August or simply take Average price paid by John is
quantity 1 for July and quantity 2 for (5X + 7Y) / (X + Y)
August. Remember, we are dealing with
proportions here and whether we take X Example 11
and 2X or 1 and 2, or 5 and 10 is
immaterial to the answer, which will be DATA SUFFICIENCY
same regardless of how we set it up.
Because we do not want to deal with “Is the average of X and Y greater than 10?”
numbers that look like Bill Gate’s
1. The average of X + 5, Y + 7, and
networth or Amazon.com’s cumulative 10 is 10.
losses since their I.P.O, let us deal with 2. X = 3Y
simple 1 and 2. Our set up will look like
this: Our N.T.K is that we need to know the value for
Month Avg. Quantity Revenue ½ •(X + Y) so that we can compare this value to
Price Sold 10 and decide whether the average is bigger
July 5.95 1 5.95 than 10 or not. The statements must help us get
a number value for this average.
August 6.40 2 12.80
Statement 1 helps us determine the value for
Total 3 18.75 X + Y, and therefore the average. The set up for
statement 1 is:
1/3 • (X + 5 + Y + 7 + 10) = 10
The average price per dozen is the Or, X + Y + 22 = 30 or X + Y = 8
ratio of the total revenue to the total We can see that it is possible to compute the
quantity sold in the two-month average by knowing the value for X + Y and
period. compare it to 10, and answer the question with a
definite yes or no. Statement 1 is good. Let us
kill B, C, and E. We retain choices A and D.
The average price/dozen = 18.75/3
Is statement 2 good for a number value for
= $6.25 per dozen. the average? No. We can only get the
average in terms of X or Y, and there is no
way we can compare it to 10.
We must choose Choice A.

Some of the materials used in this module are taken from disclosed editions of GMAT, and no monetary
value is included in the cost of this module for supply of these materials. These materials are strictly used
for illustration of concepts discussed. Page 72
©Educational Training Services, A Division of Maple Leaf International Consulting, Inc.
Not to be reproduced, copied or distributed. Intended for use only by the registrant.

500 and the standard deviation for this


Standard Deviation population is 40, approximately what percentage
of the students scored between 540 and 580 on
the test?
Standard Deviation is a concept that is used to
indicate the distribution of the values in the Solution:
population around the arithmetic mean, or the
average. The problem tells us that 67% scored between
(500-40)=460 and (500+40)=540.
Let us say that a thousand applicants vie for The problem also tells us that 95% scored
admission to a graduate school of business, and between (500-80)=420 and (500+80)=580.
the average GMAT score of all applicants is The inference we can make is that 95%-
550. Let us also say that the scores are normally 67%=28% scored in the following two ranges:
distributed and that two-thirds of the applicants 420-460 and 540-580.
submit scores that are one standard deviation
away from the average score and that 95% of We are told that the scores are normally
applicants submit scores that are within two distributed, which means that the percent of
standard deviations away from the average those scoring in the 420-460 range is the same
score. as that of those scoring in the range 540-580.
If we know that the Standard Deviation for the And the value is ½ of 28% or 14%.
population is 35, we can conclude that the
GMAT scores of two-thirds of the applicants lie
in the range between (550-35)=515 and (550 +
The answer is that 14% of test-takers
35) = 585. This is how we interpret the scored in the range between 540 and
information that the scores of two-thirds of 580.
applicants lie one standard deviation away from
the average score.
How do we read the information that the GMAT How do we find the standard
scores of 95% of applicants are 2 standard
deviations away from the mean?
deviation of a group of
We will read this information to mean that the values?
GMAT scores of 95% of applicants lie in the
range between (550- 2•35) = 480 and Step 1: Find the average of the group of values.
(550+2•35) = 620.
If the problem specified further that 99% of all Step 2: Find the deviation from the average for
applicants had their scores within three each of the values in the group.
standard deviations of the mean, we will read
this information to mean that the scores of Step 3: Square the individual deviations from
nearly all applicants were between (550 – 3•35) the mean, and add the squared deviations.
= 445 and (550 + 3•35) = 655.
Step 4: Divide the sum of the squared deviations
Standard Deviation is a concept that should by the number of values in the group.
invoke the familiar “bell-shaped” curve when
the name is dropped. You should understand Step 5: Take the square root of the value in step
this concept thoroughly because you will expect 4. You will have the standard deviation for the
to get some questions relating to this concept. group looking at you.

Example:

Students at a local town took the SAT1, and the


results were normally distributed, with 67%
scoring within one standard deviation of the
mean and with 95% scoring within two standard
deviations of the mean. If the mean score was
Some of the materials used in this module are taken from disclosed editions of GMAT, and no monetary
value is included in the cost of this module for supply of these materials. These materials are strictly used
for illustration of concepts discussed. Page 73
©Educational Training Services, A Division of Maple Leaf International Consulting, Inc.
Not to be reproduced, copied or distributed. Intended for use only by the registrant.

Example:

A group of GMAT test-takers has the FORMULAS FOR


following scores submitted by the test- COMPUTING Standard
takers:
deviation
550, 590, 620, 650, and 660.
What is the standard deviation for
this group of test-takers?

Value Deviation Squared The standard Deviation could be


from mean Deviation from computed by using any one of the
(X – 614) mean. following formulas:
550 -64 (-64)2 = 4096
590 -24 (-24)2 = 576
620 6 62 = 36
650 36 362 = 1296
660 46 462 = 2116
Mean =
3070
/5 =
614

Sum of the squared deviations (Sum of all the


values in the third column) = 8120

Let us carry out the step 4 described in the THERE ARE DIFFERENT WAYS OF
preceding page, and divide 8120 by the number COMPUTING THE STANDARD
of values, namely, five. DEVIATION OF A GROUP OF ‘N’ VALUES.
ALL THREE FORMULAS PROVIDED
BELOW ARE DIFFERENT WAYS OF
We get: 8120 + 5 = 1624
EXPRESSING THE STANDARD
The standard Deviation for this group of values FORMULA (see formula 3 below). If we
is the square root of 1624, or 40 know the SUM OF THE VALUES OR THE
(approximately). AVERAGE OF THE VALUES, and THE
(We know that 40-squared is 1600 and 1624 is SUM OF THE SQUARES OF THE
pretty close to 1600). VALUES, WE CAN COMPUTE THE
STANDARD DEVIATION. REMEMBER
In a larger population with normally distributed THIS. GMAT WILL TEST YOUR
values, 67% of the values may be distributed one UNDERSTANDING OF THESE
standard deviation away from the mean and FORMULAS
95% of the values may be distributed within 2
standard deviations of the mean. The “scaled
scores” in the GMAT bring the values within
the specified percentage ranges on the bell-
curve, and different editions of the GMAT may
involve different “bell-curving”, if you know
what we mean.

Some of the materials used in this module are taken from disclosed editions of GMAT, and no monetary
value is included in the cost of this module for supply of these materials. These materials are strictly used
for illustration of concepts discussed. Page 74
©Educational Training Services, A Division of Maple Leaf International Consulting, Inc.
Not to be reproduced, copied or distributed. Intended for use only by the registrant.

MEDIAN
Example 3:
The median of a group of values is the middle The point value of a dive is determined by
value of an ordered set of values. taking the median score of all scores allotted by
the judges and multiplying it by the degree of
We must arrange the values in an ascending or a difficulty. If a dive with a degree of difficulty of
descending order and pick the middle value such 5 received the following scores from the 5
that there is an equal number of values on either judges: 4.0, 4.2, 3,8, 3.7, and 4.3, what was the
side of the median value. This is possible only if point value of the dive?
we have an odd number of values. When we
have an even number of values, we must pick The median of the five scores is obtained by
the middle two values such that there is an equal arranging the values in an order:
number of values on either side of the middle 3.7, 3.8, 4.0, 4.2, 4.3
two values, and take the average of the middle The median value is 4.0
values.
The point value is obtained by multiplying the
Example: median value by the degree of difficulty, which
is 5.
What is the median value of the following
GMAT scores submitted by a group of
The point value is 4•5 = 20 (Answer)
applicants to an M.B.A program?
670,590,470,700,690,680,670,600,610
MEDIAN TRIVIA
Let us arrange the values in an order, starting
from the smallest to the largest. (You could do it For a set of consecutive integers
the other way around too).
(including consecutive odd,
470, 590, 600, 610, 670, 670,680,690,700 consecutive even, or any set of
We can clearly see that 670 is the median value values having the same distance
because it occurs smack in the middle of an between values), the MEAN and the
ordered set of values. MEDIAN are the same. For example,
if we consider the set 24, 26, 28, 30,
If we added one more value to the list, 450, then
the list is going to look as shown below:
32, 34, the median of this set is the
average of 28 and 30, and equal to
450, 470, 590, 600, 610, 670, 670,680,690,700 29. This is also the value for the
The median of this set of values is the average of average of the set.
610 and 670, or 640.

Example 2: The sum of any consecutive


The median of a set of six consecutive positive set of values is the average or
odd integers is how much greater than the mean
of the same group of values?
the median times the number
of values in the set. In the
Let us define the values as 2n+1, 2n+3, 2n+5, above example, the sum of
2n+7, 2n+9, and 2n+11.
The Median is the average of 2n+5 and 2n+7 = the set of values listed will be
2n+6. 29 times 6 or 174.
The average of the group of values = (12n+36)/6
= 2n+6
As you can see, the mean and the median of any
consecutive set of integers will be equal and the
difference will be ZERO.
Some of the materials used in this module are taken from disclosed editions of GMAT, and no monetary
value is included in the cost of this module for supply of these materials. These materials are strictly used
for illustration of concepts discussed. Page 75
©Educational Training Services, A Division of Maple Leaf International Consulting, Inc.
Not to be reproduced, copied or distributed. Intended for use only by the registrant.

The square root sign is a radical, and you should


Mode remember the following rules:
The mode is the most frequently occurring value Rule 1:
or values in a group. It is possible for a group of
values to have multiple modes. For example, a √x + √y is not equal to √ x + y
school can receive 100 applications each with
GMAT scores of 670 and 690. In this scenario, Rule 2:
the set of values has two modes: 670 and 690.
√x • √y = √xy
Example:
The average GMAT score of 7,000
Rule 3:
applicants to the Stanford M.B.A program
was 610. If 600 applicants submitted the x
GMAT score of 650, and if 340 applicants √x divided by √y = √ /y
submitted the GMAT score of 680, what is
the average GMAT score of the remaining Rule 4:
6,060 applicants?
√x2 + y2 is not equal to x + y
(We notice that the modal value could be 650 unless there is
another group exceeding 600 and submitting the same GMAT
score). IF we can find a way to express the values
within the radical sign in terms of factors that
The total score of 600 applicants each submitting a score of are perfect squares, we will be able to take the
650 = 390,000 numbers out of the radical sign.
The total score of 340 applicants each submitting a score of
680 = 231,200
Total score submitted by 940 applicants = Example:
390,000 + 231,200 = 621,200
Total score submitted by all 7,000 applicants
= 7,000 • 610 = 4,270,000 √32 = ?
Total score submitted by 7000-940 = 6060 applicants
= 4,270,000 – 621,200 = 3,648,800 We notice that 32 is not a perfect square and we
can tell that the answer will be more than 5 and
The average score of 6,060 applicants
= (3,648,800) + 6060 = 600 (approx.)
less than 6. In the GMAT, you will be asked to
recognize an equivalent value.

Can we find a way to express 32 in terms of two


factors, at least one of which is a perfect square?

We notice that we can write 32 as 2•16, and that


16 is a perfect square. (42)

√32 = √2.16 = √2.42 = 4√2

4√2 is a called a mixed radical, and the answer


choice will be usually in this form.
You should be looking for a choice that reads
4√2.

Radicals

Some of the materials used in this module are taken from disclosed editions of GMAT, and no monetary
value is included in the cost of this module for supply of these materials. These materials are strictly used
for illustration of concepts discussed. Page 76
©Educational Training Services, A Division of Maple Leaf International Consulting, Inc.
Not to be reproduced, copied or distributed. Intended for use only by the registrant.

Example 2:

√243 - √108 = ?
Perfect Square
We notice that neither 243 nor 108 is a perfect
square. Let us do the next best thing: Express
243 and 108 in terms of two factors, at least one A perfect square is an integer that can be
each of which is a perfect square. expressed as the square of an integer. For
example, 16 is a perfect square because it can be
expressed as 42.
√243 = √3•81 = √3•92 = 9√3
√108 = √3.36 = √3•62 = 6√3
A perfect cube is an integer that can be
√243 - √108 = 9√3 - 6√3 = 3√3 (Answer)
expressed as the cube of an integer. For
Example 3:
example, 27 is a perfect cube because it can be
expressed as 33.
√632 + 362 =?
Example:
We notice that 632 and 362 are not factors within
the radical sign, and we cannot take them out as
Data Sufficiency:
63 and 36.
If n is an integer, is n a perfect square?
When you come across problems of this type,
look for a standard form, as we will see with
1. If p is an integral factor of n, so is p2.
algebra later on, or look for the greatest
2. √n is an integer.
common factor for the values provided.
Statement 1 tells us that p2 is a factor of n. But
The greatest common factor of 63 and 36 is 9.
we are clue-less about the other factors of n. For
example, n could be 12, in which case we can
632 = (7•9)2 = 72•92
write n as n = 3•22. (Here p could be 2). We
362 = (4•9)2 = 42•92
cannot write n as a perfect square because 3 is
not a perfect square.
√632 + 362 = √92•72 + 92•42 = √92•(72+ 42)
But then, n could also be 36, in which case we
= 9√49 + 16
can write n = 32•22, where p could be 2 or 3 and
= 9√65 n could be expressed as a perfect square.
ϕ 9•8 ϕ 72 (approx.) Statement 1 is not good for a unique solution.

Our choices are limited to choice 2, choice 3 or


Example 4: choice 5.
√772 + 1102 = ? Statement 2 tells us that square root of n is an
integer. This can mean only one thing: that n is
We notice that 772 and 1102 are not factors the square of an integer, or a perfect square.
within the radical sign. We cannot take 77 and (If √n = integer, then n = (integer)2).
110 out of the radical sign. Let us do the next Statement 2 tells us in unambiguous terms that
best thing: let us express 77 and 110 in terms of n is a perfect square.
their greatest common factor of 11.
We conclude that statement 1 alone is not
772 = (7•11)2 = 72•112 sufficient but statement 2 alone is sufficient to
1102 = (10•11)2 = 102•112 answer the question.
√772 + 1102 = √ 112•(72 + 102) = 11•√149
ϕ 11• 12 ϕ 132 (approx.) We must pick the second choice.

We will revisit the radicals after we have


become familiar with some algebraic formulas.
Some of the materials used in this module are taken from disclosed editions of GMAT, and no monetary
value is included in the cost of this module for supply of these materials. These materials are strictly used
for illustration of concepts discussed. Page 77
©Educational Training Services, A Division of Maple Leaf International Consulting, Inc.
Not to be reproduced, copied or distributed. Intended for use only by the registrant.

3x•(x-7) – 2x+14 = (x-7)•(3x-2)


ALGEBRA
As you can see, we have managed to write the
Algebra is what you deal with when you set up numerator expression in terms of two factors.
equations for verbose information in word You can also see that one of the factors on the
problems. We have seen that an integer can be numerator will cancel out the denominator
represented in terms of its factors. For example, factor, leaving a simpler expression.
16 can be represented in terms of 1 and 16 or 2
and 8 and so on. 3x•(x-7) – 2x+14 = (x-7)•(3x-2) = (x-7)
(3x – 2) (3x-2)
We can also express an algebraic expression in
terms of its algebraic factors. You should be looking for a choice that says
(x-7).
Let us say that we have two algebraic factors:
(2x+1)•(3x-7)
We are allowed to break up a binomial in terms
of two monomial factors and write: GMAT UNPLUGGED
(2x+1)•(3x-7) = 2x•(3x-7) + 1•(3x-7)
or, We have seen during the programs that some
(2x+1)•(3x-7) = 6x2 – 14x + 3x – 7 participants, who have been through the Kaplan
(2x+1)•(3x-7) = 6x2- 11x – 7 process, randomly choose numbers for x and try
to see whether any of the choices will give the
In the GMAT, you will be required to proceed in same result as the original expression when the
reverse gear from a given quadratic expression value is plugged in. You cannot do that. The
or equation and rewrite it in terms of its factors. only time when you can choose to assign
numbers to variables specified in the problem is
As a rule, try to look for a simpler and more when a relationship is specified between the
manageable expression every time you are variables, relationship such as x = 2y = 3z.
dealing with an algebraic expression. All You can set z = 2 and that value will determine
GMAT algebraic expressions will be amenable the values for y and x. If z = 2, then y is 3 and x
to simplification and will lead to a simpler, and is 6. The proportional values for x, y and z are
more easy-to-deal-with form. consistent with the original definition, and will
be an acceptable methodology. IF the problem
Example 1: tells that if X is divided by 5, we get a remainder
of 4, we can choose to work with a number value
3x•(x-7) – 2x+14 = ? of 9 for X. If the problem told us that a + b = c,
(3x – 2) then we can choose to assign 1 for a, and 2 for b
so that the value for c will be 3. If the problem
We notice that the first set of items is already defines a ratio, such as A/B= 2/3, and asks to
factored in terms of 3x and (x-7), and, as a rule, verify another ratio involving the same two
We will not mess with factored expressions and variables such as (A+B)/(A-B), we can assign 2
will leave them be. for A and 3 for B and verify the new ratio. But if
the problem specifies that A/B = 2/3, and asks
Let us take a look at –2x + 14. What is common what is A+B?, then it is all-bets-are-off situation
to both –2x and 14? We see a common factor of we have.
2. We can write –2x + 14 as –2•(x-7) If the problem merely stated that x, y and z are
three values and does not specify a relationship,
Let us rewrite the numerator expression as: you should not be tempted to use plug-in
3x•(x-7) – 2x+14 = 3x•(x-7) –2•(x-7) process because you will be wasting a lot of time
in trying to reach an unreachable conclusion.
We can see that (x-7) is factor that can be taken
out so that the remaining two items can be
grouped together.
Some of the materials used in this module are taken from disclosed editions of GMAT, and no monetary
value is included in the cost of this module for supply of these materials. These materials are strictly used
for illustration of concepts discussed. Page 78
©Educational Training Services, A Division of Maple Leaf International Consulting, Inc.
Not to be reproduced, copied or distributed. Intended for use only by the registrant.

Example 2:

Let us take an example of another “clumsy” Now try the following on your own, and see if you
can simplify the each of the expressions to a more
algebraic expression:
manageable form: (Explained at the back of this
file)
4x•(x-2) - 2x + 4 = ?
1. 6x3+3x2+6x+3 =
(4x-2) (x2+1)

We notice that -2x+4 in the numerator can be


written -2•(x-2), and we recognize that (x-2) is
sitting as a factor in the first part of the numerator
expression. 2. 5x3+7x2-25x-35 =
(x2-5)

We can write the numerator as:


4x•(x-2)-2•(x-2) = (x-2)•(4x-2)
What did we do here? We noticed that (x-2) was a
common factor, and took it out. We were left with
3. 7x2(6x+5)-18x-15 =
4x from the first part and -2 from the second part, (7x2-3)
and we grouped them together to get (x-2)(4x-2)

Let us rewrite the expression in terms of these


factors as:
4. 3x(x2+2)-7x2-14 =
(3x-7)
(x-2)•(4x-2) = (x-2)
(4x-2)

This is the summary of the drill we performed:


1. We recognized common factors in parts of an
expression, and re-wrote a part of the expression in
terms of its factors.
Example 3:
2. We then took out the common factor to both parts
of the expression, and grouped the remaining values Data Sufficiency:
to write an expression in terms of two factors. What is (a + b)?
3. We found to our pleasant surprise that one of the
1. ad + bd + ac + bc = 15
factors in the numerator expression was the same as
2. c+d=5
the denominator expression. These values canceled
each other out, leaving behind a simpler, more
manageable expression.
Some of the materials used in this module are taken from disclosed editions of GMAT, and no monetary
value is included in the cost of this module for supply of these materials. These materials are strictly used
for illustration of concepts discussed. Page 79
©Educational Training Services, A Division of Maple Leaf International Consulting, Inc.
Not to be reproduced, copied or distributed. Intended for use only by the registrant.

Statement 1 is a “clumsy” algebraic expression, where x is not equal to 7.


and our first “instinctual reaction” must be to try Statement 1 is a no-brainer. We can solve for x
and simplify it. using the equation specified.

ad + bd + ac + bc = d•(a+b) + c•(a + b) = 15 Our choices narrow to choice1 or choice 4 in the


= (a + b)•(c + d) = 15 test.

We notice that we have been able to express the Statement 2 needs “manipulation”. We notice
given expression in terms of two factors but we that (–14x – 7) has 7 as a common factor and
cannot find the value for a + b unless we know can be written as –7•(2x+1)
what c + d is.
The numerator expression becomes
Our conclusion is that statement 1 alone is not X•(2x+1) –7•(2x+1) = (x-7)•(2x+1)
sufficient to answer the question. But you must
go the distance we have indicated before you can Because there is a “factor” of (x – 7) in the
come to this determination. denominator, we can see that the expression in
statement 2 will simplify to:
Statement 2 alone is also not sufficient to
answer the question because statement 2 tells us x•(2x+1) – 14x – 7 = (x – 7)• (2x + 1) = 0
nothing about a + b. (x-7) (x – 7)

We must now combine the two statements and WE get: 2x + 1 = 0


see whether the combined information will
produce a winner. We notice that this equation is the same as the
one provided in statement 1, and we can
The combined information does. If we know the determine the value for x using this statement
value of c + d, then we can determine the value alone also.
of a + b.
We conclude that we can answer the question
We must pick choice3 in the test. using either statement alone independently, and
we must pick choice 4.
Remember: you must “play around” with the
algebraic expressions before you can begin to
make sense of the information. Usually, such
“manipulation” will give you better insights into
the nature of the problem.

Example 4:

Data Sufficiency:

What is X?

1. 2x + 1 = 0
2. x•(2x+1) – 14x – 7 = 0
(x-7)

Some of the materials used in this module are taken from disclosed editions of GMAT, and no monetary
value is included in the cost of this module for supply of these materials. These materials are strictly used
for illustration of concepts discussed. Page 80
©Educational Training Services, A Division of Maple Leaf International Consulting, Inc.
Not to be reproduced, copied or distributed. Intended for use only by the registrant.

Roots of an equation We said earlier in this


discussion that in the GMAT,
The root of an algebraic the quadratic equation you will
equation is simply the value for encounter would be amenable
the variable in the equation to a form in which the
satisfying the equation. coefficient of x2 terms is a 1. If
this is the case, then you need
In the equation x + 2 = 0, the not apply the formula to
value –2 for x will satisfy the compute the two roots of the
equation and will be called the equation. We will take
root of the equation. In the recourse to “factoring” so that
equation 2x + 3 =0, the value – we can find the roots of the
3
/2 for x will satisfy the equation given equation.
and will be referred to as the
root of the equation. We have to re-write the
quadratic equation
In the GMAT, you will x2 + bx + c = 0
encounter many situations in as (x + r1)•(x + r2) = 0
which you will be required to where the values for r1 and r2
find the roots of a quadratic are such that
equation of the form c = r1•r2 and b = r1+ r2
1•x2 + bx + c = 0 After we have written the given
Notice that the coefficient of equation in terms of its factors,
the x2 terms is 1 and b and c we can determine the roots of
could be any values, usually the equation as
integers. x1 = -r1 or x2 = -r2

The standard form for a Let us take a look at an


quadratic equation is example and understand this
ax2 + bx + c = 0 process.
The quadratic equation will
have two roots, and the
formula for computing the
roots is

x12 = -b ± sqrt(b2 – 4ac)


2a Example:
Some of the materials used in this module are taken from disclosed editions of GMAT, and no monetary
value is included in the cost of this module for supply of these materials. These materials are strictly used
for illustration of concepts discussed. Page 81
©Educational Training Services, A Division of Maple Leaf International Consulting, Inc.
Not to be reproduced, copied or distributed. Intended for use only by the registrant.

hone in on –9 and +5. The product is –45 and


the sum is –4.
What are the roots of the
equation Step 3: Let us rewrite the equation as
2x2 + 6x – 56 = 0? (x – 9) • (x + 5) = 0
Step 4: The two roots are +9 and –5.
(because one of the two factors is equal to 0.
We notice that the coefficient of x2 This means that either x – 9 = 0 and x = 9 or
terms is not 1, and we must look for a x + 5 = 0 and x = -5).
way to bring it down to a 1.
EXAMPLE 3:
We notice that 2 is a common factor of
all the coefficients in the equation, and “If 3 is one root of the equation:
2
we can divide both sides of the X + 7X – P = 25, what is the other root?”
equation by 2 to eliminate the 2 in front
of the x2 term. We know that if 3 is a root, then X – 3 is a
binomial factor of the given expression
2
The equation becomes: X +7X-(P+25). We notice that the middle
value is +7 and that is the sum of the two
X2 + 3x – 28 = 0 factors of the stand-alone number. We
know that –3 is one factor of the stand
Now, let us factor the “c” value, the
alone number and the other must be +10
stand-alone number in the equation, in so that +10 – 3 = +7.
terms of two integers that will multiply to
The other binomial factor will be (X + 10)
give –28 and, at the same time, will
or the other root will be –10.
yield +3 when added together. You can also determine the other root by first
finding the value of P by writing 3 for X. You
We notice that we can pick the factors will get P = 5. If you move the number 25 to the
+7 and left and wrote the equation in standard form for
–4 so that the product is –28 and the quadratic equation, you will get:
sum of these two factors is +3. X2 + 7X – 30 = 0
We need to pick two factors of –30 that will add
We will now rewrite the given equation to give us +7. –3 and +10 will. We must write:
as: (X-3)(X+10) = 0 or X = 3 or –10.

(x + 7) • (x – 4) = 0 or x = -7 or +4.
-7 and +4 are the two roots of the given
equation.

Example 2:

Find the roots of the quadratic equation


3x2 – 12x – 135 = 0.

Step 1: Let us find a way to make the coefficient


of x2 term equal to 1. We notice that all
coefficients have 3 as a factor. Let us divide the
equation by 3 to get:
x2 – 4x – 45 = 0

step 2: Let us pick the two factors of –45 such


that those two values will add to give –4. We
Some of the materials used in this module are taken from disclosed editions of GMAT, and no monetary
value is included in the cost of this module for supply of these materials. These materials are strictly used
for illustration of concepts discussed. Page 82
©Educational Training Services, A Division of Maple Leaf International Consulting, Inc.
Not to be reproduced, copied or distributed. Intended for use only by the registrant.

give 77. We must conclude that X must be 7 and the


length must be X + 4 or 11 units.
Finding the positive root of
the variable (when we know
that the variable in the Algebraic Formulas to
equation cannot have a remember:
negative value) You will be required to memorize and remember
the following algebraic formulas, and recognize
You will have lots of opportunities for creating
expressions that are in the same standard forms
quadratic “situations” when you deal with some
as the expressions in the formulas. You may
word problems such as the distance problems
want to write down these formulas on your
and when you deal with geometry.
scratch-paper during the tutorial phase of the
test. (The good news is, the tutorial phase is not
Let us say that the problem specifies that in a
timed, and you should use this opportunity to
right triangle, one side is 3 more than the other,
“boot up” or load all the information you have
and the area is 27 square units.
learned into your memory or brain cells so that
you will be in a state of readiness for the test.)
We will set the sides as x and (x + 3) and use
the area formula: (a + b)2 = a2 + 2ab + b2
Area of a triangle = ½ • x • (x + 3) = 27 (a – b)2 = a2 – 2ab + b2
(a + b)•(a – b) = a2 – b2
When we multiply both sides by 2 so that we can or 2
a – b2 = (a + b)•(a – b)
get rid of the ½ on the left, we get:
x • ( x + 3) = 54
Example:
We know that the value for the side of a triangle What is the value of Sqrt(632 – 372)?
cannot be negative.
We notice that the value inside the radical sign
If we are aware of this “subtlety” here, we will is in the standard form a2 – b2.
not bother to turn x•(x + 3) = 54 into a quadratic
mess. We will simply look for the two positive Sqrt(632 – 372) = Sqrt[(63+37)•(63-37)]
factors of 54 such that they are 3 apart and will = Sqrt (100•26)
multiply to give us 54. We must conclude that x = Sqrt(102•26)
is equal to 6 and x + 3, by default, must be equal We notice that we have managed to express the
to 9. Time is of the essence in the GMAT, and value inside the radical in terms of two factors, a
if you can eliminate some needless steps, you step crucial to our ability to take numbers out of
will ensure that you are moving along briskly the radical sign.
through the test.
Example:
We also notice that 102 can be taken out of the
radical as 10, leaving the value of 26 inside the
The area of a rectangle is the product of its radical.
length and width. If the length of a rectangle
is 4 more than its width, and if the area is 77 Sqrt(632 – 372) = 10•Sqrt(26)
square units, what is the value for the length? We notice that the square root of 26 is
approximately 5, and the approximate value for
IF the width is X units, then the length is X + 4
the expression is:
units.
Sqrt(632 – 372) = 10•Sqrt(26) ϕ 10•5 ϕ 50
Area = X • (X + 4) = 77
Recognition is a critical requirement for success in the GMAT, and by
We also know that the value for X cannot be negative. recognizing the standard form, we have been able to get to the answer a
Therefore, we will not bother to turn this into a "quadratic lot sooner than if we had tried to expand each value and do multiple
mess”. Instead, we will simply look for the two positive arithmetic operations. Remember: Each additional operation entails the
factors of 77 such that they are 4 apart and will multiply to potential for careless mistakes.

Some of the materials used in this module are taken from disclosed editions of GMAT, and no monetary
value is included in the cost of this module for supply of these materials. These materials are strictly used
for illustration of concepts discussed. Page 83
©Educational Training Services, A Division of Maple Leaf International Consulting, Inc.
Not to be reproduced, copied or distributed. Intended for use only by the registrant.

Practice Exercises: Exercise 3:

Exercise1: What is the value of (√2 + √8)2 ?

“If (a + b)2 = a2 + b2, which of the following We recognize the form (a + b)2, and we know
must be true?” how to deal with this form.

I a=0 (√2 + √8)2 = (√2)2 + (√8)2 + 2.√2.√8


II b=0 = 2 + 8 + 2.√16
III ab = 0 = 10 + 2•4 = 18
IV (a – b)2 = a2 + b2 (√2 + √8)2 = 18

Let us expand (a + b)2: Exercise 4:

a2 + 2ab + b2 = a2 + b2 Is (a + b)2 > a2 + b2 ?


Or, 2ab = 0 or, ab = 0
1. a>0
We can see that the equation simplifies to ab = 2. b<0
0. But, do we know which value is 0? We do
not. We do know that one of the values is a 0, Let us expand the information in the stem. We
because the product is a 0. get:
Is a2 + 2ab + b2 > a2 + b2 ?
We can eliminate conditions I and II as not Or, is 2ab > 0 ?
necessarily true. Condition III is true because ab Or, is ab > 0?
must be equal to 0.
Condition IV needs to be checked after playing The question is simply one of whether the
around with the expression (a – b)2. product of a and b is positive.
We know that (a – b)2 = a2 – 2ab + b2 When will the product of two values be positive?
If ab = 0, then the middle term –2ab must be When both are positive or when both are
equal to 0, and (a – b)2 = a2 + b2. negative. Therefore, the information we are
We can clearly see that condition IV must be going to be looking for will have to deal with
valid as well. signs of a and b.

So, we must look for a choice that says “III and Statement 1 tells us that “a” is positive. That
IV only”. does not help us at all, because we are in the
dark about the value for “b”.
Exercise 2:
Statement 2 tells us that “b” is negative. Once
1 2
If x + /x = 3, what is x + /x ? 1 2 again, statement 2 fails to provide the complete
answer alone.
Let us square both sides of (x + 1/x) = 3 and see
what we get. We must proceed to combine the two statements.
When we combine the two statements, we notice
(x + 1/x)2 = 32 = 9 that a and b do not have the same sign and the
x2 + 1/x2 + 2•x•1/x = 9 product is not going to be positive. We can
x2 + 1/x2 + 2 = 9 answer the question definitively, using the
Let us move the number 2 to the right. combined information.
We get:
Remember: We should be able to answer the question one way
x2 + 1/x2 = 9 – 2 = 7. (Answer) or the other, and not necessarily in the affirmative.

We must pick choice 3.

Some of the materials used in this module are taken from disclosed editions of GMAT, and no monetary
value is included in the cost of this module for supply of these materials. These materials are strictly used
for illustration of concepts discussed. Page 84
©Educational Training Services, A Division of Maple Leaf International Consulting, Inc.
Not to be reproduced, copied or distributed. Intended for use only by the registrant.

Exercise 5: whether you get the same results. You will. In


the GMAT, know that we can go from X/Y to
x 3
“IF /y = /2, what is (X+Y)/Y by adding a 1 to the fraction. But for
1. x + y ? sake of time, work with numbers as suggested.
y Remember: We can work with real-life numbers
as long as a relationship between variables is
2. x + y ? specified, as we find is true in this problem.
x–y

Let us deal with the first part of the question. Exercise 6:

We have x/y = 3/2 (x2 + xy) / xy is equivalent to


Let us add a 1 to both sides.
what?
We get: x/y + 1 = 3/2 + 1
Let us write y/y for 1 on the left and 2/2 for 1 on
Let us divide each term on the numerator by the
the right so that the fractions will have the same
denominator. We get:
denominators.
(x2 + xy) / xy = (x2 / xy) + (xy / xy)
x (x2 + xy) / xy = (x/y) + 1 (Answer)
/y +y/y = 3/2 + 2/2
(x + y)
We get: /y = 5/2. ……………. (1)
And that is the answer.
Notice how we recognized that we can go from Algebra in word problems
x/y to (x +y)/y by simply adding a 1 to the Algebra is going to come back to haunt you
expression. Recognition is key to unsolving the every time you are looking at information
mysteries of the GMAT. specified in terms of letters and alphabets.
GMAT would like you to be as comfortable
Now, let us deal with the second part of the with algebraic notations as with real life
question. numbers.

We notice that the expression (x +y) / (x –y) is


obtained by taking the ratio of (x +y)/y to
Example 1:
(x –y)/y.
“If Pamela was x years of age
We have found the value for (x +y) / y.
Now let us find the value for (x –y) / y. y years ago, how old will
We recognize the form and conclude that we can Pamela be z years from now?”
go from x / y to (x –y) / y by simply subtracting
a 1 from x/y.
Pamela’s age today is (x + y)
We have: x/y = 3/2 years of age. z years from
Let us subtract a 1 from both sides.
We get: x/y - 1 = 3/2 – 1 now, Pamela will be (x + y + z)
We get: (x – y)/y = ½ ……………. (2) years of age.
Now, let us take the ratio of the values in
equations (1) and (2) to get:

(x +y) / (x –y) = (5/2) / (1/2) = 5

We could have dealt with the same problem by


assigning a value of 3 for X and 2 for Y. You
will find that your life would have been a lot
easier. Try assigning 3 for X and 2 for Y and see
Some of the materials used in this module are taken from disclosed editions of GMAT, and no monetary
value is included in the cost of this module for supply of these materials. These materials are strictly used
for illustration of concepts discussed. Page 85
©Educational Training Services, A Division of Maple Leaf International Consulting, Inc.
Not to be reproduced, copied or distributed. Intended for use only by the registrant.

Example 2: EXAMPLE 4:

“Which of the following has at least one root


“Manchester United won exactly 2 more games in common with the equation
than it lost in the league games. IF the team 2
X – 5X + 4 = 0?”
played a total of N games, and if there were no
ties, find an expression for the games won in 2
terms of N.” • X +1=0
3 2
• X + 2X – 7X – 14 = 0
3
• X +1=0
Let W be the number of games won, and Let L 5
• X –1=0
be the number of games lost.
• X=-4
Number of games won + lost = N
i.e. W + L = N (1) We can determine that the given equation
And W = L + 2 or L = W – 2. (2) has the following roots: 1 and 4. We can do
one of two things: We can try to replace X
Let us replace L with W –2 in (1)so that we can with 1 and 4 in the choice equations and see
write the equation in terms of W only. which one will be satisfied by at least one of
(1) becomes: these two values. The other option is to
W+W–2=N quickly determine the roots of the choice
Or, 2W = N + 2
equations because most of them are pretty
Or, W = (N + 2) / 2
easy to deal with.
And that is the expression, in terms of N,
that represents the number of games won. Choice 1 is not worth messing with, because
2
X cannot be equal to –1.
Choice 2 can be expressed in terms of two
EXAMPLE 3: binomial fators as:
2
X (X + 2) – 7 ( X + 2) = 0
DATA SUFFICIENCY: 2
Or, (X – 7) ( X + 2) = 0
Or, X = 7 or X = ± sqrt(7)
“What is (A + B) (C + D)?” We notice that none of these three roots has
anything in common with 1 or 4
1. A (C + D) = 10 Choice 3 gives us a root of –1 for X. We are
2. B (C + D) = 15. looking for +1 or +4.
Choice 4 is right on the money. If X5 = 1,
We notice that Statement 1 uses one of the then X must be 1. We must choose this
monomial terms of the first binomial option.
(A + B) as a factor with the second binomial Choice 5 is not good because –4 is not the
taken as a whole. Obviously, we cannot use same as +4.
statement 1 alone to answer the question.
We must kill choices A and D. We have just
3 choices remaining: B, C, or E. Remember:
1) Learn to recognize standard forms.
Statement 2 is also another attempt at using 2) Always “play around with” or
one of the monomial terms of the first “manipulate” algebraic expressions,
binomial (A + B) as a factor with the second
binomial. Again, we cannot answer the
and look for a simplified form. When
question using statement 2 alone. Let us kill you deal with data sufficiency
choice B. involving algebraic information, do
some minimal implementation and
If we combine the two statements, we notice get to a simplified form, which is
that amenable to making a decision.
(A + B) (C + D) = A (C + D) + B (C + D) 3) Learn to redefine the question when
We can answer the question by using the asked in data sufficiency with
combined information. We must go with
choice C.
algebraic information.

Some of the materials used in this module are taken from disclosed editions of GMAT, and no monetary
value is included in the cost of this module for supply of these materials. These materials are strictly used
for illustration of concepts discussed. Page 86
©Educational Training Services, A Division of Maple Leaf International Consulting, Inc.
Not to be reproduced, copied or distributed. Intended for use only by the registrant.

=SQRT[(100)(26)]
ALGEBRA TESTED AS
RADICALS =10•SQRT(26)
You may be expected to apply some
algebraic formulas when you deal with = 50 (APPROXIMATELY)
some problems involving radicals.
When you deal with radicals, check to
Example: see whether you can apply any standard
algebraic formula and deal with the
problem. If you cannot, then you may
(√7 – 5) (√ 7 + 5) = ? have to deal with factoring of values in
terms of integers, at least one of which
Can we associate the above binomial factors
is a perfect square and can be isolated
with any standard algebraic rule we know of?
as a common factor.
Sure. The expression is in the form:
(a + b) (a – b), and we know that For example, take a look at the following
(a + b) (a – b) = a2 – b2 problem:

In our problem, a is √ 7 and b is 5 SQRT[812 + 542] = ?


What is a2 – b2 ? Our first instinct will be to look for a standard
algebraic formula with which we can associate
this expression. Unfortunately, we cannot. If we
were to use the form (a + b)2, then we need
(√ 7 – 5) (√ 7 + 5) another term 2ab, which is missing. Let us do
the next best thing, and try to find the greatest
= (√ 7)2 – 52 common factor for 81 and 54. 81 can be written
as 3 times 27 and 54 can be written as 2 times
27. We have found the greatest common factor
= 7 – 25 = -18 between 81 and 54 – the value of 27. Let us
rewrite the given expression in terms of this
common factor and see where we go from there.

SQRT[812 + 542 ] = SQRT[32•272 + 22•272]


EXAMPLE 2: = SQRT[272•(32 + 22)
We can take 272 out of the square root sign as
SQRT(632 – 372) = ? 27. We get:
SQRT[812 + 542 ] =27• SQRT(32 + 22)
The expression within the = 27• SQRT(13)
radical sign is of the form: And that is the answer you should be looking
(a + b) (a – b). for.

Here a is 63 and b is 37. Remember: Try to look for any standard


algebraic formula you can use in the
SQRT(632 – 372) context of a problem using radicals. If
= SQRT[(63 + 37)(63-37)] you do not associate the expression with
any standard algebraic form, do the next
best thing: Look for the greatest
common factor between the values
specified under the radical.

Some of the materials used in this module are taken from disclosed editions of GMAT, and no monetary
value is included in the cost of this module for supply of these materials. These materials are strictly used
for illustration of concepts discussed. Page 87
©Educational Training Services, A Division of Maple Leaf International Consulting, Inc.
Not to be reproduced, copied or distributed. Intended for use only by the registrant.

MISCELLANEOUS EXAMPLE:
PROBLEMS USING “IF X is a multiple of Y and Y is
ALGEBRA a multiple of Z, which of the
following is not necessarily an
integer?”
EXAMPLE:
(A) (X + Y) / Z (B) (X – Y) / Z (C) X•Y/Z
“If a + b = c, which of the following is (D) (X + Z) / Y (E) X•Y•Z
equal to 1?”
If X is a multiple of Y, we can write:
I (c - b)/a II (c – a)/b III (a + b)/c X = K•Y where K is an integer.
Similarly, if Y is a multiple of Z, we can write:
This is a problem that tests your ability to Y = P•Z where P is another integer.
manipulate algebraic equations. Notice that we did not choose to use the same
integer multiple value of K for both equations
If a + b = c, then a = c – b. because X and Y could be different multiples of
If we divide both sides by a, we get Y and Z respectively.
1 = (c – b) /a You should also be able to combine the two
statements and express X in terms of Z. Can
We can see that condition I stacks up. you?

Let us play with the equation some more. If X = K•Y and Y = P•Z, then let us replace Y
IF a + b = c, then b = c – a with P•Z in the first equation. We get:
Let us divide both sides by b, and get:
1 = (c –a)/b X = K•P•Z and Y = P•Z
We can see that both X and Y are multiples of a
We can see that condition II also stacks up. common variable Z.

Let us do one more thing with the equation. WE can now continue to deal with the problem
algebraically or choose to work with numbers
If a + b = c, then let us divide both sides by c. because we have a relationship between the
We get: variables specified. IF we choose to work
(a + b) / c = c/c = 1 algebraically, we will notice that both X and Y
are multiples of Z.
We know that condition III is valid too.
We can see that choices A, B, and C use X and
We must pick a choice that says “I, II, and III”. Y are values on the numerator and Z as a
denominator. Because both X and Y are
IF you were not up to speed with algebraic multiples of Z, the Z at the bottom will cancel
manipulation, you could have answered this out the Z on the numerator.
problem by thinking up real-life numbers For example, let us examine choice A.
because there is a relationship specified between (X + Y) / Z = (K•P•Z + P•Z) / Z = K•P + P
the variables. Because K and P are both integers, K•P + P
MUST Be an integer. We can similarly confirm
IF we set a = 1 and b = 2, then c must be 3. that choices B, C and E will yield integer values
always. What about choice D? We notice that we
Now test the conditions I, II, and III by using have a Y at the bottom of the expression, and
these values and see whether the expressions that spells trouble.
specified give a value of 1. They will.

Some of the materials used in this module are taken from disclosed editions of GMAT, and no monetary
value is included in the cost of this module for supply of these materials. These materials are strictly used
for illustration of concepts discussed. Page 88
©Educational Training Services, A Division of Maple Leaf International Consulting, Inc.
Not to be reproduced, copied or distributed. Intended for use only by the registrant.

(X + Z) / Y = (K•P•Z + Z) / P•Z know how to assign values in keeping with the


= (K•P + 1) / P way things are specified. That does call for some
= K + 1/P understanding of the concept of multiples.

We notice that while K is an integer, 1/P does


not have to be an integer unless P happened to EXAMPLE 3:
be 1.
Concentration in parts per million of a chemical
Therefore, choice D does NOT HAVE TO BE at any depth X feet of a solution is given by the
AN INTEGER, because P could be any integer formula:
and not necessarily a 1.
Concentration = 4 / SQRT(3X – 2)
We must pick choice D. Notice how we were At what depth in feet will the concentration of
required to use the concepts of factors, and of the chemical be equal to 3 parts per million?
algebraic factoring in this problem. When we
dealt with the expression
(X + Z) / Y = (K•P•Z + Z) / P•Z The question is, What should be the value
We tried to separate the common factor of Z and of X, if the concentration is 3?
see where we went with the exercise.
Our set up will read:
(X + Z) / Y = (K•P•Z + Z) / P•Z 3 = 4 / SQRT (3X – 2)
= Z•(K•P + 1) / P•Z Let us cross multiply and turn this into an
Z will cancel each other out on top and bottom equation.
leaving (K•P + 1) / P We get: 3• SQRT(3X – 2) = 4
Let us square both sides to get rid of the square
Could you have done this problem by thinking root sign on the left. We get:
up real-life numbers consistent with the
relationships specified. You could have. But 9•(3X – 2) = 16
before we could begin to use real-life numbers, 27X – 18 = 16
we should have reached the following stage: 27X = 34 or X = 34/27 feet = 1 7/24 feet

X = K•P•Z and Y = P•Z That is the answer we should be looking for.

We can see that both X and Y are multiples of EXAMPLE 4:


Z. Therefore, Z will be have to be the least
value. IF we let Z be equal to 2, then Y could be The equation R = - 0.028t + 20.8 can be
4 and X could be 12. Notice that values of 2 for used to predict the world record in the
z, 4 for Y and 12 for X are consistent with the 200 meters dash, in which R stands for
relationship specified between the variables. the record in seconds and t stands for
the number of years since 1920. In what
Now use these real-life numbers to test the year will the record be 18.0 seconds?
choice expressions and see which one will NOT
necessarily give an integer value. You will also If R = 18, then what is 1920+ t ?
see that we could have chosen 1 for all X, Y, Let us replace R with 18 and write the equation
and Z because we will still be working within as:
the parameters defined. IF X, Y, and Z were 18 = - 0.028t + 20.8
each equal to 1, then all choices will give rise to Let us isolate the numbers on one side and the
an integer value. But the question is: “not variable on the other, and solve for t.
necessarily an integer”. X , Y, and Z do not -0.28t = -2.8
have to be necessarily each equal to 1. t = 2.8/0.028 = 100
Since t represents the number of years since
Remember, you can work with real-life numbers 1920, we conclude that the record of 18 seconds
if a relationship is specified but then you should will be achieved in the year 1920+100=2020.
Some of the materials used in this module are taken from disclosed editions of GMAT, and no monetary
value is included in the cost of this module for supply of these materials. These materials are strictly used
for illustration of concepts discussed. Page 89
©Educational Training Services, A Division of Maple Leaf International Consulting, Inc.
Not to be reproduced, copied or distributed. Intended for use only by the registrant.

Exponents
Exponent is a value that denotes how many times the
base repeats itself as a factor. Example 1:
2
For example, 2 tells us that 2 occurs as a factor twice. If
If 21•22•23•24•••2k = 255, what is k?
n
we have an exponential value of x , we know that x occurs
n times as a factor.

When we take the square root of an exponent, we can We notice that we are dealing with
take out of the radical one factor for every two we see
16 8
multiplication of exponents with the same base.
inside. For example, the square root of 2 will be 2 We apply the rule No. 1, and get:
because the original exponential value tells us that 2
occurs as a factor 16 times inside the radical, and we can
take one 2 out of the radical for every two we see inside. 21+2+3+4+5+•••••+k = 255
We now apply the rule number 5, and get:
What happens when we multiply two exponential values,
each with the same base?
1 + 2 + 3 + •••••• + k = 55
2
Let us say that we would like to multiply x by x
3.
The question is: how far do we have to go when
2 3 5
x •x = x•x•x•x•x = x we add consecutive integers so that the sum will
We notice that when we multiply two exponential values
with the same base, we keep the same base and simply
be 55? We need to go up to 10 so that the sum
add the exponent. of consecutive integers will be 55.
This “discovery” brings us to the first rule of exponents, We can get the same result by applying the
and to many more as shown below:
formula for the sum of consecutive integers.
The formula for the sum of consecutive integers
Rules of Exponents: from 1 through n is n•(n+1)/2.

1) xa•xb = x(a+b) In this problem, sum is equal to 55

We get: n•(n+1)/2 = 55 or n•(n+1) = 110


2) xa/xb = x(a-b) = 1/(Xb-a) Because we know that n is positive, and we do
not bother to turn this into a quadratic “mess”.
3) (xa)b = xab = (xb)a Instead, we will choose two factors of 110 such
COROLLARY: XAYA = (XY)A that they are 1 apart and will multiply to give
110. We must conclude that n must be 10.
4)
1/xa = x-a If the numbers are simple to deal with, you may
want to use your fingers to get the answer rather
a b
5) IF x = x , then a = b than apply the formula.
IF XP = YQ , AND IF X IS ODD AND Y
EVEN OR VICE VERSA, Example 2:
THEN P = Q = 0.
Notice that any exponent on an odd integer What is the smallest integer bigger than 1 that is
will yield an odd integer; any exponent on at once a perfect square and a perfect cube?
an even integer will yield an even integer.
The only way the equation will work is if The smallest number bigger than 1 is 2, and an
the exponents are equal to 0. integer that is at once a perfect square and a
perfect cube must be (22)3 = 26 = 64.
6) X0 = 1 TRIVIA:
The smallest number bigger than 1 and is
at once a perfect square, a perfect cube
and a perfect fifth is ((22)3)5 = 230 = 645

Some of the materials used in this module are taken from disclosed editions of GMAT, and no monetary
value is included in the cost of this module for supply of these materials. These materials are strictly used
for illustration of concepts discussed. Page 90
©Educational Training Services, A Division of Maple Leaf International Consulting, Inc.
Not to be reproduced, copied or distributed. Intended for use only by the registrant.

Example 5:
Example 3:
If 43x-7 = 1/512, what is x?
“IF 10-9 + 10-8 + 10-7 + 10-6 + 10-5 =
Our first instinct will be to see whether we
N•10-9, What is N?” can write 512 as an exponent with a base of
4. If we start dividing 512 by 4, we get a 2
-9 N 9
We notice that N•10 is the same as /10 as a factor in the end. That is not a good
9
situation. We must get a 4 throughout so
Let us multiply both sides by 10 so that we can get rid of
9
the 10 in the denominator of the fraction on the right side
that we can express the given number as an
of the equation. exponential value.

Let us do the next best thing, or carry out


We get: the plan B. Plan B is to express both 4 and
9 -9 9 -8 9 -7
10 •10 + 10 •10 + 10 •10 + 10 •10 + 10 •10
9 -6 9 -5
=N 512 as an exponent in terms of a base
value of 2.
9-9 9-8 9-7 9-6 9-5
10 + 10 + 10 + 10 + 10 = N
0 1 2 3 4
10 + 10 + 10 + 10 + 10 = N We have:
1 + 10 + 100 + 1,000 + 10,000 = N
11,111 = N 2 3x-7
(2 )
9
= ½ =2
6x-14
-9
(Rule 4)
Or, 2 = 2-9 (Rule 3)
or N = 11,111 (Answer) We apply the rule 5, and get the equation:
6x – 14 = -9 or
Example 4: x = 5/6 (ANSWER)

If 22x + 3 = 1/128, What is x? Example 6:


How many digits, including repetition of the
same digit, are required to express 10100 in
We notice that we have a decimal number decimal notation?
on the right-side of the equation, and an
exponential value on the left side. Our We know that 101 = 10 or 2 digits
objective is to bring values on both sides of 2
10 = 100 or 3 digits
3
the equation to a common basis. 10 = 1,000 or 4 digits
and so on.
7
We notice that 128 can be expressed as 2 .
(If you are wondering how to figure this out, We can see that the number of digits is always
start dividing 128 by 2 and see how many one more than the exponent value, if the base is
2’s you will get as you go down to 2. 10. Therefore, we will have 101 digits when
128 = 2•64 = 2•2•32 = 2•2•2•16 = 2•2•2•2•8 100
10 is expressed as a decimal number.
and so on) In fact, the exponent on top of a base of 10
specifies how many zeros there will be in the
The reciprocal of an exponential value is the decimal number, and there is a 1 in front of the
same as the base raised to negative zeros.
exponent.
7 -7
TRIVIA
1/2 = 2 The number of digits required to express 100100 is
2x+3 -7 obtained when we express 100100 as (102)100 or 10200.
So, we have: 2 =2 We can determine that we will require 200+1 or 201
digits to express 100100 as a decimal number,
We apply the rule 5 and get the equation: including repetitions of the same digit,0. The number
2x + 3 = -7 or, of digits required to express 10001000 (which is the
x = -5 (Answer) same as 103000) will be 3,001, including repetitions of
0. You get the drift, don’t you?

Some of the materials used in this module are taken from disclosed editions of GMAT, and no monetary
value is included in the cost of this module for supply of these materials. These materials are strictly used
for illustration of concepts discussed. Page 91
©Educational Training Services, A Division of Maple Leaf International Consulting, Inc.
Not to be reproduced, copied or distributed. Intended for use only by the registrant.

Example 7:

Data Sufficiency MANIPULATE THE BASES


We should be able to look at the bases of
If y = 22x + 1, what is y? exponential values, and express them as
factors so that we can decide in what
convenient manner we can rewrite the given
1. 22x = 1/64 value. For example, if we have 65, we
2. y = 2x-2 should be able to see that we can write this
as (2•3)5 or as 25•35.
Flexibility with concepts and values is what
We need to know the value of x so that we the test attempts to get the handle on. So,
can determine the value of y. get flexible.

Statement 1 lets us decide the value for x. Consider the following problem:
-6
We notice that 1/64 is the same as 2 and 21 11 N
IF 5 x 4 = 2x10 , what is N?
we can set up an equation connecting 2x
and –6. (2x = -6 or x = -3. If x is –3, then y Notice that on the right side, we have 2 and
2x+1
=2 = 2-5 = 1/32)
10 as the ‘base’ values whereas on the left,
We will conclude that statement 1 alone is
we have 5 and 4. We need to get the same
sufficient. We must not move on to examine
‘bases’ on both sides of the equation in
statement 2, and our choices are either
order to relate apples to apples. We can
choice1 or choice4. 2
write 4 in terms of 2 as 4 = 2 . We get the
following picture:
Statement 2 gives us a value for x in terms 21 2 11
5 x (2 ) = 2x10
N
of x but then we have another value for y in 21
5 x 2
22
= 2 x 10N
terms of x in the stem. If we look at the 21 21 1
5 X 2 X 2 = 2 X 10
1 N
statement 2 information in the light of the 22 21
(notice that we split up 2 as 2 times 2 so
1
stem information, we can conclude that 21
that we can combine 5 and 2 to get
21
2x+1 = x – 2 or x = -3. 21 21
(5x2) = 10 . We can thus get 10 as base
-5
Once again, y = 2 or 1/32. Statement 2 values on both sides of the equation.
alone is also sufficient to answer the
Manipulation is the name of the game
question in a unique fashion.
here0.
Either statement alone is sufficient to get We have now the following, more workable
the answer. We must pick choice 4. situation:
21 N
(5x 2) x 2 = 2 x 10
21 N
Note: You must remember to use the We get: 10 x 2 = 2x 10
statement information along with any Cancel the factor 2 on both sides, and we
21 N
get, 10 = 10 or N = 21. (answer)
information that is provided at the
outset in the stem as part of the
NOW, PLAY WITH THE FOLLOWING
question itself. PROBLEM AND SEE WHETHER YOU GET
THE ANSWER SPECIFIED:
6X X-1 3
“IF 3 = 8100, WHAT IS (3 ) ?”
(ANSWER: 10/3)
HINT: WRITE 36X AS (33X)2 = 8100 = 902 AND GET
3X X-1 3X -1 3
3 = 90. Then, write (3 )3 AS 3 TIMES 3 = 1/3
3X 3x
TIMES 3 . Then, substitute 90 for 3 to get 1/27 times
90 or 90/27 or 10/3. (all fractions must be reduced to
their simplest forms)

Some of the materials used in this module are taken from disclosed editions of GMAT, and no monetary
value is included in the cost of this module for supply of these materials. These materials are strictly used
for illustration of concepts discussed. Page 92
©Educational Training Services, A Division of Maple Leaf International Consulting, Inc.
Not to be reproduced, copied or distributed. Intended for use only by the registrant.

Multiplication is repeated will have the same base and


addition. an exponent that is one less
Multiplication is a process that than that on the left.
involves repeated addition. If
we are multiplying 2 by 5, we For example, if you take a look
get the result of 10 by adding 5 at the set-up for 59, you will
two times or by adding 2 five notice that there are 5 values
times. in an addition setup on the
right and each of those values
2•5 = 2 + 2 + 2 + 2 + 2 has the same exponent value
2•5 = 5 + 5 of 8, and that each of those
exponents on the right is 1 less
You will be required to than the value of the exponent
understand this principle and on the left.
process when you deal with
the following concept in Example:
exponents. IF 2a = 2b + 2c, then which of
the following must be true?
Let us say that we have 22 I. a>b>c
II. b=c
We can write 22 = 2•21 III. a = b + 1 = c + 1
= 21 + 21
Using the above analysis, we
3 2
We can write 2 = 2•2 can see that there are two
= 22 + 22 values in the addition setup on
the right, and that a must be 1
If we had 37, we can write: more than b and c, and that b
37 = 3•36 = 36 36 +36 and c must be equal to each
If we had 59, we can write it other. We can see that
as: condition I is not valid, and II
59 = 5•58 and III are valid. We will pick a
= 58 + 58 + 58 + 58 + 58 choice that says: II and III only.

As you can see, any


exponential value can be
written as the sum of as many
values as the base number on
the left, and each of the values
in addition set-up on the right
Some of the materials used in this module are taken from disclosed editions of GMAT, and no monetary
value is included in the cost of this module for supply of these materials. These materials are strictly used
for illustration of concepts discussed. Page 93
©Educational Training Services, A Division of Maple Leaf International Consulting, Inc.
Not to be reproduced, copied or distributed. Intended for use only by the registrant.

Of course, we could have reached the same


conclusion by writing the denominator
FACTORIAL expression as x•(x-1)! and by canceling out
Factorial is a concept that is used to the
express the product of consecutive positive (x-1)! on the denominator and the
integers from 1 all the way to the number denominator.
sitting in front of the factorial sign. Example 3:
Conventionally, we will go down in steps of
1 starting from the integer in front of the What is (x – 2)! ?
factorial sign all the way down to 1. All the (x – 4)!
consecutive integers from 1 to the value in
front of the factorial sign are factors of We notice that the value on the denominator is 2
more than that on the denominator.
given number factorial We can write (x – 2)! as (x-2)•(x-3)•(x-4)!

For example, 7! is the product of


We have:
7•6•5•4•3•2•1.
(x – 2)! = (x-2)•(x-3)•(x-4)!
Consecutive integers from 1 through 7 are
(x – 4)! (x –4)!
factors of 7!
5! is the product of 5•4•3•2•1. Consecutive
integers from 1 through 5 are factors of 5! = (x-2)•(x-3) Answer

You should also be able to see that 7! is the Example 4:


If N = 21! + 17, which of the following cannot
same as 7•6!, and is the same as 7•6•5! and
be a factor of N?
so on. I. 21
Why do we need to be able to do this? II. 17
Because, we will have to deal with factorial III. 38
values as factors when we deal with
permutation and combination problems, and Let us revisit the concept of factors here. A factor
it will make our lives a lot easier if we can must divide N a whole number of times. Any value
cancel out factors and deal with simpler that does not do so cannot be a factor of N.
values. Can 21 be a factor? No, it cannot be, because,
while 21 will divide 21! a whole number of times, it
Example: will leave a remainder of 17 when it divides N.
Therefore, N cannot be a factor of N.
(x + 1)! = ? Can 17 divide N a whole number of times?
x! Yes, It can and does. We notice that 21! has a 17 in
the run-down to 1 and will cancel out the 17 in the
We can express (x + 1)! as (x+1)•x! denominator, leaving a whole number. The stand-
alone 17 will also yield a whole number when
We have: (x + 1)! = (x + 1)•x! = (x + 1) divided by 17. Therefore, we conclude that 17 is a
x! x! factor of N.
And the answer is a simple (x + 1). Can 38 divide N a whole number of times? No,
it cannot. We notice that 38 can be expressed in
terms of its factors 2 and 19, and these values will
Example 2: cancel out the 19 and 2 that are factors of 21! But
What is (x - 1)! = ? 38 cannot divide the stand-alone 17 a whole
X! number of times, and N, when divided by 38, will
We notice that the expression on top is yield a remainder of 17, not a whole number.
one “fry short of a happy meal”. Let us Our conclusion is that only the value in II is a factor
top up the meal with the “missing” fries of N. Conditions I and III are not.
and multiply both numerator and the
denominator by x.
We get: x•(x - 1)! = x! = 1 (Answer)
x• x! x• x! x
Some of the materials used in this module are taken from disclosed editions of GMAT, and no monetary
value is included in the cost of this module for supply of these materials. These materials are strictly used
for illustration of concepts discussed. Page 94
©Educational Training Services, A Division of Maple Leaf International Consulting, Inc.
Not to be reproduced, copied or distributed. Intended for use only by the registrant.

OPERATORS AND FUNCTIONS


+4 +4
Problems involving Function statements are 5a/2 = -33/2 + 4 = - 25/2 or
usually presented in the GMAT as “operators” 5a = - 25 or a = -5
problems. A typical functions problem will go
Exercise 2:
as follows:
If x *y = xy + 2 (x +2y) for all integers x and
If F(X) = X2 – 2X + 3, What is F(-3)?
We are required to find the value for the y, then 4*(-5) = ?
function of x when x = -3. We replace the
X’s on the right side of the statement with – The operator * specifies a relationship between
3, and get the value for F(-3) as F(-3) = (-3)2 the two variables in terms of the expression
– 2(-3)+ 3 involving those two variables.
Or, F(-3) = 18.
In the GMAT, function statements are defined In our problem, x is 4 and y is -5.
in terms of “operators”. An example of such a We simply plug in these values into the
problem is shown below: expression xy + 2 (x + 2y) to get:
4.(-5) + 2 (4 + 2(-5))
Example: :The operator @ defines the = -20 + 2 (4 - 10)
relationship between a and b, for all a’s and b’s: = -20 + 2 ( -6 )

2
4*(-5) = -20 - 12 = - 32
a@b = ab – b +a/b
Exercise 3:
Here the operator represents in an For all real numbers v, the
abridged form a relationship between a and b operation v* is defined by the
equation v* = v - v/3. If (v*)* = 8,
Exercise: then V = ?
The relationship between a and b is expressed by
V* is an abbreviation for v-v/3 or 2v/3.
the following operation:
You will understand V* to stand for
a@b = ab – b2 +a/b “function of v” statement.
What is the value of if This means that every time we see *
a@2 = - 33/2 next to a v, we simply replace v by 2v/3.
Let us work on the expression within
parenthesis first to get:
How do we solve this problem ? Whenever we
(v*)* = (2v/3)*
see the operator flanked by two variables, we
can write the expression involving those two Now, we have one more operator to get
variables as specified. In this problem, we know rid of. Let us invoke the original function
that the value of b is 2. We are required to find statement: v* = 2v/3
the value for a. Instead of a v, we now have 2v/3 in
(2v/3)*. This means that we have to
2 replace v with 2v/3 to get rid of the
Let us write a@2 as a.2 – 2 + a/2
What did we do? We simply plugged in the operator.
value for b as 2 in the specified expression. We have: (2v/3)* = 2/3•(2v/3) = 4v/9
We have got (v*)* reduced to 4v/9, and
We also have a value specified for a@2,
we have a value specified for this
which is -33/2 . expression, 8. Now we set up an
We can now set up an equation as follows and equation as follows to get the value for
solve for the value of a: v.
4v/9 = 8, or
a.2 - 2.2 + a/2 = -33/2 or
v = 9.8 / 4 = 18
2a - 4 + a/2 = - 33/2 or
5a/2 -4 = - 33/2 or
The answer is: v = 18

Some of the materials used in this module are taken from disclosed editions of GMAT, and no monetary
value is included in the cost of this module for supply of these materials. These materials are strictly used
for illustration of concepts discussed. Page 95
©Educational Training Services, A Division of Maple Leaf International Consulting, Inc.
Not to be reproduced, copied or distributed. Intended for use only by the registrant.

Inequalities Ranges of values for a variable specified in an


inequality:
Inequalities make a statement that the values on
either side of the sign are not equal to each We know that if x2 = 4, then x has two roots: +2
other. The inequalities may specify that the or –2.
value on the one side is bigger than or less than
the value on the other side. ( < or >) What do we know about the ranges of values for
x if x2 > 4?
We can carry out the same arithmetic operations
– additions, subtractions, multiplication, and We should know that the values for x will lie in
division – on an inequality, but you must two ranges:
remember the following caveat: x > 2 or x < -2

When you do an operation involving a negative x x


multiplication or a negative division on an
inequality on both sides, or if you -2 0 +2
simultaneously invert the values on both sides,
the sign of the inequality will be reversed. You
must remember to do it every time such negative Another interesting fact you should know
multiplication or negative division is performed. about is that in a small range between 0 and
1, any squared value will be less than the
Example: original value. For example, if x is ½, then x2
is ¼ and ¼ < ½. This reality exists only when
If 3 – 2x < -7, find an expression for x. x is a positive fraction less than 1.

Let us move the number to the right. Example: DATA SUFFICIENCY


We get: -2x < -7 – 3 < -10
Is x2 < x?
Let us divide both sides by –2 and flip the sign.
1. x > 0
We get: x > (-10/-2) > 5
The inequality expression for x is x > 5.
2. x < 1
Example 2:
We need to know whether x lies in the following
Find an expression for x, if 5 – 7x > -2. range: 0 < x < 1, because only in this range any
-4 squared value will be less than the original value.
Statement 1 tells us that x is positive. IF x is ¾, then
x2 is 9/16, and x2 < x. But then x could be 2, in which
Let us multiply both sides by –4 and flip the case x2 > x. The statement 1 gives us “all over the
sign, because we are carrying out a negative map” solutions. Statement 1 alone is not sufficient for
multiplication. a unique solution.
We get: 5 – 7x < 8 Statement 2 tells us that x is less than 1. If x is ½,
Let us move the number 5 to the right. The then x2 is ¼ and x2< x. But if x is –3, then x2 > x
sign remains unchanged. because 9 > -3. Once again, statement 2 alone is not
We get: -7x < 3 good for a unique solution. When we combine the two
Let us divide both sides by –7, and remember statements, we notice that 0 < x < 1, and in this range
to flip the sign one more time. we know that x2 < x. We must pick choice3 in the
We get: x > -3/7 test.

Some of the materials used in this module are taken from disclosed editions of GMAT, and no monetary
value is included in the cost of this module for supply of these materials. These materials are strictly used
for illustration of concepts discussed. Page 96
©Educational Training Services, A Division of Maple Leaf International Consulting, Inc.
Not to be reproduced, copied or distributed. Intended for use only by the registrant.

is what presentations will be about in


meetings and conferences.

Now that we have established why we


need to be good with word problems, let
You should expect to get about 50 us see what kinds of word problems you
percent of GMAT problems in a Word may have to deal with in the GMAT. We
setting. The test is to see whether you have seen earlier in this module how we
can conceptualize verbal descriptions of can deal with some “word problems”
a mathematical problem and deal with it dealing with “proportions”, and algebra.
by setting up simple mathematical Let us get some more practice with word
expressions or equations. Come to think problems in this chapter.
of it, we all do it in our management
careers almost on a daily basis. In real- The word problems will test your ability
life, the word problem you may have to to pay attention to details, and to do
deal with will have the following form: simple set ups leading to a resolution of
“John was hired as a sales representative the problem.
in July 1999, and his quarterly sales
quota was $25,000. He met his quota in EXAMPLE 1:
the first two quarters after hiring but his “During a certain season, a certain
sales performance in the last quarters team won 75% of its first 100 games,
were only 80% of his quota, and he is and 40% of the remaining games. If
facing an increase in his quota come the team won 60% of all games it
January 2001 by 25%. It is unclear played for the entire season, how
whether John will be able to meet his many games did the team play in
increased quota or not. Therefore, we total?”
need to chat with him, and fire him if
necessary.” Sounds like a management When we deal with a word problem, we
case study to you? If you cannot will set as X the quantity that needs to be
understand the numbers presented in this determined. In this problem, we know
verbal diarrhea, the chances are you are that the team played 100 games and
not going to be able to have a meaningful some more. Let us call the additional
chat with your employee. What is worse, games X games. We get the following
if your employee knows that your picture:
quantitative skills are as good as
Governor Bush’s ability to pronounce INITIAL GAMES PLAYED: 100
“subliminal”, then he can run circles ADDITIONAL GAMES PLAYED: X
around you by throwing all kinds of TOTAL GAMES PLAYED: 100 + X
numbers when you meet with him belly
to belly. The bottom line is, you better Let us see what else we need to do with
come up to speed with these monstrous the information provided in the problem.
word problems. Management career does
require some level of quantitative The team won 75% of the first 100
reasoning and understanding because that games, and 40% of the additional games.

Some of the materials used in this module are taken from disclosed editions of GMAT, and no monetary
value is included in the cost of this module for supply of these materials. These materials are strictly used
for illustration of concepts discussed. Page 97
©Educational Training Services, A Division of Maple Leaf International Consulting, Inc.
Not to be reproduced, copied or distributed. Intended for use only by the registrant.

Number of games won = EXAMPLE 2:


75% of 100 + 40% of X “Salesperson A’s compensation for any week
= 75 + 0.4X is $450 plus 5% of the portion of A’s total
sales in excess of $1,000 for that week.
The problem also tells us that the number Salesperson B’s compensation is 7% of B’s
games won is equal to 60% of all games total sales for that week. For what amount of
played. total weekly sales, in dollars, would both
salespersons earn the same compensation?”
Number of games won is also =
Once again, we need to determine the
60% of (100 + X)
dollar value of sales for which both
= 0.6 (100 + X)
people will earn the same income for a
= 60 + 0.6X
given week. Let us call that sales dollar
value X.
We are looking at two different
expressions referring to the same
A’s compensation
information, namely the number of games
= 450 + 5% of (X – 1000)
won in total.
= 450 + 0.05(X – 1000)
= 450 + 0.05X – 50
Our equation is now ready to emerge:
= 400 + 0.05X
We took (X –1000) when we applied 5%
75 + 0.4X = 60 + 0.6X
because the commission is paid on the
excess over $1,000.
Let us move the numbers to one side and
the variable to the other. We get:
B’s compensation = 7% of X = 0.07X
15 = 0.2X or X = 15/0.2 = 150/2 = 75
At a sales level of X dollars, both people
make the same income.
We now know the number of additional
games played. The question asks us to
Our equation is:
determine the total number of games
played. The answer is 100 + 75 = 175.
400 + 0.05X = 0.07X
Let us move the variable to one side of
This problem was not a difficult one once
the equation and isolate the number on
we cracked it. Understand the problem
the other side.
clearly and proceed to set it up one step
at a time. (multiply both
400 = 0.02X or 40000 = 2X
sides by 100 to get rid of the decimal places after the decimal
Let us try one more and see how we can point)
translate wordy dilemma into a
or, X = $20,000
manageable mathematical form.
Both people will make the same level of
commission at a sales level of $20,000.

Some of the materials used in this module are taken from disclosed editions of GMAT, and no monetary
value is included in the cost of this module for supply of these materials. These materials are strictly used
for illustration of concepts discussed. Page 98
©Educational Training Services, A Division of Maple Leaf International Consulting, Inc.
Not to be reproduced, copied or distributed. Intended for use only by the registrant.

Now that we know what a word problem We have two equations and two
is all about, let us see what other word unknowns. Let us write 11 B for A in the
problem types we may come across in second equation, and get:
the GMAT.
11B – 3 = 21B – 63
Let us isolate the numbers on one side
AGE and B on the other side.
PROBLEM We get: 10B = 60 or B = 6

Age problem usually involves two people If B is 6, then A is 11 times B or 66.


and their ages will be specified in some Anna is 66 years of age today.
relationship. Usually, you will start off by 10 years from now, Anna
defining each person’s age as of today will be 66 + 10 = 76 years
and go from there. Also, be sure to use of age.
the first letter of the name as the variable
instead of X and Y. How can the same problem be presented
to us without numbers and in a higher
EXAMPLE: difficulty level? Take a look at the one
“Anna’s age today is 11 that follows:
times Bob’s age today. “The combined age of John and Lisa
Three years ago, Anna’s is y years of age. If John is 12 years
age was 21 times Bob’s older than Lisa, how old will John be,
age then. How old will in terms of y, y years from now?”
Anna be 10 years from
now?”
How do we begin to deal with the problem
presented above?
Let us start with the ages of the person
Let us say that J is John’s age today,
as of today.
and L, Lisa’s age today.
AGE TODAY: We have: J + L = y ……. (1)
ANNA’S =A We are also given that J = L + 12 . (2)
BOB’S =B We are asked to get John’s age y
years from now. In order to do this, we
Today, Anna’s is 11 times Bob’s age. need to find John’s age today.
The set up for this information is simple: Let us get Lisa’s age in terms of
A = 11•B John’s age from the equation (2)
above. We get L = J – 12
Three years ago, Anna was A – 3, and Let us replace L with J – 12 in (1) to
Bob was B – 3. get: J + J – 12 = y or 2J = y + 12
Or, J = ½. y + 6
Three years ago, Anna’s was 21 times The above equation gives us John’s
Bob’s age then. age as of today. How do we
The set up for this information is: determine John’s age y years from
A – 3 = 21• (B – 3) now? We simply add y to John’s
current age.
Some of the materials used in this module are taken from disclosed editions of GMAT, and no monetary
value is included in the cost of this module for supply of these materials. These materials are strictly used
for illustration of concepts discussed. Page 99
©Educational Training Services, A Division of Maple Leaf International Consulting, Inc.
Not to be reproduced, copied or distributed. Intended for use only by the registrant.

We get:
J + y = ½. y + 6 + y = 3/2. y + 6 DISTANCE PROBLEMS

John’s age, y years from now, will be


3/2.y + 6 We all know that the Distance traversed
is the product of AVERAGE SPEED
EXAMPLE 3: and time of travel.

“Is Sam were twice as old as If we traveled at an average speed of 60


he is, he would be 40 years miles per hour for 4 hours on the Interstate
older than Jim. If Jim is 10 90, we would have traveled 240 miles.
years younger than Sam, how
old is Sam?”
DISTANCE =
AVERAGE SPEED• TIME
Let Sam be S years of age today, and Jim
be J years of age today.
Average speed = Distance / Time
Time of travel =
If Sam were twice S, he would be 40 + J.
Distance / Average speed
The set up is:
2S = J + 40
Jim is 10 years younger than Sam. How do These are the three different re-
we write this information mathematically? arrangements of the same equation you
J = S –10 should remember.

In order to find out the value for S, we need It will also be useful to know the concept of
to get rid of J from the first equation. Let us relative speed, and use it when you deal
replace J with S – 10 in the first set up, and with distance problems. You will find that
get: relative speed concept helps you breeze
2S = S – 10 + 40 through some Distance problems, and get
S = 30 to the answer in short order.
Sam is 30 years of age today and Jim is 20
years of age today. If Sam were 60, he THEOREM OF RELATIVE SPEED:
would be 40 years older than 20. Our “If A and B are moving in
verification tells us that we got the correct diametrically opposite
answer. directions either away from
each other or towards each
Here is a quickie: other, then the relative
“If Lisa was X years of age speed when both A and B
Y years ago, how old was are moving is the sum of
she Z years ago?” their individual speeds.”
Lisa is today X + Y years of
age.
Z years ago, she was
X + Y – Z years of age.
Pretty simple, huh?

Some of the materials used in this module are taken from disclosed editions of GMAT, and no monetary
value is included in the cost of this module for supply of these materials. These materials are strictly used
for illustration of concepts discussed. Page 100
©Educational Training Services, A Division of Maple Leaf International Consulting, Inc.
Not to be reproduced, copied or distributed. Intended for use only by the registrant.

Let us say you are proceeding southbound


on I-95 and traveling at 65 m.p.h, and Let us see how we can apply this concept
another driver goes up North on I-95 and when we deal with some distance
doing 70 m.p.h, when you pass each other problems.
for a brief moment, it is as if you stood still
and the other “jerk” whisked past at 135 EXAMPLE 1:
miles per hour. A more gruesome way to “The Distance between Boston and
remember this concept is by understanding Toronto is 600 miles. Car A leaves Toronto
what happens in a head-on collision. If two 8 A.M. Eastern Standard Time and Car B
cars collide head-on, and if one car was leaves Boston at the same time. If Car A
doing 65 miles per hour and the other was travels at an average speed of 60 miles per
doing 70 miles per hour, the impact of the hour and Car B at an average speed of 50
collision will be the same as if either driver miles per hour, how far would Car A have
slammed into a brick wall at 135 miles per traveled before it meets Car B?”
hour. Now you know what those crash test
dummies go through. Let us do this problem the conventional
way, and then see how much faster we can
You will have to apply this concept when do it by using the relative speed concept.
you deal with a problem that specifies that
A and B are some distance apart and are We begin by attempting to construct a
moving towards each other along a straight mathematical model of this word problem:
line. The question will be: How far does Meeting Point
one person travel before he or she meets x 600-x
the other person traveling in the opposite
Toronto Boston
direction towards him or her?
S1 m.p.h S2 m.p.h
A B Car A travels at 60 mph and B at 50 mph
average speed. Let X be the distance from
D miles Toronto where the two cars meet. By
Distance traveled by A before A meets B default, the car from Boston would have
= S1/(S1 + S2) • D traveled the difference between 600 and X
Distance Traveled by B before B meets A miles or 600-X miles. Since the two cars
= S2/(S1 + S2) • D leave at the same time, they would have
traveled for the same length of time when
Distance traveled by each person will be in they meet. However, they would have
proportion to the relative speed (S1 + S2). traveled different distances because they
were traveling at different rates of speed.
Time of travel for Car A: X/60 Hours
If people are moving in the same
Time of Travel for Car B; (600-X)/50 Hr
direction, then the relative speed is the
Time of travel is the same for both cars.
difference of the two speeds. If A and B
are moving in the same direction on I- ∴ x /60 = (600 - x)/50
95, and the speeds are 65 m.p.h and 70 We have one unknown and we need just
m.p.h respectively, when they pass each one equation. We have got one equation
other, it is as if one person stood still and we couldn’t possibly ask for more.
and the other moved at 5 miles per Let us cross multiply to get:
hour. 50x = 60(600-x)
Some of the materials used in this module are taken from disclosed editions of GMAT, and no monetary
value is included in the cost of this module for supply of these materials. These materials are strictly used
for illustration of concepts discussed. Page 101
©Educational Training Services, A Division of Maple Leaf International Consulting, Inc.
Not to be reproduced, copied or distributed. Intended for use only by the registrant.

50x = 36000 - 60x What is different about this problem? We


Or 110x = 36000, or x = 36000/110 notice that Car A travels an hour longer
We solve for x and we get x = 327.2 miles. than Car B does before it meets car B.
Car A would have traveled 327.2 miles Time of travel for Car A = Time of Travel
before it meets Car B. Car B would have for B + 1 hour.
traveled 272.8 miles before it meets Car A.
The distance from Toronto where the cars
Let us do it faster with the relative speed meet is still x miles. Car B still travels 600-
concept. x miles.

60 M.P.H 50 M.P.H Time of travel for car A = x/60


TORONTO BOSTON Time of travel for Car B = (600-x)/50
600 MILES
Because the trains are moving in How do we factor this information into the
diametrically opposite directions, the new equation?
relative speed is the sum of the two speeds Time for Car A = Time for Car B + 1
or 110 miles per hour. i.e. x/60 = (600-x)/50 + 1 = (650-x)50
Let us cross multiply to get:
Distance traveled by Car A 50x = 60(650-x)
Before it meets Car B = Or 110 x = 39,000
Car A’s speed•Distance Or, x = 39000/110 = 354 miles.
RELATIVE SPEED The cars will meet at a distance that is 354
= 60•600/110 =3600/11 miles from Toronto.
= 327.2 Miles.
Let us do it the Relative speed way.
Distance traveled by Car B before it When only one car is traveling, the relative
Met Car A = Car B’s speed•Distance speed is the speed of that one car. Car A
RELATIVE SPEED gets a head start and it travels 60 miles
= 50•600/110 closer to Boston in the first hour. When Car
= 3000/11 = 272.8 MILES B starts traveling, the distance between
Can you see that Relative speed way of Toronto and Boston is no longer 600 miles
getting the distance traveled by each car but only 600-60 = 540 miles.
may be a faster way than the algebraic When both cars are moving at the same
way? But regardless of what approach you time towards each other, the distance
use, you will get the same answer. traveled by each will be computed using
this new distance separating the two cars.
Now. Let us add a small wrinkle to the Relative speed will still be the same as 110
problem and state that Car A leaves an miles per hour.
hour ahead of Car B. How far from Therefore, distance traveled by Toronto
Toronto do the cars meet under this Car = Distance traveled in Hour 1 +
changed scenario? Proportional distance traveled when both
cars are moving
Distance traveled by car A =
60 + 60•540/110 = 60 + 294
= 354 Miles. The same
answer as by the other method.

Some of the materials used in this module are taken from disclosed editions of GMAT, and no monetary
value is included in the cost of this module for supply of these materials. These materials are strictly used
for illustration of concepts discussed. Page 102
©Educational Training Services, A Division of Maple Leaf International Consulting, Inc.
Not to be reproduced, copied or distributed. Intended for use only by the registrant.

have walked the difference of 50 and 244/9


Remember: For “Distance/Speed/Time” Miles or 255/9 miles.
problems, your starting point will always
be the relationship: Can you see how the relative speed concept
Distance = speed X time helps you deal with distance problems a lot
faster when two people are walking
EXAMPLE 2: towards each other.
“Sam and Sylvia are 50 miles apart
and start walking towards each What happens if people are
other along a straight line path. moving in the same
Sam’s average speed is 5 m.p.h and direction? We said that the
relative speed is the absolute
Sylvia’s is 4 miles per hour. If Sylvia
difference of the two speeds.
leaves an hour before Sam starts
walking towards Sylvia, how far, in
Let us see how we can put this information
miles, from where Sylvia started
to use in the following problem.
does she meet Sam?”
EXAMPLE 3:
Let us do it the Relative Speed way.
“Car A and Car B leave Kalamazoo at 9
a.m., and travel towards Detroit along I-94
4 miles per hour 5 miles per hr
East. Car A’s average speed is 60 miles per
Sylvia Sam
hour and Car B’s average speed is 70 miles
per hour. An hour after starting, Car B
50 miles
pulls into a rest area, while Car A kept
traveling at the same average speed without
Because the two folks are walking in the
stopping towards Detroit. If Car B leaves
diametrically opposite directions, the
the rest area 15 minutes after it stopped at
relative speed when both are walking is 4 +
the rest area, and continued to travel at the
5 = 9 miles per hour.
same average speed as before, how far
from the rest area will Car B catch up with
Initially, Only Sylvia is walking. She would
Car A?”
have walked 4 miles closer to Sam in the
First hour, before Sam begins to walk. The
In one hour, Car B would have gone 70
effective distance between the two is now
miles from start and Car B would be 10
46 miles and not 50 miles. We will use 46
miles behind Car A because car A’s
when we compute the proportional distance
average speed is only 60 miles per hour.
Sylvia walked.
Therefore, at the end of 1 hour, car A is
ahead 10 miles of Car B.
Therefore, distance walked by Sylvia
before she met Sam =
Car B stops at the rest area for 15 minutes
Distance walked by Sylvia in the first hour
and Car A keeps traveling at the same
+ Proportional distance walked by her
average speed. In 15 minutes, Car A would
when both were walking at the same time
have gone an additional 15 miles (1/4th of
= 4 + 4•46/9 = 4 + 184/9 = 4 + 204/9
60 miles) from where it was at the end of
= 244/9 Miles
Hour 1. This means that Car A will be 5
Therefore, Sylvia walked 244/9 Miles
miles ahead of Car B when it leaves the
before she bumped into Sam. Sam would
rest area 15 minutes later. When Car A
Some of the materials used in this module are taken from disclosed editions of GMAT, and no monetary
value is included in the cost of this module for supply of these materials. These materials are strictly used
for illustration of concepts discussed. Page 103
©Educational Training Services, A Division of Maple Leaf International Consulting, Inc.
Not to be reproduced, copied or distributed. Intended for use only by the registrant.

leaves the rest area and begins traveling at


the same average speed as before, the Example:
relative speed between Car A and Car B is
10 miles per hour (difference between 70 “A salesperson travels from London to Liverpool at
miles per hour and 60 miles per hour). an average rate of speed of 40 miles per hour, and
How long does Car B take to travel 5 miles returns immediately on reaching Liverpool to
London, traveling during this leg of the journey at
at a relative speed of 10 miles per hour? (It 50 miles per hour. What is the average speed for the
is as if Car A stopped and Car B traveled at whole trip?”
a relative speed of 10 miles per hour). 30
minutes. In 30 minutes, Car B would travel We can tell that the journey is over two legs at
35 miles at 70 miles per hour speed, and different average speeds, and the distance per leg is
the answer is: Car B would catch up with the same. We need to use the formula to obtain the
average speed. In this problem, S1 = 40 miles per
Car A at 35 miles from the rest area. hour and S2 = 50 miles per hour.

Can you see how the concept of relative Average Speed


speed and some reasoning were used in
= 2S1S2 / (S1 + S2)
solving this problem? Get used to both
processes. = 2•40•50/(40 + 50)
= 2•40•50/90 = 400/9
Also, remember that the = 444/9 miles per hour
relative speed concept can be
used ONLY if two objects Notice that the average
are moving. If there is just speed is not 45 miles per
one object, we will go back hour, which is what we
to the basic concept of would have obtained had
Distance we taken the straight
= average speed • time average.

EXAMPLE 2:
TRAVEL OVER THE “Tiffany travels from A to B
SAME DISTANCE IN at an average speed of 30
TWO DIFFERENT LEGS miles per hour and returns
OF THE JOURNEY immediately to A on reaching
B, traveling back at an
average speed of 50 miles per
If the journey involves two legs at two hour. If she took 3 hours for
different average speeds S1 and S2, and if the round trip, what is the
the two legs are identical in distance, then distance between A and B?”
the average speed for the total trip
involving the two legs is obtained by using We notice that the journey
the formula: involves two legs of
identical length and two
Average Speed different average speeds.
= 2S1S2 / (S1 + S2) Average speed for the
round trip
Note that the average speed cannot be = 2•30•50/(30+50)
computed as a straight average of the two = 300/8 miles per hour.
speeds.
Some of the materials used in this module are taken from disclosed editions of GMAT, and no monetary
value is included in the cost of this module for supply of these materials. These materials are strictly used
for illustration of concepts discussed. Page 104
©Educational Training Services, A Division of Maple Leaf International Consulting, Inc.
Not to be reproduced, copied or distributed. Intended for use only by the registrant.

How do we compute the


distance? We know that In this problem, we are
the average speed for the dealing with just one average
round trip is 300/8 miles speed and some “time”
per hour, and the round information. Let us go back
trip time is 3 hours. to the basics and work our
way up.
Therefore, round trip
distance = round trip If the car takes T hours to go
average speed• round trip from Chicago to Omaha
time = 300•3/8 = 900/8 traveling at 50 miles per
miles. hour, then the same car
would take (T – 2) hours if it
One way distance between traveled at 60 miles per hour.
A and B is one half the
round trip distance. The distance traveled over T
hours at 50 miles per hour is
Distance between A and B the same as that traveled over
= ½ •900/8 = 900/16 (T – 2) hours at 60 miles per
= 56 ¼ miles hour.

Our set up is:


In some other distance
problems, we have to go 50•T = 60•(T – 2)
back to the basics and Or, 50•T = 60•T – 120
apply the rule: Or, 10•T = 120
Distance Or, T = 12 hours.
= average speed • time
The car would have taken 12
Example hours to travel from Chicago
“The distance between to Omaha at 50 miles per
Chicago and Omaha, hour. Therefore, the distance
Nebraska can be traveled can between the two cities is:
be traveled 2 hours sooner if 50 times 12 = 600 miles.
the average speed of travel
were 10 miles per hour Let us see how the same
faster. If car is traveling at an problem can be presented as
average speed of 50 m.p.h a high difficulty level
from Chicago to Omaha, question in the following
what is the distance between page.
the two cities?”

Some of the materials used in this module are taken from disclosed editions of GMAT, and no monetary
value is included in the cost of this module for supply of these materials. These materials are strictly used
for illustration of concepts discussed. Page 105
©Educational Training Services, A Division of Maple Leaf International Consulting, Inc.
Not to be reproduced, copied or distributed. Intended for use only by the registrant.

Example 2: Example 3:
“A train traveling at “m” mph arrives at its
destination 100 miles away 2 hours behind “John travels for 6 hours at an average
schedule. At what rate of speed, in terms of speed of 50 mph, and then travels T hours
m, should the train have traveled to arrive at an average speed that is 10 mph more
on time?” than that during the first 6 hours. If the
average speed for the whole journey was
How do we deal with this problem? 58 mph, how long, in hours, did John
travel?”
What is average speed? It is Total distance
traveled/ total time Once again, our starting point is the
relationship:
Let S be the speed at which the train should DISTANCE = SPEED x TIME
have traveled to arrive at its destination on What are we required to find out in this
time. Let T be the scheduled time of travel. problem? We are required to find out the
Traveling at m miles per hour, the train value for (6 + T) in hours.
takes 2 hours longer or T +2 hours.
We get 100 = m (T+2) .....................(1) How do we read, in mathematical terms,
the information in the problem ?
If the train had traveled at S mph, it would • John traveled 50 X 6 = 300 miles in the
have arrived on time, T. first hour.
We get 100 = S . T ......................... (2) • John traveled (50 + 10) .T = 60T
or S = 100/ T ........................... (3) miles in the next leg of the journey
lasting T hours.
We are asked to express S in terms of m. • Total distance traveled = 300 + 60.T
Can we express T in terms of m from(1) so
• Total time traveled: T + 6
that we can plug in that information in (3)
• Average speed = Distance/ Time
and get S in terms of m ?
• Average Speed = (300 + 60T)/(T+6)
Let us play around with (1). We have 100 • We also know that the average speed is
= mT + 2m 58 mph.
Or m T = 100 - 2m or • We are ready to set up an equation as
T = (100-2m)/m .................. (4) follows:
Let us plug in this value for T in terms of (300 + 60T)/)T+6) = 58
m in (3) to get Or 300 + 60. T = 58 ( T+6)
S = 100/T = 100 / (100-2m)/m Or 300 + 60. T = 58. T + 348
= 100m/(100-2m) Or 2 T = 48
We see a common factor of 2 in both the Or T = 24 hours.
numerator and the denominator. Let us John traveled for 6 + T =
take it out. We get 6+24 hours = 30 hours.
S = 50m/(50-m).
As you can see, we have to proceed step by
step, one step at a time, and then use the
value specified to set up an equation to
solve the problem.

Some of the materials used in this module are taken from disclosed editions of GMAT, and no monetary
value is included in the cost of this module for supply of these materials. These materials are strictly used
for illustration of concepts discussed. Page 106
©Educational Training Services, A Division of Maple Leaf International Consulting, Inc.
Not to be reproduced, copied or distributed. Intended for use only by the registrant.

Example 4: In our example, if John and David take 6


“ A salesperson travels for 3 hours at “m” hours to complete a job, then John and
miles per hour, and travels another 60 David, working together, will finish 1/6th
miles during the next 4 hours. What is the of the work per unit of time (in one hour).
average speed for the whole trip?” John, working alone, takes 10 hours to
complete the job, and will have completed
Average speed = Total 1/10th of the work in the same unit of time.
Let d be the length of time David will
distance/ total time require to complete the work, acting on his
own. Per same unit of time (in one hour),
• Distance traveled in the first 3 hours = David would have completed (1/d)th of the
3.m miles (Notice that we have to work.
compute the distance from the
time/speed information specified) Fraction of work completed by John per
• Distance traveled in the next 4 hours = hour + Fraction of work completed by
60 miles David per hour = Fraction of work
completed by John and David together.
• Total distance traveled = (3m + 60)
• Total Time traveled = 3 + 4 = 7 hours
• Average speed = The above information translates into the
Total Distance ÷ Total time following mathematical equation:
= (3m+60) ÷ 7= (3m+60)/7 (1/10) + (1/d) = (1/6)
A messy fraction. But that is answer. or (d+10)/10d = 1/6
Or 6(d+10) = 10d
WORK PROBLEMS Or 4d = 60 hours or d = 15 hours
Solving for “d”, we get 15 hours, meaning
Work Problems deal with the rates at
that David will require 15 hours to
which certain persons or machinery work
complete the work acting alone.
alone or together and you will be required
to calculate the rate at which they work RULE: If T1 is the length of time taken by
worker 1 to complete a job working alone, and if
together or alone.
T2 is the length of time taken by worker 2
complete the same job working alone, at their
Let us illustrate a typical problem dealing respective constant rates, then if worker 1 and 2
with work. work together, they will complete (T1 + T2)/
(T1.T2) of the job in 1 hour. Or, Workers 1 and
“John and David, working together, can 2, working together, will take (T1T2) / (T1 +
T2) hours to do the job, working together at
complete the job in 6 hours. If John,
their respective constant rates.
working alone, can complete the job in 10 RULE: If X machines, each working at the
hours, in how many hours will David, same constant rate, take N hours to finish a job,
working alone, complete the job?” then one machine, working alone at the same
constant rate, will require X times N hours to
The good news about this type of problem finish the job. For example, if 5 machines,
is that there is just one way to do it: You working at the same constant rate, take 4 hours
to finish the job, then one machine, working
calculate what fraction of the work will alone, will require 5 times 4 or 20 hours to
be completed per unit of time for each complete the job. If we use 3 machines to finish
person working alone and for the combined the same job, then the 3 machines will take 20/3
efforts. = 6 2/3 hours to do the job, working together.

Some of the materials used in this module are taken from disclosed editions of GMAT, and no monetary
value is included in the cost of this module for supply of these materials. These materials are strictly used
for illustration of concepts discussed. Page 107
©Educational Training Services, A Division of Maple Leaf International Consulting, Inc.
Not to be reproduced, copied or distributed. Intended for use only by the registrant.

Example 2: machine Y should be set to produce 3,000


If 8 Large hoses can fill a pool in 6 hours, nuts in an hour, or 10,000 nuts in 3.333
and 6 small ones can fill the same pool in hours so that X and Y, operating together,
10 hours, working at their respective can produce 10,000 nuts in 2 hours.
constant rates, how long would it take for 6
large hoses and 6 small hoses to fill the Example 4:
pool, working together? “A machine working at a constant rate had
produced 1/4th of the total job by 10:20
You will notice that we have information a.m. and 1/3rd of the total job by 10:40
pertaining to how long it takes for 8 Large a.m.. If the machine continue to work at the
hoses to fill the tank, but we are not going same constant rate, at what time will the
to use all 8. We will use only 6 Large hoses machine finish the job?”
to fill the pool with 6 small ones.
If it takes 6 hours to fill the pool with 8 When we deal with work problems, we
large hoses, then it would take 6 large need to compute what fraction of work is
hoses 8/6 X 6 = 8 hours to fill the same accomplished per unit of time. In this
pool. problem, the machine had done 1/3 – ¼ =
This value (8 hours) is our T1 value. 1/12th of the total job in 20 minutes
What is our T2 value? 10 hours between 10:20 a.m. and 10:40 a.m. This
corresponding to 6 small hoses. means that the machine does 1/12th of the
So. How long will it take to fill the pool for job in 20 minute increments. Working at
6 small and 6 large hoses working together this rate, the machine will take 12 times 20
at their respective constant rates? minutes from start to finish, or 240 minutes
Time to fill the pool = T1 T2 or 4 hours. The machine had done 1/3rd of
(T1 + T2) the job by 10:40 a.m. and it will take 2/3rd
= 8.10 / 18 of 4 hours to do the remaining 2/3rd of the
= 4 4/9 hours job or 160 minutes, which translate into 2
6 small and 6 large hoses, working together hours and 40 minutes beyond 10:40 a.m.
at their respective constant rates, will fill That will put us at 01:20 p.m. for the finish
the pool in 4 4/9 hours. time.

Example 3: Example 5:
“A machine started on July 2nd at 10:00
“The machine shop is required to produce a.m. and ran continuously at a constant rate
10,000 nuts in 2 hours. The foreman has at for 263 hours to finish a production lot. At
his disposal two machines, X and Y. what time and on what date did the
Machine X can produce 10,000 nuts in 5 machine finish the operation?”
hours, working alone. At what rate should
the machine Y be operating so that 10,000 263 hours are 10 days and 23 hours, or 11
nuts will be produced in 2 hours?” days less an hour. We need to go forward
11 days from 10 a.m. on July 2nd and move
back an hour. That will put us at 9 a.m. on
The Number Of Nuts required to be
July 13th. That is when the machine will
produced in one hour = 5,000 nuts.
complete the operation.
Machine X, acting alone, can produce
2,000 nuts in an hour. Therefore, the
Some of the materials used in this module are taken from disclosed editions of GMAT, and no monetary
value is included in the cost of this module for supply of these materials. These materials are strictly used
for illustration of concepts discussed. Page 108
©Educational Training Services, A Division of Maple Leaf International Consulting, Inc.
Not to be reproduced, copied or distributed. Intended for use only by the registrant.

MIXTURE PROBLEM

A mixture problem is a weighted-average Example 2:


problem, and weighted average is a How many gallons of a 80% alcohol solution
concept that you will deal with a lot in must be added to a 75% alcohol solution so that
GMAT. If you recall, Distance-speed-time the resulting solution will be 20 gallons of 77%
problem is also a weighted-average solution?
problem when you are asked to determine
the speed as a ratio of total distance and Let us say that we must mix X gallons of
total time traveled. 80% solution with 20-X gallons of 75%
solution so that we get a resulting solution
of 20 gallons.
“How many gallons of a solution that is What is the amount of alcohol in X gallons
20% alcohol must be added to 8 gallons of of 80% solution? 0.8 X gallons
a solution that is 10% alcohol so that the
resulting solution is 16% alcohol?” What is the amount of alcohol in (20-X)
gallons of 75% solution?
Let “x” represent the number of gallons of 0.75(20-X) gallons.
the 20% solution.
What is the total amount of alcohol in the
The amount of alcohol in “x” gallons of mixture ? 0.8X + 0.75(20-X)
20% solution will be (0.2x) gallons = 15 + 0.05X
The amount of alcohol in 8 gallons of 10% We want this amount of alcohol to be equal
solution will be 8(0.1) = 0.8 gallons. to 77% of the mixture volume of 20
The amount of alcohol in the resulting gallons. This requirement translates into
solution of (x + 8) gallons will be: the following equation:
(0.2x + 0.8) gallons 15 + 0.05X = 0.77 (20) = 15.4
We want this amount to be equal to 16% of Or 0.05X = 0.4 or X = 0.4/ 0.05
the total solution volume of (x+8) gallons. X = 8 gallons
The equation we get for this information We must mix 8.0 gallons of 80% alcohol
is: solution with 12.0 gallons of 75% solution
0.2x + 0.8 = 16%(x+8) = 0.16x + 1.28 to get 20 gallons of 77% alcohol solution.
Solving for “x” , we get 0.04x = 0.48 or
X = 12 gallons. If we want to truly understand why a
“mixture problem” is nothing but a
Remember: You must calculate the “weighted average” problem, we should
total solution volume, and the amount of consider the following problem.
alcohol in that solution, and set up an
equation that expresses the alcohol
content as a percent of the total solution
volume.
Some of the materials used in this module are taken from disclosed editions of GMAT, and no monetary
value is included in the cost of this module for supply of these materials. These materials are strictly used
for illustration of concepts discussed. Page 109
©Educational Training Services, A Division of Maple Leaf International Consulting, Inc.
Not to be reproduced, copied or distributed. Intended for use only by the registrant.

Example 3: Revenue from the sale of “2N” dozen


donuts in August 98 = 2N. (7.25) = 14.5N
“If one group of 10 students with an dollars.
average GMAT total score of 610 is mixed Total revenue for the months of July and
with another group of 12 students with an August 98 = 6.95N + 14.5N = 21.45 N
average GMAT total score of 590, what is dollars.
the average GMAT total score of the new
group (mixture)?” Total quantity sold during the two months
= N + 2N = 3N dozen donuts.
The total “total” score of the first group of Average price per dozen of donuts for the two-
10 students is 6,100. month period = Total Revenue / Total Quantity
= 21.45N/3N = 7.15 dollars.
The total “total” score of the second group
of 12 students is 7,080.

The size of the new group (mixture) is 22.

The sum of the two totals is: 13,180

The new average score of the mixture is:


(13,180 ÷22) =599

Example 4: Let us see what happens if we


mix sales figures for two months at a donut
shop:

“Dunkin’ Donuts sold twice as many


dozens of donuts in the Month of August
98 as in the month of July 98. If the
average price per dozen of donuts in the
month of July 98 was $6.95 and the
average price per dozen of donuts in the
month of August 98 was $7.25, what was
the average price per dozen of the donuts
for the two months?”

Solution: If the quantity sold (in dozens)


during July 98 was “N” dozens, then the
quantity sold in August 98 was “2N”
dozens.

Revenue from the sale of “N” dozen donuts


in July 98 = 6.95N dollars

Some of the materials used in this module are taken from disclosed editions of GMAT, and no monetary
value is included in the cost of this module for supply of these materials. These materials are strictly used
for illustration of concepts discussed. Page 110
©Educational Training Services, A Division of Maple Leaf International Consulting, Inc.
Not to be reproduced, copied or distributed. Intended for use only by the registrant.

INTEREST PROBLEMS:

There are two types of interest: Example 2:


Simple Interest, and Compound Interest.
An amount of money, N, was invested in
With Simple Interest, interest is computed an account paying interest at 10% per
on the principal amount only. annum compounded annually. If the
For example, 6% simple interest on a amount of money in the account at the
principal amount of $100.00 is $6.00. beginning of year 4 was $14,641, what was
the initial value of investment, N?
With compound interest, interest is
calculated on “principal plus accrued or Asset Interest Accrual
earned interest”. With 6% compound Value Earned Value
interest, compounded annually, the $100.00 N 0.1 N 1.1N
principal amount will earn $6.00 interest in 1.1N 0.11N 1.21N
the first year and $6.36 interest in the 1.21N 0.121N 1.331N
second year, the compound interest in the 1.331N = $ 14, 641 or N = $11,000
third year will be $6.74
Example: Get used to dealing with compound interest
problems by setting up a matrix as shown.
“If $10,000 is invested at 12% annual You will be using the same approach when
interest, compounded monthly, what is the you deal with “depreciation” problems,
balance in the account after 3 months of where the depreciation is calculated on the
starting the investment?” value of the asset at the beginning of the
year.
12% annual rate translates to 1% monthly
rate of interest. In a compound interest situation, interest is
calculated on the accrual value, which is
The Account Balance after Month 1: the sum of principal amount and the
$10,000 + 1% of $10,000 = $10,100 interest earned. In compound depreciation
The Account Balance after Month 2 : problem, depreciation is computed on the
$10,100 + 1% of $10,100 = $10,201 residual value of the asset, which residual
The Account Balance after Month 3 : value is the asset value at the beginning of
$10,201 + 1% of $10,201 = $10,303 the year minus the depreciation during the
year.
The formula to compute the Principal Plus
compounded interest over a period of “3”
months or years at a rate of interest per
same period of “1%” is:
P (1+1/100)³ , where P is the
principal amount.

Some of the materials used in this module are taken from disclosed editions of GMAT, and no monetary
value is included in the cost of this module for supply of these materials. These materials are strictly used
for illustration of concepts discussed. Page 111
©Educational Training Services, A Division of Maple Leaf International Consulting, Inc.
Not to be reproduced, copied or distributed. Intended for use only by the registrant.

Example 2:
DEPRECIATION PROBLEMS “An asset of value N depreciates at the
rate of 1/4 of its value at the beginning
“Depreciation” Problem is the exact of each year every year. What is the
opposite of “Interest” Problem. residual value of the asset at the end of
Depreciation refers to reduction in the year 4?”
value of an asset, and not growth.
Notice that the depreciation is not taken
Let us say that we have an asset - on the initial value of the asset, but rather
Computerized Milling Machine - that on the value of the asset at the beginning
was bought for $100,000. If the tax laws of each year. The best way to deal with
allow the company to take depreciation this type of problem is to set up a matrix
at a rate of, say, 20%, then the asset will as shown below:
lose $20,000 each year every year until Year Value Dep Residual
the asset is completely depleted in 5 Value
years. 1 N N/4 3N/4
2 3N/4 1/4(3N/4) 3/4(3N/4)
Example 1: 3 3/4(3N/4) 1/4(3/4(3N/4)) 3/4(3/4(3N/4))
“An asset valued at N dollars depreciates 4 3/4(3/4(3N/4)) 1/4(3/4(3/4(3N/4) 3/4(3/4(3/4(3N/4))
at a uniform rate of 15% of its initial Do you notice a pattern emerging here ? The
value each year. If the residual value residual value at the end of year 4 will be
(value after depreciation) after 4 years is (3.3.3.3)N / (4.4.4.4)= 81N/256
$12,000, what was the initial value of the What will be the residual value at the end of
asset?” year 5 in this problem? It will be 3/4th of
81N/256 = 243N/1024 and so on.......
How do we read and set up this
problem? The value remaining after 4
depletions- or 60% depreciation - is
$12,000. This information can mean only
one thing: That $12,000 represents (100-
60) = 40% of the value N of the asset.
Now we are ready to set up an equation:
40% of N = $12,000
OR 0.4N = $12,000
OR N = 12,000 ÷ 0.4 = $30,000
That was not a difficult problem, was it ?

What happens if the information is


specified differently as follows ?

Some of the materials used in this module are taken from disclosed editions of GMAT, and no monetary
value is included in the cost of this module for supply of these materials. These materials are strictly used
for illustration of concepts discussed. Page 112
©Educational Training Services, A Division of Maple Leaf International Consulting, Inc.
Not to be reproduced, copied or distributed. Intended for use only by the registrant.

DISCOUNT PROBLEMS: he must sell the vehicle for 150% of the


cost or 1.5 X $2,500.00 = $3,750.00
If P is the original price which is
discounted by d%, the new price Part 2 of the problem is a little more
becomes (100-d)% of P. complex. The dealer wants the profit to
Note that 10% discount on a price be 50% of the selling price. Let the
represents 90% of the original price. selling price be “s”. The Desired Profit is:
Let us consider an example: 0.5s. Therefore, the Cost Plus the
“Mary bought a mink coat for $750.00. Desired Profit must equal the selling
If that price represented a 25% discount price: We get the following equation:
on the original price, which included a $2,500.00 + (0.5)s = s
50% mark-up on the cost, what was the Solving for “s”, we get $5,000 as the
cost to the retailer of the mink coat?” selling price required to realize a profit of
50% of the selling price.
Now, try this one for size:
Mary paid $750.00 which was 75% of
the original sticker price (which is the
“An item was bought by a retailer for $1,000 and sold
same thing as saying 25% discount on for Price S, which represented a 25% discount on the
the original price). The original sticker sticker price. If the item had been sold at the sticker
price was, therefore, $750.00 ÷0.75 price, the profit would have been 40% higher than
what the retailer netted from the sale. The profit on
equal to $1,000.00 the sale of the item was what percent of the item’s
selling price ?”
This sticker price of $1,000 was 150% of Hint: Use the following relationships:
COST + MARK UP = STICKER PRICE
the cost (50% markup or profit). The STICKER(1-D/100) = SELLING PRICE
cost to the retailer was, therefore, where D is the discount percentage
$1,000 ÷ 1.5 = $666.67. SELLING PRICE - COST = PROFIT
Profit on the selling price is (S – 1000)
Because the selling price is 75% of sticker, we
A simple variation on the Discount
have:
Problem becomes a Profit Problem. Sticker ( 1-25/100) = S (selling Price)
Or ¾. Sticker = S
Profit is Selling Price minus Cost. Or Sticker = 4/3 . S
What is the hypothetical profit on the sticker
price?
“ A used car dealer buys a vehicle for Sticker – 1000 = 4/3.S – 1000
$2,500.00. We are told that the profit on the sticker price is 40% more than
the actual profit.
1. At what price should the dealer sell the We get the equation:
vehicle to realize a gross profit of 50% of 4/3S – 1000 = 1.4(S-1000)
or 4S – 3000 = 4.2S – 4,200
the cost of the vehicle? Or 0.2S = 1,200 or S = 6,000
2. At what price should the dealer sell the Profit on the selling price = 6000 – 1000 = 5000
Profit as a percent of selling price = 5000/6000 X 100
vehicle so that the profit represents 50% = 82%
of the selling price?”

Part 1 of the problem is simple. The


dealer wants 50% profit on the cost. So
Some of the materials used in this module are taken from disclosed editions of GMAT, and no monetary
value is included in the cost of this module for supply of these materials. These materials are strictly used
for illustration of concepts discussed. Page 113
©Educational Training Services, A Division of Maple Leaf International Consulting, Inc.
Not to be reproduced, copied or distributed. Intended for use only by the registrant.

minuscule chance of getting all of the answers


correct, but the odds are better than winning a lottery
STATISTICS - PROBABILITY by picking 6 numbers out of the first 49 consecutive
integers; the probability of winning a 649 lottery is 1
If there are N equally likely outcomes, and R in 14 million.
out of these outcomes are favorable to event A,
then the probability of event A is R/N. Remember: When you deal with probability
Probability is
the same as (and permutation), you will multiply when the
Proportionality. In the GMAT, you have 5 choices, and the question is about the probability of doing x AND
If 40% of a group probability of your picking a correct response by y; you will add when the problem asks you to
is made up of “winging” it is 1/5. find the probability of doing x OR y.
women, then the
probability of The probability of getting question 1 and
picking a woman What is the probability that you will get two
answers in a row correct by picking choices question 2 correct by “winging it” in the GMAT
in the group by is the product of 1/5 and 1/5, or 1/25.
random selection randomly? It is 1/5 times 1/5 or 1/25.
process is 4/10th
or 2/5th. What is the probability that you will get 6 If there is an assortment of balls in a box:
answers in a row correct by picking choices Red : 15, Blue: 20, and Green : 10
Ratios specify randomly? It is (1/5)6 or, 1/15,625. And we would like to select a ball randomly,
probability what is the probability that the ball selected will
information too:
What is the probability that you will get at least be a red one or a blue one?
If the ratio of
men to women in one of the 6 questions correctly answered by
“winging it”? This is a little tricky question. The The probability of a red ball is 15 out of 45, and
a group is 2 to 3, the probability of a blue ball is 20 out of 45.
then we know probability of getting at least one out of 6
questions correct is the same as 1 minus the Therefore, the probability that the ball selected
that men are 2
parts out of a probability of the WORST CASE will be a red or a blue ball is 35 out of 45, or 7/9.
total of 5 parts, SCENARIO of getting them all wrong. The Example 2:
and women are probability of getting them all wrong is (4/5)6 or
3 parts out of a 4096
/15625. Therefore, the probability of getting at If there is an assortment of balls in a box:
total of 5 parts. Red : 15, Blue: 20, and Green : 10
Therefore, the
least one of the six questions correctly answered
by selecting the choices randomly is 1 - 4096/15625 And we would like to select two balls randomly
probability of without replacement, what is the probability that
selecting a man Or, 11529/15625 or 0.737. Not bad at all.
the balls selected will be a red one and a blue
randomly is 2/5th
one?
and the Let us get a solid perspective on the probability
probability of of getting different numbers of correct answers Here, the order of selection is important. We
picking a woman by randomly selecting choices in the GMAT, need to consider that the first ball picked is Red
randomly is 3/5th. especially when you are about to run out of time. and the next blue or the other way around.
In GMAT, if
Let us say that we have 4 questions remaining, Because the probability of picking Red first
20% of the followed by Blue next is the same, we can
answer choices
and we decide to “shoot craps”. The probability
of getting at least 1 correct response is about 0.6 compute the probability of picking one red ball
are correct, then and one blue ball, and then multiply the result
the probability of (1 minus the probability of getting all wrong
responses), and the probability of getting at least by 2.
picking a correct
answer through 2 correct responses is about 0.18, and the Let us say that we need to compute the
random selection probability of getting at least 3 questions correct probability of getting a red ball when the first
is 20% or 1/5th. is about 0.027, and the probability of getting all ball is drawn. The probability is 15 out of 45 or
correct responses is 1/625, or 0.0016. 1/3.
How did we get these probabilities? Take a look After we have drawn a ball, we have only 44
at example 4. remaining in the box when the second ball is
drawn. The probability that the second ball will
When you are running out of time in the GMAT, and be a blue is 20 out of 44, or 5/11.
have, let’s say, 6 questions remaining, you are better The probability that the second ball drawn will
off selecting a choice for the remaining questions be a blue, given that the first one was a red is
because the unanswered questions will be counted as the product of the two probabilities: 1/3 times
incorrect responses. As you can see, you have a 5/11. Or 5/33. Because the selection could be Red
Some of the materials used in this module are taken from disclosed editions of GMAT, and no monetary
value is included in the cost of this module for supply of these materials. These materials are strictly used
for illustration of concepts discussed. Page 114
©Educational Training Services, A Division of Maple Leaf International Consulting, Inc.
Not to be reproduced, copied or distributed. Intended for use only by the registrant.

followed by Blue or vice versa, the required


probability is 2 times 5/33 = 10/33. Probability of getting at least 1 correct response
by randomly selecting answers for the remaining
Example 3: 4 questions is 1 – (4/5)4 = 1 – 256/625 = 369/625 ϕ
0.6
Joe Hitter has a batting average of
0.3. (He is likely to hit 3 ball out of 10 How did we determine that the
balls.) What is the probability that he probability of getting 4 answers in a row
will hit 4 times in a row when he wrong as (4/5)4? We know that the
comes to bat? probability of wrong answer for each
question with 5 choices is 4/5. The
The probability that he will hit 4 times in a row probability of getting the first answer
when he comes to bat is (0.3)4 or 0.081. wrong AND the second one wrong AND
The probability that he will “fail to hit” 4 times
the third one wrong AND the fourth one
in a row when he comes to bat is (0.7)4 , or wrong is the product of individual
0.2401. probabilities.
In the above problem, what is the
Finding the probability of an event probability that Sam will get at least 2
though “exceptions”. correct answers by randomly picking
choices for the 4 remaining questions?
Sometimes, the easiest way to find the
probability of an event is by subtracting The probability of 2 or more correct
the probability of a restriction or an responses is the same as 1 minus the
exception from the total probability value
of 1. We will do this every time we see
probability of all wrong responses minus
the phrase “at least” mentioned in the the probability of just 1 correct response.
problem.
We have determined that the probability
Example 4: of getting all 4 wrong responses is (4/5)4.
Sam took a test in which he had to select
responses from 5 choices to each question.
What is the probability of just 1 correct
Towards the end of the test, Sam had 4
questions remaining and he decided to randomly response?
select answers because he did not have the time There are 4 different scenarios with just
to set up the problems and work them out. 1 correct response.
I. What is the probability that Sam will Q1 Q2 Q3 Q4
get at least one question correctly
answered by taking recourse to random C W W W
Response
selection of choices for the remaining
four questions? Response W C W W
Response W W C W
The probability of getting at least 1 correct Response W W W C
response is obtained by subtracting the
probability of getting all answers incorrect.
In a multiple choice test with 5 choices per The probability for the scenario 1 as presented
question, the probability of a correct response is in row 2 in the table above is 1/5•4/5•4/5•4/5.
1/5 and the probability of an incorrect response [(43/54)]
is 4/5 when the choices are made randomly. The probability for the scenario 2 as presented
Therefore, the probability of all-wrong responses in row 3 in the table above is 4/5•1/5•4/5•4/5.
4 4
is ( /5) [(43/54)]
Some of the materials used in this module are taken from disclosed editions of GMAT, and no monetary
value is included in the cost of this module for supply of these materials. These materials are strictly used
for illustration of concepts discussed. Page 115
©Educational Training Services, A Division of Maple Leaf International Consulting, Inc.
Not to be reproduced, copied or distributed. Intended for use only by the registrant.

The probability for the scenario 3 as presented


in row 4 in the table above is 4/5•4/5•1/5•4/5.
[(43/54)] There are four different outcome scenarios:
The probability for the scenario 4 as presented HT, HH, TH, TT
in row 5 in the table above is 4/5•4/5•4/5•1/5. As a rule, if a coin is tossed N times or if N
[(43/54)] coins are tossed, the number of different
outcome scenarios will be 2N. For example, if 4
The above four scenarios have each one coins are tossed simultaneously or if the same
correct response, and three incorrect coin is tossed 4 times, then there are 24 or 16
responses. The probability of getting scenario different outcome scenarios.
1 or scenario 2 or scenario 3 or scenario 4 is
the sum of the probabilities for each scenario, What happens when we throw a die, which is a
and the probability for each scenario is the cube-shaped object with 6 square faces and each
same. Therefore, the probability of getting face is marked with different consecutive
just 1 correct response is: integers from 1 through 6?
4 • [(43/54)] = [(44/54)] = (4/5)4
The probability that we will get any of the
Now we are ready to compute values from 1 through 6 on the face showing up
will be 1/6.
the probability for 2 or more The probability of getting a 4 is 1/6.
correct responses. The probability of getting a 5 is 1/6 and so on.
Probability for 2+ correct responses =
1 – (Prob. of all wrong responses) – Let us say that we throw the same die 300 times.
(Prob. of just 1 correct response) Given that the probability of getting a 5 is 1/6,
= 1 - (4/5)4 - (4/5)4 = 1 – 256/625 - 256/625 how many 5’s should we expect in this exercise?
= 1 – 512/625 We should expect 300 times 1/6 or fifty 5’s to
= 113/625 ϕ 2/11 ϕ 0.18 show up. The probability does not mean that we
As you can see, the probability of will get a 5 exactly 50 times. The concept of
probability tells us that it is reasonable to expect
getting 2 or more responses by “crap that the value 5 will show up 50 times. The
shooting 4 questions” in the test is value may show up 70 times or only 20 times in
about 1/3rd the probability of getting at an actual exercise.
least 1 correct response. RULE: If the events can occur in
different order, we need to consider the
probability of obtaining the desired
outcomes for each of the different orders.
If we need to pick answers to two
remaining problems(each with 5 choices,
one being correct) on the GMAT or GRE,
the probability of getting one correct and
one wrong answer in a random selection is
2 times 1/5 times 4/5 because we can pick
Coin Toss and Dice throw the correct answer to the first and the
wrong answer to the second, or vice versa.

When we toss a coin, there are two


possible outcomes: Heads or tails. The
probability of getting a Heads or a Tails
is the same and is equal to ½.

What happens when we toss the same coin twice


or toss two coins simultaneously?

Some of the materials used in this module are taken from disclosed editions of GMAT, and no monetary
value is included in the cost of this module for supply of these materials. These materials are strictly used
for illustration of concepts discussed. Page 116
©Educational Training Services, A Division of Maple Leaf International Consulting, Inc.
Not to be reproduced, copied or distributed. Intended for use only by the registrant.

Example 1: = 1 – 1/32 – 5/32 = 26/32 = 13/16


“If a coin is tossed 4 times, what is the
probability that at least 1 heads will turn up?” You will see that 13/16 is also the probability for
getting at least 2 tails.
“At least 1 heads” means 1 or more heads. This
means that the “undesired” outcome scenario is
one in which it will be all tails. Example 3:

When we toss a coin 4 times, we will have 24 or If two dice are thrown at the same time, what is
16 different outcome scenarios. the probability that the sum of the values on the
faces turning up will be a 7 or a 9?
Out of these 16 different outcome scenarios,
there will be just 1 scenario in which it will be As a rule, if a die is thrown N times, or if N dice
all heads or it will be all tails. Probability of “all are thrown at the same time, the number of
tails” or “all heads” is 1/16. outcome scenarios will be 6N, where 6 is the
number of outcome scenarios per die throw.
Therefore, the probability of getting at
least 1 heads is = 1 – probability of all In this exercise, the number of different outcome
tails. scenarios will be 62 or 36.
= 1 – 1/16 = 15/16. The desired outcome scenarios are the
following:
TRIVIA: In a 5-coin toss situation (or, if the Die 1 Die 2 Sum
same coin is tossed 5 times), the number of Face value 1 6 7
different outcome scenarios is 25 or 32, and the 2 5 7
probability of getting at least 1 heads is 1 – 1/32 3 4 7
or 31/32. In this exercise, the probability of “all 4 3 7
heads” or “all tails” is 1/32. 5 2 7
6 1 7
TRIVIA: IF a coin is tossed N times or if N 3 6 9
coins are tossed, there will be exactly N outcome
4 5 9
scenarios in which there will be just 1 heads and
5 4 9
there will be exactly N outcome scenarios in
which there will be just 1 tails. 6 3 9

Probability of getting just 1 heads in the above We notice that there are 10 different outcome
exercise is N/2N. scenarios out of a total of 36 that will produce
the desired sum of 7 or 9.
For example, if a coin is tossed 4 times, the Therefore, the probability of getting a 7 or a 9 in
probability of getting just 1 heads or the the above exercise is 10/36 or 5/18.
probability of just 1 tails is 4/24 = 4/16 or ¼. We will see later on in this module that
probability question can be asked in any setting.
Example 2: Just remember that probability is the
ratio of the number of outcomes
If a coin is tossed 5 times, what is the favorable to a given event to the total
probability that at least 2 heads will show up? number of all outcomes.

We are looking at the probability of getting 2


or more heads.

Probability of getting 2 or more heads =


1 – probability of 0 heads(all tails)
- probability of just 1 heads

Some of the materials used in this module are taken from disclosed editions of GMAT, and no monetary
value is included in the cost of this module for supply of these materials. These materials are strictly used
for illustration of concepts discussed. Page 117
©Educational Training Services, A Division of Maple Leaf International Consulting, Inc.
Not to be reproduced, copied or distributed. Intended for use only by the registrant.

Example 4:
Example 5:
A club has M men and W women. 3 more men
join the club and 7 women resign their “In a certain zoo, the probability that a new-born
membership. If one person is to be randomly rhino will die is 10% for each month during a
selected for an assignment, what is the six month period. If the zoo had a new-born
probability that a woman will be selected from rhino population of 300 on June 1, 90,
the members in the club? approximately how many rhinos can be expected
to be alive at the end of month 3?”
Let us set up the information in the form of a
table so that we can get a clear perspective: In month 1, 10% of 300, or 30 are expected to
Before the After the die. Number expected to live at the end of month
change Change 1 is 300 – 30 = 270.
Men M M+3
In month 2, 10% of 270 can be expected to die.
Women W W-7 Number expected to live at the end of month 2 is
270 – 27 = 243.
Total M+W M+W-4
In month 3, 10% of 243, or 24 can be expected
No of women
Probability /Total membership to die, and approximately 243 – 24 = 219 can be
that a = (W – 7) / (M + W - 4) expected to survive.
woman will
be selected And that is the answer.

The probability that a man will be selected is


given by the expression: (M + 3) / (M + W – 4)

Problems in probability can be asked in any


setting, and occasionally you will get a
probability question that is more like a
decreasing balance depreciation problem.
Take a look at the next example on the
following page.

Some of the materials used in this module are taken from disclosed editions of GMAT, and no monetary
value is included in the cost of this module for supply of these materials. These materials are strictly used
for illustration of concepts discussed. Page 118
©Educational Training Services, A Division of Maple Leaf International Consulting, Inc.
Not to be reproduced, copied or distributed. Intended for use only by the registrant.

The probability of selecting a point that will lie


Probability questions in in the path from within the circular region
including the path is: 69π / 169π = 69/169
Geometry setting:
And that is the answer. Notice that we cannot
Example 1: simplify the above fraction further because 69
and 169 do not have any common factors.
“A circular-shaped garden is surrounded by a
circular path of uniform width 3 feet. If the RULE: ALL FRACTIONS MUST BE REDUCED TO
radius of the circular garden is 10 feet, and if a A FORM IN WHICH THERE ARE NO MORE
COMMON FACTORS FOR THE VALUES IN THE
point is to be selected randomly from within the
NUMERATOR AND IN THE DENOMINATOR.
circular-shaped region including the path, what PROBABILITY REFRESHER:
is the probability that the point chosen will PROBABILITY IS ANOTHER WAY OF
actually lie in the path?” DEFINING THE PROPORTION. If, in a
(Hint: The area of a circular region is population of N values, there are R items of
π•(radius)2). the desired kind, then the probability of
picking R in a random selection is R/N. This
is also the proportion of R in the population.
In order to answer this question, we need to find
the area of the circular path and take that area as If the probability or proportion of event A is
the ratio to the total area of the circular region P(A) and that of event B is P(B), then the
including the path. probability that A and B will both occur is
the product of two probabilities. If A and B
can occur in different order, then we need to
factor in the different order in which events
A and B can occur when computing the
probability.

EXAMPLE: There are 10 boys and 12 girls


The area shown in blue is the garden and the in a group. In a random selection of two
area around it and shown in red is the path. children, what is the probability that one boy
and one girl were picked?
The circular region includes the width of the Notice that the first selection could be a boy and
path. The radius of the circular region is equal the next a girl, or the first a girl and the next a
to the radius of the garden plus the radius of the boy. Unless the problem stipulates the order of
path. selection, we need to consider the probability
that the events may occur in any of the two
different possible orders.
The radius of the circular region = 10 + 3
= 13 feet The probability that a boy is picked first is 10/22
Area of the circular region = π•132 = 1696 or 5/11. Given that a boy was picked first, the
The radius of the garden = 10 feet. probability that a girl is picked next is 5/11
Area of the garden = π•102 = 100π times 12/21 or 5/11 times 4/7 = 20/77. Because
the selection could also be done in a different
order – girl first followed by boy- we need to
Area of the path = 169π - 100π = 69π
multiply this probability by 2 to get the required
probability: 2 times 20/77 or 40/77 We cannot
reduce this fraction any further, and will leave it
as is.

Some of the materials used in this module are taken from disclosed editions of GMAT, and no monetary
value is included in the cost of this module for supply of these materials. These materials are strictly used
for illustration of concepts discussed. Page 119
©Educational Training Services, A Division of Maple Leaf International Consulting, Inc.
Not to be reproduced, copied or distributed. Intended for use only by the registrant.

“If X is a multiple of 5, and


Example 2: also a multiple of 6, what is
the probability that X will be a
multiple of 20”?
2X
We have seen in our discussions
X on multiples that if X is a multiple
of 5 and a multiple of 6, then X
X must be a multiple of 30, the
least common multiple of 5 and
6.

X X 2X NOTE: “MULTIPLE OF” MEANS ‘ANY


Three squares are drawn as shown above. The INTEGER TIMES’. “X is a multiple of 5”
sides of the smallest square shown in green are means that X is ‘any integer times 5’.
X and X, and the sides of the middle square (in
red) are 2X and 2X, and those of the outer
square are 4X and 4X. If we want to select a This means that X must go in steps of 30. The
point randomly from within the larger square likely values are:
shaped region, what is the probability that the
point will actually lie in the L-shaped region X : 30, 60, 90, 120, 150, 180, and so on.
shown in red?
If X is equal to 30, it is not divisible by 20.
2
The area of the middle square is (2X) = 4X . 2 If X is equal to 60, then it is.
The area of the smallest inner square
that lies within the middle square = X2 We can see that for every two values, X is
Area of the L-shaped region divisible by 20 for just one of them. Therefore,
Shown in red = 4X2 – X2 = 3X2 the probability that X will be a multiple of 20,
given that it is a multiple of 5 and a multiple of
We need to find the area of the larger square 6, is ½ .
shaped region so that we can compute the
probability. The larger square has sides 4X and EXAMPLE:
4X, and
the area of the larger square is (4X)2 = 16X2. DATA SUFFICIENCY

Therefore, the probability that a point randomly “If a box contains some Red Balls, Blue Balls,
picked from within the larger square shaped and Green balls, and if one ball is drawn
region will lie in the L-shaped region shown in randomly from the box, what is the probability
red is 3/16. (The ratio of 3X2 to 16X2). that a Red ball was NOT drawn?”

EXAMPLE: 1. The box contains 25 red balls.


2. There are 65 balls in all in the box.

Our N.T.K for this problem is that we need to


know how many balls in all there are, and how
many of the non-red kind there are so that we
can computer the probability.

Statement 1 gives us nothing of what we need to


know. We must kill choices A and D and move
on to examine statement 2. We have just 3
choices remaining: B, C, and E.

Statement2 also gives us exactly one-half of


what we need to know. WE still do not know
Some of the materials used in this module are taken from disclosed editions of GMAT, and no monetary
value is included in the cost of this module for supply of these materials. These materials are strictly used
for illustration of concepts discussed. Page 120
©Educational Training Services, A Division of Maple Leaf International Consulting, Inc.
Not to be reproduced, copied or distributed. Intended for use only by the registrant.

how many of the non-RED balls we have in the


box. We must kill choice B now and proceed to
combine the two statements.

When we combine the two statements, we notice


that we know that there are 65 balls in all, and
there are 40 of non-Red kind, and we can
estimate the probability of drawing a non-Red
ball. We must pick Choice C.

Some of the materials used in this module are taken from disclosed editions of GMAT, and no monetary
value is included in the cost of this module for supply of these materials. These materials are strictly used
for illustration of concepts discussed. Page 121
©Educational Training Services, A Division of Maple Leaf International Consulting, Inc.
Not to be reproduced, copied or distributed. Intended for use only by the registrant.

Permutations and
Combinations The simplest of permutation problems involves
seating N number of people in N different seats,
Permutation is about selecting or arranging say, for a photo-shoot.
things in an ordered or role-defined way. Each
item that is ordered or selected is given a COUNTING METHODS
UNIQUE Identity. If we have to seat 5 people in THEOREM
5 row seats, then in what seat any person, say “If We can do one thing in X
person 4, is seated along with who is to his right
and who is to his left will determine the unique ways, and another thing in Y
identify for this person. That is why 12345 is a ways, we can do both things in X
different seating arrangement from 12543.
Notice that in the first seating arrangement,
times Y different ways."
person 4 is in position 4 but had 3 to his right
and 5 to his left. In the latter, he had 5 to his Let us say that we have 4
right and 3 to his left, even though he still people to seat in 4 different
occupies position 4.
Another example of a permutation problem is
seats.
when we need to make teams and give ‘titles’ or For the first seat, we have 4 choices.
‘roles’ to each member of a team. If the Having seated one person, we can choose from
problem requires that we make a team of 3 using the remaining 3 persons any one to be seated in
5 available people, and that we give titles to the the second seat.
chosen three (one person is the captain, the Having seated 2 people, we can choose from the
other vice-captain, and the third navigator), then remaining 2 people for the third seat.
we are dealing with a PERMUTATION Having seated 3 people, we have just 1 person
PROBLEM. On the other hand, if the problem remaining for the last seat.
did not require that we assign titles to the
chosen three, then it becomes a Therefore, the number of different ways we can
COMBINATION PROBLEM. seat 4 people in 4 different seats is the product
of 4, 3, 2, and 1 or 4 factorial ways.
Combination is about selection at a time and in
a non-ordered and non-role-defined fashion. FORMULAS
Each item chosen does not receive any unique 1. If we need to arrange N items in R
I.D. If we are required to make a team of 3 using positions or row-seats, then the number
5 available people, and if the problem does not of different PERMUTATIONS NPR =
specify any roles or titles for the chosen, then we N!/(N – R)!
are looking at a COMBINATION PROBLEM. 2. If we need to arrange 7 people in 3
different row seats, then we can
Think of COMBINATION as PERMUTATION determine the number of seating
adjusted for the illogical replication. For arrangements using the above formula
example, if we need to make a team of 3 using or by saying that the first seat can be
A, B, and C, then permutation problem would filled in 7 ways, the second in 6 ways,
treat ABC, ACB, BAC, BCA, CAB, CBA as 6 and the third in 5 ways. According to
different teams. Because A, B, and C cannot be the Counting methods rule, the number
in 6 different teams simultaneously (unless each of different seating arrangements is 7
has multiple personalities), we must conclude times 6 times 5 = 210.
that all 6 permutations are mere variant IF we need to make teams of R people using N
expressions of the same team ABC. available people, then the number of different
COMBINATIONS, NCR = N!/[R! (N – R)!].
NUMBER OF COMBINATIONS = (NUMBER
OF PERMUTATIONS)/(NUMBER CHOSEN)!

Some of the materials used in this module are taken from disclosed editions of GMAT, and no monetary
value is included in the cost of this module for supply of these materials. These materials are strictly used
for illustration of concepts discussed. Page 122
©Educational Training Services, A Division of Maple Leaf International Consulting, Inc.
Not to be reproduced, copied or distributed. Intended for use only by the registrant.

Example 1: We will, therefore, lose 4 seating arrangements


“Chairman Chris and four directors are to be in which A and B will be together in seat 1 and
seated for a meeting. Chris must be at the head seat 2 in different order.
of the table. How many different seating
arrangements can be made subject to this Similarly, A and B can be seated in seat 2 and
requirement?” seat 3 in two different ways next to each other,
and for each of these arrangements, D and C can
We notice that for all practical purposes, the be seated in seat 1 and seat 4 in two different
problem boils down to one of seating 4 different ways. Once again, we will lose 4 seating
people in 4 different seats because Chris is arrangements in which A and B will be next to
immovable. each other in seats 2 and 3 in different order.

We can do the seating arrangements in 4! Or 24 Lastly, A and B can be seated in seat 3 and seat
different ways. 4 in two different ways next to each other, and
for each of these arrangements, C and D can be
seated in seats 1 and 2 in two different ways. We
As a rule, If there are N people to be seated in N
will lose 4 arrangements in which A and B will
chairs, we can do it in N! different ways.
be seated in seated in seats 3 and 4 next to each
other in different order.
Example 2: Altogether, we will lose 12 seating
arrangements in which A and B will be next to
“How many different ways can we seat Anna,
each other at various seating configurations.
Bob, and Chris in a row subject to the restriction
that Bob and Chris cannot be seated next to each If we subtract the “verboten” arrangements from
other in any order?” the total of 24, we get left with 24 – 12 = 12
permissible seating arrangements in which A
We notice that since Bob and Chris cannot be and B will not be next to each other in any
seated next to each other, we must put Anna order.
smack in the middle. This configuration makes
Anna “immovable”. The problem boils down to In the GMAT, we have not so far come across a
seating 2 people in 2 different seats. We can do problem that imposes restriction on 2 people
it in 2!, or 2 different ways.
when 4 people are to be seated. But if you come
across a problem of this type, you know what to
BOB ANNA CHRIS
do. If the problem specifies seating arrangement
CHRIS ANNA BOB
for 5 people and specifies restriction that any
two people out of the 5 cannot be seated next to
Example 3: each other, just remember that we can have 72
permissible seating arrangements. How did we
“How many different ways can we seat 4
get this?
different people A, B, C, and D in 4 different
row seats subject to the restriction that A and B
As a rule, If there are N people to be seated in a
cannot be together in any order?”
row, and a restriction is imposed about seating
specified TWO entities next to each other in any
Without any restriction, we can seat A, B, C,
order, then the number of permissible seating
and D in 4!, or 24 different ways.
arrangements subject to this restriction is
obtained by using the formula:
Let us see how many seating arrangements we
lose because of the restriction imposed.
N! – 2•(N-1)!
A and B can be seated in seat 1 and seat 2 in
two different ways next to each other, and for
each of these arrangements, D and C can be Let us say that we have 6 people to be seated in
seated in seat 3 and seat 4 in two different ways. a row, and there is a restriction about seating
Some of the materials used in this module are taken from disclosed editions of GMAT, and no monetary
value is included in the cost of this module for supply of these materials. These materials are strictly used
for illustration of concepts discussed. Page 123
©Educational Training Services, A Division of Maple Leaf International Consulting, Inc.
Not to be reproduced, copied or distributed. Intended for use only by the registrant.

two specific people next to each other. The


number of permissible seating arrangements
subject to this restriction is:
EXAMPLE:
6! – 2•(6-1)!
= 720 – 2•5! “If 3 boys and 3 girls are to be seated in a
= 720 – 240 row in six seats such that no two people
= 480 of the same gender can be next to each
other in any order, how many seating
WE can seat 6! In 480 different ways subject to arrangements can be made?”
the restriction that a specified entity cannot be
seated next to another specified entity.
The restriction is that boys cannot be
If there is no restriction imposed, then we can seated together and girls cannot be right
seat 6 people in 6!, or 720 different ways. next to each other. This means that we
have to put the girls and boys in alternate
SEATING OF PEOPLE
seats
AROUND A TABLE

Instead of “row” seating, let us say that we need Boys could be in 1 – 3 – 5 or in 2 – 4 –


to seat N people around a table that seats exactly 6.
N people, the number of seating arrangements Likewise, girls could be in 2 – 4 – 6, or
we can make is (N-1)!. This is in 1 – 3 – 5.
because we need to fix one person in one seat
and keep that person as a reference for seating
the remaining persons or objects.
Boys could be seated in 1-3-5
configuration in 3 factorial or 6 ways,
However, if the problem specifies a restriction and the girls can be in 2-4-6
about seating any two people in any order configuration in 3 factorial or 6 ways.
around the table, then the number of seating We can seat the boys in 1-3-5 and girls in
arrangements subject to this restriction is:
2-4-6 in 6 times 6 or 36 different ways.
(N-1)! - 2•(N-2)! (Remember the counting methods
Example: theorem we stated at the beginning of
“If 6 people are to be seated around a this chapter?)
table that seats exactly six people, and if
any two specified people cannot be But then the boys could be in
seated next to each other in any order, configuration 2-4-6 and the girls could be
how many seating arrangements can be in 1-3-5 configuration. These two
made?” configurations give us another 6 times 6
= 36 different seating arrangements.
Number of seating arrangements
consistent with the restriction imposed Therefore, total number of seating
= 5! - 2•4! = 120 – 48 = 72 ways arrangements in two different seating
configurations for boys and girls is 36 +
36 = 72 different ways.
Let us consider another problem that
imposes a different type of restriction.
We will have to use simple reasoning and
get the answer. Let us see how we can
do it.
Some of the materials used in this module are taken from disclosed editions of GMAT, and no monetary
value is included in the cost of this module for supply of these materials. These materials are strictly used
for illustration of concepts discussed. Page 124
©Educational Training Services, A Division of Maple Leaf International Consulting, Inc.
Not to be reproduced, copied or distributed. Intended for use only by the registrant.

REMEMBER:
Permutation of digits of a
number
1. The sum of all 24 four-digit integers
formed by permutation of the digits
Let us say that we have four digits 1, 2,
1, 2, 3, and 4 without repetition is
3, and 4. If we were asked how many
66,660.
different four digit numbers can be
formed by using these digits? We need to 2. The sum of all 6 three-digit integers
know whether we are allowed to repeat formed by permutation of the digits
the digits or not. For example, we can go 1,2, and 3 without repetition is 1,332.
1111, 2222, 3333, 4444, 1112,1122, and
so on. EXAMPLE:
If repetition is allowed, then
for the first digit of the number, we have “What is the sum of all the
four different choices among the digits 1 three digit integers that can
through 4. be formed by permutation
For the second digit, we have another of the digits 2, 3, and 5
four choices. without repetition?”
For the third digit, we have yet another
four choices. We know that there are 3 factorial or 6
And for the last digit, we have still different 3 digit integers because
another four choices. repetition of digits is not allowed.

Therefore, the number of ways of In each decimal place, each of the three
forming four digit numbers by being able digits is going to occur twice exactly.
to repeat digits is 4•4•4•4 = 256 or 256
different four digit integers. In our case, the numbers are easy to set
up and add:
What happens if we are not
allowed to repeat digits? 235
253
For the first digit, we have four choices. 325
For the second digit, we have three 352
choices. 523
For the third digit, we have two choices. 532
For the fourth digit, we have just one We can see that each digit occurs in each
choice. decimal place exactly twice. The sum of
According to the Counting methods the six three-digit numbers formed by
theorem, we can form four digit integers permutation of the digits 2, 3, and 5
without repeating digits in without repetition is 2,220.
4•3•2•1 or 4 factorial = 24 ways.
We can put together 24 four-digit
numbers by using four digits without
repetition.

Some of the materials used in this module are taken from disclosed editions of GMAT, and no monetary
value is included in the cost of this module for supply of these materials. These materials are strictly used
for illustration of concepts discussed. Page 125
©Educational Training Services, A Division of Maple Leaf International Consulting, Inc.
Not to be reproduced, copied or distributed. Intended for use only by the registrant.

PERMUTATION
INVOLVING ORDERED Let us say that the problem specified the
following:
SELECTION “How many different 3-member teams can be
formed out of 4 men and 5 women so that a
If we are asked to select R woman will be the captain, and the second
items out of N items in an person selected will be the chef and the third
janitor?”
ordered or role-defined way,
we can use the Permutation
WE can choose any one of the 5 women to be
formula: the captain, and for each selection of a woman
as captain, we have to select 2 others in a role-
NP R = N! / (N-R)! defined manner from the remaining 8 people.

The number of different ways we can select the


Example 1: 3-member team subject to the specification is:
5• 8P2 = 5• 8!/(8-2)! = 5•8•7 = 280 ways

“How many different 3-member teams can be Permutation problems also give
formed from 4 men and 5 women so that one us an insight into women’s
person in the team will be the captain, the other psychology. The next time your
chef, and the third janitor?” female significant-other wants to
go shopping for yet another new
Do we see a role-definition pair of shoes, show her the
following permutation problem.
here? Yes. We must conclude
that it is a permutation
problem.
The problem is one of selecting 3 people out of
4+5 = 9 people in a role-defined manner.

We use the permutation formula:

9P3 = 9! / (9-3)! = 9.8.7.6! / 6! = 9.8.7


= 504 ways.
We can select 3 member-teams in a role-defined
manner out of 9 people in 504 different ways.

Example 2:
Some of the materials used in this module are taken from disclosed editions of GMAT, and no monetary
value is included in the cost of this module for supply of these materials. These materials are strictly used
for illustration of concepts discussed. Page 126
©Educational Training Services, A Division of Maple Leaf International Consulting, Inc.
Not to be reproduced, copied or distributed. Intended for use only by the registrant.

Example 3: combo meals that McDonald’s can offer is the


“Lisa’s wardrobe consists of the following items: product of 5, 4 and 3, or 60.
10 dresses;
12 handbags; Let us say that we place a restriction on any two
15 pairs of shoes; items in the available list. Let us say that drink
IF Lisa wears one dress, one pair of shoes and 3 and fries 2 cannot be offered together in any
carries one handbag , how many different dress- one combo meal. How many combos do we lose
shoes-handbag combos can Lisa make using the because of this restriction? For each burger, we
items in her wardrobe?” will lose one combination of Fries2-Drink3. For
5 burgers, we will lose 5 combos in all. We still
This type of problem is best dealt with as a tree. have 60-5 = 55 permissible combo meal
packages.
B1 S1
B2 S2
• • The same logic applies when we put a restriction
D1 • • on what Lisa can wear. Let us say that Lisa does
• • not like to wear D1 and Shoes 12 together.
B12 S15 (Colors don’t match, may be?). How many
D2 combinations does she lose in this process?

• She will lose one combination of D1 and S12 for
• each handbag she can carry. There are 12
D10 handbags and she will lose 12 combinations in
As we can see, Lisa can choose to wear any one all. She can still make 1800 – 12 = 1788
of the 10 dresses on a given day. For each dress combinations, after taking into account her
selection, she can choose to carry any one of the dislike for D1-S12 coordination.
12 handbags, and for each dress-handbag
selection, she can choose any one of the 15 pairs
of shoes. This means that for each dress Lisa
chooses to wear, she can make 12 times 15 or
180 combinations of shoes and handbags.

Given that Lisa has a choice of 10 dresses, she


can make 10 times 180 or 1800 different dress-
shoes-handbag combinations. Enough to make
Lisa not wear the same combination two days in
a row for close to 5 years. (Use this information
to suggest to your lady that buying another pair
of shoes may not be such a good idea. Just
kidding.)

You will notice that the number of different


assortments is simply the product of the
numbers of each of the items in the
combination. (That is what the counting
methods theorem is all about)

A similar problem can be faced by McDonald’s


manager. Let us say that McDonald’s has 5
different burgers, 4 different fries and 3 different
choices of drinks, and let us also say that a
combo meal consists of one burger, one set of
fries, and one drink. The number of different

Some of the materials used in this module are taken from disclosed editions of GMAT, and no monetary
value is included in the cost of this module for supply of these materials. These materials are strictly used
for illustration of concepts discussed. Page 127
©Educational Training Services, A Division of Maple Leaf International Consulting, Inc.
Not to be reproduced, copied or distributed. Intended for use only by the registrant.

The “combo” problem can be presented You will notice that other nodes C, D, and E
pictorially as an inverted triangular maze. Take have similarly 3 paths leading to node K along
a look at the following problem: the three middle nodes F, G, and H.

Example: We conclude that there are 5 X 3 = 15 paths that


will take the monkey from A to K.

Is there a simpler way to


figure this out?

A B C D E
A B C D E
F G H
F G H

K
K

Simply multiply the number of nodes on top by


the number of nodal points before the
Let us say that we have a really famished destination node K.
monkey at node A of the inverted-triangular We have 5 nodes A, B, C, D, and E on top
maze shown, and that it wants to get to the We have 3 nodes F, G, and H at the next level
banana at node K. If the monkey can move before the destination node K.
along any of the paths connected by straight line
So, how may different paths do we have from
segments, paths such as ABFK, ABCGK,
A to K ? 5 X 3 = 15.
ABCGFK, ABFGK, ABFGHK, and so on, how
many different paths can the monkey take to get Let us create some additional intermediate level
to the banana from node A ? (The monkey must nodes before the “banana” node and see what
take the shortest length possible for any path.) happens .
A B L C M D E
If (s)he is a smart monkey, (s)he can go down
the path AFK and get to the banana in no time. F S G T H
If (s)he is not so smart, (s)he can take any of the
other alternative paths: N P R

AFK --- Shortest route K


Now we have 7 nodal points on the top line:
A, B, L, C, M, D, & E
AFGK AFGHK
We have 5 nodal points next to the top level
ABFK ABFGK ABFGHK
F, S, G, T, & H
ABCGK ABCGFK ABCGHK
We have 3 nodal points before the destination.
ABCDHK ABCDHGK ABCDHGFK
N, P & R
ABCDEHK ABCDEHGK ABCDEHGFK
So. How many different paths can the mad
monkey take from node A to the banana node
As you Can see, the monkey can move along 15
K? Simply multiply the number of nodes in
paths to get to node K from node A.
each level: 7 X 5 X 3 = 105 paths.
If you pay attention to the bold letters in each
What if we turned this problem around and
path, you will notice that from node A, there are
made it a “Probability” question?
three paths: AFK, AFGK, AFGHK
Similarly, from node B, there are 3 paths:
ABFK, ABFGK, ABFGHK

Some of the materials used in this module are taken from disclosed editions of GMAT, and no monetary
value is included in the cost of this module for supply of these materials. These materials are strictly used
for illustration of concepts discussed. Page 128
©Educational Training Services, A Division of Maple Leaf International Consulting, Inc.
Not to be reproduced, copied or distributed. Intended for use only by the registrant.

Suppose that I asked you: “What is the


probability that the monkey will move along the
path AFNK?

We recognize that AFNK is just one path out of


105 possible paths. Therefore the probability
that the monkey will move along path AFNK is
simply 1 out of 105 or 1/105.

Let us take a look at another


type of maze, although it is not
related to permutation concept.

Some of the materials used in this module are taken from disclosed editions of GMAT, and no monetary
value is included in the cost of this module for supply of these materials. These materials are strictly used
for illustration of concepts discussed. Page 129
©Educational Training Services, A Division of Maple Leaf International Consulting, Inc.
Not to be reproduced, copied or distributed. Intended for use only by the registrant.

MAZE PROBLEMS Is there a simpler way to figure this out?

When you have a grid with three vertical lines,


start writing consecutive integers from 1 from
Imagine you are looking at a street map of
the top horizontal line, and simply add up the
Manhattan with North-South Avenues and
numbers from top to bottom as shown below:
East-West Numbered streets.
42nd 43rd 44th
42nd 43rd 44th
B
6th Avenue 1 B
6th Avenue

7th Avenue
2 7th Avenue

8th Avenue 3
8th Avenue
A 9th Ave. 4 A 9th Ave.

1+2+3+4= 10
If you are required to walk from intersection A
to intersection B along a route that is confined
If you had one more horizontal line, you will
to the square grid of four avenues and three
write 5 and add 1 through 5 to get 15.
streets shown in the map above, how many
routes from A to B can you take that have the
If you had three vertical lines and 6
minimum possible length ?
horizontal lines, you will write 1 through 6
and add them up to get 21 routes.
The best way to understand this problem is to
If you had seven horizontal lines and three
number the intersections and determine how
vertical lines, you will add 1 through 7 to get
many different combinations of streets and
28 as the value for the number of possible
avenues exist from A to B that have the same
routes with the same length .
minimum length .

42nd 43rd 44th What if you had 4 vertical lines and a


number of horizontal lines ?
9 10 B
6th Avenue
Let us say you had 4 vertical lines and 4
horizontal lines, you will add the 4
6 7 8 7th Avenue
consecutive ODD integers from 1:
1+3+5+7 = 16
3 4 5
8th Avenue
What if you had 4 vertical lines and 5
A 1 2 9th Ave. horizontal lines ? You add up 5 consecutive
ODD numbers from 1 through 9:
1+3+5+7+9 = 25
You can take any of the following routes to go
What if you had 4 vertical lines and 6
from A to B:
horizontal lines ? You will add the 6
A-1-2-5-8-B A-1-4-5-8-B A-1-4-7-8-B
consecutive ODD integers from 1:
A-1-4-7-8-10-B A-3-4-5-8-B A3-4-7-8-B
1+3+5+7+9+11 = 36
A - 3 - 4- 7-10-B A-3-6-7-8-B A-3-6-7-10-B
A-3-6-9-10-B
You get the drift, don’t you?
You can take any of these 10 routes and travel
the same distance.

Some of the materials used in this module are taken from disclosed editions of GMAT, and no monetary
value is included in the cost of this module for supply of these materials. These materials are strictly used
for illustration of concepts discussed. Page 130
©Educational Training Services, A Division of Maple Leaf International Consulting, Inc.
Not to be reproduced, copied or distributed. Intended for use only by the registrant.

exclusion. Take a look at the following


Combination problems examples. Think of
Combination
Example 2: Combination as a
Combination is selection at a time and problem that
involves order of selection is immaterial. We said earlier corrects that
SELECTION AT “ How many different 2-element subsets can be duplication
in this section that you will make a distinction formed out of the set {1,2,3,4,5,6,7} subject to
A TIME. In a involved in
permutation between a permutation problem and a the restriction that (2,5) is not a valid subset?” permutation.
situation, if we combination problem simply by paying attention Number of
to whether any role-definition or ordering is combinations =
are to seat 3 We have to find out how many different 2- Number of
people, we will specified in the problem or not. element subsets we can form out of 7 numbers permutations
seat them in 3 and exclude 1 for the invalid subset. divided by the
different seats. In Example of a permutation problem: factorial value of
the combination the number of
situation, all The problem is one of finding a number value people chosen at a
“How many different 3 member teams can be for the expression: 7C2 - 1. time.
three will occupy
selected from 5 people so that one person chosen Let us say that the
the same seat or problem is one of
the same box. will be the president, the next secretary, and the The number value for 7C2 is: choosing a group
PERMUTATION: third treasurer?” 7! = 7•6•5! = 21 of 3 from 7
people, and the
2!•5! 2•1•5! question is how
You can clearly see that the chosen people will Therefore, the number of valid 2-element many different
have roles to play. subsets is 21 – 1 = 20. And that is the answer. ‘teams of 3’ can
A B be made?
If the role definition is missing or ordering is If we started out
When you see the phrase “at least” as a permutation
missing you must treat the problem as a specified in a combination problem, we problem, the first
C combination problem. must deal with this aspect of the problem person can be
COMBINATION
by eliminating the undesired combinations chosen in 7 ways,
In the permutation from the total of all combinations. Let us the second in 6,
A the
situation, B Corder of Example of a combination problem: see how this works in the following and the third in 5
selection is example. ways for a total of
important. For 7•6•5 = 210
example, we can “How many different r member teams can be Example 3: teams of 3. But
choose to seat A first, selected out of n persons?” “How many 3-member rowing this number
B next, and C last, or involves
B first , A next, C teams can be selected from the
duplication
last and so on giving As a rule, r people can be selected at a time out pool of 4 men and 5 women so because it
rise to 6 different of n people in
permutations. In the that each team will have at least considers ABC to
Combination story, it n Cr = n! 1 woman in it?” be a different
is not material who team from ACB
was selected first as r!(n-r)! or from BAC. In
long as A, B, and C We notice that the restriction is with respect to order to make the
are together in a box. “all-men” teams. correction
How many games
Example1: required to annul
must be scheduled if “How many different 3 member teams can be Let us find out how many 3-member teams can the duplication,
5 teams play in a selected out of 5 people?” be formed out of 4+5 = 9 people without any we need to divide
tournament and each restriction. 210 by 3! To get
game is played the number of
between two teams? The problem is one of finding a number value No restriction number of teams = 9C3. Non-duplicated
This is a combination for the expression 5C3. The number value for 9C3 is 9!/3!•6! = teams of 3 as
problem because
there are 2 teams on
(9•8•7)/3•2 = 84 teams. 210/6 = 35.
the field at a given 5C3 = 5! Now, let us deal with the restriction: “all-men” Remember:
time. It does not teams are forbidden.
matter whether
3!•(5-3)! Combination is
Yankees were chosen The total of 84 teams includes 4C3 or 4 all-men non-duplicated
permutation or
to Diamondbacks or = 5.4.3! = 10 teams. If we subtract 4 from 84, we get 80 as a permutation
the other way around,
as long as Yankees 3!•2! number for 3-member teams with at least one adjusted for the
play the D-backs. woman in them. multiplicity or
for the ordered
We can make 10 different 3-member teams arrangements.
out of 5 people.

In the GMAT, you should expect combination


problems specified with a restriction or an
Some of the materials used in this module are taken from disclosed editions of GMAT, and no monetary
value is included in the cost of this module for supply of these materials. These materials are strictly used
for illustration of concepts discussed. Page 131
©Educational Training Services, A Division of Maple Leaf International Consulting, Inc.
Not to be reproduced, copied or distributed. Intended for use only by the registrant.

Example 4: We can combine 4 odd integers out of 4 odd


“If a 3-member flight crew is to be integers in just one way: We pick all of them.
chosen from a pool of 6 experienced
and 5 amateur astronauts, what is the We can choose 2 odd integers out of 4 odd
probability that an all-amateur team
integers in 4C2 or 6 different ways.
will be selected if each team that can
be formed is as likely to make it to
And, for each of these 2-value selections of odd
the mission as any other?” integers, we can choose 2 more even integers in
3C2 or 3 different ways, for a total of 6•3 = 18
ways.
We have to find out the total number of
3-member teams that can be formed from Therefore, the total number of ways we can
6+5 = 11 astronauts, and also find out combine 4 integers so that the sum will be even
the number of all-amateur teams included is 1 + 18 = 19 ways.
in the total, and take the ratio.
Number of 3-member teams = 11C3 = 165
Number of all-amateur 3-member teams See if you can compute the
= 5C3 = 10 number of different ways we
can combine 4 integers from
Probability of an all-amateur team being
{1,2,3,4,5,6,7} so that the sum
chosen = 10/165 = 2/33. (Answer) will be an odd value as 16 ways.
Example 5:
Example 7.
“In the above problem, how many
different 3-member crews can be
selected subject to the restriction that “There are six people in a room and each
each team have at least one person shakes hands with each of the
experienced astronaut in it?” others exactly once. How many
The problem tells us that we have to exclude “all handshakes are exchanged?”
amateur” teams from the total number of 3-
member teams that can be formed. The above problem is the same as asking
you how many different ways can we put
We have to get a number value for 11C3 – 5C3 together a team of 2 people out of 6
= 165 – 10 = 155.
people because when 2 people shake
155 teams will have at least one experienced hands, we are dealing with 2 people at a
astronaut in them. time out of the six people in the group.

Example 6: We also notice that the order of


handshakes is not important or material
“How many different ways can we choose 4 here. For example, A shakes hands with
distinct integers from (1,2,3,4,5,6, and 7) so that
their sum will be even?” B or the other way around will still count
Notice that there are 4 odd integers and 3 even as one handshake.
integers in the set. You can get an even number
as the value for the sum of integers by We must treat this as a combination
combining problem. We are choosing 2 at a time out
• all 4 odd integers; or by combining of 6 people.
• 2 odd and 2 even integers;
We cannot combine 3 odd and 1 even or 3 even
and 1 odd integers, because the sum will not be No of handshakes is 6C2 = 6! /2!•4! = 15.
even.
Some of the materials used in this module are taken from disclosed editions of GMAT, and no monetary
value is included in the cost of this module for supply of these materials. These materials are strictly used
for illustration of concepts discussed. Page 132
©Educational Training Services, A Division of Maple Leaf International Consulting, Inc.
Not to be reproduced, copied or distributed. Intended for use only by the registrant.

If there are 10 people in a room and each TEAM NUMBER OF


one shakes hands with each of the others GAMES WON
exactly once, there will be 10C2 or 45 A 4
handshakes. You get the idea here, don’t B 7
you? C 9
The same problem can be presented as D 2
scheduling of games problem. Take a E 2
look at the one below: X ?
“10 teams are to play in a
tournament, and each team must
According to the incomplete table above, if
play in a game each of the other
each of the 6 teams in the league played each
teams exactly once. How many
of the other teams exactly twice and there
games need to be scheduled?”
were no ties, how many games did team X
win? (Only 2 teams play in a game)
We notice that this is a problem of
choosing 2 teams at a time out of 10 (A) 4 (B) 5 (C) 6 (D) 8 (E) 10
teams. Number of games to be scheduled
6 teams to choose from and we need to choose
= 10C2 = 10!/2!•8! = 10•9/2 = 45 two teams at a time to play in a game. And there
games. is a “home” game and an “away” game. Total
number of games played:
IF the problem specified that each team
2• 6C2 = 2• 6!/2!4! = 2•(6.5)/2 = 30 games
must play each of the other teams exactly There are a total of 30 games played. This
twice, then we double 45 and get 90 as means 30 wins and 30 losses, because in each
the number of games that we need to game one team wins and the other loses. The
schedule. If the teams played each other total number of games won by teams A through
three times, we will multiply 45 by 3 to E is 24. Team X must have won the remaining
six games. We must select choice C.
get 135 games, and so on.
COMBINATION is PERMUTATION adjusted for
the illogical repetition of the same combination in
different order. We have seen how to use a formula.
WE can also use an approach that treats the
NUMBER OF COMBINATIONS as equal to
(NUMBER OF PERMUTATIONS) DIVIDIDED BY
THE (NUMBER CHOSEN)!. If the problem requires
that we compute the number of different teams of 3
that can be made using 6 people, then the number of
permutations of 6 in 3 positions is 6 X 5 X 4.
Because permutation involves repetition of the same
group in different order, we need to divide this by 3!
To adjust for this illogical repetition. Therefore, the
number of teams of 3 using 6 people is
(6X5X4)/(3X2X1) = 20 teams.

Take a look at another COMBINATION


problem.
LEAGUE RESULTS

Some of the materials used in this module are taken from disclosed editions of GMAT, and no monetary
value is included in the cost of this module for supply of these materials. These materials are strictly used
for illustration of concepts discussed. Page 133
©Educational Training Services, A Division of Maple Leaf International Consulting, Inc.
Not to be reproduced, copied or distributed. Intended for use only by the registrant.

MORE PROBLEMS IN COMBINATION AND


PERMUTATION
PROBLEM 2:
PROBLEM 1: PROBLEM 2:
“There are 10 qualified tennis players in a group. If
“A Pizza Shop allows its customers to choose a pizza the coach wants to schedule ‘doubles’ games in
with the same number of toppings as the number of which a team of 2 players plays another of 2 players,
cheese types chosen. If the shop has 4 different types how many different games can be scheduled?”
of toppings and 2 different types of cheese, how many
different pizza combinations can be made subject to . You should conceptualize the problem before you
the above restriction?” begin to attack it. We need to do the following:
1. We need to decide how many different teams of
The problem tells us that a customer can choose a 2 people can be made using 10 available people.
pizza with ONE topping and ONE type of cheese, OR 2. Then, decide how many different ‘games’
one with TWO toppings and TWO types of cheese. involving two teams of two players each must be
scheduled. We must then adjust for the ‘double
OPTION 1 count’ of the same game by dividing the total by
CHOOSE A PIZZA WITH ONE TOPPING AND 2.
ONE TYPE OF CHEESE. Let us take care of the first step first.
We can make teams of 2 people using the available
A customer can choose 1 topping out of 4 available pool of 10 people in 10•9/2•1 = 45 different ways.
toppings in 4 ways; 1 cheese out of 2 in two different
ways. A customer can choose 1 topping AND 1 Now we need to proceed to the second step.
cheese type in 4 times 2 = 8 different ways. This
means that a customer can order 8 different pizzas We can choose any ONE of the 45 different teams of
each containing a different combination of one 2 players to be one of the two teams. Because the
topping and one cheese. people chosen to be on one team cannot be in an
opposing team, we must decide how many different
OPTION 2 teams of 2 can be made using the remaining 8 people.
CHOOSE A PIZZA WITH TWO TOPPINGS We can make teams of 2 using the remaining 8
AND TWO CHEESES people in 8•7/2•1 = 28 different ways.

A customer can choose 2 toppings out of 4 toppings in Therefore, we can pick ONE team from the original
4•3/2•1 = 6 ways . Notice that this is a combination pool of 45 teams in 45 different ways. And Any one
because we made a ‘team’ of two toppings on the of these chosen teams can play any of the 28 teams of
same pizza. 2 ‘other’ players in 45 times 28 = 1,260 different
ways.
Likewise, the customer can choose 2 cheeses out of
the available 2 types in 1 way. But then, we must recognize that this number of
1,260 involves double counts of a game. For example,
A customer can choose 2 toppings and 2 cheeses in 6 AB playing CD is counted as a different game from
times 1 ways or 6 ways. CD playing AB. Therefore, we must divide the value
of 1,260 by 2 to get 630 as the number of different
A customer can order 8 different games that could be scheduled
combinations of 1 topping and 1 cheese; and
additionally order 6 different combinations of We could have also obtained the same result by doing
2 cheese and 2 toppings for a total of 8+6=14 the following: Because a game involves 4 players, 2
different pizza combinations. on each side, we can compute the number of different
selections of 4 players from 10 as 10C4 = (10!)/[4! 6!]=
210. Within a group of 4, we can create 3 different
games: For example, if the group comprises [A, B, C,
D], we can create three different games as follows:
AB PLAYS CD; AC PLAYS BD; AD plays BC.
Therefore, the total number of different ‘doubles’
games that can be scheduled by using 10 players is 3
times 210 = 630.

Some of the materials used in this module are taken from disclosed editions of GMAT, and no monetary
value is included in the cost of this module for supply of these materials. These materials are strictly used
for illustration of concepts discussed. Page 134
©Educational Training Services, A Division of Maple Leaf International Consulting, Inc.
Not to be reproduced, copied or distributed. Intended for use only by the registrant.

PROBLEMS IN CONDITIONAL As you can see, either approach gives us the same
result. Let us take a look at a few more conditional
PROBABILITY USING probability problems in which either approach is
PERMUTATION OR valid and will produce the same result.
COMBINATION APPROACH.
PROBLEM 2:
Conditional probability is unaffected whether the
problem is treated as one involving ORDER or as one A roller-coaster has 3 cars – A, B, and C. If
not involving order. Alisha rides the roller-coaster on three
different occasions and if she does not have
If the problem specifies that we need to pick specific preference for any of the three cars,
two machines out of a group of 6 machines what is the probability that she will ride in
in which 2 are defective, and requires us to each of the three cars during her three
compute the probability that we picked ONE different rides?
good machine and ONE bad machine, then the
computed probability will be unaffected whether APPROACH 1:
consider the selection of two machines as On her first visit, Alisha must ride any one of the
SIMULTANEOUS or as SEQUENTIAL involving three cars. Therefore, it is a certainty that she will use
order. one of the cars on her first visit. The probability that
she will use one of the three cars on her first visit is,
LET US EXPLAIN HOW THIS IS SO. therefore, 1. On her second trip back to the park, she
must not ride in the car that she rode the first time.
APPROACH 1: WE TREAT THE PROBLEM AS The probability that she will ride in one of the two
ONE IN WHICH TWO MACHINES ARE remaining cars is 2 out of 3 or 2/3. On her last trip,
PICKED AT A SINGLE TIME. (COMBINATION she must ride in the one car she has nor ridden in on
APPROACH). her previous two trips. The probability that she will
ride in the sole remaining car that was not used on
Number of selections of 2 machines out of 6 is the prior two trips is 1/3. Therefore, the probability
6! / 2!(6-2)! = 6!/2! 4! that she will ride in each of the three cars on her
= (6 X 5 X 4!)/2! 4! = 15 ‘teams’ of 2 three different trips is 1 times 2/3 times 1/3 or
2/9.
Number of selections of 2 machines in which one is
good and the other bad is 4C1 X 2C1 = 4 X 2 = 8 teams APPROACH 2:
of two machines in which one is good and the other We can also deal with the problem by considering the
bad. different ORDER in which she can ride the three
cars. She can ride them in any of the following 6
Therefore, the probability that a ‘team’ of 2 machines different ordered ways: ABC, ACB, BAC, BCA,
in which one machine is good and the other bad is CAB, or CBA. The probability that she will ride Car
selected out of 15 different ‘teams’ of 2 machines is A first, B next, and C third is 1/3 times 1/3 times 1/3
8/15. = 1/27. This is the same probability for each of the
remaining 5 different ordered ways in which she can
APPROACH 2: WE TREAT THE PROBLEM AS ride the three cars. Therefore, the probability that she
ONE IN WHICH MACHINES ARE PULLED IN will ride in each of the three cars during her three
DIFFERENT ORDER – BAD MACHINE different trips is 6 times 1/27 or 6/27 or 2/9.
FOLLOWED BY GOOD MACHINE or GOOD As you can see, both approaches will produce the
MACHINE FOLLOWED BY BAD MACHINE. same result.

We need to compute the probability of BAD followed


by a good machine, or Good one followed by a bad
machine selection. The probability that we will pick a
bad machine first and then a good machine is 2/6
times 4/5 = 4/15.
The probability that we will pick a good machine first
and then a bad machine is 4/6 times 2/5 = 4/5.
Because we can do the selection of two machines in
two different order, we need to add the probability of
the two events and get 4/15 + 4/15 = 8/15.

Some of the materials used in this module are taken from disclosed editions of GMAT, and no monetary
value is included in the cost of this module for supply of these materials. These materials are strictly used
for illustration of concepts discussed. Page 135
©Educational Training Services, A Division of Maple Leaf International Consulting, Inc.
Not to be reproduced, copied or distributed. Intended for use only by the registrant.

Consider another problem in which we can use either


a combination approach or a permutation approach to A group consists of 7 boys and 6 girls. If
arrive at the same result.
a team of 2 is to be randomly selected for
PROBLEM 3: an expedition, what is the probability that
a team comprising one boy and one girl
A box contains 15 Red balls, 20 blue balls, will be chosen, if all the boys and girls
and 15 green balls. If two balls are drawn are qualified to be on the team?
from the box without replacement, what is
the probability that exactly one blue ball and APPROACH 1: COMBINATION APPROACH
one green ball were picked?
The number of teams of 2 people chosen from 7 + 6=
APPROACH 1: 13 people is (13 X 12)/(2X1) = 78 teams of 2

We consider that the two balls were selected ‘at a The number of teams of 2 people in which there is
time’. This is how we will deal with a combination exactly one boy and one girl is 7C1 times 6C1 = 7X6 =
problem – selection AT A TIME. 42.

WE can see that there are 50 balls in the box, and the Therefore, the probability that the randomly selected
number of 2 ball ‘teams’ chosen from 50 is (50 X 49) team will have exactly one boy and one girl in it is
divided by 2! Or 1225 different selections of 2 balls 42/78 or 21/39 or 7/13.
out of 50 balls.
APPROACH 2: ORDERED SELECTION
Number of 2-ball ‘teams’ in which there is exactly
one BLUE ball and one GREEN ball is 15 times 20 The first member of the team could be a boy and the
or 300 selections of 2 balls of specific color as next a girl, or vice versa. The probability that we
required. will choose a boy first and then a girl to be on the
team is 7/13 times 6/12 = 7/26.
Therefore, the probability that the set containing one
green and one blue ball was picked is 300/1225 or We can also reverse the order of selection and choose
12/49. a girl first followed by a boy. The probability that we
will pick a girl first followed by a boy is 6/13 times
APPROACH 2: 7/12 = 7/26.

We can pull the two balls in two different order: Therefore, the probability of picking a team in which
BLUE FIRST AND THEN GREEN there is exactly one boy and one girl is the sum of the
Or two probabilities of the two different ordered
GREEN FIRST AND THEN BLUE. selections. The answer is: 7/26 + 7/26 = 14/26 =
7/13.
If we pull green first and then blue, the probability of
this occurrence is 15/50 times 20/49 = 6/49. Two different approaches producing the same result.

If we pull BLUE first and then green, the probability On the following page, we will take a look at a few
of this occurrence is 20/50 times 15/49 = 6/49. more conditional probability problems that are typical
of the ones that appeared on the recent GMAT/GRE
Because we can select two balls in two different tests.
order, we need to add the probabilities of the two
different ordered scenarios to compute the total
probability of the desired event. Therefore, the
probability that one blue ball and one green ball will
be picked in any order is 6/49 + 6/49 = 12/49.
The approach is not critical as long as we engage in
sound reasoning.

CONSIDER ANOTHER PROBLEM and solidify


your understanding of Conditional probability.

PROBLEM 4:

Some of the materials used in this module are taken from disclosed editions of GMAT, and no monetary
value is included in the cost of this module for supply of these materials. These materials are strictly used
for illustration of concepts discussed. Page 136
©Educational Training Services, A Division of Maple Leaf International Consulting, Inc.
Not to be reproduced, copied or distributed. Intended for use only by the registrant.

PROBLEM 5 Peter goes on a camping trip on Friday


evening and will return from the camp-
A gardener is planting two red rose
ground at the end of the day it rains on
bushes and two white rose bushes in a
the camp ground. If the probability that it
row. If the gardener is to select each of
will rain on the camp ground on any day
the bushes randomly, and one at a time,
is 1/5, what is the probability that Peter
what is the probability that he will plant
will not return from the camp-ground
both red rose bushes in the middle of the
until the END OF the day on the
row?
following Monday?
The desired order of planting is W – R –
R – W. According to the problem, Peter should not
return on Saturday or on Sunday but must
return on Monday. Accordingly, it must not
The probability that the first bush is
rain on Saturday or on Sunday but must rain
White is 2/4. on Monday on the camp-ground.
The probability that the first bush is
white and the second a Red is 2/4 times The probability that it will not rain on the
2/3 = 1/3. camp-ground is 1/5. Therefore, the
probability that it will rain on the camp-
The probability that the first is white, the ground on any day is 4/5.
second red, and the third red is 2/4 times
2/3 times ½ = 1/6. The required probability is obtained by the
following operation:
The probability that the first bush is
white, the second a red, the third also PROBABILITY THAT IT WILL NOT
red, and the last a white is 2/4 times 2/3 RAIN ON SATURDAY TIMES
PROBABILITY THAT IT WILL NOT
tmes ½ times 1 = 1/6. (If we have
RAIN ON SUNDAY TIMES
planted one white and two red already,
PROBABILITY THAT IT WILL RAIN ON
then the probability that the last one will MONDAY = 4/5 x 4/5 x 1/5 = 16/125 =
be a white is a certainty having a 0.128
probability of 1).

The required probability is 1/6.

PROBLEM 6:

Some of the materials used in this module are taken from disclosed editions of GMAT, and no monetary
value is included in the cost of this module for supply of these materials. These materials are strictly used
for illustration of concepts discussed. Page 137
©Educational Training Services, A Division of Maple Leaf International Consulting, Inc.
Not to be reproduced, copied or distributed. Intended for use only by the registrant.

What are some of the other mutually exclusive


events?
Males and Females,
We are going to be dealing
Smokers and non-smokers,
with mutually exclusive Rented and not-rented rooms in a
events, and independent hotel,
Yes and No answers to a question,
events when we deal with Pass and Fail a test, and so on.
statistical events..
We will represent the independent events by
What are mutually exclusive events? intersecting circles.

Let us say we toss a coin. We have two Math


possible events: heads and tails. When one
coin is tossed, heads and tails are mutually
exclusive because if “heads” occur, then the
“tails” do not. Two events are said to be Physics
Chemistry
mutually exclusive when if one occurs, then the
other does not.

What are independent events?

In the set of independent events shown in the venn


Independent events are those that diagram above, a person can take Math only, or
do not depend on the occurrence Physics only, or Chemistry only, or can take any two
subjects, or all three subjects.
or non-occurrence of the other
events for their own occurrence. Probability of mutually exclusive events adds up to
Let us say that we toss two coins. 1.0. For example, if the Probability of Heads is 0.5,
and that of Tails is 0.5, then the total probability of
Then the heads and tails are no these mutually exclusive events must add up to 1.0
longer mutually exclusive, but are (Later on in this section, we will see how we can deal
independent. This means that we with two sets of mutually exclusive events by setting
up a table. For example, let us say that we have two
can have heads occurring mutually exclusive items: Males and Females, and
together and tails occurring each of these items can be one of the two mutually
together on the two faces exclusive items: Smokers and Non-smokers. We will
deal with the above situation by setting up a table
showing up.
along the following lines:
Males Females Total
We will represent mutually exclusive
events by two circles that do not Smokers
intersect.
Non-
smokers
Total

The sum of the probabilities of independent


Heads Tails
events will be greater than 1.0 if they overlap.

Some of the materials used in this module are taken from disclosed editions of GMAT, and no monetary
value is included in the cost of this module for supply of these materials. These materials are strictly used
for illustration of concepts discussed. Page 138
©Educational Training Services, A Division of Maple Leaf International Consulting, Inc.
Not to be reproduced, copied or distributed. Intended for use only by the registrant.

Sets and Venn Diagrams Approach 2: Venn Diagrams


Some wise guy said that a picture is worth a
thousand words. We would concur with that
aphorism and go one step further and state that
If is a set of numbers 1,2,3,4,5,.....,n, “one good circle deserves another.”
then we define the set S as follows:
S ={1,2,3,4,.....n} Two intermeshed circles, dividing the
graduating class into three sets, become a Venn
Venn diagrams describe the relationship Diagram, after the guy who invented it.
among the members of a set by circles. The Venn Diagram for our problem will look
like this:
Let us consider a simple problem dealing Marketing Both Finance
with a set of numbers:

22-n n 18-n
“ Each of 30 students in a graduating
class were enrolled in marketing, finance \
or both. If 22 were enrolled in marketing,
18 in finance, how many were enrolled in The number of students enrolled in
both subjects?” marketing alone was 22-n. The figure for
finance was 18-n. The figure for “both”
There are two approaches to solving a was n. Since the Total cannot be more
problem of this type: than 30, 22-n + 18-n + n = 30, or n =10.

Approach 1: Marix approach to sets


we will try to solve this problem problems.
without the Venn Diagrams. Let us say that we have two genders: Males and
Females, and two sub-classifications: Smokers
Let “n” represent the number of people and Non-Smokers.
Or that we have two questions in a survey: Q1
enrolled in both Marketing and Finance.
and Q2, and two possible responses to each
question: Yes and NO.
Marketing enrollment of 22 includes “n”. Or that we have two types of rooms: air-
Finance Enrollment of 18 includes “n” as conditioned and non-air-conditioned rooms,
well. and each type of room could be Rented or
Vacant.
Since we know that the total size of the
class is 30, we construct the following You will deal with problems involving two sets and
two sub-sets using a table or matrix approach. An
equation: example of such a problem is discussed next. You
(22-n) + (18-n) +n = 30 will see that these “matrix” problems involve
“plugging” in the values provided, and computing the
values for the remaining cells through a process of
Solving for “n”, we get n=10. simple addition and subtraction:
The values in the cells along a row must add up to
give the row totals, and those in the cells along the
columns must add up to give the column totals.

Some of the materials used in this module are taken from disclosed editions of GMAT, and no monetary
value is included in the cost of this module for supply of these materials. These materials are strictly used
for illustration of concepts discussed. Page 139
©Educational Training Services, A Division of Maple Leaf International Consulting, Inc.
Not to be reproduced, copied or distributed. Intended for use only by the registrant.

Let us illustrate what we mean by that


with an example. We pick E as the correct answer choice.
Example 2:
“A survey was conducted among a sample size
“One night a certain motel rented 3/4 of its of N persons. 1/4 answered Yes to Question 1
rooms, including 2/3 of its air-conditioned and 1/3 of those who answered Yes to Question
rooms. If 3/5 of its rooms were air-conditioned, 1 answered Yes to Question 2. Which of the
what percent of the rooms that were not rented following expressions represents the number of
were air-conditioned?” people interviewed who did not answer “Yes” to
both questions?”
The best way (and probably the only way) to
deal with this problem is as follows: (A) N/7 (B) 6N/7 (C) 3N/12 (D) 7N/12
Let us say that the motel had 100 rooms. (E)11N/12
The motel rented 3/4th of its rooms, or 75 Although you could answer this question without
setting up a table/matrix, you may want to do so
rooms. By inference, we can conclude that the
because you will get a better perspective on the
Motel did not rent 1/4 of its rooms, or 25 rooms. information specified in the problem. Later on in the
3/5th of the motel rooms are air-conditioned, or assignment, you will come across an extension of this
60 are air-conditioned. problem with additional information, and you will
By inference, 2/5th of its rooms, or 40, are not have to set up a matrix before you can make sense of
air-conditioned. all the details provided.
We recognize that there are two sets of mutually
Sets A/C Non A/c Total exclusive categories of information: Yes and No
answers to questions 1 and 2. Remember that Q1 and
Rented 75 Q2 are not mutually exclusive because the respondent
Vacant 25 is required to answer both.
Total 60 40 100
We will set up a matrix with the mutually exclusive
rd values Q1Yes and Q1No as row headings and with
The problem states that 2/3 of its air-
Q2Yes and Q2No as column headings. (We can set
conditioned rooms were rented. This means that them up the other way around too, as long as we set
2/3rd of 60, or 40 of the motel’s rooms were mutually exclusive values along the rows and
rented. That leaves 1/3rd or 60, or 20 rooms Air- columns.)
conditioned and not rented. Q2Yes Q2No Total
We complete the rest of the cells in the table as
1
follows: Q1Yes /3•¼N ¼N
=1/12 N
Sets A/C Non A/c Total Q1No ¾N
Rented 40 35 75
Total N
Vacant 20 5 25
Total 60 40 100
We are given that ¼th of N people said Yes to Q1. By
inference, ¾th of N people said No to Q1.
Let us take a look at the question now: We are also told that of the ¼ N people answering
Yes to Q1, 1/3rd also said Yes to Q2. That gives us
“What percent of the rooms that were not rented
1/12 of the people interviewed answering yes to both
were air-conditioned?” Q1 and Q2.

The question is: What percent is 20 of 25? The question is: How many people did not answer yes
The answer is 20/25 •100% or 80%. to both questions?
That is, 80% of the not-rented rooms were
air-conditioned. And that is the answer. # not answering yes to both questions =
N - # answering yes to both q’s.
Let us take a look at the answer choices: = N – 1/12 N = 11/12 N.
Choice E.
(A) 20% (B) 331/3% (C) 35% (D) 40%
(E) 80%
Some of the materials used in this module are taken from disclosed editions of GMAT, and no monetary
value is included in the cost of this module for supply of these materials. These materials are strictly used
for illustration of concepts discussed. Page 140
©Educational Training Services, A Division of Maple Leaf International Consulting, Inc.
Not to be reproduced, copied or distributed. Intended for use only by the registrant.

We recognize that this is a “table” problem


Practice Problems involving Sets because we have two sets of variables, and
proceed to set up the matrix as below:

Seen X Not seen Total


X
Seen Y ???? 75%
Not Seen 25% 25%
Y
Total 40% 60% 100%

Since 75% of the members have seen


the movie Y, we can compute that
25% must have “not seen Y”.
Minimum value for “Seen X-Seen Y”
The figure above shows enrollment in the is obtained if the entire 25% who have
three clubs at a local college. How many not seen Y have seen X.
students are enrolled in just one club? We conclude that 40% - 25% = 15% of the
members must have seen both movies, at
We notice that there are 10 students enrolled least. In fact, the percentage of members who
in all three clubs. How do we get the number have seen both movies lies in the range 15%
for Math only? to 40%, with 15% as the minimum value.
We notice that 17 students are enrolled in
math and Drama; and 14 in Math and
Science; and 10 in all three clubs for a total of Note: We can get the answer by determining by
17 + 10 + 14 = 41. We subtract this number what percentage the total of X-seen and Y-seen
from 68, the total number of students taking exceeds 100%. The total of X-seen and Y-seen
math, to get the number for “math only”. We is 115% or 15% more than 100%. This tells us
get : 68-41 = 27 students for math only. that at least 15% must have seen both movies.
Similarly, How do we get the number for
Drama only? We notice that 17 are taking Remember: When we have two
drama and Math; 21 are taking drama and independent sets, the percentage
science; and 10 are taking all three for a total value by which the total of the
of 17+10+21=48. We subtract this number sets exceeds 100% must give us
from 79, the total number taking drama to the value for the overlapping
get: 79 - 48 =31 for the number taking drama area.
only.
Similarly, we can determine that the number Example:
taking Science only is 87 - (21+10+14) = 42. “In a town of 5300 residents, 3900 people own
pets. The number of people owning cats is
We have 27 students taking Math only; 31 2,970, and the number of people owning dogs is
taking Drama only; and 42 taking Science 1,530.
only for a total of 100 students participating How many people own a cat and a dog?”
in just one activity.
We notice that the total of people owning dogs
Example 2: and cats is 4,500 and it exceeds the number of
“40% of all members in a club have seen pet owners by 4500 – 3900 = 600.
movie X; 75% of all members have seen
movie Y. At least what percent of the We conclude that 600 folks have both dog and
members must have seen both movies?” cat for pets.

Some of the materials used in this module are taken from disclosed editions of GMAT, and no monetary
value is included in the cost of this module for supply of these materials. These materials are strictly used
for illustration of concepts discussed. Page 141
©Educational Training Services, A Division of Maple Leaf International Consulting, Inc.
Not to be reproduced, copied or distributed. Intended for use only by the registrant.

EXAMPLE:

“If x@y denotes the number of people owning x


only or y only but not both, what is the value of EXAMPLE:
x@y if the number of people owning x is 450,
the number of people owning y is 560 and the Of the 32 applicants for a job,
total people in town is 900? (People in town own 15 had at least 5 years of
either x or y or both.)” experience, and 18 had graduate
degrees. If 4 applicants had
We notice that x@y denotes the excess over the neither the 5 year experience nor
total population of x and y. a degree, how many had both 5
years of experience or better and
x and y add up to 1010, and the excess over 900 a degree?
is 110. Therefore, the value of x@y is the value
for X only + the value for Y only = (450 – Once again, we can deal with this problem by
110)+ + (560 – 110) = 340 + 450 = 790. (X using a matrix because we see two mutually
only is computed by subtracting from the exclusive sets of categories:
population of X the value that belongs to both X
and Y; ditto for Y only) 5+ Exp. 5+ Exp. TOTAL
YES NO
EXAMPLE: DEGREE 5 13 18
YES
“Out of 65 cars in a lot, 25 DEGREE 10 4 14
have air-conditioning and 45 NO
have power windows. If 20 TOTAL 15 17 32
cars have both power
windows AND air- The values in the gray band are derived on the
conditioning, how many cars basis of the other values in white cells, values
in the lot have neither air- specified in the problem.
conditioning nor power
windows?” Therefore, the number of people having BOTH
experience and a degree is 5 among the
applicants.
We notice that there are two sets of mutually
exclusive categories, and we can deal with this EXAMPLE
problem by setting up a matrix. If A@B denotes the number of items that are in
A/C YES A/C NO TOTAL A only or in B only but not in both, and A
contains 130 items, and B contains 75 items,
P/W 20 25 45 and if there are 25 items common to both A and
YES B, how many items are in A@B?
P/W 5 15 20
NO A@B = Items in A only + Items in B only
TOTAL 25 40 65 If A contains 130 items and 25 are common to
both A and B, items in A only = 130- 25 = 105
If B contains 75 items, and 25 are common to
The values in the gray band are the values both A and B, items in B only = 75 – 25 = 50
derived from the information specified in the Therefore, A@B = Items
problem. We can see that 15 cars have neither in A only + Items in B
A/C nor Power windows. only = 105 + 50 = 155.

Some of the materials used in this module are taken from disclosed editions of GMAT, and no monetary
value is included in the cost of this module for supply of these materials. These materials are strictly used
for illustration of concepts discussed. Page 142
©Educational Training Services, A Division of Maple Leaf International Consulting, Inc.
Not to be reproduced, copied or distributed. Intended for use only by the registrant.

Word Problems Involving Geometry 1. Northern Region’s performance in Quarter 3


fell short of its target by what percent?
A B The Figure to the left is a 2. Which region shows the most fluctuation
C D piece of land. All angles over 4 quarters in performance (target
h=200 E F are right angles. What is achievement)?
perimeter of the land? 3. Which region shows the most stable
H G performance in terms of target achievement?
L=200
You will find that questions of this type test your
ability to read a chart, or a graph, or a table and
Can you recognize that the piece of land obtain the relevant information. Piece of Cake.
will have the same perimeter as if it were
a square piece of land with sides 200 m?

Since the angles are all right angles,


AB + CD + EF = GH
BC + DE + FG = AH

Perimeter = 2(AH + GH) = 2(200+200) = 800 meters.

DATA INTERPRETATION

GMAT Problem Solving section might include


questions involving interpreting data based on a
table, graph or a chart.

90
80
70
60
50 East
40 West
30
North
20
10
0
1st 2nd 3rd 4th
Qtr Qtr Qtr Qtr

The performance of three regional divisions of


a company is shown for the 4 quarters of 1995.
The figures in Y axis represent percentage
achievement of sales targets. Based on the
information presented in the graph, answer the
following questions:

Some of the materials used in this module are taken from disclosed editions of GMAT, and no monetary
value is included in the cost of this module for supply of these materials. These materials are strictly used
for illustration of concepts discussed. Page 143
©Educational Training Services, A Division of Maple Leaf International Consulting, Inc.
Not to be reproduced, copied or distributed. Intended for use only by the registrant.

false. (Read Condition I to say “x must


be equal to y”).

We can quickly conclude that condition


Must-be-true problems that you will II is false as well because, as we have
encounter in the GMAT problem-solving shown in the previous paragraph, y does
section require you to make the most not have to be equal to 1 to satisfy the
sense of the given information, consider requirement that 3x/y be a prime number
an exception, and then answer the greater than 2. (Read condition II to say
question. A “must-be-true” condition is “y must be equal to 1”).
valid under all circumstances and if you
can think up an exception, you will be We can eliminate condition III just as
ready to test the conditions. easily because 3x/y can be a prime
number greater than 2 for values of x=21
and y = 9. None of these values
“Must-be-True” problem takes the represents a prime number. (Read
following form: condition III to say: “x and y must be
prime numbers”).
“ If y ≠ 3 and (3x)/y is a prime integer
greater than 2, which of the following We have concluded that conditions I, II
must be true?” and III need not be true.

I. x = y II. Y = 1 III. x and y are What is the answer choice? None of the
prime integers. above. We pick and mark (A) as the
correct answer.
(A) None. (B) I only (C) II only
(D) III only (E) I and III
Let us do another question to be more
comfortable with “must be true”
The best and probably the only way to questions:
approach this type of problem is to test
each of the conditions against “real-life” “If x and y are integers and xy² is a
situations. Also note the word “must”. It positive, odd integer, which of the
will not be highlighted or appear in bold following must be true?”
in the test.
I. xy is positive
3x/y is a prime integer greater than 2. II. xy is odd
Which means that it could be 3, 5, 7, 11, III. x+y is even.
13, 17, etc.
(A) I only (B) II only (C) III only
Let us pick a prime number: 7. (D) I and II (E) II and III

The expression 3x/y can produce 7 for This question tests our knowledge of
the following values of x = 21 and y=9. properties of integers - particularly odd and
even integers. (Discussed at the outset of this
Therefore x does not have to be equal to module).
y. So we conclude that the condition I is Given: xy² is a positive, odd integer.
Some of the materials used in this module are taken from disclosed editions of GMAT, and no monetary
value is included in the cost of this module for supply of these materials. These materials are strictly used
for illustration of concepts discussed. Page 144
©Educational Training Services, A Division of Maple Leaf International Consulting, Inc.
Not to be reproduced, copied or distributed. Intended for use only by the registrant.

What do we know about odd integers?


(A) I only (B) II only (C) III only
For xy² to be a positive, odd integer, the
(D) I and II (E) II and III
following conditions must be met:

1. x must be positive and odd. From the relative positions of q, r, s, t,


2. y² must be (and will be) positive and must and u, we can construct the following
be, in addition, odd. line:
This condition requires that y be an odd q r t s? u s?
integer - positive or negative so that the
square of the integer will be a positive and
odd integer. For example, if y is -5, then y² The relative positions of q through u are a given
and they will look like those as shown along the
will be 25, an odd, positive integer. If y is 5,
continuum.
then y² will still be a positive and odd.
Now, let us look at the “must be” statements.
Now Let us look at the “must be” statements:
1. The only given information about s is that it
1. We conclude that the condition I is false
is greater than t. Which means that it occurs on
because xy² will be a positive, odd integer
the continuum after t but does it occur before u
even for negative odd values of y. xy can be
or after u? We do not know from the available
negative and
information. If s occurs before u, which will still
still the requirement that xy² be a positive
satisfy the condition that s be greater than t,
and odd integer can be met.
then condition I will not be met. Therefore the
condition I fails.
2. xy must be odd is a valid and true
requirement. Both x and y must be odd to
2. Conditions II and III must be true as can be
satisfy the requirement that xy² is odd and
seen from the positions of q, r, s and u from the
positive. xy can be odd and negative and still illustration.
satisfy the requirement of the question. II is
true. We pick (E) as the answer choice.
3. Condition III must be true as well because Note: The only “trick” condition in this question
if x and y are both odd , then their sum is is the statement s is greater than t but there is
even. (Remember odd plus odd gives even; nothing given about s and u to define their
odd plus even gives odd; etc.) relative positions.
So. What do we have now? Conditions II and
1. Make as much sense as possible of the given
III must be true to satisfy the requirement
information before you go to deal with the
that xy² be odd and positive. So we pick (E)
conditions stipulated.
as the answer choice. 2. Remember: Must-be-true condition must be
valid for all values of the variables defined.
Let us deal with just one more question Therefore, if you can think of an exception -
before we move on the basics of just one - you know that the condition
Geometry. stipulated is not valid.
3. Do not assume that a number is a whole
number, and positive. Unless the numbers are
specified as such, they could be anything.
“If u>t, r>q, s>t, and t>r, which of the
following conditions must be true?”
I. u>s
II. s>q Go to Assignment on next page.
III. u>r
Some of the materials used in this module are taken from disclosed editions of GMAT, and no monetary
value is included in the cost of this module for supply of these materials. These materials are strictly used
for illustration of concepts discussed. Page 145
©Educational Training Services, A Division of Maple Leaf International Consulting, Inc.
Not to be reproduced, copied or distributed. Intended for use only by the registrant.

(F) x.y is a positive integer. True / False


ASSIGNMENT
Why?___________________________
_________________________________
_________________________________
1. If x+y is a prime number, then
which of the following “must be” true?

(A) x = 2 True / False 2. If x²y³ is a negative odd integer, then


which of the following must be true?
Why?___________________________
_________________________________ (A) y is a negative integer. True/ False.
_________________________________
Why?____________________________
(B) Either x or y is 2. True / False _________________________________
_________________________________
Why?___________________________
_________________________________ (B) x can be a fraction. True / False
_________________________________
Why?____________________________
(C) Both x and y are prime numbers. _________________________________
_________________________________
True/ False
(C) x can be positive or negative, integer
Why?___________________________
or a Fraction. True / False
_________________________________
_________________________________ Why?____________________________
_________________________________
(D) x.y is not a prime number.
_________________________________
True / False
(D) Both x and y are odd numbers -
Why?___________________________ fractions or integers. True / False
_________________________________
Why?____________________________
_________________________________
_________________________________
(E) x-y is a prime number. _________________________________

True / False (E) x² + y³ is a positive even integer.


True / False
Why?___________________________
_________________________________ Why?____________________________
_________________________________ _________________________________

Some of the materials used in this module are taken from disclosed editions of GMAT, and no monetary
value is included in the cost of this module for supply of these materials. These materials are strictly used
for illustration of concepts discussed. Page 146
©Educational Training Services, A Division of Maple Leaf International Consulting, Inc.
Not to be reproduced, copied or distributed. Intended for use only by the registrant.

Assignment Continued

3. Thames Toyota Dealership has 120 4. Machine A produces bolts at a


cars in their lot. The cars come in two uniform rate of 120 every 40 seconds,
colors - Red or Blue and two sizes - mid- and machine B produces bolts at a
size and large. There are exactly 10 large uniform rate of 100 every 20 seconds. If
sized Red cars in the lot. If there are the two machines run simultaneously,
three times as many mid-sized cars as how many seconds will it take for them
there are large ones and the red to blue to produce a total of 200 bolts?
cars ratio in the lot is 3:2, How many
cars in the lot are :

(A) Red and Mid-size?

(B) Blue and Large?

(C) Blue and Mid-size?

Directions: You are required to construct 5. A survey involving two questions Q1


a table as discussed and solve the and Q2 was answered by N persons. 1/3
problem. of those participating answered YES to
Q1, and of these 1/5 also answered YES
to Q2. If the number of those answering
YES to Q1 but NO to Q2 is twice that
answering NO to Q1 and YES to Q2,
what is the number answering NO to
both questions Q1 and Q2?

Hint: Plug in the values given and


compute the remaining values to agree
with the totals specified.
YES TO Q2 NO TO Q2 TOTAL
YES TO Q1

NO TO Q1

TOTAL

Some of the materials used in this module are taken from disclosed editions of GMAT, and no monetary
value is included in the cost of this module for supply of these materials. These materials are strictly used
for illustration of concepts discussed. Page 147
©Educational Training Services, A Division of Maple Leaf International Consulting, Inc.
Not to be reproduced, copied or distributed. Intended for use only by the registrant.

GEOMETRY Two angles whose measures add up to


90º are referred to as complementary
angles. If the two measures add to
180º, they are known as
1. LINES: supplementary angles.

A Line is a Straight Line. It is a no-


brainer.
Two straight lines intersecting each other
A B at 90º are referred to as perpendicular
lines. A right angle symbol at the
AB is called a line segment and A and B intersection denotes perpendicular lines.
are the end points of the segment. Both
the segment and the length of the Parallel Lines are two lines in the same
segment are denoted by the notation AB. plane that do not intersect. The lines are
supposed to meet at infinity. A third line
intersecting two parallel lines produces
the following relationship among the
2. INTERSECTING LINES AND angles of intersection:
ANGLES:
xº yº

yº xº

xº yº

yº xº
The opposite angles are referred to as the
Vertical angles and will have the same xº+ yº =180º
measure. S

A Polygon is a closed plane figure


P xº yº R formed by 3 or more line segments,
referred to as the sides of the polygon.
Q Each side intersects exactly two other
sides at their end points called the
vertices. Each interior angle of a convex
polygon has a measure of less than 180º.
PQR is a straight line or straight angle
(180º). Angle PQS and SQR are A Polygon with 3 sides is a triangle; with
adjacent angles and add up to 180º. four sides is a quadrilateral; 5 sides is a
(xº + yº = 180º). pentagon and six sides is a hexagon.

Some of the materials used in this module are taken from disclosed editions of GMAT, and no monetary
value is included in the cost of this module for supply of these materials. These materials are strictly used
for illustration of concepts discussed. Page 148
©Educational Training Services, A Division of Maple Leaf International Consulting, Inc.
Not to be reproduced, copied or distributed. Intended for use only by the registrant.

A triangle has 3 angles and the sum of


the angle measures is 180º.
The Perimeter of a polygon is the sum of the
The general rule for the sum of the angle lengths of its sides.
measures of a polygon with “n” sides is:
(n-2)180º A triangle with sides 3, 4 and 5 will have a
perimeter of 3+4+5=12 units.
1. What is the sum of the angle
The area of a figure lying in a plane will
measures of a triangle? mean the area of the region enclosed by that
A triangle is a polygon with 3 sides. figure.
∴n=3 The sum of its angle measures
is: (3-2)•180º = 180º

2. What is the sum of the angle


measures of a Quadrilateral? Q

A quadrilateral is a polygon with 4 sides.

∴ n = 4. The Sum of its angle measures 5 5


is:
P xº yº R
(4-2)•180º = 360º
The above triangle is referred to as
Triangle PQR. PQ, PR and QR are the
3. What is the sum of the angle
measures of a pentagon? sides of the triangle. In our illustration,
PQ =QR.
A pentagon is a polygon with 5 sides.
If none of the sides in the triangle is the same
∴N=5. length as any other, then we are dealing with a
scalene triangle.
The sum of its angle measures is:
If two sides of a triangle are equal, then the
(5-2)•180º = 540º triangle is called an “isosceles” triangle. If all
three sides are equal , i.e. PQ=QR=PR, then the
4. What is the sum of the angle triangle is called an equilateral triangle.
measures of a hexagon?
In an isosceles triangle, the two angles opposite
the two sides of equal length will have the same
A hexagon is a polygon with 6 sides. measure. In our illustration xº = yº. The third
∴N=6 angle will be equal to 180º-(xº+yº) = 180º-
2xº = 180º - 2yº since xº and yº are equal.
The sum of its angle measures is: Also, the vertical line drawn from the vertex
where the two line segments of equal length
(6-2)180º = 720º meet will divide the opposite side (base) in two
equal halves. Such a vertical line will also
divide the vertex angle in two equal halves.
What is the sum of the angle measures
of a polygon with 9 sides? In an equilateral triangle, all three angles are
equal to 60º each. All sides will be the same
length. A vertical line drawn from any vertex of
Some of the materials used in this module are taken from disclosed editions of GMAT, and no monetary
value is included in the cost of this module for supply of these materials. These materials are strictly used
for illustration of concepts discussed. Page 149
©Educational Training Services, A Division of Maple Leaf International Consulting, Inc.
Not to be reproduced, copied or distributed. Intended for use only by the registrant.

an equilateral triangle to the opposite side (base) L1 and L2 are parallel lines, and we choose
will divide the base in two equal halves. Such a points A and B on L2. If we use the line
vertical line will also divide the vertex angle in segment AB as the base for all triangles ABC,
two equal halves. ABE, ABE, ABF, the area of these triangles
must be equal. Why ? The area of a polygon is
30o 30o determined by the vertical height. Since these
triangles share the same base, and have the
same vertical heights (shown in dotted lines), it
follows that these triangles must have equal
areas as well.

30o 30o 30o 30o The only set of information we need to know to
be able to compute the area of a triangle is
The sides are divided in two equal halves, and information pertaining to the base value and the
the line segments shown are equal length. The vertical height.
vertex angles are all equal and the vertical lines
drawn to the opposite “bases” will divide the As long as we know that the triangles
vertex angles in equal halves. In fact, the vertex
angles are each 60 degrees in an equilateral
share the same base and are drawn
triangle, and the vertical lines will divide the between two parallel lines, we will know
vertex angles to two 30 degree angles. that these triangles have the same area.

Triangles Trivia:

Did you know that if you have two parallel lines,


and if you take any two points on one of the
lines and draw triangles by picking points on the
other parallel line, such triangles will all have
equal areas?

L1 C D E F

L2

A B

Some of the materials used in this module are taken from disclosed editions of GMAT, and no monetary
value is included in the cost of this module for supply of these materials. These materials are strictly used
for illustration of concepts discussed. Page 150
©Educational Training Services, A Division of Maple Leaf International Consulting, Inc.
Not to be reproduced, copied or distributed. Intended for use only by the registrant.

PRACTICE EXERCISE 1

3x-2 x 2x+6

A B

3x+4

An isosceles triangle ABC shown


above has its sides as marked. IF AC =
BC, and if the vertical height is x
units, what is the area of the triangle?

Answer: An isosceles triangle has two


sides of equal value. In this problem AC
and BC have equal values. We can,
therefore, write: 3x-2 = 2x+6 or x = 8

Knowing the value for x = 8, we can


determine that the base must measure
(3 X 8 + 4) = 28 units.

The base is 28 units and the vertical


height is 8 units. We have all the
information that we require to find the
area.

Area of isosceles triangle ABC = 1/2.


28.8 = 112 square units.

Some of the materials used in this module are taken from disclosed editions of GMAT, and no monetary
value is included in the cost of this module for supply of these materials. These materials are strictly used
for illustration of concepts discussed. Page 151
©Educational Training Services, A Division of Maple Leaf International Consulting, Inc.
Not to be reproduced, copied or distributed. Intended for use only by the registrant.

A right triangle is a triangle with one Leg 1 Leg 2 hypotenuse


right angle (90º). The side opposite the
right angle is referred to as the 3 x (n) 4 x (n) 5 x (n)
hypotenuse; the other two sides are the
legs. 5 12 13

An important property of a right triangle 7 24 25


is provided by the Pythagorean
theorem. 9 40 41

Pythagorean theorem states that, in a 11 60 61


right triangle, the square of the length of
What do you notice? The square of
the hypotenuse is equal to the sum of the
the smallest length of a leg is the same
squares of the lengths of the legs.
as the sum of the lengths of the other
leg and the hypotenuse. This is an
B
important property arising from
5 Pythagorean theorem.

3 The notation x (n) in the table


indicates that if you multiplied the
A 4 C lengths of all sides by the same
positive integer, the relationship
⊇ABC is a right triangle. AB and AC are among the sides still holds.
the legs of the triangle; BC is the
hypotenuse. For instance, 3, 4,5 is a right triangle.
So is 6, 8, 10. So is 9, 12, 15.
The legs have lengths 3 and 4; the
hypotenuse must have length 5, based on 5, 12, 13 is a right triangle. So is
Pythagorean theorem. 10,24,26. So is 15, 36, 39.

AC² +AB² = BC² You should also remember that 45-45-


90, and 30-60-90 triangles have their
3² + 4² = BC² = 9 + 16 =25 sides in specific proportions as shown
below:
BC = √25 = ± 5
Angles Sides
Since the length of a figure in a plane
cannot be a negative number, BC must 45º -45º - 90º 1 : 1 : √2
be 5.
30º -60º -90º 1 :√3 : 2
You should also know that if the triangle
has its sides in any of the following If the angles are 45º, 45º and 90º,
standard triples, its must be a right then the opposite sides to these angles
triangle. are in the ratio 1:1:√2.

Some of the materials used in this module are taken from disclosed editions of GMAT, and no monetary
value is included in the cost of this module for supply of these materials. These materials are strictly used
for illustration of concepts discussed. Page 152
©Educational Training Services, A Division of Maple Leaf International Consulting, Inc.
Not to be reproduced, copied or distributed. Intended for use only by the registrant.

To illustrate, consider the following 30º-60º-90º Right Triangle


right triangle:
In 30º-60º-90º right triangles, the
3 xº 3√2 √2 = 1.414 lengths of the sides are in the ratio
1:√3:2 (√3 = 1.732)
xº xº=45º

3 If one angle in a right triangle C


is given as 45º, the other two angles
must be 45º and 90º. If one side in the √3 30º 2
above right triangle with one angle 45º
90º 60º
is given as 3, the other two sides must be
3 and 3√2. A 1 B
1. If in the above right triangle, The side AB is opposite the 30º angle
one leg length is 5, the other two sides and if it measures 1 unit, then the side
must be 5 and 5√2. AC opposite the 60º angle must
measure √3(=1.732) units and the
2. If in the above right triangle,
hypotenuse (always) opposite the 90º
one leg length is 6, the other two sides
must be ______ and _________. angle must measure 2 units.

1. If in the above right triangle, the


3. If in the above right triangle,
length of side opposite the 60º angle is
one leg length is 7, the other sides must
be ______ and __________. 5√3. The other two sides must be _____
and ________.
4. If the hypotenuse in a right
triangle with one angle 45º is 12√2, the 2. If in the above right triangle, the sides
leg lengths must be _______ and are 4, 4√3 and 8. Identify the angles
_______. opposite each side:

5. In a right triangle, the sides The angle opposite side 4 units long is
____º. The Angle opposite side
are as follows: 4, 4, 4√2. The angles of
the triangle must be ____, _____, 4√3units long is ___º. And the angle
_____. opposite side 8 units long is _____º.

6. In a right triangle with one 3. If in the above right triangle, the


angle 45º, the hypotenuse measures hypotenuse is 14 units long. The other
25√2. The angle opposite the hypotenuse two sides must measure _____ and
must measure ______º. ____.

Answers: (1) 5, 10 (2) 30º,60º, 90º (3)


Answers: (2) 6, 6√2 (3) 7, 7√2 (4 )12,
7, 7√3.
12 (5) 45º,45º,90º (6) 90º

Some of the materials used in this module are taken from disclosed editions of GMAT, and no monetary
value is included in the cost of this module for supply of these materials. These materials are strictly used
for illustration of concepts discussed. Page 153
©Educational Training Services, A Division of Maple Leaf International Consulting, Inc.
Not to be reproduced, copied or distributed. Intended for use only by the registrant.

6 30º 6
The area of a triangle is calculated
using the formula: A 60º D 60º B

(length of the altitude) X (length of the base) 6

2 From the properties of a 30º - 60º - 90º


triangle (Which is what CDB is), and
Altitude: Denotes the segment going from the knowing that DB=3 inches because the
vertex perpendicular to the side opposite that altitude from vertex C divides the base in two
vertex. This side is referred to as the Base. equal halves, we know that the altitude must
be 3√3 (Remember the sides are in the ratio
C of 1:√3:2).

90º E ∴ The Area is (CD X AB) ÷ 2 = (3√3 X 6)÷2

A 90º B
The Area is: 9√3
D
2. An isosceles right triangle has its
10 hypotenuse measuring 5√2. What is its
area?
In the above triangle, CD is the altitude with We know that in an isosceles right
respect to the base AB and AE is the altitude triangle, the 3 angles are 45º, 45º and
with respect to the base CB.
90º and the sides opposite these angles
If the base length is 10 units and the altitude are in the ratio 1:1:√2.
is 5 units, the area is :
In this exercise, the hypotenuse is 5√2.
10 X 5 We know that the other two sides must
be 5 and 5.
2

The area is 25 square units. We can compute the area to be:

Exercises: (5 X 5)÷2 = 12.5

1. What is the area of an equilateral triangle of NOTE: In a right triangle, the two legs
side 6 inches? constitute the altitude and the base and we can
compute the area by multiplying the two leg
We know that the equilateral triangle has all 3 lengths and dividing the product by 2. The area
sides of equal measure, in this case 6 inches. All for any polygon is determined by the vertical
3 angles are equal to - 60º each. The altitude
height .
from a vertex to the base will divide the vertex
angle equally 30º - 30º and the base equally - 3
inches - 3 inches in length.

Some of the materials used in this module are taken from disclosed editions of GMAT, and no monetary
value is included in the cost of this module for supply of these materials. These materials are strictly used
for illustration of concepts discussed. Page 154
©Educational Training Services, A Division of Maple Leaf International Consulting, Inc.
Not to be reproduced, copied or distributed. Intended for use only by the registrant.

ASSIGNMENT Answer: _________________º

7. An isosceles triangle has two sides


10 and 10. The base where the altitude
1. A triangle has its 3 sides of from the vertex meets measures 16.
following lengths: 3, 3, 3. What are the What is the area of the triangle?(Draw
3 angles? triangle, mark sides and angles
appropriately, and solve the problem).
Answer:______º, ______º, ________º

2. A right triangle has its hypotenuse


measuring 8 units. One of its angles is
30º. What are the other two side
lengths?
Answer: ____________________
Answer:__________, _____________
8. If AB X CE = 24, What is the area
3. A triangle has 3 sides of lengths 27,
of the triangular shaped region ABD
36, 45. What type of a triangle is this?
shown below?(AB and CD are parallel
Answer:_________________________ lines)

4. An equilateral triangle has sides C D


measuring “a”. What is the area of the
equilateral triangle in terms of its side
“a”?

A E B K
Hint: The vertical height DK shown in dotted
lines will determine the area of the triangle ABD.
Area is:__________________________ You will notice that the altitude for the triangle
ABD is the same as CE.
5. A right triangle has one leg
measuring 4 and the other 6. What is
the hypotenuse length? (use
Pythagorean theorem and show steps).

Answer:

A
nother important rule with
The Hypotenuse is: __________ regard to the side lengths of a
triangle that you should be
6. A triangle has two angles measuring
aware of is stated here.
55º and 43º. What is the measure of
the third angle?
Some of the materials used in this module are taken from disclosed editions of GMAT, and no monetary
value is included in the cost of this module for supply of these materials. These materials are strictly used
for illustration of concepts discussed. Page 155
©Educational Training Services, A Division of Maple Leaf International Consulting, Inc.
Not to be reproduced, copied or distributed. Intended for use only by the registrant.

side can have more than one value and still satisfy the
If the triangle is a right triangle, then the requirement that it is part of a triangle.)
relationship between the values of the
side measures is specified by the I. 5 II. 8 III. 11
Pythagorean theorem. For example, Can
line segments of lengths 4, 7, and 9 be Answer Choices: (A) II only (B) III only
the values for the sides of a right (C) I and II only (D) II and III only (E)
triangle? We know that in a right I,II and III.
triangle, the hypotenuse is the longest
side (because it is opposite the largest We know the values of two sides to be 3
angle). Of these three values - 4, 7, and 9 and 8. For the third side to be part of this
- 9 must be the hypotenuse value if these triangular region described, it must lie
line segments can indeed form a right within the following values:
triangle. The only test we will apply is:
(3+8) > the third side > (8-3) which
Is 42 + 72 = 92 ? We notice that 16 + 49 means that the third side can have values
is not equal to 81, and we conclude that greater than 5 and less than 11 and be a
4, 7, and 9 could not be the values for part of the triangular region.
the sides of a right triangle.
From the answer choices, we notice that
Could these be the values for the sides of there is just one value - 8 - that satisfies
a normal triangle (one that is not right this requirement and condition and we
angle triangle)? To determine that, you choose (A) as the correct answer choice.
will apply the following test:
Note: Greater than 5 means that 5 cannot
If two sides of a triangle are known, then be a value. Similarly, less than 11 means
the third side must have its length that 11 cannot be a value.
measure less than the sum of the other
two side lengths and greater than the Let us do one more exercise to clarify this rule for
a normal triangle.
difference of the other two side lengths.

In other words, if a, b and c are the three


sides of a triangle and if a and b are
known quantities, then, for a,b and c to
be the sides of a triangle, c must satisfy
the following condition:

(a+b) > c > (+ve difference of a and b)

Let us consider an example from a recent


GMAT:

“If 3 and 8 are the lengths of two sides of


a triangular region, which of the
following can be the length of the third
side? (Notice the “can be” condition here because the third
Some of the materials used in this module are taken from disclosed editions of GMAT, and no monetary
value is included in the cost of this module for supply of these materials. These materials are strictly used
for illustration of concepts discussed. Page 156
©Educational Training Services, A Division of Maple Leaf International Consulting, Inc.
Not to be reproduced, copied or distributed. Intended for use only by the registrant.

The area of a parallelogram is the


product of its altitude (CD) and its
“A skating rink is in the shape of a triangle. The two
base (AB).
sides of the rink measure 80 ft and 90 ft.
A parallelogram is composed of two
1. What is the possible range of values for the third triangles, in our example ABD and AED.
side? ABD and AED will have equal areas. If
the area of ABD is 24, AED will have an
Answer: The third side can have values in the area of 24 and the parallelogram will
following range: 170> third side <10. The third side have an area of 48.
can have any value greater than (but not including)10
ft and less than (but not including) 170 feet.
Do the Diagonals of a parallelogram
intersect each other at right angles? What
do you think?

Answer: The diagonals of a simple


parallelogram do not bisect each other at
Quadrilaterals right angles. The diagonals of a rhombus
We know that a quadrilateral is a do.
polygon with 4 sides.
What is a Rhombus? A rhombus is a
What do you call a quadrilateral in which parallelogram with all sides of equal
both pairs of opposite sides are parallel? lengths. In our illustration,
Simple: A parallelogram. What else do
we know about parallelograms? The If AB=BD=DE=EA, then we have a
opposite sides of a parallelogram have rhombus for a parallelogram. In a
equal lengths. rhombus, all four sides are considered to
be “congruent” and the diagonals bisect
E D the vertex angles and each other at right
angles.
F
IN other words, ∠AEF = ∠DEF; ∠EAF
A B C = ∠BAF; ∠EDF = ∠BDF.

In the above parallelogram ABDE, ∠EFD = 90º = ∠BFD

AB || ED and AB = ED; ∠BED=∠ABE EF=BF=AF=FD

AE || BD and AE = BD:∠BAD=∠ADE A parallelogram with right angles is a


rectangle and if all sides of a rectangle
If AB=6, then DE =6; If BD =5, then are of equal length, it becomes a
AE=5. square. Is a square a rhombus with
all angles equal to 90º. You could say
The diagonals of a parallelogram bisect
that.
each other. i.e. AF=FD & EF=BF

Some of the materials used in this module are taken from disclosed editions of GMAT, and no monetary
value is included in the cost of this module for supply of these materials. These materials are strictly used
for illustration of concepts discussed. Page 157
©Educational Training Services, A Division of Maple Leaf International Consulting, Inc.
Not to be reproduced, copied or distributed. Intended for use only by the registrant.

The area of a rectangle is the product of or congruent, then the figure is a


its altitude and its side. In a rectangle, the square.)
altitude happens to be another side The
area of a square is the square of its side. Can a quadrilateral have two parallel
If one side of a square is 6, then all sides sides and the other two sides not parallel
are 6 and the area is 6² = 36. to each other. Yes. Consider the
following figure: It is called a trapezoid.
If a rectangle has two sides of value 4 A trapezoid has two parallel sides and
each and the other two sides of value 6, two non-parallel sides.
then its area is 6 times 4 = 24.
C 6 ft D
The perimeter of a parallelogram is the
sum of the lengths of its sides. A square 4’ 4’
of side 6 will have its perimeter equal to
24. A rectangle with sides 4,4,6,6 will A 3’ E 6‘ F 3’ B
have it perimeter equal to 20.
How do you calculate the area of the
trapezoid?

What else do we need to know about If you take a look at the trapezoid again,
quadrilaterals? you will notice that it has:

We said that a rectangle has 4 right 1. A rectangular shaped region.


angles. Can a quadrilateral have 2 right
2. Two triangular shaped regions.
angles and not be a rectangle? You bet it
can be. Consider the following shape of a The area of a trapezoid will be the sum
quadrilateral: of the areas of the rectangle and the two
triangles.

In our illustration, the area of the


trapezoid will be:
90º 90º
1. Area of the rectangle = 24 Plus

2. Areas of two triangles = ½.. (3)(4) +


We can see that the above shape is not ½.(3)(4) = 12
that of a rectangle.
Equal to :36 (See below for another way to get this)
Can a quadrilateral have 3 right angles
Note: The two triangular regions may
and not be a rectangle? NO. Because, the
not be congruent or have the same area
internal angles add up to 360º
in a trapezoid.
(Remember: (n-2)180º). If the three
angles are 90º each, then the fourth
angle of a quadrilateral must be 90º as
well. It must, therefore, be a rectangle or
a square. (If all sides are given as equal
Some of the materials used in this module are taken from disclosed editions of GMAT, and no monetary
value is included in the cost of this module for supply of these materials. These materials are strictly used
for illustration of concepts discussed. Page 158
©Educational Training Services, A Division of Maple Leaf International Consulting, Inc.
Not to be reproduced, copied or distributed. Intended for use only by the registrant.

“A soccer field is in the shape of


trapezoid with a rectangular portion in
the middle with sides 120 yards and 60
yards; the rectangular area is flanked by
two triangular regions of total area 450
sq.yards. The total area of the
trapezoidal soccer field will be 7,650
sq.yds. (120X60 + 450 = 7,650 sq.yds).

The area of a trapezoid can also be


calculated using the formula:
½ . (sum of bases)(height).

In our illustration, the area will be:

½. (12 +6)(4) = ½.(72) = 36 which is


the answer we obtained by computing
and adding individual areas of rectangle
and the two triangular regions.

Some of the materials used in this module are taken from disclosed editions of GMAT, and no monetary
value is included in the cost of this module for supply of these materials. These materials are strictly used
for illustration of concepts discussed. Page 159
©Educational Training Services, A Division of Maple Leaf International Consulting, Inc.
Not to be reproduced, copied or distributed. Intended for use only by the registrant.

Answer: __________
ASSIGNMENT
4. If the length and the width of a
rectangular garden plot were each
increased by 25%, what would be the
1. If ∠ABC in the figure below is a right percent increase in the area of the plot?
angle, what is the value of external xo?
(A) 25% (B) 33% (C) 45% (D) 56.25%
xº C (E) cannot estimate from the information

B 55º D

A ∠ABD = 55º
Answer: ______________

5. The trapezoid shown below is the


cross section of a rudder of a ship. If the
Answer: xº = ______º distance from A to C is 25 feet, what is
the area of the cross section of the
2. If a triangular region has two sides of rudder in square feet? C
value 17 and 25, which of the following
cannot be a value for the third side? D

(A) 38 (B) 45 (C) 11 (D) 17 (E) 41 A B

The lengths are: AD= 4’ BC=7’.

(A)168 (B) 175 (C) 100 (D) 132 (E) 28

Answer: ________ (Draw diagram here and show your working)

3. Which of the following groups of


numbers could be the lengths of the sides
of a triangle?
Answer: ________________
I. 1, 4. √17 II. 4, 7, √11 III. 4, 9,6

Choices: (A) I only (B) I and II only (C)


I and III only (D) II and III only (E) I, II
and III. (Explain your answers)

Some of the materials used in this module are taken from disclosed editions of GMAT, and no monetary
value is included in the cost of this module for supply of these materials. These materials are strictly used
for illustration of concepts discussed. Page 160
©Educational Training Services, A Division of Maple Leaf International Consulting, Inc.
Not to be reproduced, copied or distributed. Intended for use only by the registrant.

9. The shaded portion ABC of a


rectangular lot shown below is a flower
bed. If the area of the flower bed is 24
Assignment Continued... sq.yds and BC=AB + 2, then AC equals:

6. If a rectangular photograph that is 10 (A)√13 (B)2√13 (C) 6 (D) 8 (E) 10


inches wide by 15 inches long is to be
enlarged so that the width will be 22
inches and the ratio of length to width
C
will be unchanged, then the length in
inches of the enlarged photograph will be

(A) 33 (B) 32 (C) 30 (D) 27 (E) 25 A B

Answer: ________

7. If L and W are the dimensions of a Answer: _________


rectangular region that has area 42 and if
L and W are integers such that L > W, 10. A rectangular region has the same
What is the total number of possible area as a square region : 64 sq. yards. If
values of L? one length of one side of the rectangle is
half the length of the side of the square
(A) Two (B) Three (C) Four (D) Five and the other side of the rectangle is
(E) Six twice the length of square side, what are
the dimensions of the rectangle and the
square?(rectangle length, rectangle width, square side)
Answer: _______
(A) 8,8,8 (B) 4,16,8 (C) 8,12,12
8. In the above question, if the area were (D) 4,8,8 (E) 16,8,8
45, and L and W are integers such that
L >W, how many possible values for W
exist?

(A) Two (B) Three (C) Four (D) Five


(E) Six
Answer: ____________

Answer: __________
Assignment Continued...

Some of the materials used in this module are taken from disclosed editions of GMAT, and no monetary
value is included in the cost of this module for supply of these materials. These materials are strictly used
for illustration of concepts discussed. Page 161
©Educational Training Services, A Division of Maple Leaf International Consulting, Inc.
Not to be reproduced, copied or distributed. Intended for use only by the registrant.

11. Which of the following inequalities is house. If the cafe is 7 miles closer to
equivalent to 10-2x > 18? Mary’s house than to Jenny’s house, how
many miles is the cafe from Jenny’s
(A) x > -14 (B) x > -4 (C) x >4 (D) x < 4 house? (Hint: Use Pythagorean theorem and right triangle
(E) x < -4 properties discussed.) Draw diagram and show steps.

(A) 12 (B) 20 (C) 6 (D) 10 (E) 10√3

Answer: ___________

12. Which of the following inequalities is


equivalent to 5x-10y < -20
Answer: ______________
(A) 2y < -(4+x) (B) x < 2y+4
(C) x >2y+4 (D) 2y>4+x 15. Which of the following equations
(E) None of the above. has one root in common with x²-6x+5=0

(A) x² + 1=0 (B) x²-x-2 = 0 (C)x²-10x-


5=0 (D) 2x² - 2=0 (E) x²-2x-3=0

Answer: ____________

Answer:________________
13. If n is an integer and
16. How many positive integers n are
n = (2.3.5.7.11.13) ÷ 91p there such that 100n is a factor of:

Which of the following could be the (2³ )(5)(5³ )


value of p?
(A) None (B) Five (C) Seven (D) Six (E)
(A) 22 (B) 26 (C) 35 (D) 54 (E) 60 Eleven

Answer: ________ Answer: ______________________

14. Mary and Jenny live 13 miles apart.


They meet at a cafe that is directly north
of Mary’s house and due east of Jenny’s
Some of the materials used in this module are taken from disclosed editions of GMAT, and no monetary
value is included in the cost of this module for supply of these materials. These materials are strictly used
for illustration of concepts discussed. Page 162
©Educational Training Services, A Division of Maple Leaf International Consulting, Inc.
Not to be reproduced, copied or distributed. Intended for use only by the registrant.

In our illustration, the area of the circle


Circles and Solids of diameter 10 cm will be:

Imagine, if you will, a set of points in a π.5² = 25π = π. (10²÷4)


plane that are all located equidistant from
a fixed point and you have just imagined If you traveled around the
up a circle. circumference of the circle, you will
have traversed a complete revolution
A chord of a circle is a line segment that with the number of degrees equal to
has its end points on the circle. When the 360º. In other words, the number of
chord passes through the center of the degrees of arc in a circle is 360º.
circle, it becomes the diameter. A radius
is a line segment going from the center of The length of arc of a circle is
the circle to a point on the circle. calculated by determining the angle
the arc forms at the center of the
A radius is ½. of the diameter. r is a circle.
common notation for the radius and d
for the diameter. T

The circumference of a circle is the xº S


distance around the circle. The
circumference is (2πr) or (πd) where π is R
a constant and has a value of 22÷7 or
If the angle that the arc RST forms at the
3.14 center of the circle is 60º, then the arc is 1/6
of the circumference of the circle. i.e. (60º÷
360º) . circumference

P Q R If each vertex of a polygon is located on


a circle as in Figure A, then the polygon
is inscribed in the circle and the circle is
The line segment shown in red is the circumscribed about the polygon.
chord of the circle. The blue line is the
diameter. Q is the mid point of the line
PR and denotes the center of the circle.

PQ =QR = r = radius of the circle

If PR = 10 cm, then PQ=QR=5 cm

The circumference of the circle is 2πr


=10π or 31.4

The area of the circle is expressed by the


formula: πr² or (πd²÷4)

Some of the materials used in this module are taken from disclosed editions of GMAT, and no monetary
value is included in the cost of this module for supply of these materials. These materials are strictly used
for illustration of concepts discussed. Page 163
©Educational Training Services, A Division of Maple Leaf International Consulting, Inc.
Not to be reproduced, copied or distributed. Intended for use only by the registrant.

We are now ready to answer the rest of


If each side of a polygon is tangential to the questions.
the circle (meets the circle at just one
point) as in Fig.B, then the circle is said 2. Area of the circle: πd²/4 = 6.25π
to be inscribed in the polygon and the
polygon is circumscribed about the 3. Circumference of the circle: πd=5π
circle.
4. Perimeter of the Triangle =3+4+5=12
If a triangle is inscribed in a circle such
that one of its sides is the diameter of the As you can see, what is required is your
circle, then the triangle is a right triangle. ability to quickly determine that the
inscribed triangle is a right triangle and
C 90º from the Pythagorean theorem , to see
that the triangle must be 3-4-5 and apply
A B the formula to calculate the required
information.
⊇ABC is a right triangle with ∠ACB
=90º. AB is also the diameter of the Try this one out, and see if you can get
circle. the answer:

Exercise: “Within a circle of radius 6.5 inches is


inscribed a right triangle. If one of the
“A triangle is inscribed in a circle such other sides is 5 inches, then (1) What is
that its hypotenuse is the diameter of the the area of the inscribed triangle? (2)
circle. If the legs of the triangle are 3, What is the area of the circle? (3) What
and 4 inches, (1) What is the diameter of is the circumference of the circle? (4)
the circle? (2) What is the area of the What is the ratio of the circumference of
circle? (3)What is the circumference of the circle to the perimeter of the
the circle? (4) What is the perimeter of triangle?”
the triangle?”
(Answers: (1) 30 (2) (169π÷4) (3) 13π
From the properties of right triangle and (4) 13π:30 )
the Pythagorean theorem, we know that
Did you know that the circumference of a semi-
if the two legs of a right triangle are 3, circular shaped region is ½ the circumference of
the other two sides are 4 and 5 and the the full circle PLUS the diameter? You need to
and 4, the hypotenuse is 5 inches. traverse across the diameter to complete the loop.

When you find values of 3 and 4 or any


multiple thereof for the measure of the
legs, you know that the hypotenuse must
be 5 or multiple thereof.

∴The diameter of the circle is 5.


(Answer to Q1)

∴ The radius of the circle is 2.5.

Some of the materials used in this module are taken from disclosed editions of GMAT, and no monetary
value is included in the cost of this module for supply of these materials. These materials are strictly used
for illustration of concepts discussed. Page 164
©Educational Training Services, A Division of Maple Leaf International Consulting, Inc.
Not to be reproduced, copied or distributed. Intended for use only by the registrant.

Also, if L, W, and H are the three


dimensions of the rectangular box, then
the maximum length of a diagonal
connecting two points on the box is
3 given by the formula:

4 8 SQRT(L2 + W2 + H2)

Rectangular Solid Cylinder If the dimensions of a rectangular box are


5, 7, and 9 inches, then the maximum
length of a diagonal is SQRT(52 + 72 +
92) = SQRT(25 + 49 + 81) = SQRT(155)

Pyramid Cone Sphere

A rectangular BOX has 6 faces and its CUBE


surface area is the sum of the surface
areas of all its faces. In our illustration, A CUBE IS A RECTANGULAR BOX
the rectangular BOX has dimensions 3, THAT HAS ALL SIDES MADE OF
4 and 8. Therefore the surface area of the SQUARES. The surface area of a cube is
BOX is 2(3.4)+2(3.8)+2(4.8) = 136. 6 times (side)2

Notice that there are 3 sets of parallel The capacity or the maximum volume of
faces and, therefore, you need to a cube is (side)3.
calculate the surface areas of 3 faces and
multiply the result by 2 to get the total REFER TO GEOMETRY FORMULAS
surface area for the solid. FILE FOR A DETAILED
DISCUSSION OF THE PROPERTIES
To calculate the volume of the OF A CUBE.
rectangular solid, multiply length , width
and height. (3.4.8 = 96)

Some of the materials used in this module are taken from disclosed editions of GMAT, and no monetary
value is included in the cost of this module for supply of these materials. These materials are strictly used
for illustration of concepts discussed. Page 165
©Educational Training Services, A Division of Maple Leaf International Consulting, Inc.
Not to be reproduced, copied or distributed. Intended for use only by the registrant.

Exercise: A circular cylinder of height


CYLINDERS 20 has a base area of 64π. What is the
(1) Surface area of the cylinder? (2)
Volume of the cylinder?

The base area is 64π. ∴ πr² = 64π;


r²=64. r =8 The height “h” is 20.

The surface area of the cylinder is twice


the base area plus 2πrh =2(64π)+2π.8.20
In a right circular cylinder shown above,
there are two circular bases of equal size Surface Area = 128π+320π=448π
connected by a curved surface.
Volume of cylinder =πr²h =π.8².20 =
The area of the cylinder is the area of the 1280π
two circular bases plus the area of the
curved surface. The curved surface is
nothing but a rectangular sheet metal that
CONE
has been rolled along its width. What A
Cone is a
was once width of a rectangular sheet truncated
metal has become now the circumference cylinder. The
of the curved surface. The area of the surface area of a
curved surface is, therefore, 2.pi.r.h cone is ½ that of
a cylinder of
The area of the circular bases are each
identical radius
πr². That gives 2πr². The area of the
and height, and the capacity or
curved surface is 2πrh.
maximum volume is 1/3 that of a
The Surface area of a circular cylinder is: cylinder of identical radius and height.

2πr² + 2πrh

In our illustration, if, say, the radius, r=5


and the height, h=10, the surface area
will be:

2.( π5²) + 2π.5.10 = 50π+ 100π = 150π

The volume of the cylinder is πr²h


which is the same as the product of base
area and the height.

In our illustration, the volume of the


cylinder is : π (5²).10 = 250π

Some of the materials used in this module are taken from disclosed editions of GMAT, and no monetary
value is included in the cost of this module for supply of these materials. These materials are strictly used
for illustration of concepts discussed. Page 166
©Educational Training Services, A Division of Maple Leaf International Consulting, Inc.
Not to be reproduced, copied or distributed. Intended for use only by the registrant.

box, which is 8 inches. The capacity of


SPHERE such a cylinder is: π•5•5•8 = 200π

We can see that the cylinder of 10


inches diameter will have the
maximum capacity. We will discuss
more practice problems in geometry
A SPHERE is later on in this chapter.
a ball whose capacity is given by the
formula:

4/3 • π•RADIUS3

You are not likely to get questions


about Spheres in the GMAT, but you
should expect questions about
Cylinders, cubes and rectangular
boxes.

EXERCISE:

“What is the diameter of a cylinder,


which will have the maximum
capacity, and which can be placed
inside a rectangular box of dimensions
8 inches by 10 inches by 12 inches?”

We can place the cylinder on any of


the following three sets of faces:

• 8 inches by 10 inches. The maximum


diameter of a cylinder that can be placed
on this side is 8 inches, and the height of
this cylinder could be the third dimension
12 inches. The capacity of such a cylinder
will be π•4•4•12 = 192π

• 8 inches by 12 inches. The maximum


diameter is only 8 inches and the height is
10 inches. We know that this cylinder is
going to have less capacity than the one
discussed earlier.

• 10 inches by 12 inches. The maximum


diameter of a cylinder is 10 inches, and
the height is the third dimension of the
Some of the materials used in this module are taken from disclosed editions of GMAT, and no monetary
value is included in the cost of this module for supply of these materials. These materials are strictly used
for illustration of concepts discussed. Page 167
©Educational Training Services, A Division of Maple Leaf International Consulting, Inc.
Not to be reproduced, copied or distributed. Intended for use only by the registrant.

base to the vertex. The volume is


expressed in measurement units,
cubed, like cubic inches. See if you
Pyramids can imagine little cubes filling up the
interior space of the shape.
A pyramid is a polyhedron that has This formula is true for pyramids of
only one base. (The base is the any shape base. As long as you can
"bottom" of the Egyptian pyramids.) find the area of the base and you
The other faces are all congruent know the height, you can calculate
triangles, and they share a common the volume.
vertex, which is the top point. The
base can be any type of polygon. If What is the volume of a pyramid with
the base is a triangle, then the a square base with sides of 5 cm, and
pyramid has a total of four faces. The a height of 3 cm?
Egyptian pyramids have square bases

and four triangles as faces

V= (5cm x 5cm) x 3 cm
Volume = 25 cm3

These are two illustrations of


pyramids, one with a triangular base
and triangular faces, and the other
with a rectangular base and triangular
faces.
The volume of a pyramid is a measure
of how much it would take to fill the
shape. For a pyramid, the formula is:

Where B is the area of the base


figure, and h is the height from the
Some of the materials used in this module are taken from disclosed editions of GMAT, and no monetary
value is included in the cost of this module for supply of these materials. These materials are strictly used
for illustration of concepts discussed. Page 168
©Educational Training Services, A Division of Maple Leaf International Consulting, Inc.
Not to be reproduced, copied or distributed. Intended for use only by the registrant.

The surface area is calculated by The surface areas of the triangular faces
computing the area of all faces that are the same.
constitute a pyramid. Since all faces
except possibly the base are triangles, Surface area of 4 triangular faces= 4.1/2. .bh
you will find Pythagorean theorem handy
in solving pyramid problems. In this exercise, b is 10; the height is 13.

A Surface Area of triangular faces=260

S 12 R ∴The Surface of the pyramid =100+200


=360
B C
Exercise: Try this one out on your own.
P 10 Q
In the subject illustration, change the
PQRS is the base of this pyramid and is a value of the base length to 14 and the
square. ASP, APQ, AQR and ASR are height AB to 24. What are the new
the four triangular faces. How do we find values for (1) AQ; (2) Surface area of the
(1) the measure of AQ=AP=AS=AR?; pyramid?
and (2)the area of this pyramid?
(Answer: (1)√674 or 26 (approx.) Note: In
the test, you will not be required to find the square
We are given that the altitude AB root of 674. In all probability, the answer choice will
measures 12 and the sides of the square be √674.
base are 10 (therefore, BC=5). Using
Pythagorean Theorem, let us compute (2) 896
the value for AC.

AC= √AB²+BC² = √144+25=√169=13.

AC is the altitude of the triangular face


AQR. We know that QC = 5. (Base is
10 on each side).

AQ = √QC²+AC² =√13²+5²=√194..(1)

Now, we need to calculate the surface


area of the pyramid.

To recap, the surface area of the


pyramid is the surface area of the base
plus the surface areas of the 4
triangular surfaces.

The Area of the square base is 10²=100

Some of the materials used in this module are taken from disclosed editions of GMAT, and no monetary
value is included in the cost of this module for supply of these materials. These materials are strictly used
for illustration of concepts discussed. Page 169
©Educational Training Services, A Division of Maple Leaf International Consulting, Inc.
Not to be reproduced, copied or distributed. Intended for use only by the registrant.

Area of equilateral triangle


= 1/2. Base . height = 1/2. b. \/3.b/2 =
\/3.b2/4
Practice Problems in Since the area of the triangle is the same
Geometry as that of the square, we are ready to set
up the following equation:
1. An equilateral triangle of side b and a \/3.b2/4 = s2 or s2 / b2 = \/3 /
square of side s have the same area. The 4. Choice E.
ratio of s2 to b2 is:

(A) 1 (B) \/2/3 (C) \/3 (D) \/3/2 2. A square of side s is inscribed in a
(E) \/3 / 4 circle of diameter d. The area of the
circle in terms of s is:
We know that the area of the square is s2
, where s is the side of the square. (A) π.s2 (B) π. s2 /2 (C) π.s2 / 4
(D) π. s2 /8 (E) Cannot be determined
The area of a triangle is 1/2. Base .
height. Solution:

How do we express the height of an We know that when a square is inscribed in a


circle, then the diameter of the circle is the
equilateral triangle in terms of its side diagonal of the square. Is s is the side of the
value? square, then the diagonal is \/2.s because the
30o 30o diagonal divides the square into two 45-45-90
b b triangles, and the sides are in proportion 1:1:\/2.
60o 60o If the sides are s and s, the diagonal must be
b/2 b/2 \/2.s

Therefore, diameter of the circle must be \/2.s as


well.
As we can see, the altitude divides the
base in two equal halves: b/2 and b/2 Area of the circle in terms of diameter = π d2 /4
We also notice that the altitude converts = π (\/2.S)2 /4 = π.4.s2 /2 = π.s2 /2. Choice
the equilateral triangle into two 30-60-90 B
degrees triangles, with a value of b for
the side opposite 90o angle, and a value
Note: if the problem asks you for the ratio of the
of b/2 for the side opposite 30o angle. area of square to that of circle, then you would
What should be the value for the altitude, set it up as follows:
the side opposite the 60o angle? \/3. b/2
(Remember: the sides of a 30-60-90 Area of the square = s2
degree triangle are in proportion
Area of the circle = pi.d2 /4 = pi.s2/2
1: \/3 : 2 )
Ratio of area of square to that of circle = 2/pi
We now know the value for the altitude (Take the ratio of s2 and π.s2/2 and we have s2
in terms of b, the value for the sides of canceling out leaving 2 on the numerator and π
the equilateral triangle. on the denominator).

Some of the materials used in this module are taken from disclosed editions of GMAT, and no monetary
value is included in the cost of this module for supply of these materials. These materials are strictly used
for illustration of concepts discussed. Page 170
©Educational Training Services, A Division of Maple Leaf International Consulting, Inc.
Not to be reproduced, copied or distributed. Intended for use only by the registrant.

3. A cube’s volume is 3 times its 5. Cylinder X has twice the capacity of


surface area. What is the volume cylinder Y. If cylinder X is 1/3 filled
with oil, and cylinder Y is 3/4 filled
of this cube?
with the same oil, and if all the oil in Y
Solution: is poured into X, to what capacity is X
filled after this transfer?
If we know the relationship of a cube’s volume
to its surface area, we can determine all the Solution:
properties of the cube.
We have: X = 2.Y or Y = 1/2. X
If a is the side of the cube, we have: If Y is 3/4 filled, then 3/4 Y is the volume of
Volume = a.a.a = a3 oil in Y.
And Surface area = 6. a2 And 3/4 . Y = 3/4. 1/2. X = 3X/8
If all the liquid from Y is transfered to X, we
If the volume is 3 times the surface area, we get: are, in effect, adding 3/8 more to the volume
Volume = 3 X Surface Area of liquid already in X (=1/3. X).
i.e. a3 = 3. 6a2 or a = 18 Volume of liquid in X after the transfer =
1/3. X + 3/8. X = 17/24. X
Let us plug in this value for a into the “formula” Cylinder X will be filled to 17/24 of its
for volume, a3 to get: capacity after all the liquid in Y is poured
Volume = 183 = 5,832 cubic units. into X.

4. The radius and the height of 6. The radius of circular rim X is


Cylinder X are each twice the twice that of circular rim Y. If the
circular rims X and Y move the same
values of those of Cylinder Y.
distance per second, what is the ratio
What is the ratio of volume of X to of the number of revolutions per
that of Y ? second that X makes to the number of
Solution: revolutions per second that Y makes?
Cylinder X Cylinder Y Solution:
Radius 2r r
Height 2h h Circular Rim X Y
Volume π.(2r)2.2h π.r2.h Radius 2R R
(pi.(rad)2.(height)) Circumference 2.pi(2R) 2.pi.(R)
Volume 8. πi.r2.h π.r2.h Number of revs
per second N1 N2
We notice that cylinder X has 8 times the Distance Moved
capacity of cylinder Y. per second 4.pi.R..N1 2.pi.R.N2
(Circumference X
The ratio of volume of X to that of Y is: No. Of revs/sec)
8. π.r2.h/ π.r2.h = 8:1 = 8 Since both X and Y move the same distance per
second by making different number of revs per
Note: You can also choose Radius =1 and second, we have 4.pi.R. N1 = 2.pi.R. N2
Height = 1 for one cylinder, and R=2, and H=2 Or N1 / N2 = 2/4 = 1/2
for the other, and compute the same ratio,
without having to mess with stuff like π and R.

Some of the materials used in this module are taken from disclosed editions of GMAT, and no monetary
value is included in the cost of this module for supply of these materials. These materials are strictly used
for illustration of concepts discussed. Page 171
©Educational Training Services, A Division of Maple Leaf International Consulting, Inc.
Not to be reproduced, copied or distributed. Intended for use only by the registrant.

7. “What is the greatest length of a line 10. “If the ratio of surface area of cube X to that
segment that can be placed inside a of cube Y is 1/16, what is the ratio of the
capacity of X to that of cube Y?”
rectangular box of dimensions 4 inches
by 6 inches by 8 inches?” IF X is the side of cube X and Y that of cube Y,
then the ratio of surface areas is given by the
The greatest length of a line segment that following set up:
can be placed inside a rectangular box is
the same as the greatest length of the 6X2 /6Y2 = 1/16 or X2 / Y2 = 1/16
Let us take square root on both sides to get:
box’s diagonal. The greatest diagonal of X/Y=¼
a rectangular box is given by the formula:
SQRT(L2 + W2 + H2) Capacity is side cubed, and the ratio of capacity
= SQRT(42 + 62 + 82) of X to that of Y is:
= SQRT(16 + 36 + 64) X3 / Y3 = (X/Y)3 = (1/4)3 = 1/64
The ratio of capacity of X to that of cube Y is
=SQRT(116)
1/64.
=SQRT(4.29) Notice how this problem combined geometry
=2•SQRT(29) and Exponents in one go.

The greatest length of a line segment that can be 11. “A Rectangular frame of size 18 inches in
placed inside the rectangular box of the length and 12 inches in width encloses a
specified dimensions is 2•⊕29 rectangular picture whose area is ½ the total
area of the frame on which it is mounted.
Also, the length and width of the picture are
8. “What is the greatest length of a line in the same proportion as the overall length
and width of the frame. What is the length
segment that can be placed inside a
of the picture in inches?”
circle of radius 5 inches?”

The greatest length of a line segment that can be The “picture” painted by this word problem will
placed inside a circle is the same as the length of look like this:
the diameter. The diameter of the specified
circle is 10 inches, and that is the greatest
length of a line segment we can place inside the
circle.
The diameter is also called the longest chord of 12”
a circle.

9. “Samantha wants to put up a fence around 18”


the rectangular backyard of length 35 feet The pink outline shows the picture inside tha
and width 20 feet, and she wants to leave frame. The green band shows the area of the
one side of width 20 feet unfenced. How frame surrounding the picture.
many feet of fencing does she need to Let L be the length of the picture, and W its
erect?” width. We have two pieces of information,
which translate into the following two
The length of the fence is the length of the equations:
perimeter minus 20 feet. Or, it is twice length L•W = ½ •18•12 = 108
plus one width for a total of 90 feet. And L/W = 18/12 = 3/2, or W = 2L/3
That is how many feet of fencing she needs to Let us write 2L/3 for W in the first equation and
put up. get: L•2L/3 = 108 Or, L2= 162
or L = √162. = √81•2 = 9√2

Some of the materials used in this module are taken from disclosed editions of GMAT, and no monetary
value is included in the cost of this module for supply of these materials. These materials are strictly used
for illustration of concepts discussed. Page 172
©Educational Training Services, A Division of Maple Leaf International Consulting, Inc.
Not to be reproduced, copied or distributed. Intended for use only by the registrant.

12. What is the shaded area shown within a We can see that
circle of radius 6 inches?
X = 2(L + 3)
And X = 3L
Therefore, we can set up
2(L + 3) = 3L
50o Or, 2L + 6 = 3L
Or, L = 6 feet.
The length of the table top is 6 feet.

14. If a square region has area X square


We notice that the shaded area represents 100 feet, what is the length of its diagonal
degrees out of a total of 360 internal degrees in a in feet?
circle. Therefore, the shaded area will be
We know that the area of a square in
100/360th of the total circular area.
Area of the circle = pi.radius2 = pi.62 = 36pi terms of its diagonal is
Area of the shaded part ½ •(diagonal)2 = X
= (100/360) • 36pi Or, diagonal2 = 2X
= 10pi Or, diagonal = SQRT(2X)
Side of the square is SQRT(X)

13. “A rectangular table top consists of a piece 15. The size of a television screen is
of laminated wood bordered by a thin metal specified by the length of its diagonal.
strip along its four edges. The surface area For example, a 15 inches television
of the table top is X square feet, and the has diagonal measuring 15 inches.
total length of the strip before it was
How many square inches more is the
attached was also X feet. IF the table top is
3 feet wide, what is the length of the table area of a 19 inch square, flat
top?” television screen than that of a 21
inch square, flat television screen?
L feet
The area of a square region = ½ •(diag)2
Area of 19” diagonal square = ½ •192
3 ft Area of 21” diagonal square = ½ •212
Difference in area = ½ •212 – ½ •192
= ½ (212 – 192)
The Blue line around the rectangular table top = ½ •(21+19)(21-19)
shows the strip that was attached to the table (We used the algebraic formula here. Can
top. Let L be the length of the table top. you notice it? A2-B2 = (A+B)(A-B) )
We also notice that the length of the strip Difference in area
corresponds to the perimeter of the rectangular = ½ •40•2 = 40 sq.inches
table top. Perimeter of rectangle = 2•(L + W)
Because W = 3, Perimeter
= 2•(L + 3) = X
We also know that the area of the table top is
X. Area is L•W = 3L = X

Some of the materials used in this module are taken from disclosed editions of GMAT, and no monetary
value is included in the cost of this module for supply of these materials. These materials are strictly used
for illustration of concepts discussed. Page 173
©Educational Training Services, A Division of Maple Leaf International Consulting, Inc.
Not to be reproduced, copied or distributed. Intended for use only by the registrant.

FORMULAS IN GEOMETRY

REFER TO THE GEOMETRY


FORMULAS FILE FOR A
DETAILED DISCUSSION ON THE
FORMULAS AND RULES
RELEVANT TO GEOMETRY

Some of the materials used in this module are taken from disclosed editions of GMAT, and no monetary
value is included in the cost of this module for supply of these materials. These materials are strictly used
for illustration of concepts discussed. Page 174
©Educational Training Services, A Division of Maple Leaf International Consulting, Inc.
Not to be reproduced, copied or distributed. Intended for use only by the registrant.

C
Coooorrddiinnaattee G
Geeoom
meettrryy

You can expect with reasonable certainty In Quadrant I, both x and y values are
at least one question from this chapter in positive. In Quadrant II, x is negative and
each of the two problem solving sections y is positive. In Quadrant III, x and y are
and in the Data Sufficiency section of the both negative. In Quadrant IV, x is
GMAT. positive and y is negative.

The coordinate geometry deals with To find the distance AB, use
information in the coordinate plane. You Pythagorean theorem by completing a
have a horizontal line called the x-axis triangle as shown by the dotted lines.
and a vertical line called the y-axis. The Coordinate geometry is conducive to
point at which these axes meet is called easy measurement: We can see that
the origin, usually designated O. The AC=CB are both 4 units. Applying the
axes divide the plane into 4 Quadrants, Pythagorean theorem, we can conclude:
designated I, II, III and IV.
AB² = AC² + CB² = 4²+4²=32
y-axis 3
AB = √32 =4√2
II A 2 P I
Exercise:
1 x-axis
II I
-4 -3 -2 -1 0 1 2 3 4
III IV
III -1 IV
In the rectangular coordinate system
C -2 B shown above, which quadrant, if any,
contains no point (x,y) that satisfies the
-3 inequality 2x-3y ≤ -6 ?

Each point in the plane has an x- (A) None (B) I (C) II (D) III (E) IV
coordinate and a y-coordinate. A point in
the plane is denoted and identified by an We are required to find a quadrant, if one
ordered pair of numbers, conventionally exists, such that no point in that plane
denoted (x,y), where the x-coordinate will satisfy the inequality 2x-3y≤ -6.
value is the first number and the y-
Working with -6 on the right side of the
coordinate value is the second number.
inequality is a little clumsy. So we
The point P in the Quadrant I is denoted multiply both sides by -1 and reverse the
by (2,2). Point B is specified by the order of inequality as follows: 3y-2x≥6.
coordinates (3, -2). Point C is specified
by the coordinates (-1,-2). To get a handle on a problem of this
type, we have to use real-life numbers

Some of the materials used in this module are taken from disclosed editions of GMAT, and no monetary
value is included in the cost of this module for supply of these materials. These materials are strictly used
for illustration of concepts discussed. Page 175
©Educational Training Services, A Division of Maple Leaf International Consulting, Inc.
Not to be reproduced, copied or distributed. Intended for use only by the registrant.

into the inequality and see if the Now, try to do the following on your
condition is met. own and figure out which Quadrant will
not have a point such that the inequality
In Quadrant I, both x and y are positive. -5x + 3y -8 will be satisfied.
We can always choose a value of x and y
such that 3y-2x will be greater than 6. II y I
For instance, if we chose y=4 and x=1,
3y-2x has a value of 10, which is greater x
than 6. Therefore, there does exist at
least one point in Quadrant I that can III IV
satisfy the inequality. Therefore the
answer choice (B) is not valid. We move The Correct answer is Quadrant II.
on. Use the same argument that we have
discussed earlier to arrive at this
In Quadrant II, x is negative and y is conclusion.
positive. If we chose a point in Quadrant
II : (-1,3), then we have for 3y-2x a
value of 11, which is greater than 6.
Therefore answer choice (C) is not valid.

In Quadrant III, both x and y are


negative. If we chose a point (-5,-1),
then we have for 3y-2x a value of 7,
which is greater than 6. Therefore, we
can find at least one point in Quadrant III
such that the inequality is true. We can
eliminate choice (D). We move on.

In Quadrant IV, x is positive and y is


negative. No point exists in this quadrant
such that the inequality is satisfied, The fastest way to deal with such problems is
because there is a negative sign in front to identify the quadrant where the signs
attached to x and y values are such that the
of a positive value x and a positive sign
inequality will not be satisfied. In our
in front of a negative value y in the problem, If -5x + 3y is required to be less than
inequality being tested: 3y-2x ≥ 6. For all or equal to -8, then look for a quadrant where
values of x and y, the expression on the x values will be negative and y values positive
left side of the inequality will always be so that the inequality expression -5x + 3y will
negative in this Quadrant and the always be positive. Only quadrant II has -ve x
values and +ve y values.
inequality cannot be satisfied.

Therefore, we pick Quadrant IV as the If you had difficulty understanding


one containing no point to satisfy the this concept, let us know.
inequality, the correct answer. The
correct answer choice, therefore, is (E).

Some of the materials used in this module are taken from disclosed editions of GMAT, and no monetary
value is included in the cost of this module for supply of these materials. These materials are strictly used
for illustration of concepts discussed. Page 176
©Educational Training Services, A Division of Maple Leaf International Consulting, Inc.
Not to be reproduced, copied or distributed. Intended for use only by the registrant.

COORDINATE What is the equation for the straight line in the


GEOMETRY(Contd.) x-y plane connecting the points (5,27) and
(20,18) ?
The equation of a straight line in an x-y plane is
defined by the equation : y = mx + c Y = -3/5 . x + 30 or 3x + 5y = 150
Where m is the slope of the line, and c is the y How will this line look in an x –y plane ?
intercept or the value of y when x is 0. In other
SLOPE defines words, c is the y coordinate value of the point on
the constant the y axis where the straight line meets. +y
ratio of the (0,30)
vertical height How do we find the equation of a straight line in
to the an x-y plane? We require to be given
horizontal information pertaining to the slope and the y -x +x
distance of any intercept. (50,0)
point on the
straight line. In the alternative, we may simply want to know -y
The ratio of the the x and y values of at least 2 points on the As you can see, the straight line will pass
vertical height straight line. Let us say that we have two points through quadrants I, II and IV, but will not pass
of any point on (5,27) and (20,18) in an x-y plane and want to through quadrant III. How did we get the points
the straight line find out the equation for a straight line (0,30) and (50,0) ? We get (0,30) by writing 0
to the connecting these two points. How do we for x in the equation. A point that lies on the y
horizontal proceed? We start at the starting point, and say axis will have x value equal to 0. Similarly, we
distance of that that “Let the equation for the straight line be got (50,0) by writing 0 for y in the equation,
point from the y = mx + C because a point that lies on the x axis will have
end of the line We will find out the values for m and c by using y value equal to 0.
where the line the information about the points that lie on this
meets X axis is line.
If in the GMAT, you got a question that asks:
a constant. “The straight line 3x + 5y = 150 in an x-y plane
If (5,27) is a point on this line, then we can
will pass through all of the following quadrants
A write 5 for x and 27 for y. We get
except:
B 27 = m.5 + c ……………… (1)
(A) QI (B) QII (C) QIII (D) QIV (E) None”
C Similarly, if (20,18) is another point on the
How will you answer this question ?
same line, then we can write 20 for x and 18 for
D E F G y.
We get 18 = m.20 + c……………….(2) We can answer this question by constructing a
AD/DG =
straight line in an x-y plane as shown or by
BE/EG =
What do we see with this picture ? We have two reasoning that the x and y values in QIII are
CF/FG =
equations and two variables. Can we get the both negative so that 3x + 5y will be a negative
SLOPE (m)
values for m and c ? You bet. value for all points in quadrant III. Therefore,
the straight line defined by the subject equation
By setting up Let us subtract equation 2 from 1, so that we can will NOT pass through quadrant III.
an equation for get rid of c and get an equation in terms of m The slope of a straight line can also be
a straight line only. We get computed from the following formula:
connecting two = m (5-20) Or 9 = -15.m “If (x1, y1) and (x2, y2) are two points on a
specified points, Or m = -9/15 = -3/5 ……(3) straight line in an x-y plane, then the slope of
we are able to the line is: (y1 - y2)
get the handle Now that we have determined the value for m, (x1 - x2)
on many more let us get the value for c by plugging in the value Remember to use the same order for x and y
points on the for m in either equation – 1 or 2. values: If you start with y1 , then you should
straight line.
start with x1 for the denominator.
We get : 27 = -3/5 . 5 + c = -3 + c SEE ADDITIONAL DISCUSSION OF THE
Or, c = 30 ……………………… (4) RULES IN GEOMETRY FORMULAS FILE.

Some of the materials used in this module are taken from disclosed editions of GMAT, and no monetary
value is included in the cost of this module for supply of these materials. These materials are strictly used
for illustration of concepts discussed. Page 177
©Educational Training Services, A Division of Maple Leaf International Consulting, Inc.
Not to be reproduced, copied or distributed. Intended for use only by the registrant.

Now, try this question for size: Which means that the (x,y) values of D must be
“What is the equation for a (9,15).
straight line in an x-y plane
connecting the points (-3, 45) Notice how this problem combined the
and (–12,30) ?” Also, properties of triangles and those of co-ordinate
determine what quadrant, if geometry, and forced us to use a fair bit of
any, the line will NOT pass reasoning to arrive at the (x,y) values of the
through. point D ?

Another problem frequently encountered in the


GMAT goes like this:
Answer:(The equation is: y = 5x/3 + 50 and “If (a, 2b) and (a + 4, 2b +k) are two points on
the line will NOT pass through QIV) a straight line defined by the equation
x = -2y + 7, what is the value of k ?”

You should also expect to get problems in You can solve this problem in two different
GMAT testing your ability to combine concepts ways.
from more than one area, say triangles and co- APPROACH 1
ordinate geometry. Let us take a look at such a What is the slope of the line defined by the
problem: y equation x = -2y + 7 ? We need to express
D the equation in the form y = mx + c.
We get -2y = x - 7
Or -y = ½.. X - 7/2
-x +x Or y = - 1/2 .x + 7/2
A(-6,0) B C (24,0) We conclude that the slope is - ½ and the y
intercept is 7/2.
-y BD = BC
Now, let us take a look at the points on the
The above figure shows an isosceles triangle straight line: (a, 2b) and (a+4, 2b + k).
ADC such that AD = DC. The triangle lies in an We can see that for every 4 units movement of
x-y plane. What are the co-ordinates of point x value, the y value moves by k. The slope of the
D? line in terms of k must be the ratio of the y
increment to that of x increment.
What do we know about the properties of an Slope of the line = k/4 = - ½ or k = -2
isosceles triangle? We know that the vertical
line drawn from the vertex where the two sides APPROACH 2
of equal value meet to the opposite side divides If (a,2b) is a point on the straight line x = -2y+7,
the opposite side (base) in two equal halves. In then we can write a for x and 2b for y.
our problem, AB = BC = ½.. AC We get a = -2. 2b + 7 .................. (1)
Since A lies 6 units to the left of the center of Or a = - 4b + 7 ................ (2)
the x-y plane, and C lies 24 units to the right of Similarly, if (a+4, 2b+k) is another point on
the center, we determine that AC must be 30, the same line, we can write a+4 for x and 2b +k
and that AB = BC = ½.. 30 = 15 for y. We get
a + 4 = -2 (2b + k) + 7 ............. (3)
IF AB is 15 units, and knowing that A lies 6 Or a + 4 = -4b - 2k + 7 ................. (4)
units to the left of 0, we conclude that B MUST Subtract equation (2) from (4) to get
lie 15-6 = 9 units to the right of 0. Which a+ 4 - a = -4b - 2k + 7 -(-4b + 7)
means that the co-ordinates of B are (9,0) where 4 = -4b - 2k + 7 + 4b - 7
9 is the distance from the center along the x axis Or 4 = -2k or K = -2
and 0 is the y coordinate value of point B.
You can use either approach, but if you are not
The problem also specifies that BD = BC = 15. sure about determining the slope of the equation
correctly, then use the approach 2 suggested.
Some of the materials used in this module are taken from disclosed editions of GMAT, and no monetary
value is included in the cost of this module for supply of these materials. These materials are strictly used
for illustration of concepts discussed. Page 178
©Educational Training Services, A Division of Maple Leaf International Consulting, Inc.
Not to be reproduced, copied or distributed. Intended for use only by the registrant.

PROBLEM:
B(0,30)

A(50,0) A is a point in an x-y plane with (x,y) values


(-3, 56) and B is another point in the same plane
How many points exist on the line AB with coordinate values (24,-7). The straightline
between A and B, inclusive, such that the x connecting A and B intersects the Y axis at C
and y co-ordinate values of the points are and the X axis at D.
integers?
We recognize that the slope of the line is -3/5, 1. What is the length of the straightline
(how do we get this? Use the formula for connecting points A and B?
computing slope using two points) which means 2. What is the equation for the straight line
that you can go down y axis in increments of 3 connecting A and B?
and along x axis in increments of 5. If you went 3. What is the slope of the line connecting A
down along y-axis in steps of 3, and drew a line and B?
to the sloping line, you will find a point on the 4. What are the coordinates of C and D?
sloping line that has both x and y values 5. How many points exist along the line segment
integers. You can go down along the y axis in CD such that the x,y values of each of these
steps of 30,27,24,21,18,15,12,9,6,3,0 or there points will be integers?
are 11 points along AB with integer values for 6. If (6,35) and (12, 35+k) are two points on the
(x, y).. Or, you can move along the X-axis in line AB, what is the value for k ?
steps of 5: 0, 5,10,15,20,25,30,35,40,45,50 . A(-3,56)
11 points. C(?,?)

The other approach is to set up the equation for


the straight line. We notice that the y intercept D(?,?)
is 30, and the slope is –3/5.
The equation is: y = -3/5. X + 30 (24,-7) B
If x and y are required to be integers, then we
must find x values that are multiples of 5
between 0 and 50. The number of integers that
are multiples of 50 between 0 and 50, inclusive,
is 11.

ASSIGNMENT:

Some of the materials used in this module are taken from disclosed editions of GMAT, and no monetary
value is included in the cost of this module for supply of these materials. These materials are strictly used
for illustration of concepts discussed. Page 179
©Educational Training Services, A Division of Maple Leaf International Consulting, Inc.
Not to be reproduced, copied or distributed. Intended for use only by the registrant.

COORDINATE GEOMETRY 2. The equation for the straight line


connecting A and B is given by the
ASSIGNMENT EXPLAINED standard form: y = mx + b where m is the
A is a point in an x-y plane with (x, y) values (- slope and b is the y-intercept.
3, 56) and B is another point in the same plane The slope of the line is given by the ratio:
with coordinate values (24,-7). The straight-line (y1-y2)/(x1-x2)
connecting A and B intersect the Y-axis at C Where x1 and y1 values are the coordinates of
and the X-axis at D. any of the points, A or B, and x2 and y2
values are the coordinates of the other point.
1. What is the length of the straight-line
connecting points and A and B ? Slope of the line connecting A
2. What is the equation for the straight- line and B is : (56 – (-7)) / (-3 – 24) = -
connecting A and B ? 63/27 = -7/3
3. What is the slope of the line connecting A
and B ?
The equation for the straight-line can now be
4. What are the coordinates of C and D ?
written with this slope value as:
5. How many points exist along the line segment
Y = - 7/3 x + b
CD such that the x,y values of each of these
How do we get the value for b, the y-intercept ?
points will be integers ?
We can use any of the x, y values of point A or
6. If (6, 35) and (12, 35+k) are two points on the
B to get the value for b. Let us write –3 for x
line AB, what is the value for k ?
and 56 for y to get:
A(-3,56)
56 = - 7/3. (-3) + b = 7 + b
C(?,?)
Or b = 49
Therefore, the equation for the straight-line
connecting A and B is:
D(?,?)
Y = - 7/3 X + 49
M (24,-7) B
3. The slope of the line is -7/3.
4. The coordinates of C are (0,49).
Notice that the y-value of C is the y-
intercept, b, value determined above.
1.The length of the straight-line connecting
5. To get the coordinates of D, we have to
points A and B is obtained by using the
write y = 0 and find out the value for the x-
Pythagorean theorem:
coordinate. Why do we do this? Because D
lies on the X-axis, where the y -value is
AB2 = AM2 + BM2 (1)
zero.
How do we get the values for AM and
0 = -7/3. X + 49 or X = 21.
BM? The value for AM is obtained by
subtracting the y coordinate values of A and B: Therefore, the coordinates
(Notice that the y value of B is the same as the y of D are (21,0)
value of M) 6. We notice that the slope of the line is –7/3.
Similarly, we get the distance BM by Which means that we can go down y axis
subtracting the x values of A and B. (Notice that from C in steps of 7 or move along x-axis in
the x value of M is the same as the x value of steps of 3 from 0 to D, and that is how
A.) many points we will find on the line
We get AM = 56 – (-7) = 63 between C and D with (x,y) values whole
We get BM = 24 – (-3) = 27 number values.
Let us plug these values in the equation We can find (0,49), (3,42), (6,35), (9,28),
(1) above, and get the value for the length of (12,21), (15,14), (18,7), and (21,0) on the
AB: line segment between C and D – a total of 8
AB = √63.63 + 27.27 = 9√58 points.
Answer

Some of the materials used in this module are taken from disclosed editions of GMAT, and no monetary
value is included in the cost of this module for supply of these materials. These materials are strictly used
for illustration of concepts discussed. Page 180
©Educational Training Services, A Division of Maple Leaf International Consulting, Inc.
Not to be reproduced, copied or distributed. Intended for use only by the registrant.

can determine what common points


7. If (6,35), and (12, 35+k) are two points on exist for the two curves. Consider the
the straight-line AB, the slope in terms of k
is given by the expression: (35-35-k) / (6-
following problem:
12) = k/6
But we know that the actual slope is - 7/3, “If F(X,F(X)) is a point common to
and we can write: k/6 = -7/3 or both F(X1) = X12 + 8, and
F(X2) = X2 + 10, then F(X,F(X)) could
k = -14 be what?”

The common point must satisfy both


QUADRATIC
equations. Therefore, the common
FUNCTION IN AN X-
Y PLANE point, when substituted into each of
the two equations, will yield:
We have seen that a linear equation of the type
Y=mX + b gives rise to a straight line. IF we F(X) = X2 + 8 AND
have a quadratic function of the type
Y = X2 – 4, we will get a parabola if we plot X F(X) = X + 10
and Y points for various values of X and Y.
As we can see, both equations are for
If Y = X2 – 4, then the curve will be a parabola
that looks as follows:
the common F(X), and we can write:
X2 + 8 = X + 10
PARABOLA Or, X2 – X – 2 = 0 or (X-2)(X+1) = 0
QUADRATI C FUNCTI ON
IF X=0, F(X) = 2 OR -2 X = 2 or X = -1
6
5
4 The corresponding values for F(X)
3 are:
2
1
F(X)

0 If X = 2, then F(X) = X+10 = 12


-1
-2 IF X = -1, then F(X) = X+10 = 9
-3
-4
-5 Therefore, F(X,F(X)) could be (2,12)
-4 -3 -2 -1 0 1 2 3 4 or (-1, 9). The parabola and the
X VALUES straight line will intersect at these two
points. In the final analysis, a
problem of this type is simple algebra
Notice that Y has been replaced with
asking you to set a quadratic
F(X) because Y is in face a function of
expression equal to a linear
X even in a linear situation. Instead of
expression, and then to solve for X.
plotting Y values in the Y axis, we plot
Once we get X, find the corresponding
the F(X) values.
value for the other variable such as
If we are given a linear equation of the F(X).
type F(X1)= mX+b and a quadratic
function of the type F(X2) = X2- 4, we

Some of the materials used in this module are taken from disclosed editions of GMAT, and no monetary
value is included in the cost of this module for supply of these materials. These materials are strictly used
for illustration of concepts discussed. Page 181
©Educational Training Services, A Division of Maple Leaf International Consulting, Inc.
Not to be reproduced, copied or distributed. Intended for use only by the registrant.

statements: 1 and 2. Your task is to


determine if you can answer the question
asked based on the information presented
in statement 1 alone or statement 2 alone
or using the information presented in
both the statements.
The Data Sufficiency section of the
GMAT does not require you to solve a We will illustrate the appropriate strategy
problem presented; but, instead, to for this section with a couple of
determine at what stage or point do examples.
you have sufficient information
pertaining to the problem so that the 1. If √x / y = n, what is the value of x?
problem can be solved.
(1) y•n=10 (2) y=40 and n=1/4
The Data Sufficiency section will
require all the information that you Let us rearrange the given information as
have learned in this module thus far; follows: √x = n•y = y•n (commutative
the only difference is that, in data rule of algebra)
sufficiency, you are going to be given
information in a piece-meal fashion, Let us look at the statement 1 now. It
and you will be required to do the says that yn=10 which is equal to √x. We
following: have sufficient information to determine
the value of x. The answer choice will be
Be sure to make a note of any either A or D. Let us make a check mark
standard properties pertaining to against (1) and move on to statement (2).
the problem. For instance, if the
problem is about a 30-60-90 When you read statement (2), do not let
triangle, you should know that the the information presented in (1) influence
sides are in a standard proportion, you. In other words, put your blinkers on
and you will be required to use this with regard to statement 1 while reading
knowledge in the context of statement 2.
determining whether you can
answer the question. Statement 2 gives values for y and n
from which we can determine the value
Be sure to make a note of any of x. Statement 2 alone is sufficient too.
information that is furnished along
with the question itself. You are Therefore, we have a situation where
required to combine this Each statement ALONE is sufficient to
information with the additional answer the question. We pick answer
information in statements 1 and 2 choice D as the correct one.
independently, and determine
whether a unique solution is Remember: Even though we reached a
possible. point where we could answer the
question based on information in
In this Section of the test, you will be statement 1 alone, the test directions
given a question with two accompanying
Some of the materials used in this module are taken from disclosed editions of GMAT, and no monetary
value is included in the cost of this module for supply of these materials. These materials are strictly used
for illustration of concepts discussed. Page 182
©Educational Training Services, A Division of Maple Leaf International Consulting, Inc.
Not to be reproduced, copied or distributed. Intended for use only by the registrant.

require you to analyze statement 2 before We must pick E as the answer choice,
you decide if you must pick A or D. meaning that statements (1) and (2)
together are not sufficient.
Let us do one more to clarify this
section:

“Is the prime number p equal to 37 ?” Let us try one more for the road:

(1) p = n² + 1 where n is an integer. “Is x < 0 ?”

(2) p² is greater than 200. (1) -2x > 0 (2) x³ < 0”

Let us look at statement 1 alone. Since n Let us redefine the question. The
is an integer capable of taking on any question is: Is x a negative number?
value, p may be a prime number if n=6
but if n=5, p is not only not a prime Consider statement (1) alone. For -2x to
number but is nowhere close to 37. be a positive number, x must be a
negative number. We can, therefore,
We conclude that statement 1 alone is conclude on the basis of statement 1
not sufficient and we make a cross mark alone that x is less than 0. We put a
against statement 1. check mark against (1). The answer
choice is A or D.
Let us look at statement 2 and consider it
solely on its own merits. Now we consider statement 2 alone.
That x³ is a negative number (less than 0)
The fact that p² is greater than 200 indicates that x must be a negative
indicates that p is greater than 14. Which number.(Cube of a positive number is
means that p could be 17 or 19 or 23 or positive and a negative number is
29 or 31 or 37 or 41. Since there is no negative). Therefore statement 2 alone is
unique value for p we can determine sufficient to answer the question. We
based on the information provided, we place a check mark against statement 2
conclude that statement 2 alone is not also.
sufficient. We put a cross mark against 2.
What do we have now? We have a
Can we combine the information in 1 and situation where either statement alone is
2 and then be in a position to determine sufficient and we pick D as the answer
if p is 37 ? choice.
p could be a prime number with a value We will deal with specific Data
of n=4 in the first statement. Such a Sufficiency reasoning techniques later on
value will satisfy the requirement in in Module II supplement, which follows
statement 2 as well. Therefore we cannot after the Problem Solving Exercises.
conclude on the basis of combined
information in both statements that p is Now, go to the assignment involving
37. problem solving exercises from recent
GMAT test sections.

Some of the materials used in this module are taken from disclosed editions of GMAT, and no monetary
value is included in the cost of this module for supply of these materials. These materials are strictly used
for illustration of concepts discussed. Page 183
©Educational Training Services, A Division of Maple Leaf International Consulting, Inc.
Not to be reproduced, copied or distributed. Intended for use only by the registrant.

Some of the materials used in this module are taken from disclosed editions of GMAT, and no monetary
value is included in the cost of this module for supply of these materials. These materials are strictly used
for illustration of concepts discussed. Page 184
©Educational Training Services, A Division of Maple Leaf International Consulting, Inc.
Not to be reproduced, copied or distributed. Intended for use only by the registrant.

Y
ou are going to be constantly Solving with Equations:
tested with regard to your ability Assignment
to reduce word descriptions of
problems to mathematician-speak, a.k.a 1. X is twice the value of Y and three
equations, and solve for as yet unknown times the difference of Z and Y (Z > Y).
quantities. This Section of exercises will If Z is 4, what are X and Y?
test your ability to do precisely that.

Example: Of the three varieties of


lettuces, the Romaine variety has 10
times the nutrients content of the Iceberg
variety and 5 times the combined
nutrients content of the Iceberg and
Boston varieties. If the Boston variety (Answer: x=24/5 and y = 12/5)
has the nutrients content of 20, what is
the nutrients content of the Romaine 2. In a weight-lifting competition, King
lettuce? Kong had the weights of his first three
lifts total 900 pounds. If the combined
Step 1: Reduce the verbosity into weight of his second and third lifts is 50
equations: Let R, I and B represent the pounds less than three times the weight
nutrients content respectively of the of his first lift and the weight of his third
Romaine, Iceberg and the Boston lift is 50 pounds more than his first lift,
varieties of Lettuce. what was the weight of King Kong’s
second lift?
R = 10. I --------- (1)

R = 5. (I + B) ------(2) & B = 20 --(3)

Step 2: Reduce the number of equations


by substituting known values into
equations. When we substitute 20 for B
in (2), we get R = 5(I + 20) -------(4)
<answer: x=237 ½ y=375 z=287 ½ >
We have 2 expressions for R (1 and 4)
and they are equivalent. We get:
10.I = 5 (I+20) = 5.I + 100 or I = 20.

From (1), we calculate R = 10.20 = 200

You are going to need this skill to do


well in the GMAT. Get used to it.

Please complete the following


assignment.
3. The average (arithmetic mean) of x, y
and z is 6. The product of the three
Some of the materials used in this module are taken from disclosed editions of GMAT, and no monetary
value is included in the cost of this module for supply of these materials. These materials are strictly used
for illustration of concepts discussed. Page 185
©Educational Training Services, A Division of Maple Leaf International Consulting, Inc.
Not to be reproduced, copied or distributed. Intended for use only by the registrant.

positive numbers is 192. If x is 4, what


is yz? What is y+z?

<Answer: yz=48; y+z = 14>

Some of the materials used in this module are taken from disclosed editions of GMAT, and no monetary
value is included in the cost of this module for supply of these materials. These materials are strictly used
for illustration of concepts discussed. Page 186
©Educational Training Services, A Division of Maple Leaf International Consulting, Inc.
Not to be reproduced, copied or distributed. Intended for use only by the registrant.

4. The equation N = d.q + r is used to MODULE II SUPPLEMENT


denote the relationship between the
dividend(N),divisor(d), the quotient(q)
and the remainder(r). 1. A recreation club has 40 members, How many
male members does the club have?
Express the following numbers as an
(1) Exactly 1/4 of its male members take
equation with the relationship shown swimming lessons.
above.
(2) Exactly 1/5 of its female members take
(A) N = 78 d = 9 swimming lessons.

<78 = 9•8 + 6> In a data sufficiency problem, you are required


to determine at what point you have sufficient
(B) N = 108 d = 7 information to answer the question with a
unique value. Which means that each statement
must be good for a unique value for the
information sought: How many male members
are there in the club?
<108 = 7•15 + 3>
Let us first examine the problem statement itself
(C) N = a d = 6 before we move on to examine the statements
(1) and (2).

What information do we have from the stem?


< a = 6•q + r> Simply this: There are 40 members in the club.
We have no other information given.
5. A jar contains candies in three colors:
Let us move on to examine statement (1).
Red, Blue and Green. There are exactly
120 candies in the jar. If there are twice Statement (1) tells us that exactly 1/4 of the
as many red candies as green ones and club’s members take swimming lessons.
three times as many blue candies as green
ones, how many red candies are there in What do we understand this statement to mean?
Simply this: The number of male members in
the jar?
the club is a multiple of 4. Which means that the
possible numbers are 4, 8, 12, 16, 20, 24, 28,
32, and 36. Why is 40 not a possible number for
male members? Let us be chivalrous and leave
<Red=40; Green=20; Blue = 60> some room for the female members, OK?

6. If John’s age n years ago was m, how Do we have a unique value generated from
old will John be p years from now? information in statement (1) ? Nope.
(Express John’s Age, J = )
What do we do at this stage of the game? We
simply eliminate statement (1) as sufficient by
itself and must move on to examine statement
(2).
<J = m+n+p>
What possible answer choices do we have at this
juncture? B, C, or E.

Some of the materials used in this module are taken from disclosed editions of GMAT, and no monetary
value is included in the cost of this module for supply of these materials. These materials are strictly used
for illustration of concepts discussed. Page 187
©Educational Training Services, A Division of Maple Leaf International Consulting, Inc.
Not to be reproduced, copied or distributed. Intended for use only by the registrant.

Let us make the following notations against the


question so that we will stay focused as to our
possible choices:
1. A recreation club has 40 members, How
many male members does the club have?

1. A recreation club has 40 members, How many


male members does the club have? X (1) Exactly 1/4 of its male members
take swimming lessons.

X (1) Exactly 1/4 of its male members


B,C,E (2) Exactly 1/5 of its female members
take swimming lessons.
take swimming lessons.

B,C,E (2) Exactly 1/5 of its female members When we combine the information (1) and (2),
take swimming lessons. what do we see?

What did we just do? We put a cross mark Statement (1) tells us that the possible values for
against statement (1) meaning that we have male membership are 4, 8, 12, 16, 20, 24, 28,
eliminated that as a possible choice and even as 32, and 36.
we begin to examine statement (2), we want to
keep in focus that our possible choices at this Statement (2) tells us that the possible values for
point are B,C, or E. So we write B, C,or E against male membership are 35, 30, 25, 20, 15, 10,
statement (2). and 5.

Let us examine statement (2). Do we see a common number in both


statements? Yes, we do. 20.
Statement (2) tells us that exactly 1/5 of the
female members are taking swimming lessons. What does that mean? Simply this: When we
combine statement (1) and (2) we can determine
What do we understand this statement to mean? that the number of male members in the club is
The number of female members in the club is a 20, a unique value.
multiple of 5. Which means that the possible
numbers are 5, 10, 15, 20, 25 30, and 35. As a Therefore the combined information is good
corollary, we understand that this statement enough to answer the question with precise
means that the possible values for male information and we pick C as the answer
members are 35, 30, 25, 20, 15, 10, or 5. choice.

Do we get a unique value for the male 1. A recreation club has 40 members, How
membership from this statement? Hardly. We many male members does the club have?
have a range of values for male membership
which is not good enough.
X (1) Exactly 1/4 of its male members
So what do we conclude? We conclude that take swimming lessons.
statement (2) alone is not sufficient to answer
the question: How many male members are
there in the club?
B,C,E (2) Exactly 1/5 of its female members
take swimming lessons.
Now, what possible answer choices do we have?
C or E. Do you understand the reasoning process
involved with data sufficiency problems and the
Let us cross out the B against Statement (2) to procedure involved in picking the correct
indicate that we have eliminated B is no longer answer choice?
a viable answer choice.
Some of the materials used in this module are taken from disclosed editions of GMAT, and no monetary
value is included in the cost of this module for supply of these materials. These materials are strictly used
for illustration of concepts discussed. Page 188
©Educational Training Services, A Division of Maple Leaf International Consulting, Inc.
Not to be reproduced, copied or distributed. Intended for use only by the registrant.

Remember: Each statement alone or together


must be good for a UNIQUE value for the
information sought. If you get a range of values
from the statements, either alone or together,
then you must choose E.

Remember the following drill:

You pick Answer A if statement (1) alone


generates a unique value for the information
sought but statement (2) alone does not.

You pick Answer B if statement (2) alone is


good for a unique value for the information
sought but statement (1) alone is not.

You pick Answer D if statement (1) and (2),


each considered alone, are good for a unique
value for the information sought.

You pick Answer C if statement (1) and (2)


together generate a unique value for the
information sought.

You pick Answer E if none of the above is true.


That means that the statements (1) and (2),
either alone or together, cannot generate a
unique value for required information.

Some of the materials used in this module are taken from disclosed editions of GMAT, and no monetary
value is included in the cost of this module for supply of these materials. These materials are strictly used
for illustration of concepts discussed. Page 189
©Educational Training Services, A Division of Maple Leaf International Consulting, Inc.
Not to be reproduced, copied or distributed. Intended for use only by the registrant.

Let us illustrate the process and the procedure 2. A jar contains 30 marbles, of which 20 are
with another data sufficiency problem. red and 10 are blue. If 9 of the marbles are
removed, how many of the marbles left in the jar
2. A jar contains 30 marbles, of which 20 are are red?
red and 10 are blue. If 9 of the marbles are
removed, how many of the marbles left in the jar A, D (1) Of the marbles removed, the ratio of
are red?
√ the number of red ones to the number
(1) Of the marbles removed, the ratio of of blue ones is 2 : 1
the number of red ones to the number
of blue ones is 2 : 1 (2) Of the first 6 marbles removed, 4
are red.
(2) Of the first 6 marbles removed, 4
are red. Let us proceed to examine statement (2) now.

What is the information we are seeking? From this statement we know that the first 6
How many marbles left in the jar are red after marbles removed had 4 red ones. We do not
removing 9 marbles ? know what the next 3 were. They could have
been all blue or all red or any combination of the
Before we go to examine the statements, two color.
remember that each statement alone or together
must be good for a unique value for the Since we cannot precisely estimate the value for
information sought. the red marbles removed - and, therefore, the
value for the red ones remaining in the jar - we
Let us examine statement (1) eliminate Statement (2) as sufficient
information.
The ratio of red ones to the blue ones is 2:1 of
the 9 marbles removed. Which means that 2/3 In the process of eliminating statement (2) as
of the marbles removed were red and 1/3 of the sufficient, we have eliminated D as an answer
marbles removed were blue. (Or 6 red ones and choice. We therefore pick A and mark it on the
3 blue ones). Remember: We do not have to answer sheet or use the mouse to click the
bother to compute how many red ones were answer choice (Only for GMAT-CAT)
removed etc. Suffice it to know that the
information given is good for a determination of 2. A jar contains 30 marbles, of which 20 are
the value. red and 10 are blue. If 9 of the marbles are
removed, how many of the marbles left in the jar
Once we can determine from the information are red?
how many red ones were removed, we can
determine how many were left behind in the jar.
A, D (1) Of the marbles removed, the ratio of
So what do we conclude? That statement (1) is √ the number of red ones to the number
sufficient for the purposes of answering the of blue ones is 2 : 1
question with a unique value.

What are the answer choices we have at this


X (2) Of the first 6 marbles removed, 4
are red.
point in time? A or D. Remember: We still have
to examine statement (2) to determine if it can
generate a unique value for the information
sought too, in which case the choice will be D. Let us take another question to get familiar with
the reasoning process and the elimination
So we proceed to make the following markings procedure for the answer choices.
on the test booklet against the statements.

Some of the materials used in this module are taken from disclosed editions of GMAT, and no monetary
value is included in the cost of this module for supply of these materials. These materials are strictly used
for illustration of concepts discussed. Page 190
©Educational Training Services, A Division of Maple Leaf International Consulting, Inc.
Not to be reproduced, copied or distributed. Intended for use only by the registrant.

If w and z are consecutive odd integers and if


3. If w + z = 28, what is the value of wz ? they add up to 28, then the only set of values for
w and z are 13 and 15. (If w is 13, then z is 15
(1) w and z are positive integers. or vice versa). Can we now determine the value
of wz precisely? You bet.
(2) w and z are consecutive odd
integers. So what do we conclude? We conclude that
statement (2) ALONE is sufficient to answer the
What is the information being sought? questions precisely and we cross out C and E to
The value of wz. pick B as the answer choice.

Remember: Each statement alone or together We go to the answer sheet and blacken the
must be good for a unique value for wz. If you corresponding oval, or move the mouse to click
get a range of possible values, then what do we on the answer choice (GMAT-CAT).
do to that statement? We deep-six it; we say
“good riddance to bad rubbish”; we say adios;
we say sayonara and move on. 3. If w + z = 28, what is the value of wz ?

Now. Let us start with the statement (1). X (1) w and z are positive integers.

We understand from statement (1) that w and z B, C, E (2) w and z are consecutive odd
can have any range of positive values. For integers.
instance these are the possible values for w and
z: Let us consider another question from GMAT.

w 1 2 ........... 25 26 27 4. Last Friday a certain shop sold 3/4 of the


z 27 26 3 2 1 sweaters in its inventory. Each sweater sold for
$20. What was the total revenue from the sale of
Do you see what is wrong with this picture here? these sweaters?

Can we generate a unique value for wz from this (1) When the shop opened for business
statement? No way, Jose. last Friday, there were 160 sweaters in
its inventory.
So what do we do? We conclude that statement
(1) is not sufficient to answer the question (2) All but 40 sweaters in the shop’s
precisely. We, therefore, put an X against inventory were sold last Friday.
statement (1) and write B, C, E against
statement (2) to indicate that those are the
What is the information we are seeking?
possible choices we are faced with at this
Revenue. What is revenue? Number of sweaters
juncture.
sold times the price of each sweater. What do we
know from the information given to us on a
3. If w + z = 28, what is the value of wz ? silver platter? We know the selling price:
$20.00. What information do we need now? We
X (1) w and z are positive integers. need to know how many sweaters were sold in
precise numbers.
B, C, E (2) w and z are consecutive odd
integers. You get the drift here? We have to determine
ahead of time what is the nature of “sufficient”
information we are seeking before we move on
Let us move on to examine statement (2).
to examine the statements. Here the “sufficient”
information is the quantity of sweaters sold and

Some of the materials used in this module are taken from disclosed editions of GMAT, and no monetary
value is included in the cost of this module for supply of these materials. These materials are strictly used
for illustration of concepts discussed. Page 191
©Educational Training Services, A Division of Maple Leaf International Consulting, Inc.
Not to be reproduced, copied or distributed. Intended for use only by the registrant.

that will help you calculate revenue using the


selling price of each sweater specified.

Let us examine statement (1) 4. Last Friday a certain shop sold 3/4 of the
sweaters in its inventory. Each sweater sold for
Statement (1) tells us that the shop had 160 $20. What was the total revenue from the sale of
sweaters when it opened its doors for business these sweaters?
that day. Combine this with the fact that 3/4 of
the sweaters in the inventory were sold, we can (1) When the shop opened for business
determine how many were sold. (3/4 of 160).
A, D last Friday, there were 160 sweaters in
its inventory.
Is statement (1) sufficient information? YES.
So what do we do now? We write A, D against
statement (1) in the test booklet to indicate that √ (2) All but 40 sweaters in the shop’s
those are the choices we have at this point in inventory were sold last Friday.
time and move on to examine statement (2)
4. Last Friday a certain shop sold 3/4 of the Let us consider one more to drive home the
sweaters in its inventory. Each sweater sold for reasoning process before we move on to
$20. What was the total revenue from the sale of assignments.
these sweaters?
5. What is the value of x?
(1) When the shop opened for business
A, D last Friday, there were 160 sweaters in (1) x is a prime number
its inventory.
(2) 31 x 37
(2) All but 40 sweaters in the shop’s
inventory were sold last Friday. What is the information we are seeking?
Value of x . A precise value of x . No less.

What does statement (2) tell us? Remember we Let us examine statement (1)
have to examine statement (2) independently of Statement (1) tells us that x is a prime number.
our knowledge of statement (1) information. Is there just one prime number? No. How many
prime number values we can think of for
Statement (2) tells us that 40 sweaters were left possible values of x ? Endless.
unsold. From the information specified in the
question, what fraction of sweaters were unsold? Since we cannot get a precise or unique value
1/4 . How do we know this? Because the for X from statement (10, what do we do? We
problem states that the shop sold 3/4 of its conclude that it is not sufficient and move on to
inventory leaving 1/4 unsold. examine statement (2).

If 40 represents 1/4 of the inventory, we can What are the possible answer choices we have at
determine how many were sold. (3 times as his stage of the game? B, C, or E.
many).
Let us do the drill to make sure that we stay
Is statement (2) good for the information we are focused on our options.
seeking. You bet.

So what do we have now? We have a situation


in which both statements standing alone are
good for the information we are seeking.

In this situation what is our answer choice? D.

Some of the materials used in this module are taken from disclosed editions of GMAT, and no monetary
value is included in the cost of this module for supply of these materials. These materials are strictly used
for illustration of concepts discussed. Page 192
©Educational Training Services, A Division of Maple Leaf International Consulting, Inc.
Not to be reproduced, copied or distributed. Intended for use only by the registrant.

5. What is the value of x? The reasoning process and the procedures


involved in choosing and eliminating answer
choices need to be mastered by you until it
X (1) x is a prime number
becomes second nature. How long does it take to
do that? 30 minutes for some and 60 minutes for
B, C, E (2) 31 x 37 most others.

Statement (2) specifies a narrow range of The following assignment is intended to give
possible values for x. But then it is a range and you the much-needed practice with the
not a unique value. So we conclude that reasoning process, and the procedure involved
statement (2) alone is not sufficient to answer in choosing and eliminating answer choices.
the question. We eliminate B as a possible
answer choice and move on to examine the
combined information in statements (1) and (2).
The answer choices are limited to C or E at this
stage.

5. What is the value of x?

X (1) x is a prime number

B, C, E (2) 31 x 37

When we combine statements (1) and (2), we


understand that x is a prime number and in the
range specified in statement (2), we have at least
two prime numbers: 31 and 37.

Once again, the combined information at best


gives us 2 possible values for x. Is that good
enough. No way. We need a single, precise,
unique value for the variable from either or the
combined statements.

Since we do not get that, we have to conclude


that E is the best choice.

5. What is the value of x?

X (1) x is a prime number

B, C, E (2) 31 x 37

We go and mark (or click with the mouse)


answer choice E on the answer sheet (computer
screen).

Some of the materials used in this module are taken from disclosed editions of GMAT, and no monetary
value is included in the cost of this module for supply of these materials. These materials are strictly used
for illustration of concepts discussed. Page 193
©Educational Training Services, A Division of Maple Leaf International Consulting, Inc.
Not to be reproduced, copied or distributed. Intended for use only by the registrant.

In this assignment, explain your reasoning and


the procedures for eliminating and choosing
answer choices as discussed earlier in this
module for each question.

1. What is the capacity, in liters, of a certain


aquarium?

(1) Three liters is 1/2 of the capacity of the


aquarium.
(2) One-half liter is 1/12 of the capacity of the
aquarium.

2. In Triangle PQR, what is the measure of


angle P?

(1) Angle Q is a right angle.


(2) The measure of angle R is 17 degrees.

Some of the materials used in this module are taken from disclosed editions of GMAT, and no monetary
value is included in the cost of this module for supply of these materials. These materials are strictly used
for illustration of concepts discussed. Page 194
©Educational Training Services, A Division of Maple Leaf International Consulting, Inc.
Not to be reproduced, copied or distributed. Intended for use only by the registrant.

5. What is the value of x?

3. What amount did Jean earn from the (1) 2x + 3y = 12


commission on her sales in the first half of (2) 5x + 7y = 29
1988?

(1) In 1988 Jean’s commission was 5 percent of


the total amount of her sales.
(2) The amount of Jean’s sales in the second
half of 1988 averaged $10,000 per month more
than in the first half.

6. Does x = y?

(1) | x | = | y |
(2) x2 = y2

4. A certain car went from one town to another


without stopping. What was the car’s average
speed for the trip?

(1) The car traveled the 90-mile trip in 2 hours.


(2) The car traveled the first 40 miles of the trip
in 1 hour.

Some of the materials used in this module are taken from disclosed editions of GMAT, and no monetary
value is included in the cost of this module for supply of these materials. These materials are strictly used
for illustration of concepts discussed. Page 195
©Educational Training Services, A Division of Maple Leaf International Consulting, Inc.
Not to be reproduced, copied or distributed. Intended for use only by the registrant.

7. Dan took 20-question multiple-choice test in


psychology. If Dan answered every question, did
he answer at least 12 questions correctly? 10. How many people did Apex company
employ in 1990?
(1) Dan answered fewer than 40 percent of the
questions incorrectly. (1) The company employed 538 more people in
(2) Dan answered at least 25 percent of the 1991 than in 1990.
questions incorrectly. (2) The company employed 20 percent more
people in 1991 than in 1990.

8. If a < x < b and c < y < d, Is x < y?


11. Of the four numbers represented on the
(1) a < c number line below, is r closest to zero?
(2) b < c

q r s t

(1) q = -s
(2) -t < q

9. Is (3x + 8)/ (x + 2 ) an integer?

(1) x is an integer
(2) x = 0

Some of the materials used in this module are taken from disclosed editions of GMAT, and no monetary
value is included in the cost of this module for supply of these materials. These materials are strictly used
for illustration of concepts discussed. Page 196
©Educational Training Services, A Division of Maple Leaf International Consulting, Inc.
Not to be reproduced, copied or distributed. Intended for use only by the registrant.

12. Is the integer n divisible by 20?


14. If x is not equal to - 1 , which is greater,
(1) n is divisible by 5 1/(x + 1) or x / 2?
(2) n is divisible by 6
(1) x ≥ 0
(2) x < 3

13. If Mark saved an average (arithmetic


mean) of $80 per week for 3 consecutive weeks,
how much did he save the second week?

(1) The average amount that Mark saved per


week for the first 2 weeks was $60. 15. In a certain two-digit integer, the ratio of
(2) The amount that Mark saved the first week the units digit to the tens digit is 2 to 3. What is
was 1/2 the amount he saved the second week the integer?
and 1/3 the amount he saved the third week.
(1) The tens digit is 3 more than the units digit.
(2) The product of the two digits is 54.

Some of the materials used in this module are taken from disclosed editions of GMAT, and no monetary
value is included in the cost of this module for supply of these materials. These materials are strictly used
for illustration of concepts discussed. Page 197
©Educational Training Services, A Division of Maple Leaf International Consulting, Inc.
Not to be reproduced, copied or distributed. Intended for use only by the registrant.

that statement 2 alone is also not sufficient to


answer the question. We have eliminated B as a
viable choice at this juncture.
We must now combine the two statements, and
see if we can make a determination of what P
must be. When we combine the two statements,
we notice that P+Q+R = 180o and Q+R = 107o.
1. What is the capacity, in liters, of a certain We must conclude that P must be 63o. A unique
aquarium? solution. We must pick C as the answer.

(1) Three liters is 1/2 of the capacity of the


aquarium. 3. What amount did Jean earn from the
(2) One-half liter is 1/12 of the capacity of the commission on her sales in the first half of
aquarium. 1988?
Statement 1 tells us that ½ (Capacity) = 3 liters.
We can determine that the capacity of the (1) In 1988 Jean’s commission was 5 percent of
aquarium is 6 liters. Statement 1 alone is the total amount of her sales.
sufficient to answer the question. We still need (2) The amount of Jean’s sales in the second
to examine statement 2 independently, and half of 1988 averaged $10,000 per month more
determine whether we can answer the question than in the first half.
using statement 2 alone also. Our choices
narrow to A or D. We must pre-determine what information will
Statement 2 tells us that 1/12 (Capacity) = ½ help us answer the question here. We require the
liter. We can determine that the capacity of the following information to be able to answer the
aquarium is 12 liters. Statement 2 is also question:
independently sufficient to answer the question. ◊ Dollar value of sales in the first half of 1988
We must pick Choice D.
◊ Jean’s commission rate

Statement 1 tells us that the commission rate is


2. In Triangle PQR, what is the measure of
5%. We have no information about the dollar
angle P ?
value of sales. We must conclude that statement
1 alone is not sufficient. Our choices are B,C or
(1) Angle Q is a right angle.
E.
(2) The measure of angle R is 17 degrees.
Statement 2 tells us that
We need to use our knowledge of the properties Sales in 2nd half = Sales in 1st half + $60,000
of a triangle while examining the statements. If This information alone is not sufficient to
we have information about 2 angles, we can determine what was the dollar value of sales in
determine the third angle value. the first half. Also, we do not know from this
Statement 1 tells us that Q is a right angle. The statement at what rate Jean was paid
other two angles must add up to 90 degrees as commission. We must find statement 2 also
well, but we have no means of knowing what P wanting. Our choices are C or E.
must be. All that we know is that Angle P +
Angle R = 900. This information is not enough Even when we combine the two statements, we
to let us determine what P must be. We conclude do not have any information that helps us
that Statement 1 alone is not sufficient. We must determine the dollar value of sales in the first
move on to examine statement 2. Our choices half. We cannot answer the question using the
are B, C and E. statements independently or combined. We must
pick E.
Statement 2 tells us that angle R is 170. We can
conclude that P + Q = 1630. This information is
not sufficient to answer the question because P
and Q can have any number of combinations of
values adding up to 1630. We must conclude
Some of the materials used in this module are taken from disclosed editions of GMAT, and no monetary
value is included in the cost of this module for supply of these materials. These materials are strictly used
for illustration of concepts discussed. Page 198
©Educational Training Services, A Division of Maple Leaf International Consulting, Inc.
Not to be reproduced, copied or distributed. Intended for use only by the registrant.

4. A certain car went from one town to another 6. Does x = y?


without stopping. What was the car’s average
speed for the trip? (1) | x | = | y |
(2) x2 = y2
(1) The car traveled the 90-mile trip in 2 hours.
(2) The car traveled the first 40 miles of the trip Statement 1 tells us that the absolute
in 1 hour. values of x and y are equal. We must
We need to know the total distance traveled, remember that the absolute value
and the total time it took to travel the signifies the distance of the value on a
distance. Average speed = Distance/Time number line from the center, and that
Statement 1 tells us that the total distance
the absolute values of 3 and –3 are the
traveled was 90 miles, and the total time of same. WE cannot conclude with any
travel was 2 hours. We can determine the certainty that x and y are equal to
average speed using these two items of each other. Our choices are B,C or E.
information. Our choices are A or D. Statement 2 tells us that x and y have the same
absolute values. For example, x could be +2 and
Statement 2 is not so precise. This y could be –2, and x2 = y2. Or, x and y could
statement tells us that a distance of 40 miles both be +2 and +2. We have no means of
was traveled in the first hour. We do not knowing whether the former scenario is valid or
know how many more miles or how much the latter. Statement 2 is also not good for a
longer the car traveled. Statement 2 is not unique determination. We have eliminated B as
good for a unique solution. a viable choice. We are left with C and E.
When we combine the two statements, we notice
We must pick choice A. that we are dealing with the same information in
two different forms, and that we cannot answer
5. What is the value of x? the question with any certainty. We must pick
choice E.
(1) 2x + 3y = 12
7. Dan took 20-question multiple-choice test in
(2) 5x + 7y = 29 psychology. If Dan answered every question, did
he answer at least 12 questions correctly?
Statement 2 gives us an equation with two
variables, x and y. We require at least 2 sets (1) Dan answered fewer than 40 percent of the
of independent information to determine the questions incorrectly.
value for x. (2) Dan answered at least 25 percent of the
Our choice are B, C or E. questions incorrectly.
Statement 2 also gives us a two variable The question is: Did Dan answer at least
information, and we cannot determine what 60% of the questions correctly.
x must be using statement 2 alone. We have Statement 1 tells us that Dan answered less than
eliminated B as a viable option now. 40% incorrectly. This means that Dan answered
When we combine the two statements, we more than 60% correctly. Statement 1 is good
notice that we have two INDEPENDENT for a unique determination of the answer to the
sets of information about x and y, and we question. Our choices are A or E.
can solve for x and y. We must pick choice Statement 2 is not good for a unique solution.
C The fact that Dan answered more than 25%
incorrectly could mean that he answered 30%
incorrectly (70% correctly) or all 100%
incorrectly. Statement 2 gives us “all over the
map” solution. We must pick choice A.

Some of the materials used in this module are taken from disclosed editions of GMAT, and no monetary
value is included in the cost of this module for supply of these materials. These materials are strictly used
for illustration of concepts discussed. Page 199
©Educational Training Services, A Division of Maple Leaf International Consulting, Inc.
Not to be reproduced, copied or distributed. Intended for use only by the registrant.

8. If a < x < b and c < y < d, Is x < y?


10. How many people did Apex company
(1) a < c employ in 1990?
(2) b < c
(1) The company employed 538 more people in
Statement 1 is not good for a unique 1991 than in 1990.
determination because a could be 1, x 2, c 5 (2) The company employed 20 percent more
and y 6. (In this case x < y). Or, a could be 5, people in 1991 than in 1990.
x 10, c 6 and y 7. (in which case, x > y).
Statement 1 gives us “may be, but not Statement 1 tells us that
necessarily” solution to the question. We Number in 1991 = Number in 1990 + 538
must move on. Our choices are B, C or E. We cannot determine a unique value for the
Statement 2 is right on the money. IF b < c, number of people employed in 1990 using this
we can combine the two inequalities into one information alone.
whole: a<x<b<c<y<d. Our choices are B, C or E.
This picture tells us that x < y. Statement 2 is
good for a unique answer. We must pick B. Statement 2 tells us that
Number in 1991 = Number in 1990+0.2 (Number in 1990)
WE cannot determine the number employed in
1990 using this information alone.
9. Is (3x + 8)/(x + 2) an integer?
Our choices narrow to C or E.
(1) x is an integer
(2) x = 0 When we combine the two statements, we notice
that
0.2 X (Number employed in 1990) = 538
We are looking for a value for x so that we
We can conclude that the number employed in
can answer the question definitively.
538 must be 5 times 538.
The combined information is good for a unique
Statement 1 tells us that x is a whole number.
determination of the number of people employed
If x = 0, then the expression is an integer. If
in 1990.
x=1, another integer, the expression is not a
We must pick C as the choice.
whole number. We are beginning to see
conflicting answers right here. We must
conclude that statement 1 is not good for a
unique answer. We must move on to examine
statement 2. Our choices are B, C or E.

Statement 2 tells us that x = 0. Using this


information, we can determine one way or the
other whether the given expression is an
integer or not. We can answer the question
uniquely using this information alone. We
must pick B. (Statement 1 alone is not
sufficient, but statement 2 alone is sufficient.)

Some of the materials used in this module are taken from disclosed editions of GMAT, and no monetary
value is included in the cost of this module for supply of these materials. These materials are strictly used
for illustration of concepts discussed. Page 200
©Educational Training Services, A Division of Maple Leaf International Consulting, Inc.
Not to be reproduced, copied or distributed. Intended for use only by the registrant.

11. Of the four numbers represented on the


number line below, is r closest to zero? 12. Is the integer n divisible by 20?

(1) n is divisible by 5
q r s t (2) n is divisible by 6
The question is: Is n a multiple of 20?
(1) q = -s
(2) -t < q
Statement 1 tells us that n is a multiple of 5.
Statement 1 tells us that q and s have the same What are the possible values for n?
absolute values, and that q and s lie the same N could be 5, 10, 15, 20, 25, 30, 35, 40,……
distance from 0 on either side of 0. This means If n is 5, then it is not a multiple of 20. If n is 20
that 0 must occur smack in the middle of q and or 40 or 60 (all multiples of 20), then it is. We
s, and the only value that is between q and s is r, are looking at a “may be, but not necessarily”
and r must be closest to 0. Statement 1 alone is type of an answer. Not good. Our choices are B,
sufficient to answer the question definitively. C or E.
We must now examine statement 2 to determine Statement 2 tells that n could be
whether we can answer the question also using 6,12,18,24,30,… 60,…..
the statement 2. Our choices are A or D. If n is 12, it is not a multiple of 20. If n were 60,
it is. Once again, “may be, but not necessarily”
Statement 2 tells that the mirror image of t kind of an answer. We must now combine the
occurs to the left of q on the number line shown. two statements and see if we can make any sense
We know that 0 occurs midway between t and – of the statements.
t, but we have three values – q, r, and s –
between t and –t. Any one of these three values When we combine the two statements, we
could be close to 0. Statement 2 is not good for a determine that n is a multiple of 30 (least
unique determination as to whether r is the common multiple of 5 and 6).
closet value to 0. If n is a multiple of 30, n could be 30, 60, 90,
120, and so on.
We must pick choice A. (Statement 1 alone is If n is 30, it is not a multiple of 20
sufficient but statement 2 alone is not sufficient). If n were 60, then it is.
Once again, we are looking at a “may be, but not
necessarily” answer here.
We must give up, and pick E.

Some of the materials used in this module are taken from disclosed editions of GMAT, and no monetary
value is included in the cost of this module for supply of these materials. These materials are strictly used
for illustration of concepts discussed. Page 201
©Educational Training Services, A Division of Maple Leaf International Consulting, Inc.
Not to be reproduced, copied or distributed. Intended for use only by the registrant.

13. If Mark saved an average (arithmetic 14. If x is not equal to - 1, which is greater,
mean) of $80 per week for 3 consecutive weeks, 1/(x + 1), or x/2?
how much did he save the second week?
(1) x ≥ 0
(1) The average amount that Mark saved per (2) x < 3
week for the first 2 weeks was $60.
(2) The amount that Mark saved the first week We need to know the actual value of x so that
was 1/2 the amount he saved the second week we can answer the question.
and 1/3 the amount he saved the third week. Statement 1 tells us that x is greater than, or
equal to, 0. If x is 0, then 1/(x+1) > x/2
When you are dealing with a data sufficiency IF x is 1, then the two expressions are equal
question, be sure to make a note of any If x is 2, then x/2 > 1/(x+1)
information that is provided in the stem. You We are getting “all over the map” answers here.
will be required to use this information in Not unique. We must conclude that statement 1
addition to the information in the statements 1 alone is not sufficient. Our choices narrow to B,
and 2, and to decide whether you can make a C or E.
unique decision about the question posed. Statement 2 tells us that x < 3.
If x is 2, then x/2 > 1/(x+1)
Let us say that W1, W2, and W3 are the savings IF x =1, then the expressions are equal.
in the weeks 1,2, and 3 respectively. IF x = 0, then 1/(x+1) > x/2.
We have: W1 + W2 + W3 = 3X80 =$240 Another “all over the map” information.
(An average of $80 per week for 3 weeks Statement 2 alone is also not sufficient.
gives us a total of $240) We have choices C or E to contend with.
We notice that we have a 3-variable equation When we combine the two statements, we notice
here. We will require 3 independent sets of that we are examining the same scenarios that
information to be able to answer the question we did under statements 1 and 2 independently.
about W2. Even the combined information is not sufficient
to answer the question in a unique fashion.
Let us now examine the statements one by one. We must pick E.

Statement 1 tells us that W1 + W2 = 120 15. In a certain two-digit integer, the ratio of
This statement alone is not sufficient, even in the units digit to the tens digit is 2 to 3. What is
the light of the stem information, because Week the integer?
1 savings could be 0 and week 2 savings could
have been all 120, or the other way around (not (1) The tens digit is 3 more than the units digit.
to mention other scenarios in which W1 and W2 (2) The product of the two digits is 54.
can have any combinations of values adding up The information in the stem tells us that we are
to $120). We conclude that statement 1 alone is dealing with one of the following three 2-digit
not sufficient. Our choices narrow to B, C or E. numbers: 32, 64, or 96. Only these numbers
Statement 2 gives us two additional sets of have the digits in the specified ratio. (units’ to
independent information: tens’ is 2 to 3).
W1 = ½ . W2 and W1 = 1/3 . W3 Statement 1 tells us that the tens digit is 3 more
We have to use these independent equations than the units’. The only two digit number that
along with the original equation in the stem. We satisfies this condition and the ratio specified is
have 3 equations and 3 variables. We can find 96. Our choices are A or D.
the values for Week 1, Week 2, and Week 3 Statement 2 tells us that the number must be 96
savings, and answer the question in a definitive because the product of the digits cannot be 54 if
way. We conclude that statement 2 alone is the number were 32 or 64.
sufficient, but statement 1 alone is NOT. We Either statement is sufficient alone. We must
must pick B as the answer here. pick Choice D.

Some of the materials used in this module are taken from disclosed editions of GMAT, and no monetary
value is included in the cost of this module for supply of these materials. These materials are strictly used
for illustration of concepts discussed. Page 202
©Educational Training Services, A Division of Maple Leaf International Consulting, Inc.
Not to be reproduced, copied or distributed. Intended for use only by the registrant.

M
Moodduullee IIII aassssiiggnnm
meennttss E
Exxppllaaiinneedd:: Now try the following on your own and see if you
can simplify the expression to a more manageable
form:
If X is a value such that A and B are the two
factors, we can write X = A.B

In algebraic setting, A and B could stand for


algebraic expressions. For example, A could be
(n-3) and B could be (3n + 7), in which case X
could stand for (n-3) (3n+7) = 3n2 - 2n - 21 1. 6x3+3x2+6x+3 = 6x3+6x+3x2+3
(x2+1) (x2+1)
When you come across an algebraic expression, = 6x(x +1) + 3(x2+1)
2

your first task is to determine if the expression can (x2+1)


be simplified and reduced to a more manageable
form. = (x2+1) (6x+3)
(x2+1)
Let us take an example of a “clumsy” algebraic
expression: = 6x+3 = 3(2x+1)

4x(x-2)-2x+4 = ??
(4x-2) 2. 5x3+7x2-25x-35 = x2(5x+7) –5(5x+7)
(x2-5) (x2-5)
We notice that -2x+4 in the numerator can be
written -2(x-2), and we recognize that (x-2) is = (5x+7)(x2-5)
sitting as a factor in the first part of the numerator (x2-5)
expression.
= (5x+7)
We can write the numerator as:
4x(x-2)-2(x-2) = (x-2) (4x-2)
3. 7x2(6x+5)-18x-15 = 7x2(6x+5) – 3(6x+5)
What did we do here? We noticed that (x-2) was a
(7x2-3) (7x2-3)
common factor, and took it out. We were left with
4x from the first part and -2 from the second part,
= (6x+5)(7x2-3)
and we grouped them together to get (x-2)(4x-2)
(7x2-3)
Let us rewrite the expression in terms of these
=(6x+5)
factors as:

(x-2)(4x-2) = (x-2)
(4x-2)
4. 3x(x2+2)-7x2-14 = 3x(x2+2) –7(x2+2)
This is the summary of the drill we performed:
(3x-7) (3x-7)
1. We recognized common factors in parts of an
expression, and re-wrote a part of the expression in
terms of its factors. = (x2+2)(3x-7)
2. We then took out the common factor to both parts (3x-7)
of the expression, and grouped the remaining values
to write an expression in terms of two factors. = (x2+2)
3. We found to our pleasant surprise that one of the
factors in the numerator expression was the same as
the denominator expression. These values canceled
each other out, leaving behind a simpler, more
manageable expression.

Some of the materials used in this module are taken from disclosed editions of GMAT, and no monetary
value is included in the cost of this module for supply of these materials. These materials are strictly used
for illustration of concepts discussed. Page 203
©Educational Training Services, A Division of Maple Leaf International Consulting, Inc.
Not to be reproduced, copied or distributed. Intended for use only by the registrant.

ASSIGNMENT
(A) x = 2 True / False
1. If x+y is a prime number, then
which of the following “must be” true? x does not have to be equal to 2
Before we begin to take a look at the (B) Either x or y is 2. True / False
conditions specified, let us set up some
scenarios, and test possible real life x and y could be negative fractional
values for x and y. Remember: “Must be values
true” questions should be answered on
the basis of an exception and not on the (C) Both x and y are prime numbers.
basis of a solitary agreement. But if we
can find one exception, then we are ready True/ False
to say “no” to the condition specified.
X and y do not have to be prime integers.
We notice that x and y are not specified See the scenarios we have created at the
as whole numbers. We must be sure to outset.
check positive values, negative values,
whole number values and non-whole (D) x.y is not a prime number.
number values for x and y such that the
True / False
sum will be a prime integer.
x.y could be a prime number because if x
x 1 -1 ½ +7¼ -25
is 1 and y is 2, then the sum is 3, a prime
y 2 +4 ½ -2 ¼ +36 number and the product is 2, a prime
number. We have an exception here, may
x+y 3 3 5 11 be the only exception we will ever have,
but an exceptional scenario is sufficient
Notice that the sum row is full of prime to say “no” to the condition specified.
integers but the x’s and y’s could be
anything we choose them to be. (E) x-y is a prime number. True / False

Now, let us take a look at the conditions x-y does not have to be prime. See the
and see whether they stack up against the scenarios we set up at the outset.
scenarios specified above. If we find one
(F) x.y is a positive integer. True / False
exception, we will say “no” to the
x or y could be negative and
condition stipulated.
fractional, and the product does not
have to be an integer, let alone a
positive one.

Some of the materials used in this module are taken from disclosed editions of GMAT, and no monetary
value is included in the cost of this module for supply of these materials. These materials are strictly used
for illustration of concepts discussed. Page 204
©Educational Training Services, A Division of Maple Leaf International Consulting, Inc.
Not to be reproduced, copied or distributed. Intended for use only by the registrant.

2. If x²y³ is a negative odd integer, then


which of the following must be true?
(D) Both x and y are odd numbers -
Once again, we notice that x and y are fractions or integers. True / False
not specified as whole numbers. We
should be sure to check out fractional see scenarios we set up.
values as possible values for x and y
consistent with the outcome specified.

Because x2y3 is specified as a negative (E) x² + y³ is a positive even integer.


odd integer, we must conclude that y True / False
must be a negative number but x could
x2+y3 does not have to be an integer, let
be positive or negative because of the
alone a positive even integer, because
“power of 2” attached to it.
either x or y could be a fraction.

x -1 1/3 -9

x2 1 1/9 81

y -3 -3 -1/3

y3 -27 -27 -1/27

x2y3 -27 -3 -3

We notice that x and y could be


fractional values as well as whole number
values.

Now, let us take a look at the conditions


specified, and check how they stack up.

(A) y is a negative integer. True/ False.

y is negative but does not have to be an


integer.

(B) x can be a fraction. True / False

see scenarios above

(C) x can be positive or negative, integer


or a Fraction. True / False

see scenarios above


Some of the materials used in this module are taken from disclosed editions of GMAT, and no monetary
value is included in the cost of this module for supply of these materials. These materials are strictly used
for illustration of concepts discussed. Page 205
©Educational Training Services, A Division of Maple Leaf International Consulting, Inc.
Not to be reproduced, copied or distributed. Intended for use only by the registrant.

4. Machine A produces bolts at a


Assignment Continued uniform rate of 120 every 40 seconds,
and machine B produces bolts at a
3. Thames Toyota Dealership has 120 uniform rate of 100 every 20 seconds. If
cars in their lot. The cars come in two the two machines run simultaneously,
colors - Red or Blue and two sizes - mid- how many seconds will it take for them
size and large. There are exactly 10 large to produce a total of 200 bolts?
sized Red cars in the lot. If there are
three times as many mid-sized cars as Machine A produces 3 bolts per second.
there are large ones and the red to blue Machine B produces 5 bolts
cars ratio in the lot is 3:2, How many per second. Together, A and B
cars in the lot are : produce 8 bolts per second.
(A) Red and Mid-size? To produce 200 bolts, working
together, A and B will take
(B) Blue and Large?
200/8 =25 seconds.
(C) Blue and Mid-size?
5. A survey involving two questions Q1
Values indicated in red are specified in and Q2 was answered by N persons. 1/3
the problem. The rest of the values are of those participating answered YES to
obtained by subtracting from the column Q1, and of these 1/5 also answered YES
and row totals. to Q2. If the number of those answering
YES to Q1 but NO to Q2 is twice that
LARGE MID TOTAL answering NO to Q1 and YES to Q2,
what is the number answering NO to
RED 10 62 72 both questions Q1 and Q2?
BLUE 20 28 48 .
TOTAL 30 90 120 YES TO Q2 NO TO Q2 TOTAL
YES TO Q1 1/5(N/3) 4N/15 N/3

NO TO Q1 2N/3
2N/15

TOTAL N

The values in the red cells are computed


from the information in the problem.(Yes
to Q1 and No to Q2 is TWICE Yes to
Q2 and No to Q1)

Some of the materials used in this module are taken from disclosed editions of GMAT, and no monetary
value is included in the cost of this module for supply of these materials. These materials are strictly used
for illustration of concepts discussed. Page 206
©Educational Training Services, A Division of Maple Leaf International Consulting, Inc.
Not to be reproduced, copied or distributed. Intended for use only by the registrant.

6. A triangle has two angles measuring


ASSIGNMENT 55º and 43º. What is the measure of
the third angle?

Answer: _82º
1. A triangle has its 3 sides of
following lengths: 3, 3, 3. What are the 7. An isosceles triangle has two sides
3 angles ? 10 and 10. The base where the altitude
from the vertex meets measures 16.
Answer:__60_º, _60__º, ____60_º What is the area of the triangle?(Draw
triangle, mark sides and angles
2. A right triangle has its hypotenuse appropriately, and solve the problem).
measuring 8 units. One of its angles is
30º. What are the other two side 10 10
lengths?
8 8
Answer:___4___, ____4\/3___
16
3. A triangle has 3 sides of lengths 27,
We notice that the altitude divides the base
36, 45. What type of a triangle is this?
into two equal halves, and the whole triangle
into two 3:4:5 triangles, with the hypotenuse
Answer:____Right Triangle (3:4:5 measuring 10 and one side measuring 8. We
proportions) must conclude that the altitude must measure
6 corresponding to proportion 3. Knowing
4. An equilateral triangle has sides that the height is 6 and the base is 16, we can
measuring “a”. What is the area of the calculate the area: ½ . (6)(16) = 48 sq.units.
equilateral triangle in terms of its side
“a”? 30o 30o Answer: _____48_______
The sides
of a 30-60-
a \/3.a/2 a 90 triangle 8. If AB X CE = 24, What is the area
are in of the triangular shaped region ABD
proportion
60o a/2 a/2 60o 1:\/3:2 shown below?(AB and CD are parallel
a lines)

Area is: \/3/4. (side)2 = \/3/4. a2 C D

5. A right triangle has one leg


measuring 4 and the other 6. What is
the hypotenuse length?(use
Pythagorean theorem and show steps).
A E B K
\/42+62 = \/52 = 2\/13 Hint: The vertical height DK shown in dotted
lines will determine the area of the triangle ABD.
You will notice that the altitude for the triangle
ABD is the same as CE. Area= ½ (24) = 12
sq.units
Some of the materials used in this module are taken from disclosed editions of GMAT, and no monetary
value is included in the cost of this module for supply of these materials. These materials are strictly used
for illustration of concepts discussed. Page 207
©Educational Training Services, A Division of Maple Leaf International Consulting, Inc.
Not to be reproduced, copied or distributed. Intended for use only by the registrant.

ASSIGNMENT Similarly, the sets of values in II and


III will check out ok too. (Try that).
1. If ∠ABC in the figure below is a right You may notice that the values in I
angle, what is the value of external xº? are the values for a right triangle as
well.
xº C
Answer: ___E_____
B 55º D
4. If the length and the width of a
A ∠ABD = 55º rectangular garden plot were each
increased by 25%, what would be the
Angle CBd is 90-55=35o. The external percent increase in the area of the plot?
angle x is the supplementary angle,
and must be 180-35 = 145o (A) 25% (B) 33% (C) 45% (D) 56.25%
(E) cannot estimate from the information
Answer: xº = __145º
The area of a rectangle is the product of
2. If a triangular region has two sides of the two sides. IF L becomes 5/4.L and
value 17 and 25, which of the following W becomes 5/4 .W, then the new area is:
cannot be a value for the third side? 5/4.L.5/4.W = 25/16.LW = 1.5625LW.

(A) 38 (B) 45 (C) 11 (D) 17 (E) 41 We notice that the new area is 56.25%
more than in the original scenario.
The third side must lie in the following
range: 8 < Third Side<42. The only Answer: _____D_____
value that is not in the range is 45.
5. The trapezoid shown below is the
Answer: ___B__ cross section of a rudder of a ship. If the
distance from A to C is 25 feet, what is
3. Which of the following groups of the area of the cross section of the
numbers could be the lengths of the sides rudder in square feet? C
of a triangle?
D 25
I. 1, 4. √17 II. 4, 7, √11 III. 4, 9,6
A 4 7B
Choices: (A) I only (B) I and II only (C)
I and III only (D) II and III only (E) I, II The lengths are: AD= 4’ BC=7’.
and III. (Explain your answers)
(A)168 (B) 175 (C) 100 (D) 132 (E) 28
We apply the range rule here and notice
In the right triangle ABC, AC, hypotenuse, is
that all sets of values will satisfy the rule. 25 and one of the sides BC is 7. We must
For example, the set of values in I are recognize the classical triple 7:24:25 proportion,
consistent with the range rule and conclude that AB must be 24.
requirements: 3<\/17 <5

Some of the materials used in this module are taken from disclosed editions of GMAT, and no monetary
value is included in the cost of this module for supply of these materials. These materials are strictly used
for illustration of concepts discussed. Page 208
©Educational Training Services, A Division of Maple Leaf International Consulting, Inc.
Not to be reproduced, copied or distributed. Intended for use only by the registrant.

The area of a trapezoid is ½ (sum of two parallel 8. In the above question, if the area were
sides).(height separating the two parallel sides) 45, and L and W are integers such that
L >W, how many possible values for W
Area of trapezoid = ½ .24.(4+7) = 132 Choice
D exist?

(A) Two (B) Three (C) Four (D) Five


(E) Six
Assignment Continued... When the area is 45, we can go: 45X1,
15X3, and 9X5. Three combinations
6. If a rectangular photograph that is 10 consistent with the specification that
inches wide by 15 inches long is to be L>W and L.W=45.
enlarged so that the width will be 22
inches and the ratio of length to width B is the answer.
will be unchanged, then the length in
inches of the enlarged photograph will be 9. The shaded portion ABC of a
rectangular lot shown below is a flower
(A) 33 (B) 32 (C) 30 (D) 27 (E) 25 bed. If the area of the flower bed is 24
sq.yds and BC=AB + 2, then AC equals:
This is a simple proportion problem:
(A)√13 (B)2√13 (C) 6 (D) 8 (E) 10
Let X be the new Length.

10/15 = 22/X or X = 33 inches.


C
Choice A
(X+2)

A X B
7. If L and W are the dimensions of a
rectangular region that has area 42 and if We have: ½ .X.(X+2) = 24, or
L and W are integers such that L > W, X.(X+2)=48
What is the total number of possible
values of L? Knowing that X is positive, can we think
of two values that are 2 apart and will
(A) Two (B) Three (C) Four (D) Five multiply to give 48? Yes, 6 and 8. AB
(E) Six must be 6 and BC must be 8. We
recognize the 3:4:5 proportion here, and
The question is: How many different
must conclude that AC must be 10
ways can we set up 42 as the product of
(6,8,10 triangle in 3:4:5 proportion).
two integers? We can go: 42X1, 21X2,
14X3, 7X6 (the first value is that of L Answer: E
and the second that of W). We have 4
possible choices here. C is the answer.

Some of the materials used in this module are taken from disclosed editions of GMAT, and no monetary
value is included in the cost of this module for supply of these materials. These materials are strictly used
for illustration of concepts discussed. Page 209
©Educational Training Services, A Division of Maple Leaf International Consulting, Inc.
Not to be reproduced, copied or distributed. Intended for use only by the registrant.

10. A rectangular region has the same


area as a square region : 64 sq. yards. If 12. Which of the following inequalities is
one length of one side of the rectangle is equivalent to 5x-10y < -20
half the length of the side of the square
and the other side of the rectangle is (A) 2y < -(4+x) (B) x < 2y+4
twice the length of square side, what are (C) x >2y+4 (D) 2y>4+x
the dimensions of the rectangle and the (E) None of the above.
square?(rectangle length, rectangle width, square side)
We notice that the answer choices are in
(A) 8,8,8 (B) 4,16,8 (C) 8,12,12 terms of both X and Y. We need to
(D) 4,8,8 (E) 16,8,8 express the inequality in terms of both
and decide which one matches the
It takes more time to read this answer.
problem than answer it. If the area of
a square is 64, we know that the side 5x-10y < -20 or x –2y <-4
of the square must be 8 units. If the
rectangle has the same area and has or, x < 2y-4 ………………. (1)
one side that is twice the side of the
or, -2y < -4-x or 2y > 4+x ……. (2)
square, we know that the length must
be 16 and the width MUST be 4. We notice that (2) matches choice D.
Choice B. 13. If n is an integer and

n = (2.3.5.7.11.13) ÷ 91p
11. Which of the following
inequalities is equivalent to 10-2x > 18? Which of the following could be the
value of p?
(A) x > -14 (B) x > -4 (C) x >4 (D) x < 4
(E) x < -4 (A) 22 (B) 26 (C) 35 (D) 54 (E) 60

10-2x > 18 or –2X > 8 or -X > 4 This is a problem testing your knowledge
of factors. Let us try to simplify the
Now let us divide both sides by –1 and expression to the right of = sign: We
flip the sign of the inequality. notice that 91 cancels out the 7 and 13
on the numerator leaving us with a
We get X < -4. Choice E simpler n = (2.3.5.11)/p

Knowing that n is an integer, we


conclude that p must be such that n must
continue to be an integer. The only value
for p that will produce an integer value
for n is 22. Choice A.

Some of the materials used in this module are taken from disclosed editions of GMAT, and no monetary
value is included in the cost of this module for supply of these materials. These materials are strictly used
for illustration of concepts discussed. Page 210
©Educational Training Services, A Division of Maple Leaf International Consulting, Inc.
Not to be reproduced, copied or distributed. Intended for use only by the registrant.

14. Mary and Jenny live 13 miles apart. 15. Which of the following equations
They meet at a cafe that is directly north has one root in common with x²-6x+5=0
of Mary’s house and due east of Jenny’s
house. If the cafe is 7 miles closer to (A) x² + 1=0 (B) x²-x-2 = 0 (C)x²-10x-
Mary’s house than to Jenny’s house, how 5=0 (D) 2x² - 2=0 (E) x²-2x-3=0
many miles is the cafe from Jenny’s
house? (Hint: Use Pythagorean theorem and right triangle We notice that the roots of the given
properties discussed.) Draw diagram and show steps. equation are 1 and 5.

(A) 12 (B) 20 (C) 6 (D) 10 (E) 10√3 (x2-6x+5) = (x-1)(x-5) = 0 or x =1 or 5.

We must set up Mary and Jenny on the Now, check to see whether 1 or 5 will
hypotenuse of a right triangle and the satisfy the choice equations. Notice that
configuration will appear as follows: Choice A is not worth messing with
because the x does not have a real root.
Jenny X (x2 = -1, and that cannot be). We must
Cafe not waste time checking out choice A.
We must move on. We will see that
choice D has a root of 1 in common with
the stem equation. D is the answer.
13 (X-7)
Answer:________________

Mary 16. How many positive integers n are


there such that 100n is a factor of:
We see a 13 on the hypotenuse of a right
triangle and must check to see whether (2³ )(5)(5³ )
the other two sides are consistent with
the classical triple, 5:12:13. If one side is (A) None (B) Five (C) Seven (D) Six (E)
7 more than the other, then the sides Eleven
must be 5 and 12. We conclude that the
Café is 5 miles from Mary’s house and When we express the exponential
12 miles from Jenny’s house. expression as a decimal number, we get
5000. If 100n is a factor of 5000, then
Remember: You must learn to recognize
the triples so that you do not waste time 5000/100n = integer or 50/n = integer.
by setting up quadratic equations and
finding the roots. n can have all the positive values that are
factors of 50.
Choice A
Factors of 50 are: 1,50,2,25,5,10.

n can have 6 positive values. D is the


answer.

Some of the materials used in this module are taken from disclosed editions of GMAT, and no monetary
value is included in the cost of this module for supply of these materials. These materials are strictly used
for illustration of concepts discussed. Page 211
©Educational Training Services, A Division of Maple Leaf International Consulting, Inc.
Not to be reproduced, copied or distributed. Intended for use only by the registrant.

9. How many factors or divisors do the


following integers have? What are they?
Assignment: Integers
Explained (p.44) (A) 81 : 1,81, 3, 27, 9
(B) 140:1,140;2,70;4,35;5,28;7,20,10,14.
1. Integer 9 is not a divisor of which of
(C) 256:1,256; 2,128; 4,64; 8,32, 16
the following values of y?
(D) 171: 1, 171; 3,57; 9,19;
(A) 63 (B) 108 (C) 36 (D) 200 (E) 126
10. The difference of two consecutive
even integers is ____2_____________
Your Answer:______D _________
11. The difference of two consecutive
2. Even integer when added to an Odd
odd integers is ______2____________
integer results in _ODD__ integer.
12. What is the next integer in the
3. Even integer when multiplied by an
following sequence of even integers?
even integer results in
___EVEN_____integer.
2n, 2n+2, 2n+4, 2n+6,___2n+8______
4. An odd integer when subtracted from
13. Which of the following are odd, even
an even integer results in
or indeterminate integers? (n is not 0)
_____ODD________ integer.
Odd Even Can’t Say
5. A prime number n is an integer that
(A) 2n+3 x O O
has exactly _TWO_______ different
(B) n+5 O O X
__POSITIVE_ divisors and the divisors
(C) 2n+24 O X O
are 1 AND ITSELF.
(D) 2n-3 X O O
(E) 2(n-7) O X O
6. Which of the following numbers
(F) 3(2n-1) x O O
cannot be prime number(s) and why?
(G) (2n+15) -
(2n-4) X O O
(A)3 (B)169 (C) 32 (D) 83 (E) 59
(H) (2n+64) +
(3n+5) O O X
Your Answer: 32 AND 169 ARE
(I) n(2n-11)÷n X O O
DIVISIBLE BY MORE THAN 2
FACTORS. (J) (2n+1)(2n+6) O X O

7. The only even prime number Hint: n takes on all values (except 0),
positive and negative.
is___2____.

8. The expression n/n=1 is true for all 14. If x and y are prime numbers and
values of n except ___0_________ (x+y) is a also a prime number, then x or
y must be equal to _____2______

Some of the materials used in this module are taken from disclosed editions of GMAT, and no monetary
value is included in the cost of this module for supply of these materials. These materials are strictly used
for illustration of concepts discussed. Page 212
©Educational Training Services, A Division of Maple Leaf International Consulting, Inc.
Not to be reproduced, copied or distributed. Intended for use only by the registrant.

Some of the materials used in this module are taken from disclosed editions of GMAT, and no monetary
value is included in the cost of this module for supply of these materials. These materials are strictly used
for illustration of concepts discussed. Page 213
©Educational Training Services, A Division of Maple Leaf International Consulting, Inc.
Not to be reproduced, copied or distributed. Intended for use only by the registrant.

Some of the materials used in this module are taken from disclosed editions of GMAT, and no monetary
value is included in the cost of this module for supply of these materials. These materials are strictly used
for illustration of concepts discussed. Page 214

S-ar putea să vă placă și